You are on page 1of 443

PROBLEMAS, CUESTIONES Y EJERCICIOS DE

FSICA

JAIME CARRASCOSA ALS SALVADOR MARTNEZ SALA


Doctor en Didctica de las Ciencias Licenciado en Ciencias Fsicas
Catedrtico de Fsica y Qumica Catedrtico de Fsica y Qumica
Aprender a resolver problemas 2

NDICE

Presentacin ----------------------------------------------------------- 1
Aprender a resolver problemas ------------------------------------- 5
1. Cinemtica del punto --------------------------------------------- 15
2. Dinmica del punto ----------------------------------------------- 57
3. Trabajo y Energa -------------------------------------------------- 115
4. Sistemas de partculas -------------------------------------------- 149
5. Slido rgido -------------------------------------------------------- 189
6. Campo gravitatorio ------------------------------------------------ 261
7. Campo elctrico ---------------------------------------------------- 293
8. Movimiento ondulatorio ------------------------------------------ 325
9. Naturaleza de la luz ----------------------------------------------- 363
10. Fsica nuclear ----------------------------------------------------- 389
Anexo: Vectores ------------------------------------------------------ 415

1997. Jaime Carrascosa Als


y Salvador Martnez Sala.
Generalitat Valenciana.
Valencia (Espaa)
Versin revisada. Valencia. 29 de febrero de 2016.
Aprender a resolver problemas 3

PRESENTACIN
Qu es este libro? Uno ms de problemas?

ste es un libro de cuestiones, ejercicios y problemas de Fsica, elaborado por profesores con
una larga experiencia docente. Nuestro inters en este tema tiene su origen en las graves difi-
cultades que muchos estudiantes tienen para resolver problemas de Fsica. Una reflexin crti-
ca de nuestro propio trabajo en el aula nos ha llevado a cuestionar no pocas de nuestras ideas
y comportamientos iniciales respecto a las diversas causas de dichas dificultades as como a la
bsqueda de nuevas orientaciones que nos permitan, a profesores y alumnos, hacer frente de
una forma ms eficaz al problema de los problemas.

Afortunadamente, en ese proceso de bsqueda de alternativas hemos contado con la ayuda de


otros colegas que tambin estn trabajando sobre el mismo tema y hemos tenido acceso a
muchas de las investigaciones ms importantes que desde la Didctica de las Ciencias se han
realizado en el campo de la resolucin de problemas de lpiz y papel. Durante varios aos
hemos tratado de plasmar las nuevas orientaciones didcticas en ejemplos concretos de reso-
lucin de problemas, que hemos ensayado con nuestros propios alumnos, desarrollando un
trabajo que creemos satisfactorio y gratificante tanto para ellos como para nosotros. En este
libro hemos incluido gran parte de esos problemas, acompaados tambin de numerosos ejer-
cicios de manejo y de cuestiones para la clarificacin de conceptos fsicos importantes.

Cules son sus contenidos y en qu orden se presentan?

Los contenidos que se incluyen en este libro corresponden a los siguientes temas:

Aprender a resolver problemas 6. Campo gravitatorio


7. Campo elctrico
1. Cinemtica del punto 8. Movimiento ondulatorio
2. Dinmica del punto 9. Naturaleza de la luz
3. Trabajo y energa 10. Fsica nuclear
4. Sistemas de partculas
5. Slido rgido Anexo sobre clculo vectorial

Nuestro propsito es revisar continuamente estos contenidos tratando de mejorarlos y am-


pliarlos incluyendo ms problemas complementarios y tambin otros nuevos captulos.

El libro cuenta con una introduccin (Aprender a resolver problemas: una asignatura pen-
diente) del profesor Daniel Gil, en la que se exponen de forma breve y concreta las orienta-
ciones que hemos intentado dar a muchos de los problemas que en l se incluyen y cuya lectu-
ra, o mejor an, realizacin de las actividades que en l se proponen, recomendamos vivamen-
te antes de comenzar a trabajar en los restantes temas.

El orden en que se presentan los distintos captulos as como la secuenciacin de los propios
contenidos de cada uno de ellos, obedece a una lgica interna que permite ir avanzando en la
construccin de un cuerpo coherente de conocimientos de Fsica. Ello hace aconsejable que
los captulos se vayan trabajando siguiendo el orden en que se presentan en el ndice. No obs-
tante, dado que el clculo vectorial constituye un prerrequisito para gran parte de los temas
Aprender a resolver problemas 4

tratados, es necesario asegurarse primero de que se domina este tema, por lo que conviene,
que despus de la introduccin, se resuelvan los ejercicios sobre vectores que se incluyen en el
anexo al final del libro.

A quienes va dirigido?

El contenido se ajusta bastante al programa oficial para las asignaturas de Fsica y Ampliacin
de Fsica de segundo de Bachillerato, pero el libro puede ser utilizado tambin con provecho
por estudiantes de un primer curso de Fsica en la Universidad.

Cmo conviene utilizarlo?

Los problemas, ejercicios y cuestiones estn resueltos en su mayora y el resto viene con las
soluciones. En los problemas, la resolucin se ajusta esencialmente a las orientaciones bsicas
que se detallan en la introduccin. En general se ha tratado de incluir, siempre que ha sido
posible, un planteamiento cualitativo inicial, considerando diversas estrategias de resolucin y
analizando los resultados obtenidos, huyendo de la simple aplicacin de frmulas y de los
clculos numricos precipitados.

Con el propsito explcito de contribuir a que el estudiante no sea un simple consumidor pasi-
vo, se hacen frecuentes preguntas (en letra cursiva) en las que se invita a interrumpir la lectu-
ra y pensar sobre ellas tratando de contestarlas. Despus hay que comparar el trabajo personal
con lo que se presenta en el texto. Este procedimiento suele producir bastante satisfaccin
cada vez que se da una concordancia, pero, aunque en ocasiones no sea as, vale la pena reali-
zarlo ya que los aprendizajes que se producen despus de haber intentado seriamente resolver
una cuestin, son siempre mucho ms fructferos. El libro ser pues especialmente til a aque-
llos que estn dispuestos a realizar el esfuerzo que conlleva un aprendizaje que vaya ms all
de la simple memorizacin.

Coherentemente con las orientaciones anteriores, hemos optado por no incluir en los distintos
captulos ningn formulario y recomendar que el libro se complemente con un buen texto
terico y/o unos buenos apuntes del trabajo realizado en el aula. No obstante, es preciso tener
en cuenta que en cada captulo los problemas no se presentan aleatoriamente sino que siguen
una secuencia bastante precisa, cubriendo una serie de conocimientos tericos de manera pro-
gresiva. Ello hace muy conveniente ir trabajando los ejercicios de cada captulo de forma or-
denada.

En la seleccin y secuenciacin de los contenidos de cada captulo y del libro en general,


hemos intentado evitar los tratamientos puntuales y favorecer que los fenmenos fsicos que
se estudian as como las magnitudes fsicas que se introducen para ello, puedan ser compren-
didas e integradas por los estudiantes. Ello nos ha hecho dar una importancia especial a todas
aquellas expresiones que tienen un campo de validez ms amplio, es decir que se pueden ma-
nejar en situaciones muy diversas.

Agradecimientos y recomendaciones finales

Los autores queremos expresar nuestro agradecimiento a todos los compaeros que, con sus
crticas y discusiones han contribuido a la elaboracin de este libro, a nuestros alumnos del
Aprender a resolver problemas 5

Instituto de Bachillerato Cid Campeador de Valencia que nos han permitido darnos cuenta
de muchos errores de todo tipo y especialmente al profesor Daniel Gil por su colaboracin y
sus acertadas sugerencias.

Casi todo el mundo sabe que aprender bien algo exige un cierto esfuerzo. Los autores de
este libro no queremos engaar a nadie diciendo que con este texto se aprende Fsica sin
esfuerzo. Por el contrario, quienes lo utilicen y quieran aprovecharlo, han de estar dispues-
tos a realizar un trabajo serio. Por nuestra parte hemos intentado orientar los contenidos
del libro para que se ese trabajo se pueda hacer bien y esperamos que al finalizarlo no solo
hayis aprendido algunas cosas sobre Fsica y resolucin de problemas, sino que adems
haya aumentado vuestro inters por esta materia y pensis que el esfuerzo realizado ha
valido la pena. Si eso se cumple, este libro habr sido algo ms que un libro ms de pro-
blemas.

Textos tericos recomendados:

Por su coherencia con los contenidos y metodologa empleados en este libro, recomenda-
mos como obras de consulta y profundizacin los textos siguientes:

Fsica y Qumica de 1 de Bachillerato, de Jaime Carrascosa Als, Joaqun Martnez To-


rregrosa, Salvador Martnez Sala y Juan Jos Ruz.

Fsica de 2 de Bachillerato, de Jaime Carrascosa Als, Salvador Martnez Sala y Manuel


Alonso Snchez.

Los autores manifestamos que:

Los contenidos de esta obra se pueden reproducir total o parcialmente de forma


libre y gratuita. Los autores no solo lo autorizamos expresamente sino que nos
congratulamos de ello. Tan solo pedimos que se indique la fuente y que, por fa-
vor, siempre que sea posible, se colabore en su difusin dndolos a conocer a
otras personas a las que tambin pudieran resultar tiles.

Nuestro objetivo es contribuir, en lo que podamos, a la mejora de la enseanza y


aprendizaje de la Fsica y Qumica.

PERMITIDA LA REPRODUCCIN TOTAL O PARCIAL DE ESTE LIBRO,


CITANDO SIEMPRE AUTORES Y FUENTE
Aprender a resolver problemas 6

APRENDER A RESOLVER PROBLEMAS: UNA ASIGNATURA PENDIENTE

Los problemas "de lpiz y papel" constituyen, como es bien sabido, la causa principal del fra-
caso de muchos estudiantes en el aprendizaje de materias como la Fsica. A menudo se pre-
tende explicar dicho fracaso con argumentos que se refieren a las insuficiencias de capacidad
y de esfuerzo de los estudiantes ("no dominan la teora") o a los "fallos de atencin" cuando
leen un enunciado.

Pero, hasta qu punto ese fracaso generalizado puede atribuirse nicamente a los estudian-
tes? De hecho, muchos de los que no logran resolver un problema muestran un conocimiento
correcto de "la teora", estn interesados en aprender y resuelven sin dificultad muchos ejerci-
cios estndar. Las explicaciones habituales sobre el fracaso en la resolucin de problemas
parecen, pues, insuficientes.

Ello ha dado lugar a una abundante investigacin didctica con objeto de profundizar en las
razones de esta preocupante situacin y buscar vas de solucin. Los resultados de estas inves-
tigaciones permiten afirmar que el fracaso en la resolucin de problemas de Fsica, o de otras
materias como las Matemticas o la Qumica, tiene otras causas y, lo que es ms importante,
que es posible proporcionar a los estudiantes unas estrategias de resolucin ms efectivas
que les permitan enfrentarse con xito a problemas que inicialmente puedan parecerles in-
abordables.

Este captulo est destinado a que los estudiantes se apropien de esas nuevas estrategias, que
podrn poner en juego reiteradamente en el resto de los captulos del libro hasta familiarizarse
plenamente con ellas, comprobando cmo su capacidad para resolver problemas aumenta
notablemente y que es posible hacer mucho ms que reconocer y repetir problemas ya
hechos...o abandonar.

1. ALGO MS QUE DEFICIENCIAS DE LOS ESTUDIANTES

Conviene comenzar tomando conciencia de algunos defectos habituales en la forma de resol-


ver problemas que pueden pasar inadvertidos. Proponemos para ello el siguiente pequeo
ejercicio, cuya realizacin (inicialmente de forma individual) favorece una fecunda discusin
posterior:

Un objeto se mueve a lo largo de su trayectoria segn la ecuacin: e = 25 + 40t - 5t2 (e en


metros si t en segundos). Qu distancia habr recorrido durante los 5 primeros segundos?

El resultado que obtienen muchas personas es, o bien 100 m o bien 75 m. Eso es lo que ocurre
en general cuando ese "sencillo ejercicio" se propone a estudiantes, tanto de secundaria como
universitarios, en ms del 90 % de las respuestas. Si este ha sido tambin tu caso, sin entrar en
la discusin de esta discrepancia, te proponemos:

Calcular la distancia recorrida por el mismo mvil a los seis segundos.


Aprender a resolver problemas 7

Los resultados obtenidos ahora son, normalmente, 85 m (quienes antes obtuvieron 100 m) o
60 m (quienes obtuvieron 75 m). Algo, pues, va mal: El mvil no puede haber recorrido en
ms tiempo menos distancia! La resolucin de este aparente enigma es, por supuesto, sencilla
y te proponemos que t mismo intentes descifrarlo:

Discute con otros compaeros el ejercicio propuesto y las posibles razones de los resultados
obtenidos replanteando su resolucin para superar las incoherencias aparecidas.

Una pequea reflexin permite comprender que la ecuacin dada (e = 25 + 40t - 5t2) corres-
ponde al movimiento de un objeto cuya posicin inicial est a 25 m del origen y que avanza
con rapidez decreciente (v0 = 40 m/s y aceleracin tangencial negativa a = -10 m/s2 ) hasta
pararse y comenzar a retroceder. Es lo que ocurre, por ejemplo, cuando lanzamos un objeto
hacia arriba: el objeto va perdiendo velocidad, se para y cae....

Se obtienen as los resultados correctos, que son 85 m a los 5 s (80 m hacia delante y 5 m
hacia atrs) y 100 m a los 6 s (80 m hacia delante y 20 m hacia atrs). Pero lo que nos interesa
aqu, es reflexionar sobre el hecho de que un problema tan sencillo conduzca a resultados
errneos de forma tan mayoritaria.

A qu cabe atribuir unos resultados errneos tan generalizados en un problema como el


anterior? De qu pueden ser ndice? Qu sugieren?

Los resultados del ejercicio que acabamos de comentar ponen en cuestin la orientacin dada
habitualmente a la resolucin de problemas, caracterizada, entre otros, por los siguientes as-
pectos:

- La falta de reflexin cualitativa previa, o, dicho de otro modo, el operativismo mecnico con
que se abordan habitualmente los problemas, incluso, a veces, por los mismos profesores.
Conviene recordar a este respecto las palabras de Einstein:

"Ningn cientfico piensa con frmulas. Antes que el fsico comience a calcular debe tener en su cerebro el curso de
los razonamientos. Estos ltimos, en la mayora de los casos, pueden ser expuestos con palabras sencillas. Los
clculos y las frmulas constituyen el paso siguiente".

Sin embargo, insistimos, la orientacin habitual de la resolucin de problemas suele impulsar


el manejo abstracto de frmulas, buscando ecuaciones que relacionen datos e incgnitas y
ponindose a realizar clculos inmediatamente.

- Un tratamiento superficial que no se detiene en la clarificacin de los conceptos. As, en el


problema considerado, se producen evidentes confusiones entre diferentes magnitudes como
e (espacio o posicin sobre la trayectoria), e (desplazamiento espacial) y la distancia reco-
rrida. Ms an: se manejan casi exclusivamente situaciones que favorecen las confusiones. En
el caso que nos ocupa, por ejemplo, la mayor parte de los problemas sobre mviles toman
como sistema de referencia (explcita, o, mas a menudo, implcitamente) el punto e instante
en que el movimiento se inicia y sentido positivo el del movimiento, con lo cual el espacio e
coincide con el desplazamiento; si adems no hay retrocesos, el valor de la distancia recorri-
Aprender a resolver problemas 8

da coincide tambin. De este modo, los problemas as planteados, en vez de ayudar a romper
con visiones confusas, favorecen su afianzamiento.

Una discusin como la anterior permite que, tanto estudiantes como profesores, tomemos
conciencia de las deficiencias de la orientacin habitual de la resolucin de problemas y com-
prendamos la necesidad de un replanteamiento en profundidad de la misma.

2. NECESIDAD DE UN REPLANTEAMIENTO EN PROFUNDIDAD

Las mayores dificultades que a menudo ha encontrado el desarrollo de una ciencia han deri-
vado de supuestos implcitos, aceptados sin cuestionamiento alguno, que escapan as a la
crtica. En tales casos se impone -como la historia de las ciencias ha mostrado reiteradamente-
un replanteamiento en profundidad que analice crticamente hasta lo que parece ms evidente.
Por lo que se refiere a la orientacin de la resolucin de problemas, ello supone descender
hasta la clarificacin misma de la idea de problema. Esta ser, pues, la siguiente actividad:

Qu hemos de entender por problema?

Existe un acuerdo general en entender como problemas aquellas situaciones que plantean
dificultades para las que no se poseen soluciones hechas. As Polya, un conocido investigador
en el campo de la resolucin de problemas, seala que:

"Resolver un problema consiste en encontrar un camino all donde previamente no se conoca tal, encontrar una
salida para una situacin difcil, para vencer un obstculo, para alcanzar un objetivo deseado que no puede ser
inmediatamente alcanzado por medios adecuados".

Conviene ahora que nos planteemos la relacin entre dichas ideas sobre lo que son los pro-
blemas y lo que se hace en clase:

En qu medida la forma en que suele abordarse la resolucin de problemas est de acuerdo


con su naturaleza de tarea desconocida para la cual, de entrada, no se posee la solucin?

Esperamos que la discusin propiciada por esta actividad os habr permitido poner en cues-
tin la prctica habitual de resolucin de problemas. En efecto, los problemas suelen ser abor-
dados como algo que se debe saber hacer, algo cuya solucin ha de conocerse (si se sabe la
teora correspondiente y se tiene la base matemtica necesaria) y no generar dudas ni exigir
tentativas. De hecho, cuando un profesor resuelve un problema (bien sea en la pizarra o en las
pginas de un texto), conoce la situacin (para l no es un problema) y la explica "con toda
claridad", sin dudas de ningn tipo; consecuentemente, alumnos y alumnas pueden aprender
dicha solucin y repetirla ante situaciones idnticas o casi idnticas, pero difcilmente apren-
den a abordar un verdadero problema y cualquier pequeo cambio puede suponer dificultades
insuperables provocando el abandono. Podemos dar un paso ms y plantearnos:

Si un problema es una situacin para la que no se tiene respuesta elaborada, cmo habr
que enfocar su resolucin?

Si se acepta la idea de que todo problema es una situacin ante la cual se est inicialmente
perdido, una posible orientacin consistira en preguntarse qu hacen los cientficos en este
Aprender a resolver problemas 9

caso? Con ello planteamos concretamente qu es lo que hacen los cientficos delante de lo
que para ellos constituye un verdadero problema. La respuesta es "simplemente" que... se
comportan como investigadores, enfrentndose a situaciones, que les resultan desconocidas,
de una forma tentativa, construyendo hiptesis con ayuda de los conocimientos que poseen (y
tambin de su inventiva), ponindolas cuidadosamente a prueba, rectificndolas o aban-
donndolas a la luz de sus resultados y de los de otros investigadores, etc.
No hay, pues, ningn mtodo preciso con reglas rgidas, no hay un camino que podamos
aprender y seguir mecnicamente para resolver problemas en general... ni para cada problema
en particular. Intentar aprender el camino, la receta, para resolver cada problema, est abo-
cado al fracaso, pues cada pequea variacin desconcierta y hace que nos sintamos perdidos.
Se trata, por el contrario, de aceptar que estamos enfrentndonos a una situacin nueva, ms o
menos familiar pero no completamente conocida... y aprender a trabajar tentativamente como
hacen los cientficos, imaginando (a la luz de los conocimientos tericos existentes) posibles
soluciones y probndolas, analizando los resultados, etc.
sta nueva orientacin ha sido ensayada reiteradamente con resultados muy satisfactorios,
transformando la actividad de los estudiantes en una actividad ms creativa, interesante y efi-
caz. Proponemos, pues, analizar esa orientacin investigativa (que presentamos detalladamen-
te en el prximo apartado) y ponerla en prctica cada vez que os encontris delante de lo que
para vosotros constituya un verdadero problema.

3. LA RESOLUCIN DE PROBLEMAS COMO INVESTIGACIN

Digamos ante todo que las orientaciones que vamos a exponer con cierto detalle a continua-
cin no constituyen una receta que pretenda guiar paso a paso vuestra actividad. Al contrario,
se trata de indicaciones genricas destinadas a llamar la atencin contra ciertos "vicios" muy
frecuentes que nos alejan de un comportamiento cientfico, como, p.e., la tendencia a caer en
un manejo ciego de datos y ecuaciones (ponindonos a calcular precipitadamente) o pensar en
trminos de certeza, lo que se traduce en no buscar caminos alternativos de resolucin, en no
poner en duda y analizar cuidadosamente los resultados, etc.
Conviene advertir que no se trata de que intentis aprender "en abstracto" las orientaciones
que siguen: es mucho ms eficaz verlas "en acto" y, sobre todo, comenzar a utilizarlas contan-
do con la ayuda de vuestro profesor. Hemos incluido, a ttulo de ejemplo, el tratamiento deta-
llado de un problema, que permitir comprender mejor el sentido de las propuestas realizadas:

Sobre un mvil de 1000 kg de


masa que se desplaza con una
rapidez de 108 km/h, comienza
a actuar una fuerza de frenado
constante de 7500 N
Qu distancia recorrer hasta
pararse?

*Considerar cul puede ser el inters de la situacin problemtica abordada


Aprender a resolver problemas 10

El clculo de la distancia que recorre un mvil hasta pararse tiene un indudable inters, por
ejemplo, en el aterrizaje de aviones o en el trfico de vehculos en general (por carretera y
ciudad), en donde interesa asegurar una distancia mnima de frenada para evitar accidentes, lo
que lleva a limitar la velocidad mxima, guardar una cierta distancia entre vehculos, perfec-
cionar los sistemas de frenado, etc.

*Comenzar por un estudio cualitativo de la situacin, intentando acotar y definir de manera


precisa el problema, explicitando las condiciones que se consideran reinantes, etc.

En este problema un mvil lleva una cierta velocidad inicial y frena hasta que se para, pidin-
donos la distancia que recorre durante este proceso. Las fuerzas que actan sobre el mvil son

tres: el peso P , la fuerza normal R ejercida por la carretera y la fuerza F de frenado. Las dos

primeras son perpendiculares a la trayectoria y se anulan entre ellas, mientras que F es tangen-
te. Para simplificar el problema, supondremos que la fuerza de frenado es constante, de modo
que el movimiento ser rectilneo y uniformemente acelerado. Como la fuerza de frenado
tiene sentido contrario al movimiento, la rapidez inicial del mvil ir disminuyendo lineal-
mente desde v0 hasta 0. En trminos de trabajo y energa se podra describir diciendo que el
trabajo realizado por la fuerza de frenado tiene como efecto disminuir la energa cintica hasta
hacerla igual a 0.

La figura anterior representa un esquema de la situacin, en donde podemos ver el punto que
se toma como origen de espacios, las fuerzas que se ejercen sobre el mvil y cmo la veloci-
dad del mismo se va haciendo cada vez ms pequea hasta que finalmente el mvil se para.

En el problema se nos pide la distancia D que recorre el mvil desde que comienza a frenar
hasta que se para, lo cual, si tomamos como origen de espacios el punto O de la figura y como
sentido positivo el del movimiento, equivale a determinar el valor de e en el instante en que
la rapidez valga 0. En trminos de trabajo y energa dicha distancia coincidir con el mdulo
del desplazamiento experimentado por el mvil desde que comienza a frenar hasta que se
para.

*Emitir hiptesis fundadas sobre los factores de los que puede depender la magnitud buscada
y sobre la forma de esta dependencia, imaginando, en particular, casos lmite de fcil inter-
pretacin fsica.

En el ejemplo que nos ocupa, podemos pensar que la distancia D, va a depender de la rapidez
a la que vaya el mvil en el momento que empiece a frenar, de modo que (a igualdad de los
restantes factores) cuanto mayor sea sta, ms grande ser la distancia que recorrer hasta
pararse. Otro factor que ha de influir es la fuerza resultante de frenado que acte sobre el
vehculo. Parece evidente, que cuanto mayor sea dicha fuerza menor distancia precisar para
Aprender a resolver problemas 11

pararse (siempre a igualdad del resto de los factores). Tambin podemos pensar en la influen-
cia de la masa. Algunas personas creen que si tiene una masa muy grande se parar antes, pero
si reflexionamos un poco nos daremos cuenta que sera al contrario, ya que, por ejemplo,
cuesta ms parar un camin que vaya a 100 km/h que una motocicleta a la misma velocidad,
por tanto, cuanto mayor masa tenga, ms distancia precisar para parar. (En ocasiones se se-
ala tambin el tiempo que dure la frenada, sin embargo este factor est implcito ya en los
factores enunciados). As pues, la distancia recorrida hasta pararse podr expresarse en fun-
cin de las magnitudes citadas:

D = D (v0, m, F)

Las magnitudes anteriores (v0, m, F) constituyen, de hecho, los datos que se suministran en el
enunciado del problema. Adems de aventurar la forma en que cabe esperar que influyan en
D, podemos considerar alguna condicin lmite, como por ejemplo, que si la fuerza resultante
de frenado fuese nula no se parara, es decir, D sera infinita o que si v0 = 0, D tendr que va-
ler 0 (ya estara parado).

*Elaborar y explicitar posibles estrategias de resolucin antes de proceder a sta, evitando el


puro ensayo y error. Buscar distintas vas de resolucin, para posibilitar la contrastacin de
los resultados obtenidos y mostrar la coherencia del cuerpo de conocimientos de que se dis-
pone.

Una posible estrategia de resolucin es utilizar las ecuaciones cinemticas y dinmicas co-
rrespondientes al movimiento del vehculo:

Al ser la trayectoria fija y conocida podemos realizar un tratamiento escalar para resolver el
problema. Si escogemos como origen de espacios el punto donde comienza a frenar, origen de
tiempos el instante en que lo hace y sentido positivo el del movimiento, tendremos:

Aceleracin tangencial del mvil (constante): at = -F/m (1)


Rapidez del mvil en cualquier instante t del movimiento: v v0 a t (2)

a t2
Posicin del mvil en cualquier instante t del movimiento: e e0 v0t (3)
2

La determinacin de la distancia D (e en el instante en que v = 0) podramos efectuarla me-


diante la ecuacin del movimiento (3), si supisemos el instante t en que el mvil se para. ste
ltimo lo podemos hallar fcilmente haciendo v = 0 en la ecuacin (2) y despejando t. Para
ello hemos de calcular previamente la aceleracin mediante la ecuacin (1).

Otra posibilidad es utilizar la ecuacin que nos relaciona el trabajo realizado por la fuerza
resultante a lo largo del desplazamiento que abarca el frenado, con la variacin de energa
cintica (teniendo en cuenta que la energa cintica final ser 0) y despejar el mdulo del des-
plazamiento, que en este caso coincide con la distancia recorrida D.

*Realizar la resolucin propiamente dicha.


Aprender a resolver problemas 12

De las estrategias expuestas vamos a desarrollar la segunda (basada en consideraciones de


trabajo y energa), designando como estado A el que corresponde a la situacin del mvil
cuando empieza a frenar (vA = 108 km/h = 30 m/s), y como B cuando se para (su velocidad se
hace 0).

La ecuacin que relaciona el trabajo realizado por la fuerza resultante con la variacin de
energa cintica es:

1 1
Wres A Ec BA Ec B Ec A = mv 2B mv 2A
B

2 2

En nuestro caso, la fuerza resultante es la de frenado F (tangente a la trayectoria), que siempre
tiene sentido contrario al movimiento, la energa cintica en la situacin B es 0 (se para) y el
mdulo del desplazamiento coincide con la distancia recorrida D. Introduciendo estas condi-
ciones en la ecuacin anterior:

1 mv 2A
F D mv 2A y despejando D obtenemos: D
2 2F

1000 900
Ahora, basta sustituir los datos numricos y operar para obtener D = = 60 m
15000
.

*Analizar cuidadosamente los resultados a la luz de las hiptesis elaboradas y, en particular,


de los casos lmite considerados.

Si analizamos el resultado del problema que estamos resolviendo, podemos darnos cuenta en
primer lugar que el valor numrico obtenido (60 m) parece normal (est de acuerdo con nues-
tras experiencias cuando frena un automvil). Si hubiese sido, por ejemplo, 06 m o 6000 m
habra que dudar y revisar el problema.

mv 2A
Si nos detenemos en el resultado literal: D
2F

podemos ver que la ecuacin es dimensionalmente homognea (dimensiones de una longitud


a ambos lados de la igualdad). Ello no es garanta de que el ejercicio est bien resuelto, pero si
no se diese dicha homogeneidad s que sera indicativo de alguna equivocacin. Por otra parte,
en el resultado se contemplan las hiptesis de partida y es fcil ver que cuando la masa y/o la
rapidez inicial aumentan (para una cierta fuerza de frenado), la distancia D necesaria para
pararse tambin aumenta (necesidad de limitar la velocidad mxima a la que pueden circular
los vehculos, especialmente los de gran tonelaje), que si la fuerza F aumenta (manteniendo
fijas la masa y rapidez inicial) D disminuir (conveniencia de disear sistemas de frenada
apropiados) o que si vA = 0, la distancia D = 0, etc.

Adems, dicho resultado nos permite aprender que, en contra de lo que piensan algunas per-
sonas, si la rapidez inicial con que se mueve un vehculo se duplica, la distancia que recorre
hasta pararse no se hace tambin el doble sino 4 veces mayor, ya que la rapidez inicial est
elevada al cuadrado, es decir, es la variable que ms influye.
Aprender a resolver problemas 13

Finalmente, si queremos, podemos resolver el problema mediante la otra estrategia (basada en


un tratamiento puramente cinemtico-dinmico) y comprobar que el resultado es el mismo.

*Considerad las perspectivas abiertas por la investigacin realizada contemplando, por


ejemplo, el inters de abordar la situacin a un nivel de mayor complejidad o considerando
sus implicaciones tericas (profundizacin en la comprensin de algn concepto) o prcticas
(posibilidad de aplicaciones tcnicas). Concebir, muy en particular, nuevas situaciones a
investigar, sugeridas por el estudio realizado.

Al igual que ocurre en una verdadera investigacin, los resultados pueden ser origen de nue-
vos problemas. Sera conveniente que los estudiantes (y los profesores) llegasen a considerar
este aspecto como una de las derivaciones ms interesantes de la resolucin de problemas,
poniendo en juego de nuevo su creatividad.

En el caso que estamos considerando, podemos plantearnos, a la luz del resultado obtenido,
nuevos problemas de inters prctico, como por ejemplo:

Qu fuerza se ejerce sobre un coche en un choque frontal contra un obstculo fijo?

La fuerza que se pide ser la que ejerce el obstculo y para determinarla, siguiendo con nues-
tro ejemplo, basta con despejar F en la ecuacin anterior, teniendo en cuenta que si el obstcu-
lo es fijo y grande (por ejemplo un muro) la distancia recorrida se reduce a lo que da de s la
carrocera del vehculo. As si suponemos D = 2 m, la fuerza sera:

mv 2A 1000 900
F = 225000 N
2D 4

Como se puede ver fcilmente, en la situacin descrita el valor de la fuerza que el obstculo
ejerce sobre el coche, resulta ser igual que el peso de un objeto de casi 23 toneladas de masa,
lo que permite comprender las consecuencias fatales de un choque frontal a esa velocidad. El
problema se agrava cuando en lugar de ser contra un muro es contra otro vehculo que va en
sentido contrario (en cuyo caso vA sera la suma de los mdulos de ambas velocidades).

Otra cuestin interesante es:

Qu es lo que ocurre cuando un vehculo frena tan intensamente que bloquea las ruedas
impidiendo el giro de stas?

En la nueva situacin planteada, la fuerza F de frenado ser la fuerza de rozamiento mxima


por deslizamiento entre las ruedas y la carretera, cuyo mdulo viene dado por la expresin:
Froz = N siendo N el mdulo de la componente normal de la fuerza que el vehculo ejerce
sobre la carretera (cuyo valor, en este caso, coincide con el del peso, mg).

Sustituyendo F por Froz = mg, el resultado se convierte en:

mv 2A mv A2 v2
D = A
2F 2F 2F
Aprender a resolver problemas 14

Este ltimo resultado nos muestra que ningn vehculo puede frenar y parar en el acto, sino
que siempre recorrer una cierta distancia, tanto ms grande cuanto menor sea el coeficiente
de rozamiento con el suelo y mayor sea la velocidad a la que vaya (de ah el peligro que repre-
senta conducir a gran velocidad sobre un pavimento mojado o con placas de hielo). Por otra
parte, nos remite a la necesidad de que entre los vehculos se respete una distancia mnima de
seguridad (que hay que calcular en funcin de la rapidez mxima permitida, el coeficiente de
rozamiento y el tiempo que tarda un conductor en reaccionar ante la vista de un obstculo).
Adems es necesario tener en cuenta que lo que es verdaderamente limitante no es la fuerza
con que se pueda frenar sino la velocidad a la que se circula, ya que si sta es muy grande, el
hecho de parar en muy poca distancia podra matar al conductor por la enorme aceleracin a
la que se vera sometido.

Como podis comprobar, las orientaciones para aprender a resolver problemas que hemos
tratado de fundamentar en este captulo, son bastante coherentes con algunas de las caracters-
ticas del trabajo cientfico y particularmente con los aspectos ms importantes comunes a la
investigacin cientfica, como son la invencin de hiptesis, la elaboracin de diferentes dise-
os para contrastarlas, el anlisis de los resultados, el planteamiento de nuevas interrogantes,
etc. Se trata pues de una propuesta investigativa que conviene aplicar siempre en la medida
de lo posible para resolver problemas. No obstante, ello, no invalida el importante papel que
pueden representar en el aprendizaje de la Fsica otro tipo de actividades como pueden ser los
ejercicios de aplicacin o las cuestiones destinadas a la clarificacin de conceptos, que tam-
bin se han incluido abundantemente en este libro.
1. CINEMTICA DEL PUNTO


1. El vector de posicin de un mvil viene dado por: r 2t 2i (2t 1) j 2k m (si t en
s). Determinad:

a) Su posicin en los instantes 0, 1, 2, 3, y 4 (todos ellos expresados en segundos).


b) La ecuacin de su trayectoria.

sol: a) (0,1,2) m; (2,3,2) m; (8,5,2) m; (18,7,2) m; (32, 9, 2) m respectivamente.


b) x = 2t2, y = 2t+1, z = 2

2. Representad la funcin e(t) del mvil de la figura adjunta (cada divisin es 1 m).

Como sabemos, la funcin e(t) determina la posicin que ocupa, en cada instante, un
mvil que se desplaza sobre cierta trayectoria conocida. Esta funcin, como todas, admite
una doble representacin: analtica y grfica y en la prctica se puede pasar, con los cono-
cimientos matemticos adecuados, de una a otra forma.

En un caso como el que nos ocupa, vemos que con la informacin que se nos da resulta
imposible obtener directamente la expresin analtica, pero, en cambio, s que podemos
obtener la representacin grfica (esto suceder siempre que partamos de datos experi-
mentales).

A partir de la figura del enunciado, podemos construir una tabla con los valores que estn
presentes en ella como datos:
Cinemtica 16

t (s) 0 1 2 3
e (m) -3 -1 1 3

Si representamos los datos anteriores en una


e (m)
grfica e-t, observaremos que aparecen cuatro
puntos alineados segn una recta y, extrapolando
los resultados, podemos imaginar que si hubise-
mos conocido ms puntos tambin estaran igual-
t (s)
mente alineados, lo que significa que la grfica
ser una recta (funcin lineal).

Es cierto que si solo tenemos en cuenta los datos


que nos dan podra suceder que la funcin e(t)
tuviese la forma de la figura de la derecha, pero
es fcil imaginar que dar 4 valores aleatoriamen-
te y que estn en lnea recta es ms que una ca-
sualidad. Con esto queremos decir que, aunque
es posible, no es probable que as suceda.

Si la representacin grfica de la funcin es una recta, ser sencillo obtener, a partir de


ella, la forma analtica. En nuestro caso ser: e = 2t-3.

Conviene tener en cuenta que la forma de la grfica e (t) no tiene nada que ver con la tra-
yectoria que sigue el mvil ya que lo nico que nos indica es su posicin en el transcurso
del tiempo. En este ejercicio, por ejemplo, en el enunciado se muestra claramente que la
trayectoria seguida es curva, mientras que la forma de la grfica e (t) es una lnea recta.
Cinemtica 17

3. Una partcula se mueve sobre la trayectoria de la figura adjunta pasando por los
puntos A y B en los instantes que marcan los relojes. Se pide:

a) Expresad los vectores de posicin en los instantes representados en la figura.


b) Vector desplazamiento y su mdulo en el intervalo de tiempo representado.
c) Sabiendo que el espacio del punto A es 1 m y el del punto B es 5 m, determinad el
desplazamiento espacial (o desplazamiento sobre la trayectoria) en el intervalo consi-
derado. Si la partcula se desplaza siempre en el mismo sentido, cul ser la distan-
cia recorrida en dicho intervalo?
d) Si la partcula, una vez que alcanza B, retrocede de nuevo al punto A llegando a l
cuando el cronmetro indica 9 s Qu valdra ahora el desplazamiento espacial en el
intervalo (1, 9 ) s? Y la distancia recorrida?

sol: a) rA (4, 6) m; rB (6, 3) m . b) r (2, - 3) m; r 13 m
c) e = 4 m; D = 4 m. d) e = 0; D = 8 m

4. Un mvil se desplaza sobre la trayectoria de la figura segn: e = -t2 +6t -2 m (si t


en s). Representad grficamente la funcin e(t).

Z
(+)
O

Y
X

sol: A diferencia del ejercicio 2, aqu la trayectoria es recta mientras que la forma de la
grfica e(t) es una curva (parbola).
Cinemtica 18

5. Utilizando los datos del ejercicio 3, se pide: a) Rapidez media, vector velocidad
media y su mdulo. Todo ello en el intervalo (1, 5) s; b) Razonad por qu el valor (ab-
soluto) de la rapidez media y el mdulo de la velocidad media no son iguales.

sol: a) vm = 1 m/s; v m (3/ 4, 1/2) m/s; v m 0'9 m /s
b) Porque nicamente coinciden si se trata de valores instantneos. Los valores
medios solo coinciden si la trayectoria es rectilnea, y en este caso no lo es.

6. Determinad la rapidez de un mvil en los instantes 0, 1, 2 y 3 (todos ellos en segun-


dos) sabiendo que: e = 5t+1 m (si t en s) y construid la grfica e(t).

sol: v = 5m/s

7. Determinad la rapidez de un mvil en los instantes 0, 1, 2 y 3 (todos ellos en segun-


dos), sabiendo que: e = -2t2 + 4t - 2 m (si t en s). Representad la funcin e(t).

sol: 4 m/s, 0, -4 m/s y -8 m/s respectivamente.


8. Determinad el vector velocidad
v de un mvil sabiendo que su vector de posicin

viene dado por: r 2t i t 2 j 5 k m (si t en s).

sol: v 2 i 2t j

9. Sabiendo que un mvil se desplaza segn la ecuacin: e = 3t3 +2t+1 m (si t en s),
determinad su rapidez v y la aceleracin sobre la trayectoria a t

sol: v = 9t2 +2 ; at =18t


10. Dado el vector de posicin r 2t i 5 j (t 1) 2 k m (si t en s), determinad los

vectores velocidad v y aceleracin a .

sol: v 2 i 2(t 1) k ; a 2k m/s2


11. Dado el vector de posicin r 2 i t 3 j 2t 2 k m (si t en s), determinad los vecto-

res velocidad v y aceleracin a .

sol: v 3t 2 j 4t k ; a 6t j 4 k
Cinemtica 19

12. Un cuerpo se mueve a lo largo del eje OX segn la ecuacin: x = 2t 3 + 5t2 +5 (m, si

t en s). Determinad las expresiones del vector de posicin r , el vector velocidad v , el

vector aceleracin a , la aceleracin tangencial at y la aceleracin normal an en cual-
quier instante t.

Antes de proceder a operar, conviene reflexionar sobre la siguiente cuestin:

Qu implicaciones tiene el hecho de que el cuerpo se mueva a lo largo del eje X?

Es evidente que las componentes Y y Z del vector de posicin sern nulas. Lo mismo ocu-
rrir con el vector velocidad, ya que ste siempre es tangente a la trayectoria. Por otra par-
te, al tratarse de una trayectoria rectilnea, el vector velocidad no cambia de direccin y
por tanto no puede haber aceleracin normal, luego, si existe aceleracin, solo podr ser
tangencial.

Para resolver el problema podemos expresar el vector de posicin r en funcin de sus


componentes cartesianas y derivando respecto al tiempo obtener sucesivamente los vecto-
res velocidad y aceleracin:

dr dv
r = (2t +5t +5, 0, 0) v
3 2
= (6t +10t, 0, 0) a
2
= (12t +10, 0, 0)
dt dt

Del anlisis de los vectores anteriores concluimos que el cuerpo se mueve siguiendo una tra-
yectoria recta segn OX, de manera que se va alejando del origen de coordenadas en sentido
positivo y los mdulos de su velocidad y aceleracin van aumentando con el tiempo.

Cmo podramos calcular at y an ?

Aplicando las expresiones at = dv/dt y an = v2/R, donde R es el radio de curvatura. Para


poder realizar las derivadas anteriores, es necesario conocer en primer lugar la expresin
del mdulo de la velocidad (v), en cualquier instante:

De la expresin de v , como solo tiene una componente, es evidente que su mdulo v,
coincidir con el valor absoluto de dicha componente:

v = 6t2 +10t m/s (si t en s)

at = dv/dt = d (6t2 +10t)/dt = 12t +10 m/s2 (si t en s).

an = v2/R = (6t2+10t)2/R = 0, ya que al tratarse de una trayectoria rectilnea, su radio de


curvatura es . Por tanto, tal y como habamos sealado, toda la aceleracin ser tangen-
cial.
Cinemtica 20


13. El vector de posicin de un mvil es: r 2t i t 2 j 3 k (m, si t en s). Se pide:

a) Los vectores velocidad y aceleracin.


b) Las componentes intrnsecas de la aceleracin y el radio de curvatura.

Este problema difiere del anterior en que ahora la trayectoria descrita no es rectilnea y,
por tanto, al ir cambiando la direccin del vector velocidad existir aceleracin normal.
No obstante, el procedimiento de resolucin es el mismo:
Comenzaremos en primer lugar por calcular la expresin de la velocidad y aceleracin en
cualquier instante:

dr dv
v = d (2t, t , -3)/dt = (2, 2t, 0) y derivando de nuevo: a
2
= (0, 2, 0) m/s.
dt dt

Para calcular at, aplicaremos la expresin at = dv/dt determinando primero la expresin del
mdulo del vector velocidad en cualquier instante:

dv 8t 2t
v v x2 v y2 4 4t 2 m/s at
dt 2 4 4t 2
1 t2

A pesar de que el vector aceleracin es constante, al ser a t variable con el tiempo se tratar

de un movimiento variado y no uniformemente acelerado (el mdulo de v cambia pero no
lo hace regularmente, de forma lineal).

Cmo podramos calcular an?


Podramos pensar, en principio, en utilizar la expresin a n = v2/R, sin embargo, ello no es
posible puesto que desconocemos el radio de curvatura R. Para evitar este inconveniente,
recordemos que la expresin del vector aceleracin en funcin de sus componentes intrn-

secas, puede escribirse como: a at an a t uv an un en la que u v es un vector unitario

que siempre tiene la misma direccin y sentido que v y u n otro vector unitario que siem-

pre es perpendicular a v y con sentido hacia el centro de curvatura O.

Analizando la figura adjunta, es fcil darse cuenta de la


relacin existente entre el mdulo de la aceleracin y las
componentes at y an de dicho vector: a a t2 a n2

Si expresamos el vector a en componentes cartesianas,
podemos calcular su mdulo como a a x2 a y2 a z2 =
2 m/s2.
Cinemtica 21

Dado que el mdulo de un vector es independiente del sistema de coordenadas que utili-
cemos para representar dicho vector, podemos sustituir su valor en a a t2 a n2 y despe-
jar la aceleracin normal, con lo que:

4t 2 2
a n a 2 a t2 = 22 2
1 t 1 t2

Ahora, podemos determinar fcilmente la expresin del radio de curvatura R en cualquier


instante, despejando R de la expresin an = v2/R. En efecto:

4 4t 2
R = v2/an y sustituyendo nos queda que: R 2 (1+t2)3/2
2
1 t2

14. Determinad las expresiones de la aceleracin tangencial a t, la aceleracin normal


an y el radio de curvatura R, para el caso del ejercicio 10.

2t 2 2 3
sol: a t ; an ; R 2 ( t 2 2 t 2) 2

t 2t 2
2
t 2t 2
2


15. Dado el vector velocidad v =(3, 2t, 0) m/s (t en s), obtened el vector de posicin r y

el vector aceleracin a sabiendo que en el instante 0, la posicin es r0 = (0, 0, 2) m.

sol: r = (3t, t2, 2); a = (0, 2, 0) m/s2


16. Dado el vector velocidad v = 3i 2(t 1) j 3t 2 k (m, si t en s), determinad el vec-

tor de posicin r y elvector aceleracin a , sabiendo que en el instante 2 s el vector de

posicin es r0 = j 2k m.

sol: r (3t 6) i (t 1) 2 10 j t 3 10 k ; a 2 j 6t k

17. La aceleracin de un mvil viene dada por a = (2, 4, 6) m/s2. Obtened la expresin
del vector de posicin en cualquier instante, sabiendo que en el instante inicial el
mvil se encontraba en reposo en el origen de coordenadas.

sol: r = (t2, 2t2, 3t2 )

18. La aceleracin de un mvil viene dada por a = (2, 4, 0) m/s2. Obtened la expresin
del vector de posicin en cualquier instante, sabiendo que en el instante 1 s, la veloci-
dad es (0, 2, 1) m/s y el vector de posicin (1, 0, 0) m.
Cinemtica 22


En este ejercicio conocemos el valor de a y se nos pide que calculemos el de r . Para re-
solverlo necesitamos buscar la forma de relacionar ambas magnitudes.

Qu relacin existe entre a y r ?


En principio, a y r no estn directamente relacionados ya que, como sabemos, r est
dr
relacionado con la velocidad v mediante la expresin: v . Esto significa que podra-
dt
mos conocer r integrando el vector v . Para ello es necesario conocer v , pero lo que co-

nocemos es a . No obstante,
dado que los vectores velocidad y aceleracin estn relacio-
dv
nados mediante a , podemos obtener v sin ms que integrar a .
dt

As pues, la forma de proceder ser integrar primero a para obtener v e integrar des-

pus v para obtener r .

Procederemos a realizar la primera integracin:



dv
a
dt
dv a dt e integrando en ambos miembros: dv a dt v a dt


Se trata pues de resolver la integral v (2,4,0) dt que podemos abordar directamente o
bien descomponerla en tres integrales escalares:

v x 2 dt 2t C1

v y 4 dt 4t C2

vz 0 dt 0 C3


y volviendo a la expresin vectorial escribir v = (2t + C1 , 4t + C2 , C3), que como vemos
se trata de una expresin indeterminada a causa de las constantes de integracin. Para

conocer v ser pues necesario calcular primero el valor de dichas constantes. Ello solo

puede hacerse si conocemos el valor de v en un instante dado. En este caso, se nos dice
que el mvil, en el instante 1 s se mova con una velocidad de (0, 2, 1) m/s, por tanto:

v1 (2t0 +C1 , 4t0 +C2 , C3) = (2 +C1 , 4 +C2 , C3) = (0, 2, 1), de donde identificando las
componentes nos queda:

2+C1 = 0 C1 = 0-2 = -2; 4+C2 = 2 C2 = 2-4 = -2; C3 = 1



As pues, la velocidad en cualquier instante t, viene dada por: v = (2t-2, 4t-2, 1)

Otro procedimiento para calcular v , consiste en resolver la integral definida que resulta de
introducir los limites de integracin (el primero de los cuales corresponder siempre al
instante conocido que nos den y el segundo a cualquier instante posterior):
Cinemtica 23



dv a dt
v t
=
v0 t0

y descomponiendo los vectores:

dv x a x dt vx 2 dt 2t 2
vx t t

0 1 1

dv y a y dt v y 2 4dt 4t 2
vy t t

2 1 1

dvz az dt vz 1 0 1
vz t

1 1


Obtenemos as directamente el mismo resultado anterior: v = (2t-2, 4t-2, 1)

Para obtener el vector
de posicin r , seguiremos un procedimiento similar: A partir de la
dr
expresin v , sabemos que dr v dt . Considerando las componentes escalares de
dt
sta ltima expresin e integrando, teniendo en cuenta que en el instante 1 s la posicin

del mvil viene dada por r1 = (1, 0, 0) m, nos queda que:

drx vx dt 1 drx 1 vx dt rx 1 (2t 2)dt t 2 2t 2


x t

dry v y dt 0 dry 1 vy dt ry 1 (4t 2)dt 2t 2 2t


y t t

drz vz dt 0 drz 1 vz dt rz dt t 1
z t


Por tanto: r = (t2-2t+2, 2t2-2t, t-1)

19. Un mvil se desplaza segn la ecuacin de movimiento e = 2t 3 + 1 m (si t en s). De-


terminad la rapidez v y la aceleracin sobre la trayectoria a t.
sol: v = 6t2 ; at =12t

20. La aceleracin sobre la trayectoria de un mvil viene dada por a t = 2t +1 m/s2 (si t
en s). Determinad la ecuacin del movimiento e(t) sabiendo que en el instante t = 0 su
rapidez es de -2 m/s y la posicin 4 m.
sol: e = t3/3 + t2/2 -2t + 4

21. La aceleracin sobre la trayectoria de un mvil viene dada por a t = 3t-2 m/s2 (si t
en s). Determinad su posicin en los instantes 0 y 5 s sabiendo que a los 2 s su rapidez
es de 4 m/s y su espacio es de -1 m.
Cinemtica 24

Para conocer la posicin del mvil en un instante dado, hemos de obtener primero la
ecuacin que de la posicin en cualquier instante t, llamada ecuacin del movimiento.
Cmo podemos obtener dicha ecuacin?
Como la aceleracin sobre la trayectoria (a t) no es constante (movimiento variado), no
existen unas ecuaciones ya conocidas para v y e (como sucede con los movimientos uni-
forme y uniformemente acelerado), siendo necesario integrar la a t para obtener v = v (t) y
despus integrar v para obtener e = e (t).
dv
Para resolver la primera integral partiremos de la expresin at , de modo que:
dt
3t 2
dv = atdt dv a t dt v (3t 2) dt 2 t C1
2
Para determinar C1 hay que tener en cuenta que en el instante t 0 =2 s, la rapidez vale v0 =
4m/s. Introduciendo estos datos en la ecuacin anterior nos queda que:

4 = 322/2 -22 + C1 C1 = 2 y, por tanto, v = 3t2/2 -2t + 2

Conocida v = v(t), podemos volver a integrar y obtener la ecuacin e = v(t).


de
En efecto, a partir de v nos queda que:
dt
3t 2 t3
de = vdt de v dt e ( 2 t 2) dt t 2 2 t C2
2 2

Para determinar C2 hay que tener en cuenta que en el instante t0 =2 s, la posicin vale e0 = -
1m. Introduciendo estos datos en la ecuacin anterior nos queda que:

-1 = 23/2 - 22 + 22 + C2 C2 = -5 y por tanto e = t3/2 -t2 + 2t -5 m (si t en s).

Al mismo resultado se puede llegar mediante la utilizacin de integrales definidas, en las


que el lmite inferior vendra fijado por las condiciones conocidas y el superior coincidira
con las incgnitas a determinar:

v t v t 3t 2 3t 2
dv a t dt dv a t dt dv (3t 2) dt v 4 2t 2 v 2t 2
v0 t0 4 2 2 2
Integrando ahora en de = vdt, nos queda que:

3t 2 t3
de dv de (
e v e t
2t 2)dt e 1 t 2 2t 4
e0 v0 1 2 2 2

y de aqu obtenemos que: e = t3/2 - t2 + 2t -5


Cinemtica 25

La ecuacin de e obtenida, nos permite calcular la posicin del mvil en cualquier instan-
te mediante la simple sustitucin del valor de t. As pues:

Posicin del mvil en el instante t = 1 s: e 1 = -5 m


Posicin del mvil en el instante t = 5 s: e 5 = 53/2 - 52 + 25 - 5 = 425 m

22. Un mvil se desplaza con una at = -5 m/s2. Sabiendo que en el instante 0 s se encon-
traba en la posicin e = 40 m y con una rapidez de 25 m/s.
a) Describid el tipo de movimiento de que se trata.
b) Obtened las ecuaciones de v y de e en funcin del tiempo.
c) Hallad el instante que la rapidez es nula y su posicin en dicho instante.

sol: a) Movimiento uniformemente acelerado, en el que inicialmente el mvil se encuentra


a 40 m del origen de espacios desplazndose en sentido positivo con una rapidez de 25
m/s. Su rapidez decrece linealmente con el tiempo en 5m/s cada segundo, de forma que
llega un momento en que el mvil se para y a partir de entonces comienza a retroceder.
b) v = -5t + 25; e = -5/2 t2 +25 t + 40; c) para t = 5 s y en el punto dado por e = 1025 m.

23. Un mvil recorre una circunferencia de 20 cm de radio de modo que su posicin


en cualquier instante viene dada por = t/4 rad (si t en s). Haced un dibujo indican-
do las posiciones ocupadas por el mvil en los instantes: 0, 1, 2, 3, 4, 5, 6 y 7 s.

24. Un mvil recorre la trayectoria circular de la


figura adjunta pasando por los puntos A y B en
los instantes 0 y 3 s, respectivamente.
a) Expresad los vectores de posicin en los instan-
tes 0 y 3 s.
b) Determinad el vector desplazamiento en el in-
tervalo (0, 3) s y su mdulo.
c) Hallad el desplazamiento angular y espacial en
el intervalo (0, 3) s.

sol: a) rA = (025, 043) m; rB = (047, 009) m.

b) r = (022, -034) m; r = 040 m.
c) = 087 rad; e = 044 m

25. Un mvil se desplaza por una trayectoria circular, de 60 cm de radio, con una
aceleracin angular = 2/3 rad/s2. Sabiendo que cuando el cronmetro indica 1 s su
posicin viene dada por un ngulo es de 600 y su rapidez es nula, determinad su rapi-
dez angular y su ngulo en funcin del tiempo.
Cinemtica 26

El movimiento descrito es uniformemente acelerado ya que, el cuerpo se mueve con una


aceleracin constante = 2/3 rad/s2, es decir su rapidez angular aumenta regularmente
en 2/3 rad/s cada segundo que pasa. Tambin sabemos que en el instante t0 = 1 s, el cuer-
po se encontraba en reposo en la posicin dada por o = 60.

Cmo podemos calcular w = w(t) y = (t)?

Como sabemos la aceleracin angular y los datos correspondientes a la posicin y la


rapidez en un instante dado (t = 1 s), podemos obtener fcilmente w = w(t) integrando en
=dw/dt y despus el ngulo = (t) integrando en w = d/dt.

w t w t 2 2
dw dt dw dt dw dt w ( t 1)
w0 t0 0 1 3 3

60 2
En el instante t0 = 1 s el ngulo vale 0 = 60 = radianes
360 3

2
d w dt d w dt d
t t
(t 1) dt (t 2 2t 1)
0 t0 1 3 3 3
3


Despejando en la ecuacin anterior nos queda: = ( t 2 2t 2)
3

Otra forma, ms cmoda, de resolver este problema, consiste en considerar que las inte-
graciones de y , en el caso de ser constante, tienen siempre la misma forma. Bastar
pues con utilizar directamente esas expresiones, conocidas como las ecuaciones del mo-
vimiento circular uniformemente acelerado y sustituir en ellas los datos.

w = w0 + (t -t0 )
1
0 w0 (t t 0 ) (t t 0) 2
2
2
Sustituyendo en la primera ecuacin se tiene: w (t 1) s-1
3
1 2
Sustituyendo en la segunda: ( t 1) 2 ( t 2 2 t 2)
3 2 3 3

26. Un mvil describe una trayectoria circular, de radio R=3 m, segn la ecuacin
=3t3 +2 radianes (si t en s). Determinad la rapidez angular w, la aceleracin angular
, la rapidez lineal v y la aceleracin tangencial a t.

sol: w = 9t2 ; =18t; v = 27t2 ; at = 54 t


Cinemtica 27

27. Un disco de radio 2 m gira en torno a un eje perpendicular por su centro, con ra-
pidez constante. Sabiendo que invierte 4 s en dar un giro completo, se pide:
a) La rapidez angular w y lineal v de dos puntos, uno situado a 1 m de su centro (A) y
otro en la periferia (B).
b) Frecuencia del movimiento.
c) ngulo girado en un tiempo de 6 s.
d) Distancia recorrida por los puntos A y B en 6 s.

a) Se trata de un cuerpo extenso que gira en torno a un eje con rapidez angular constante.
En este caso todos los puntos del slido describen trayectorias circulares centradas en el
eje y todos giran tambin el mismo ngulo (suponemos que el slido es indeformable) en
el mismo tiempo, es decir, poseen la misma rapidez angular.

Y qu ocurre con la rapidez lineal?

Si observamos los puntos A y B de la figura anterior, podemos darnos cuenta de que am-
bos describen ngulos iguales en tiempos iguales, pero el arco recorrido por B es siempre
mayor, de manera que aunque ambos giran con la misma rapidez angular, la rapidez lineal
de B ser mayor que la de A. En efecto, la rapidez lineal de un punto cualquiera del disco
viene determinada por la expresin v = w R donde w es la rapidez angular (la misma
para todos los puntos del disco) y R el radio de la circunferencia que describe dicho punto
en particular, con lo que cuanto ms grande sea R mayor ser el valor de la rapidez v.

Como se trata de un movimiento circular uniforme, podemos aplicar las ecuaciones co-
rrespondientes:

=0
w = constante
= 0 + w (t-t0 )

Para conocer w bastar con despejar de la ltima ecuacin, con lo que nos queda:
Cinemtica 28

0 2
w rad/s
t t0 4 2

Para conocer la rapidez lineal aplicamos v = w R a cada uno de lo puntos:


Para el punto A: vA = w RA = 1 m/s. Para el B: vB = w RB = 2 m/s
2 2 2

Como vemos la rapidez lineal es justo el doble para el punto B, situado en la periferia que
para el punto A cuyo radio de giro es la mitad. Esto quiere decir que aunque ambos dan el
mismo nmero de vueltas cada segundo, la distancia recorrida por el B ser siempre el
doble que la recorrida por el A.

b) Como el disco gira con movimiento circular uniforme, se trata de un movimiento pe-
ridico en el que a intervalos regulares de tiempo (periodo) cada punto vuelve a estar en la
misma posicin y con el mismo estado de movimiento.

Cul es el valor del periodo T para el movimiento de giro de cualquier punto del disco?

Es evidente que ser precisamente el tiempo que tarde en realizar una vuelta completa (un
ciclo), que en este caso es de 4 s.

La frecuencia , mide el nmero de vueltas completas (ciclos) que se realizan cada segun-
do. Se trata, por tanto, de una magnitud inversa del periodo, es decir: = 1/T. Su unidad
internacional es el hertzio (Hz), de forma que un valor de 1 Hz significa que se realiza un
ciclo completo cada segundo.

= 1/T = 1/4 = 025 Hz Qu quiere decir este resultado?

El resultado anterior se interpreta diciendo que, mientras se mantenga constante dicha


frecuencia, cualquier punto del disco dar un cuarto de vuelta cada segundo.

c) Para calcular el ngulo girado en 6 segundos, utilizamos la expresin = 0 + w (t-t0 )


en la forma = w t, en donde corresponde al ngulo girado en un intervalo de

tiempo t. As pues: = w t = 6 3 rad
2
Cinemtica 29

d) La distancia recorrida en los 6 segundos se puede obtener fcilmente utilizando la rela-


cin existente entre la longitud del arco de circunferencia y el ngulo correspondiente
expresado en radianes: e = R

Para el punto A: eA = RA = 3 1 = 3 m
Para el punto B: eB = RB = 3 2 = 6 m

28. Desde la boca de un pozo de 20 m de profundidad, se lanza verticalmente y hacia


arriba una piedra con rapidez de 10 m/s. Determinad con qu rapidez chocar contra
el fondo del pozo.

La piedra comienza a subir con una rapidez de 10 m/s. Dicha rapidez, debido a la grave-
dad terrestre, ir disminuyendo hasta que alcanzada la mxima altura, la piedra se parar
momentneamente para comenzar a descender. Mientras baja, debido a la gravedad, se
mover cada vez ms aprisa, hasta que impacte contra el fondo del pozo con la rapidez
que lleve justo en ese instante. Para resolver el problema, vamos a despreciar el rozamien-
to del aire con la piedra, de esta forma el movimiento ser uniformemente acelerado (des-
de que se lanza hasta el instante en que choca contra el fondo del pozo).

Como la trayectoria es conocida, podemos aplicar un tratamiento escalar. Para ello es im-
prescindible seleccionar (si no est preestablecido en el enunciado) un origen de espacios
(O), el sentido que se toma como positivo y un origen de tiempos, indicndolos expresa-
mente en un esquema lo ms claro posible. Dicha seleccin es arbitraria, pero conviene
procurar que resulte siempre lo ms cmoda posible. As, en este caso, podramos hacer lo
siguiente:
Cinemtica 30

e1 ?
t1 ?
Altura mxima v1 0


a v
(+) e0 = 0 m
v0
t0 = 0 s
Boca del pozo O
v0 = 10 m/s
(-)

e2 20 m
Fondo del pozo t2 ?
v2 ?

Como se puede ver en el esquema, se han seleccionado tres instantes del movimiento: el
"t0" o inicial (cuando se lanza desde la boca del pozo), el instante "t 1" (cuando alcanza su
altura mxima) y el instante "t2" o final (en el momento en que llega al fondo del pozo).
La aceleracin tangencial se puede considerar durante todo el proceso como constante,
vertical y dirigida hacia abajo y por tanto (de acuerdo con el criterio de signos escogido)
su valor es el mismo y negativo, tanto en la subida como en la bajada . La designaremos
como at = -g (aceleracin de la gravedad). Se trata, pues, de un movimiento uniformemen-
te acelerado.
Las ecuaciones correspondientes a un movimiento uniformemente acelerado son:
v = v0 + at (t-t0) e = e0 + v0 (t-t0) + 1/2 at (t-t0)2
Si sustituimos en ellas los datos de este problema nos queda:
g 2
1) v v 0 g t 2) e v0 t t
2
que sern las ecuaciones que representan el movimiento que lleva a cabo la piedra.

Cmo podemos hallar la rapidez v en el instante en que choca contra el fondo?


Para ello podramos utilizar la ecuacin v v 0 g t que nos da la rapidez de la piedra en
cualquier instante t, pero no es posible ya que, en principio, desconocemos en qu preciso
instante t2 se produce el choque contra el fondo del pozo. Para calcular el valor de t 2 , po-
g
demos utilizar e v0 t t 2 sustituyendo "e" por -20 m y "g" por 10 m/s2 :
2
g
e v0 t t 2 para t = t2, queda como: 20 10t2 5t22 t22 2t2 4 0
2

Resolviendo la ecuacin de segundo grado, obtenemos que el instante en que la piedra


choca contra el suelo del fondo del pozo es t 2 = 324 s. Sustituyendo ahora este valor en la
ecuacin que nos proporciona la rapidez en cualquier instante, obtendremos la rapidez de
la piedra justo cuando choca contra el fondo v2 :
Cinemtica 31

v v0 g t v2 = v0 - gt2 v2 = 10 - 10 32 = -224 m/s

Otra forma (ms general) de resolver el problema, sin tener que recordar las ecuaciones
del movimiento uniformemente acelerado, consiste en obtener las ecuaciones v = v(t) y e
= e(t) a partir de la aceleracin tangencial, integrando convenientemente:

at = dv/dt = -10 m/s2 dv = -10 dt 10 dv 0 10 dt v 10 10t v 10 10t


v t

de (10 10t ) dt e 10t 5t


e t
v = de/dt de = v dt 0 0
2

Una vez obtenidas las ecuaciones anteriores, se procedera siguiendo los mismos pasos
que hemos realizado anteriormente.

A qu se debe el signo negativo del resultado?

Como hemos elegido (arbitrariamente), espacios crecientes hacia arriba, la rapidez (que
mide lo aprisa que cambia e y se expresa como de/dt) ser positiva siempre que el cuerpo
suba (porque entonces la e va aumentando y toma valores cada vez mayores, con lo que al
ser el valor posterior ms alto que el anterior, el cambio o variacin de e resultar positi-
vo) y negativa cuando el cuerpo baje (porque entonces la e va disminuyendo tomando
cada vez valores ms pequeos, con lo que al ser el valor posterior menor que el anterior,
el cambio o variacin de e resultar negativo).

Como en el momento de chocar con el fondo del pozo, la piedra se est moviendo hacia
espacios decrecientes (de acuerdo con el criterio de signos escogido), la rapidez ser nega-
tiva. Conviene no confundirse y pensar, errneamente, que es negativa por encontrarse en
la parte negativa de la trayectoria, ya que si, manteniendo igual todo lo dems, hubise-
mos lanzado la piedra desde el fondo del pozo, la rapidez mientras sube sera positiva, a
pesar de encontrarse en espacios negativos, porque la posicin "e" ira aumentando. Otra
cuestin a tener en cuenta es que el signo negativo de un dato solo hay que ponerlo una
vez, ya que es bastante comn el error de ponerlo en un smbolo y luego volver a ponerlo
en el valor numrico.

Para terminar el problema, comprobad que la altura mxima alcanzada por la piedra es
de 5 m sobre la boca del pozo.

29. Suponiendo que la Tierra gire en torno a su eje con movimiento de rotacin cons-
tante, se pide:
a) Rapidez lineal, rapidez angular y aceleracin de un punto de latitud L.
b) Calcular los valores de las magnitudes anteriores en los polos y en el ecuador.

Debido al movimiento de rotacin de la Tierra alrededor de su eje, todos los puntos de su


superficie (excepto los dos que coinciden con el eje de rotacin), giran con la misma rapi-
dez angular (movimiento circular uniforme), describiendo circunferencias de distinto ra-
dio (mximo en los puntos del ecuador y disminuyendo hacia los polos).
Cinemtica 32

Cunto vale dicha rapidez angular?

Como la Tierra da una vuelta completa cada 24 h, podemos aplicar la ecuacin del movi-
miento circular uniforme expresada en la forma = w t, despejar w y sustituir (po-
niendo previamente la vuelta en radianes y las 24 h en segundos):

2
= w t w rad/s
t 86400

As pues cualquier punto de la superficie terrestre gira con la rapidez angular anterior. Sin
embargo, como sabemos, no podemos decir lo mismo respecto a la rapidez lineal.

Cmo podramos hallar la rapidez lineal del punto P situado en un paralelo de latitud L,
como el que se indica en la figura?

P
P

La rapidez lineal correspondiente a P, vendr dada por v = w R, siendo R el radio de la


circunferencia descrita por P alrededor del eje. Cmo podemos expresar dicha rapidez en
funcin de la latitud L?

Se trata de relacionar el radio R con la latitud L. Analizando la figura podemos ver que cos L
= R/RT, de manera que R = RT cos L. Sustituyendo esta ltima expresin en la de la rapidez
2
lineal nos queda: v = wRTcos L = 6350 10 3 cos L v = 46179 cos L
86400

Con qu aceleracin se mueve el punto P?

Como el punto P describe un movimiento circular y uniforme, estar sometido exclusiva-



mente a una aceleracin normal, de modo que a a n . Como sabemos la aceleracin nor-
mal se puede calcular como an = v2/R.

2
2

an = v2/R = w2 R = w2 RT cos L = 6350 10 cos L an = 003 cos L


3
86400

Qu ocurrira con las magnitudes anteriores en los polos y en el ecuador?


Cinemtica 33

Como es lgico, al coincidir los polos con puntos del eje, tanto su rapidez angular como la
lineal debern de ser nulas y lo mismo ocurrir con la aceleracin normal.

En cuanto al ecuador, cabe pensar que la rapidez lineal tome su valor mximo (en el mis-
mo intervalo de tiempo se describe un arco mayor), y la aceleracin normal alcance tam-
bin su valor mximo (ya que all el vector velocidad cambia ms rpidamente de direc-
cin).

Si los razonamientos cualitativos anteriores son correctos, los resultados que se obtengan
al imponer la condicin de que P est situado en el polo o en el ecuador, a las expresiones
generales que hemos deducido, tendrn que ser coherentes con los mismos:

En uno de los polos:

v = 46179 cos L, como en el polo L = 90 queda que vP = 46179 cos 90 = 0

wP = 0, ya que ningn punto del eje gira alrededor del mismo.

an = 003 cos L, como en el polo L = 90 queda que anp = 003 cos 90 = 0

En el ecuador:

v = 46179 cos L. En el ecuador L = 0 ve = 46179 cos 0 = 46179 m/s

Segn el resultado anterior, todos los puntos del ecuador giran a algo ms de 1600 km/h
alrededor del eje terrestre.

2
w rad/s (constante, excepto en los mismos polos donde es nula).
t 86400

an = 003 cos L = 003 cos 0 = 003 m/s2

30. Un coche de polica pretende alcanzar a otro vehculo que circula con rapidez de
108 km/h. El vehculo policial arranca desde el reposo con aceleracin de 2 m/s 2 hasta
que su rapidez es de 180 km/h y luego prosigue con movimiento uniforme hasta dar
alcance al otro vehculo. Calculad:
a) Dnde lo alcanza si se puso en marcha 2 s despus de ser rebasado.
b) Representad en un mismo diagrama e-t ambos movimientos.
Cinemtica 34

Se trata de un problema de dos mviles, el coche de la polica (que llamaremos A) y otro


coche (que llamaremos B). Tomaremos como origen de espacios el punto donde se encon-
traba A en reposo y como origen de tiempos el instante en que B pasa por su lado. El
mvil B se desplaza con movimiento uniforme, el A lo hace primero con movimiento uni-
formemente acelerado y cuando alcanza una rapidez de 180 km/h sigue con movimiento
uniforme. Los datos de partida sern:

Para el mvil A

Primer tramo (movimiento uniformemente acelerado): a t = 2 m/s2; t0 =2 s; v0 = 0; e0 = 0.

vA = at (t-t0) = 2 (t-2)
eA = 1/2 at (t-t0)2 = (t-2)2

Segundo tramo (movimiento uniforme): v = 180 km/h = 50 m/s.

Para el mvil B

Movimiento uniforme: at = 0; t0 = 0; e0 = 0; v = 108 km/h = 30 m/s.

vB = 30m/s = constante
eB = vB t = 30 t

Dado que el mvil A acelera hasta alcanzar una rapidez de 50 m/s y luego la mantiene
constante, es seguro que alcanzar al mvil B, que se desplaza con una rapidez constante
de 30 m/s. Lo que conviene saber, en primer lugar, es si dicho alcance se producir antes
o despus de que A cambie de movimiento.

Una forma de averiguarlo sera calculando en qu instante A alcanzara a B, suponiendo


que A no cambiase nunca de movimiento. Despus habra que comparar el valor de t ob-
tenido, con el tiempo que tarda A en alcanzar la rapidez lmite de 50 m/s. Si el primer
valor fuese menor que el segundo, supondra que A sobrepasa a B antes de dejar de acele-
rar y cambiar a movimiento uniforme.

Cmo podemos determinar dicho instante?

En el momento en que se produjese el encuentro, la posicin de ambos mviles respecto


del O sera la misma y por tanto: eA = eB (t-2)2 = 30 t t2 -34t + 4 = 0

Resolviendo la ecuacin anterior se obtienen dos valores de t. De ellos solo uno tiene sen-
tido fsico segn las condiciones del problema y vale 3388 s, pero en ese instante la rapi-
dez que llevara A sera: vA = 2 (t-2) = 2 (3388 - 2) = 6376 m/s, que supera los 50 m/s,
por lo que hemos de concluir que el alcance se producir despus de que el coche de po-
lica A haya cambiado su movimiento de uniformemente acelerado a uniforme. En qu
instante y dnde ocurre esto?

Haciendo v = 50 m/s en la ecuacin vA = 2 (t-2) nos queda 50 = 2t - 4 y despejando t obte-


nemos t = 27 s. Su posicin en ese instante ser e = (t-2)2 = (27-2)2 = 625 m.
Cinemtica 35

De acuerdo con lo anterior, las caractersticas del segundo tramo (movimiento uniforme)
para el coche A, son: at = 0; e0 = 625 m; t0 = 27 s; v = 50 m/s, y su ecuacin de movimien-
to vendr dada por e = 625 + 50 (t - 27).

Para hallar la posicin en que B ser alcanzado, necesitamos hallar primero en qu ins-
tante se produce dicho alcance. Para ello, como sabemos, basta igualar las posiciones:

eA = eB 625 + 50 (t-27) = 30 t t = 3625 s

Ahora basta sustituir el valor obtenido en una de las ecuaciones que dan la posicin, para
obtener a qu distancia del origen de espacios A da alcance a B:

eB = 30 t = 30 3625 = 10875 m

Podemos ahora, construir la tabla de valores de "e" frente a "t", para cada uno de los
mviles y representar en una misma grfica.

31. La rapidez angular de una rueda disminuye uniformemente desde 1000 r.p.m.
hasta 500 r.p.m. en 10 s. Determinad:
a) Nmero de vueltas efectuadas en esos 10 s.
b) Distancia recorrida en los 10 s por un punto "A", situado a 50 cm del eje de giro.
c) Tiempo necesario para detenerse.

El que la rapidez angular de la rueda disminuya uniformemente significa que la acelera-


cin angular es constante. En ese caso el movimiento de cualquier punto de la misma, es
uniformemente acelerado y, como sabemos, las ecuaciones que rigen este tipo de movi-
miento son:

= constante

w = w0 + (t-t0)
1
0 w(t t 0 ) (t t 0 ) 2
2
Cinemtica 36

En el momento en que la rueda empieza a frenar comenzaremos a contar el tiempo (t 0 = 0)


y consideraremos la posicin de la rueda en ese instante como origen de ngulos (0), y
sentido positivo el de giro, de manera que:

= constante; t0 = 0; w0 = 1000 rev/min = 1047 rad/s; 0 = 0.

Introduciendo estos datos en las ecuaciones generales anteriores nos queda que:

(+) 1
= w0 t + t2
2
A O w = w0 + t

v0

Cmo podemos calcular el nmero de vueltas N1 que da la rueda durante los 10 prime-
ros segundos desde que comienza a frenar?

1
Mediante la ecuacin = w0 t + t 2 podemos hallar el ngulo descrito (en radianes)
2
por cualquier punto de la rueda en esos 10 segundos (1 ). Luego, como cada vuelta equi-
vale a 2 radianes, bastar que dividamos por 2 para tener el nmero de vueltas que nos
piden.
1
w0 t1 t12
N1 = 1 / 2 = 2
2

El problema es calcular el valor de 1 en el instante t1 = 10 s, ya que, en principio, no co-


nocemos cunto vale la aceleracin angular . Sin embargo, s que sabemos que en el
instante t1 la rapidez angular es de w1 = 500 r.p.m = 500 (2/60) = 5235 rad/s .

Con estos datos podemos hallar . Sustituyendo el valor de t1 en la ecuacin de w:

w1 w0
w1 = w0 + t1 = = (1047 - 5235)/10 = -524 rad/s2
t1

El signo negativo de la aceleracin se debe a que, de acuerdo con el criterio de signos es-
cogido, como la rueda frena, la rapidez angular (positiva) va decreciendo. Sustituyendo
ahora este dato en la expresin anterior, podemos calcular N1:

1
w 0 t1 t12
2 104'7 10 2'62 100 785
N1 = 1 / 2 = = 124'9 vueltas.
2 2 2
Cinemtica 37

Para calcular la distancia recorrida en ese mismo tiempo por un punto A situado a 50 cm
del eje, hemos de tener en cuenta la relacin e = R. De manera que:

e1 = 1 R = 1 R = 785 05 = 3925 m.

Qu podemos hacer finalmente, para calcular el tiempo total que la rueda tarda en de-
tenerse desde que comenz a frenar?

En el instante en que la rueda se detenga, la rapidez angular alcanzar el valor 0. Introdu-


ciendo esta condicin en la ecuacin general w = w 0 + t y despejando t, podemos obte-
ner el tiempo que tarda en detenerse:

0 = w0 + t t = -w0 / = -1047/ -524 = 20 s

32. Un mvil describe un movimiento circular de 50 m de radio con una at de 2 m/s2.


Sabiendo que cuando el cronmetro indica 4 s el mvil se encuentra en un ngulo de 3
rad movindose con una rapidez de 6 m/s. Determinad:

a) Posicin y rapidez en el instante inicial.


b) Aceleracin total a los 3 s.

Se trata de un movimiento circular uniformemente acelerado del que conocemos que a t =


2 m/s2; t0 = 4 s; v0 = 6 m/s y 0 = 3 rad. Como vemos algunos datos son de magnitudes
angulares y otros de magnitudes lineales. Para resolver el problema conviene decidir con
cules de ellas vamos a trabajar. Si nos inclinamos por utilizar magnitudes lineales, ten-
dremos:

at = 2m/s2; t0 = 4 s; v0 = 6m/s; e0 = 0 R = 3 50 = 150 m y las ecuaciones sern:

v = v0 + a (t -t0) v = 6 + 2 (t - 4)
1
e e0 v 0 (t t 0 ) a (t t 0 ) 2 e = 150 + 6 (t - 4) + (t - 4)2
2

Con estas ecuaciones podemos responder a cada una de las preguntas planteadas:

Cules sern la posicin y la rapidez en el instante inicial?

Bastar con sustituir t por 0 en las ecuaciones e = e (t) y v = v (t) anteriores:

v = 6 + 2 (t - 4) = 6 + 2 (0 - 4) = - 2 m/s
e = 150 + 6 (t - 4) + (t - 4)2 = 150 + 6 (0 - 4) + (0 - 4)2 = 142 m

Cul ser el vector aceleracin a los 3 s?



Como sabemos, el vector aceleracin a se puede calcular derivando el vector velocidad

v . Sin embargo en este caso eso no es posible dado que no conocemos v . Otra opcin es
Cinemtica 38


obtener a en componentes intrnsecas como: a at an n en donde es un vector
unitario tangente a la trayectoria y cuyo sentido coincide siempre con el tomado como

positivo (espacios crecientes), mientras que n es un vector unitario perpendicular a la
trayectoria y dirigido en todo momento hacia el centro de curvatura. En la expresin ante-
rior at se obtiene derivando la rapidez v respecto del tiempo y a n como v2/R. (R es el radio

de curvatura en cada instante). As pues, podemos tratar de expresar el vector a en fun-
cin de at y an :

En este caso, at es constante y vale 2 m/s2 mientras que an no lo es, pero podemos calcular
su valor en cada instante mediante a n = v2 / R.

Como nos piden el vector a en el instante t = 3 s, tenemos que hallar el valor de la rapi-
dez v en dicho instante. Para ello utilizamos de nuevo la expresin v = 6 + 2 (t - 4) susti-
tuyendo t por 3, con lo que obtenemos v = 6 + 2 (3 - 4) = 4 m/s. Sustituyendo ahora este
valor en la expresin de an nos queda que en el instante 3 s, an = v2 / R = 16 / 50 = 032

m/s2. Introduciendo los resultados anteriores en a at an n obtenemos que el vec-

tor aceleracin en el instante t = 3 s vendr dado por: a 2 0'32 n m/s2.

33. Un mvil A describe una trayectoria circular de radio 50 metros con rapidez an-
gular de 03 rad/s. A los 5 s de partir A, sale en su persecucin otro mvil B, desde el
reposo y con aceleracin de 01 rad/s2. Determinad:

a) Posicin e instante en que B alcanzar a A.


b) Rapidez angular y lineal de B en el momento de alcanzar a A.
c) Aceleracin tangencial, aceleracin normal y mdulo del vector aceleracin de B
en el momento del encuentro.

sol: a) 426 rad y 14'24 s; b) 092 rad/s; 46 m/s; c) 5 m/s2; 423 m/s2; 426 m/s2

34. Un objeto posee un movimiento armnico simple de amplitud 1 m y periodo 2 s.


Determinad: a) Ecuacin del movimiento. b) Posicin en los instantes 1/2, 1, 3/2 y 2
(todos ellos en s). c) Grficas x (t), v (t) y a (t)

Podemos representar esquemticamente el movimiento armnico simple de una partcula


como un movimiento de trayectoria rectilnea en el que dicha partcula vibra respecto a un
punto medio O de equilibrio, como se indica en la figura, movindose entre dos posiciones
extremas A y -A, de forma que la aceleracin tangencial a que est sometida en cada instante
es proporcional a la elongacin x (posicin respecto al punto O) en ese mismo instante, es
decir: a = -Cx, donde C es la constante de proporcionalidad llamada constante armnica.

a
-A x A X+
O

Si tomamos el origen de tiempos cuando la partcula se encuentra en el punto O des-


plazndose hacia X+, las ecuaciones que describen su movimiento son:
Cinemtica 39

x = A sen wt, que nos da la posicin en cualquier instante.


v = Aw cos wt, que nos da la rapidez en cualquier instante.
a = -Aw2 sen wt = -Cx, nos da la aceleracin en cualquier instante o posicin (C = w 2 ).

Donde w es una constante caracterstica de cada movimiento y viene dada por:


w = 2/T = 2 , siendo T el periodo y la frecuencia del movimiento.
Partiendo de las ecuaciones anteriores y sustituyendo los datos que nos dan en el enuncia-
do, obtendremos las ecuaciones que describen el movimiento del cuerpo al que se refiere
el problema:

x = A sen wt = sen t
v = Aw cos wt = cos t
a = -Aw2 sen wt = -2 sen t
Ahora, simplemente manejando estas ltimas ecuaciones es posible contestar a todas las
preguntas que se hacen en el enunciado del problema:
a) La ecuacin del movimiento, es la ecuacin que nos da la posicin del mvil en cual-
quier instante x = sen t m (si t en s).

b) Cmo hallar la posicin del mvil en los instantes que se piden?


Bastar sustituir en la ecuacin del movimiento la t por cada uno de los valores co-
rrespondientes, con lo que tendremos:

Para t = 1/2 s x = sen /2 = 1 m y v = 0 (la partcula se encuentra en x = A)


Para t = 1 s x = sen = 0 m y v = - m/s (se encuentra en el origen O, movindose hacia X-)
Para t = 3/2 s x = sen 3/2 = -1 m y v = 0 (la partcula se encuentra en x = -A).
Para t = 2 s x = sen 2 = 0 y v = m/s (se encuentra en el origen O, movindose hacia X+)

De los resultados anteriores es fcil comprobar que, como cabe esperar, cada periodo de
tiempo T, la posicin de la partcula se repite. En el siguiente apartado lo veremos me-
diante una grfica y analizaremos tambin que les ocurre al resto de las magnitudes.

c) Construccin de las grficas de x = x (t), v = v (t) y a = a (t).

Para construir dichas grficas hemos de elaborar primero las tablas correspondientes me-
diante las ecuaciones x = x (t), v = v (t), a = a (t) y luego representar:

t (s) 0 05 1 15 2
x (m) 0 1 0 -1 0
Cinemtica 40


t (s) 0 05 1 15 2
v (m/s ) 0 - 0
-

2
t (s) 0 05 1 15 2
a (m/s2) 0 -2 0 2 0
-2

Qu conclusiones se pueden extraer de las grficas anteriores?


Podemos ver reflejadas en ellas las principales caractersticas de un movimiento armnico
simple. As, por ejemplo, en el instante en que x vale 0 (punto de equilibrio), la velocidad
es mxima y la aceleracin (y por tanto la fuerza) es nula. Sin embargo, cuando la x al-
canza el valor mximo A, la velocidad es nula y la aceleracin (y por tanto la fuerza) son
mximas (en valor absoluto). Este tipo de movimiento constituye un claro ejemplo de
cmo un objeto puede moverse con su mxima velocidad y ser nula la fuerza resultante
sobre el mismo (o estar momentneamente en reposo y la fuerza tomar su valor mximo).

35. Determinad el periodo de una partcula vibrante que posee una aceleracin de
-2m/s2 cuando se encuentra a 50 cm de su posicin de equilibrio.

sol: T = s

36. Un mvil de 4 kg de masa que realiza un movimiento armnico simple, lleva una
rapidez de 80 m/s en el punto medio de su trayectoria y de 50 m/s cuando su elonga-
cin x es de 2m. Calculad la ecuacin de su movimiento x = x (t).

La ecuacin del movimiento viene dada por x = A sen(wt), donde A es la amplitud y w la


frecuencia angular (w = 2/T). As pues para obtener x necesitamos saber A y w.
Como, tanto A como w son constantes caractersticas de cada movimiento, podemos tratar
de hallarlas sustituyendo los datos particulares del enunciado en las ecuaciones generales
x = x (t), v = v (t) y a = a (t).
Para el instante t1 (cuando el mvil se encuentra en el punto medio de la trayectoria):
x1 = A sen wt1 0 = A sen wt1 sen wt1 = 0
v1 = Aw cos wt1 80 = Aw cos wt1 cos wt1 = 80/Aw
Cinemtica 41

Para el instante t2 (cuando el mvil se encuentra en x = 2 m):

x2 = A sen wt2 2 = A senwt2 sen wt2 = 2/A


v2 = Aw cos wt2 50 = Aw sen wt2 cos wt2 = 50/Aw

Si tenemos en cuenta que sen2 + cos2 = 1, podemos establecer las siguientes ecuacio-
nes:

02 + (80/Aw)2 = 1

(2/A)2 + (50/Aw)2 = 1

De la primera de las ecuaciones anteriores obtenemos que Aw = 80. Sustituyendo este


valor en la segunda de ellas, obtenemos que A = 256 m, con lo que si Aw = 80 y A =
256, nos queda que w = 80/256 = 3125 rad/s.

As pues, la ecuacin de movimiento pedida es: x = 256 sen (3125 t)

37. Una pelota rueda sobre una


mesa horizontal a 15 m de altura
del suelo, cayendo por el borde de
la misma. Si choca con el suelo a
una distancia de 18 m, medidos
horizontalmente desde el borde
de la mesa. Cul es la rapidez
con la que sali de la mesa?

Dado que la trayectoria es, en principio, desconocida, ser necesario un tratamiento vecto-

rial para resolver el problema, utilizando los vectores a , v y r .

De igual manera que en los problemas que admitan un tratamiento escalar, tendremos
que establecer ahora un sistema referencial en el que queden definidas las distintas magni-
tudes. En este caso, podemos tomar unos ejes de coordenadas cartesianas cuyo origen co-
incida con la base de la mesa y el suelo con el eje OX, tal y como se aprecia en el esque-
ma siguiente, en donde hemos dibujado mediante una lnea de puntos la trayectoria que
intuimos seguir la pelota una vez que abandone la mesa. El origen de tiempos lo situa-
remos en el instante en que la pelota deja de tener contacto con la mesa y se ve sometida a
la aceleracin de la gravedad.

Como datos tenemos pues: a (0,g ) , t0 = 0; v 0 (v 0 , 0) ; r0 (0, h); y r1 ( A, 0) , sien-
do h la altura de la mesa y A el alcance horizontal, en donde ya hemos tenido en cuenta
los signos correspondientes de cada uno de dichos datos.
Cinemtica 42

Y (m)

0s
v0

a v
r
r0 a
a

O
X A
(m)
Cmo podemos hallar la velocidad inicial con que sali la pelota?

Para conseguirlo, podemos comenzar por determinar las ecuaciones v = v (t) y

r = r (t). Despus podemos tratar de introducir en dichas ecuaciones los datos que conoce-
mos para obtener as la velocidad inicial.

dv v t t
A partir de a
dt
obtenemos que dv a dt v v

v0 0
0 (0, g ) dt
0


y resolviendo la integral nos queda que v (v0 , 0) (0, gt ) (v0 , gt )


dr r t t
A partir de v , obtenemos que
dt dr v dt r r ( v

r0 0
0
0
0 , gt ) dt

1 1
y resolviendo la integral nos queda que r (0, h) ( v0 t , gt 2 ) ( v0 t , h gt 2 )
2 2

Analizando la expresin obtenida para r = r(t), nos damos cuenta que las coordenadas de
cualquier punto de la trayectoria seguida por la pelota, segn el sistema de referencia que
hemos escogido, vienen dadas en cualquier instante del movimiento por:

x v0 t

1 2
y h gt
2

Podemos ahora particularizar x e y para el instante t 1 en que la pelota choca contra el sue-
lo y despejar v0 . En efecto:

A = v0 t1 v0 = A/t1 y aunque no sabemos t1, podemos hallarlo haciendo y = 0 en la ex-


1 2h
presin y = h - gt2 de forma que 0 = h - gt12 t1 (el mismo tiempo que si la
2 g
pelota hubiese cado verticalmente al suelo).
Cinemtica 43

Sustituyendo ahora t1 en la expresin anterior v0 = A/t1 , podemos determinar v0 :

A
v0
2h
g

Cmo podemos analizar el resultado anterior?

En primer lugar, es posible darse cuenta de que lo que hay a la izquierda del signo igual,
tiene las mismas dimensiones que lo que hay a la derecha (L/T en ambos casos). Ello no
es una garanta de que el resultado sea correcto pero si la ecuacin no resultara dimensio-
nalmente homognea, s que podramos tener la seguridad de haber cometido algn error y
deberamos de revisar el problema.

En segundo lugar, vemos que el resultado contempla adecuadamente ciertas condiciones


lmite evidentes, como por ejemplo:

Si el alcance A es nulo, v0 ha de ser nula

Por otra parte, cuanto mayor sea el alcance conseguido (manteniendo constantes los valo-
res de las restantes magnitudes) mayor habr sido la velocidad inicial con que se lanz.
Cuanto mayor sea la altura h desde la que se hace el lanzamiento para conseguir un cierto
alcance, con menor velocidad inicial habr que hacer el lanzamiento, etc.

Ahora podemos sustituir los datos de que disponemos y calcular el valor de v0 segn:

A 18
'
v0 = = = 327 m/s
2h 2 15
'
g 10

El resultado, podemos expresarlo tambin en forma vectorial como v 0 (3' 27, 0) m/s. Es
preciso tener en cuenta que al sustituir el valor de g, hemos puesto 10 y no -10 porque el
signo negativo que le corresponda a esta componente escalar del vector aceleracin
(segn el sistema de referencia escogido) ya lo hemos tenido en cuenta al principio po-

niendo -g en la expresin de dicho vector aceleracin a (0, g ) m/s2.

Otra cuestin que podemos plantearnos a raz de este problema es si se trata o no de un


movimiento uniformemente acelerado. Un anlisis superficial de la situacin podra llevar
a pensar que puesto que la aceleracin es constante s lo es. Sin embargo, si obtenemos la
expresin de at nos podemos dar cuenta de que no es constante y que, por tanto, no se trata
de un movimiento uniformemente acelerado y el mdulo de la velocidad (o la rapidez)
vara de forma no lineal con el tiempo. El hecho de que, en este caso, la aceleracin to-
tal sea constante sin que lo sea la aceleracin tangencial, se debe a que la aceleracin

normal cambia de tal modo que la suma a t a n a s que lo es.
Cinemtica 44

38. Un jugador de rugby patea el baln y ste sale con rapidez de 18 m/s formando
un ngulo de 30 con la horizontal. Otro jugador que se encuentra a 40 m de distan-
cia en la direccin del baln, corre en ese mismo instante a por l. Cul debe ser su
rapidez, supuesta constante, para cogerlo justo antes de que llegue al suelo?

Se trata de un problema que plantea una situacin relacionada tambin con otros deportes
en los que se utiliza un baln o una pelota a la que se le puede lanzar oblicuamente, como,
por ejemplo ocurre en el ftbol, golf, etc. Otro inters es el lanzamiento de proyectiles (en
cuyo caso no es deporte, desgraciadamente, lo que se practica). El estudio (por unas u
otras razones), del tiro oblicuo y del horizontal tuvo un papel histrico importante en el
desarrollo de la mecnica.

En el caso concreto que se nos plantea, el mvil A, describe una trayectoria que, en prin-
cipio es desconocida (aunque nuestras experiencias cotidianas nos hagan pensar que
tendr una forma parablica, no sabemos cul ser concretamente). Durante su movimien-
to se ve sometido a la aceleracin de la gravedad. B tiene un movimiento rectilneo y uni-
forme sobre el eje x, con el fin de coger el baln en el momento que llegue al suelo.

Para resolver el problema, conviene que utilicemos un sistema de referencia espacio-


temporal comn para los dos mviles, lo que nos lleva a aplicar un tratamiento vectorial
para describir el movimiento de ambos.

Tomaremos como sistema de referencia espacial unos ejes de coordenadas cartesianas cuyo
origen coincida con el punto desde donde se lanza el baln, tal y como se indica en el esque-
ma siguiente y como origen de tiempos el instante en que sale el baln. Haremos la aproxima-
cin de considerar a A y B como masas puntuales y la friccin con el aire nula.


Cinemtica 45

Los datos para cada uno de los mviles son:



Mvil A (baln): a A (0, g ) m/s2 ; t0 =0; v 0A (v 0A cos , v 0A sen ) m/s

r0A (0, 0) m.

Mvil B (contrario): a B (0, 0) ; t0 =0; v B (v B , 0) m/s; r0B (x 0B , 0) = (40, 0 ) m.
Donde vB es la incgnita a determinar.

Cabe esperar que, en las condiciones que se han considerado, la rapidez v B dependa de
alguna manera de la rapidez inicial vOA con que se lanza el baln, del ngulo con que se
lance, de la distancia inicial x OB existente entre ambos jugadores, y la aceleracin de la
gravedad g. Es decir: vB = vB (vOA, , xOB, g).

De acuerdo con la situacin planteada, no estamos seguros de qu hara el jugador B,


podra ser que tuviera que correr hacia la derecha, quedarse en donde est o correr hacia la
izquierda, dependiendo del alcance mximo del baln. As, por ejemplo, si B se moviera
hacia la derecha, podemos pensar que, a igualdad de los restantes factores, v B ser tanto
mayor cuanto mayor sea v0A y menor x0B. Adems, debera tomar un valor mximo para
= 45 ya que para este valor el alcance es mximo, por lo que debera correr ms rpido
para alcanzar a recoger el baln antes de que ste llegase al suelo.

Con los datos anteriores podemos determinar las ecuaciones v = v (t) y r = r (t) para cada
mvil, integrando a partir de la aceleracin. Mediante dichas ecuaciones podremos cono-
cer la velocidad o la posicin de cada uno de ellos en cualquier instante. Posteriormente,
podramos resolver el problema considerando que en el momento en que B recoja el
baln, la posicin de ambos mviles (que consideramos como masas puntuales) habr de
ser la misma.

Ecuaciones de movimiento para el mvil A:

A partir del vector aceleracin :



dvA vA t
aA dvA a A dt dvA (0, g ) dt . Resolviendo estas integrales:
dt v0A 0

v A (v 0A cos , v 0A sen ) + (0, - g t) v A (v 0A cos , v 0A sen - gt)

A partir del vector velocidad:



drA rA t
vA drA vA dt drA vA dt
dt 0 0

rA 0 (v 0A cos , v 0A sen g t ) dt rA (v 0A cos t , v 0A sen t - gt 2 )


t 1
2

Ecuaciones de movimiento para el mvil B:

Dado que se mueve con movimiento uniforme sobre el eje X tendremos que:
Cinemtica 46


v B (v B , 0) m/s y el vector de posicin: rB ( x0B v B t , 0) m

En las ecuaciones anteriores vB es la incgnita y por eso no le hemos puesto signo.


Adems, no sabemos a priori en qu sentido se ha de mover B para recoger la pelota.

Qu podemos hacer ahora para calcular con qu rapidez v B se debera de mover B para
coger el baln justo cuando llegue al suelo?

En el momento en que A y B se encuentren, los vectores de posicin de ambos mviles


sern los mismos (tengamos en cuenta que hemos escogido un nico sistema de referencia
posicin-tiempo). Si designamos como t1 a ese instante, e igualamos las coordenadas co-
rrespondientes, podremos obtener vB :

En el momento del encuentro: xA = xB v0A cost1 = x0B + vBt1 Despejando vB obte-


v cos t1 x 0 B
nemos que: v B 0A , con lo que para determinar vB necesitamos saber t1
t1
o instante en que el baln llega al suelo.

Como en el instante en que el baln toca el suelo, su ordenada "y" vale 0, podemos igualar
a 0 la expresin general de dicha ordenada y despejar t 1, con lo que:

1 2 2v 0A sen
v 0A sen t1 -gt1 0 de donde obtenemos que t1 y sustituyendo este
2 g
valor en la expresin de vB anterior nos queda finalmente:

x0B g
v B v 0A cos
2v0A sen

La expresin obtenida nos sirve para calcular, en las condiciones que se dan en el enun-
ciado del problema, qu velocidad constante deber llevar el jugador del equipo contrario
para recoger la pelota cuando llegue al suelo.

Analizad el resultado obtenido. Reflexionad, en particular, sobre el significado de que v B


pueda ser negativa, nula o positiva

En primer lugar, podemos darnos cuenta de que la ecuacin es dimensionalmente


homognea y que en ambos lados del signo igual tenemos L/T.

En segundo lugar, como el resultado es una resta de dos trminos. Si nos sale un nmero

negativo implica que la nica componente escalar del vector v B es negativa y por tanto el
jugador se mueve hacia la izquierda porque el alcance del baln ser inferior a la distancia
inicial que separa a los dos jugadores. Si la resta es 0, quiere decir que el jugador no se
mueve y que, por tanto el baln va a caer justo donde l se encuentra y finalmente, si la
resta es positiva, quiere decir que el jugador se dirige hacia la derecha, porque el alcance
Cinemtica 47

del baln es superior a la distancia que le separa del punto de lanzamiento. Sustituyendo
los datos numricos, podremos saber en cul de los tres casos nos encontramos:

x 0B g 40 10
vB = v 0A cos = 18 cos 30 = - 661 m/s
2 v0A sen 2 18 sen 30

Por tanto, el baln no sobrepasar al jugador contrario y ste ha de correr hacia la izquier-

da con v B ( 6'61, 0) m/s para darle alcance en el momento justo en que llegue al suelo.

39. Desde el suelo se dispara un proyectil A con rapidez de 100 m/s y ngulo de 37.
Un segundo ms tarde y desde 320 m ms all, se lanza verticalmente y hacia arriba
otro proyectil B con rapidez v. Determinar el valor de v para que choque con el pri-
mer proyectil.

sol: v = 683 m/s.

40. Una bola de nieve se desliza por el tejado de un edificio con rapidez de 10 m/s en
el momento que lo abandona. Teniendo en cuenta las caractersticas geomtricas de la
figura adjunta y el sistema referencial indicado, determinad la ecuacin de la trayec-
toria y el punto de impacto.


30

v0
2'3 m


Este problema es similar al tiro horizontal ya estudiado aunque la velocidad inicial v 0 no

es horizontal como all suceda. El proceso pues ser el mismo, con la diferencia que v 0
tiene ahora dos componentes. Tomaremos como sistema de referencia los ejes de coorde-
nadas cartesianas que se incluyen en el dibujo y como origen de tiempos el instante en que
la bola de nieve abandona el tejado. De acuerdo con ello los datos del problema seran los
siguientes:

a (0,g ) ; t0 = 0; v 0 (v 0 cos , v0 cos ) ; r0 (0, 2'3) m

siendo y los ngulos directores (ved anexo sobre vectores) del vector v 0

La bola en cuanto pierde contacto con el tejado se ve sometida nicamente a la acele-


racin de la gravedad (para simplificar ignoramos el efecto del rozamiento con el aire).
Cinemtica 48

Cmo podemos determinar la ecuacin de la trayectoria y el punto de impacto?


Como conocemos la aceleracin y las condiciones iniciales, podemos determinar me-

diante integracin las ecuaciones v v (t ) y r r (t ) , que nos dan la velocidad de la bola
de nieve y sus posicin en cualquier momento desde que sale del tejado hasta que impacta
finalmente contra el suelo.

Una vez que obtengamos la ecuacin del movimiento r r (t ) , tenemos tambin las coor-
denadas x = x (t) e y = y (t). Podemos hallar el alcance o distancia horizontal desde la base
del edificio hasta el punto del impacto. Para ello hay que tener en cuanta que el valor de
dicha distancia coincide con el de la coordenada x en el instante t 1, en que se produce el
choque contra el suelo (es decir, en el momento en que "y" toma el valor 0).

dv v t
Integrando a partir de a dv a dt dv a dt
dt v0 t0


v v 0 (0, g ) dt v (v 0 cos , v 0 cos ) (0, gt ) v (v 0 cos , v 0 cos gt )
t


dr r t
Integrando a partir de v dr v dt dr v dt
dt r0 t0

r r0 0 (v 0 cos , v 0 cos g t ) dt (0, h) (v 0 t cos , v 0 t cos gt 2 )


t 1
2
1 2
r (v 0 t cos , h v 0 t cos gt )
2
La ecuacin de la trayectoria vendr dada (en paramtricas) por:

x = v0 t cos = 10 t cos 30 x = 87 t

y = h + v0 t cos -1/2 gt2 = 23 + 10 t cos 120 - 5 t2 y = 23 - 5t -5t2

Podemos eliminar t en las ecuaciones anteriores y dejar as y en funcin de x (ecuacin de


la trayectoria en explcitas):

t = x/87 y = 23 - 057 x - 0066 x2 , que es la ecuacin de una parbola.

Para hallar el alcance, sabemos que en el punto en que impacte la bola sobre el suelo se
cumplir que x = v0 t1 cos, siendo t1 lo que marque el reloj en ese instante. Como adems
la coordenada y ser 0 (la bola llega al suelo), podemos hallar t 1 mediante:

1 2
0 h v 0 t1 cos gt1 5t12 5t1 cos120 2'3 0 5t12 5t1 2'3 0
2

Con lo que nos queda la ecuacin 5t12 5t1 2'3 0 , que resolvindola nos da: t1 = 034 s

Sustituyendo este valor en la expresin x = x (t), obtenemos que el alcance vale:

A = v0 t1 cos = 10 034 cos 30 = 2'94 m


Cinemtica 49

41. Una barca que se desplaza a 6 m/s sale de la


orilla de un ro de 60 m de ancho en una direccin
que forma un ngulo de 60 con dicha orilla (en el
sentido en que se desplaza la corriente). Deter-
minad a qu punto de la otra orilla llegar la bar-
ca suponiendo que la rapidez de la corriente sea
constante y su valor 2 m/s.

Presentacin de la situacin problemtica, discusin de su posible inters, precisin del pro-


blema y anlisis cualitativo de la situacin.

En el esquema siguiente hemos representado la situacin que plantea el problema. El pun-


to (1) es al que llegara la barca en caso de que no hubiese corriente y el ngulo valiese
90. El punto (2) corresponde tambin al caso de que no hubiese corriente, pero con un
ngulo de 60 (que es el rumbo que toma la barca). Sin embargo, como s que existe co-

rriente, la barca sufre una deriva en el rumbo inicial ya que a la velocidad de la barca v B

hay que sumar la velocidad de la corriente v C , lo que hace que se dirija hacia el punto (3)

de la otra orilla, con una velocidad resultante de v v B v C .

El problema nos pide a qu punto de la otra orilla llegar la barca. Se trata de un problema
que tiene un indudable inters en aquellas situaciones en las que hay que cruzar una ex-
tensin grande de agua y se desea conocer el rumbo para llegar a un punto determinado.
Dicho punto puede determinarse mediante la distancia d existente entre los puntos (1) y
(3). Para ello, simplificaremos el problema despreciando lo que ocurre en el instante ini-
cial (salida de la barca) y final (llegada a la otra orilla) en los que habr una cierta acele-
racin y consideraremos el movimiento como uniforme a lo largo de todo el trayecto.

De qu factores depender el valor de la distancia d?


Cinemtica 50

Podemos suponer que dicha distancia depender de la anchura A del ro, del ngulo , de

la velocidad v B de la barca y de la velocidad v C de la corriente, de manera que si, mante-
niendo constantes los restantes factores, aumentase, por ejemplo, la velocidad de la co-
rriente, d tambin aumentara y lo mismo ocurrira si aumentase la anchura del ro o disminu-
yese el ngulo .

Diseo de posibles estrategias de resolucin.

Si escogemos un sistema de coordenadas cartesianas como el que se representa en la figu-


ra anterior, cuyo origen coincida con la posicin inicial de la barca, podemos darnos cuen-
ta de que la distancia d coincidira con la componente cartesiana r x del vector de posicin

de la barca r (rx , ry ) en el preciso instante en que sta llegue a la otra orilla, es decir,
cuando la componente ry coincida con la anchura del ro (ry = A). Por tanto, una forma de
obtener d, sera determinar en primer lugar la ecuacin del movimiento de la barca

r r (t) y a continuacin tratar de calcular rx en el instante en que ry = A. La ecuacin de

movimiento puede determinarse integrando la funcin v v (t) .

Otra posible forma de resolver el problema (que no desarrollaremos aqu), es mediante


consideraciones geomtricas (semejanza de tringulos).

Resolucin, anlisis de los resultados, implicaciones y nuevas perspectivas.

Resolveremos el problema segn la primera estrategia. De acuerdo con la figura anterior,


el vector velocidad resultante con que se mueve la barca puede expresarse como:

v (v x , v y ) (v B cos v C , v B sen ) .

dr
Integrando en v (teniendo en cuenta que para t = 0 s, r0 = 0 y que v es constante),
dt
nos queda que:

r (rx , ry ) = (vB cos v C )t, v B t sen que nos da el vector de posicin de la barca en
cualquier instante del tiempo que dura el trayecto. Descomponiendo ahora segn los ejes:

(1) rx = (vB cos + vC) t


(2) ry = vB t sen

Igualando ry a la anchura del ro A, obtenemos de la expresin (2) el tiempo t correspon-


A
diente al instante en que la barca llega a la otra orilla: t =
v B sen

(v B cos v C )A
Sustituyendo ahora t en la expresin (1) tendremos rx = d y d
v B sen

Finalmente, sustituyendo los valores numricos llegamos obtenemos: d = 5774 m


Cinemtica 51

Analizad el resultado literal obtenido

En primer lugar podemos comprobar que la expresin obtenida es dimensionalmente


homognea ya que en ambos lados de la igualdad las dimensiones son las de una longitud.
Por otra parte, el resultado contempla algunos casos bien conocidos como, por ejemplo,
que si el ngulo disminuyese la distancia d aumentara tendiendo a cuando tendiese
a 0. Tambin podemos constatar que si = 90 y vC = 0, se obtiene d = 0. Este ltimo as-
pecto nos permite plantear una cuestin particularmente interesante desde el punto de
vista prctico, consistente en determinar cunto debera valer para que la barca llegase
justo enfrente de donde se halla inicialmente:

Como la corriente del ro desva a la barca deberemos de tener en cuenta su efecto y tomar
un ngulo mayor de 90. En efecto, si llega justo enfrente se deber de cumplir que la dis-
tancia d valga 0, luego haciendo d = 0 en el resultado del problema, podemos despejar y
obtener el ngulo pedido:

(v B cos v C ) A vC vC
d 0 v B cos v C cos arccos 109'47
v B sen vB vB

42. Un vehculo que se desplaza sobre un colchn de aire detiene su propulsin, que-
dando sometido exclusivamente a la fuerza de friccin con el aire que le provoca una
aceleracin proporcional a la rapidez del vehculo. Determinad la variacin temporal
de la rapidez.

Al quedar el mvil sometido exclusivamente a una aceleracin de frenado que acta en


sentido contrario a su velocidad, la trayectoria ser rectilnea (lo que significa que no
habr aceleracin normal). Adems, la rapidez ir disminuyendo con el tiempo y en con-
secuencia, el valor absoluto de la aceleracin tambin lo har. Se trata, pues, de un movi-
miento rectilneo y variado (la aceleracin tangencial no es constante).

Si establecemos los orgenes de espacios y tiempos cuando comienza a actuar exclusivamente


la aceleracin de frenado, un posible esquema representativo sera:

a v0
+
O

De acuerdo con todo lo anterior: at = -kv; t0 = 0; v0 ; e0 = 0

Que podemos hacer para calcular la expresin de la rapidez v en funcin del tiempo?

Se trata, evidentemente, de proceder como ya lo hemos hecho en otros problemas anterio-


res, integrando a partir de la aceleracin:
Cinemtica 52

dv
at dv at dt dv kv dt
dt

En este caso, sin embargo, la segunda integral no puede resolverse ya que la rapidez que
figura en ella no es una constante (precisamente es su variacin temporal lo que se de-
manda). Para poder solucionar este inconveniente, hemos de proceder a separar las varia-
bles de forma adecuada antes de integrar:

dv = -kv dt dv/v = -k dt. Ahora s que podemos integrar:

v dv t v v
v0 v
k dt ln v ln v0 k t ln k t
0 v0 v0
e -kt v v0 e kt

Qu forma tendra la funcin anterior si procediramos a su representacin grfica?


Podemos comprobar que si representramos v frente a t, obtendramos una curva como la
de la figura en la que como puede verse, la rapidez decrece aproximndose a 0 asin-
tticamente:

v (m/s)
v0

t
(s)
Cmo podramos obtener la ecuacin del movimiento e = e (t)?
de
Para ello tenemos que integrar de nuevo a partir de v v 0 e -kt
dt
e t v0
de
0 0
v0 e -kt dt e
k
(1 e -kt )

Si representramos grficamente la funcin obtenida obtendramos:

e (m)
v0 /k

t (s)

43. La aceleracin tangencial de un mvil es a t= -kv2. Sabiendo que para t = 0 la rapi-


dez es v0 y se encuentra en el origen, determinad la ecuacin de movimiento e (t)

sol: e = ln (ktvo +1)/ k


Cinemtica 53

PROBLEMAS COMPLEMENTARIOS

C-1. Se lanza un cuerpo hacia arriba. Qu altura mxima alcanzar?

La situacin abierta que se plantea en el enunciado se puede relacionar con el problema


del lanzamiento de proyectiles y comentar brevemente la importancia histrica de los tra-
bajos de Galileo en este campo y sobre la cada de graves en general.

Podemos precisar el enunciado considerando el lanzamiento vertical desde el suelo de un


objeto pequeo y compacto de forma que podamos ignorar el efecto de rozamiento con el
aire. Por otra parte supondremos que no llega tan alto como para que podamos medir nin-
guna variacin en el peso de dicho objeto.

Al lanzar el cuerpo hacia arriba desde el suelo ste sale con una rapidez inicial v 0 pero
debido a la gravedad sube cada vez ms lentamente hasta que llega un momento en que se
detiene y comienza a descender cada vez ms aprisa. Tanto en la subida como en la bajada
el objeto se halla sometido nicamente a la fuerza peso y se mueve con la aceleracin de
la gravedad (que suponemos constante y dirigida siempre verticalmente hacia abajo).

En principio podemos suponer que el valor de la altura mxima alcanzada depender de la


rapidez inicial con que se lance y del valor de la aceleracin de la gravedad. Estas ideas se
pueden resumir mediante la ecuacin:
hmax = h (v0, g)

Podemos incluso, tratar de profundizar un poco ms haciendo alguna hiptesis respecto a


cmo van a influir cada una de esas variables en la altura mxima alcanzada (siempre
suponiendo que las dems permanecen constantes). As, por ejemplo, cabe pensar que:

-Cuando v0 aumente (se lance hacia arriba con mayor rapidez) ms alto llegar.
-Cuando la gravedad disminuya la aceleracin del objeto tambin ser menor y su rapidez
ir disminuyendo ms lentamente, por lo que la altura mxima aumentar (esto ocurrira,
por ejemplo, si el lanzamiento se realizar en la Luna en lugar de hacerlo sobre la superfi-
cie terrestre).

Es posible que adems de las variables anteriores se consideren otras, como el tiempo que
est subiendo y la masa del objeto. En cuanto a la primera, es fcil darse cuenta de que se
encuentra ya implcita en las dos variables consideradas (no es posible, por ejemplo, va-
riar la v0 con que se lanza un objeto en un lugar dado y mantener constante el tiempo que
dura la subida). Respecto a la posible influencia de la masa, no hay ningn inconveniente
en mantenerla como una hiptesis ms de trabajo pero sin olvidar analizarla a la luz del
resultado obtenido.

Dado que la trayectoria es conocida (lnea recta perpendicular al suelo), podemos aplicar
un tratamiento escalar para resolver el problema. Para ello escogeremos arbitrariamente
un punto de la trayectoria como origen de espacios (por ejemplo el punto del suelo desde
donde se lanza) y un sentido como positivo (por ejemplo hacia arriba), tal y como se indi-
ca en la figura adjunta.
Cinemtica 54

La aceleracin tangencial es constante y segn el esquema anterior ser negativa e igual a


la aceleracin de la gravedad. Se trata, pues, de un movimiento uniformemente acelerado:

v = v0 + a (t-t0) para la rapidez v en cualquier instante t.


a
e e0 v0 ( t t 0 ) ( t t 0 ) 2 para la posicin e en cualquier instante t.
2
Teniendo en cuenta las condiciones imperantes en el problema, los datos a considerar seran:
Un objeto de masa m que en el instante t 0 = 0 se lanza desde el suelo (e0 = 0) verticalmen-
te hacia arriba con una rapidez inicial v0 (positiva) y que se mueve con una aceleracin
sobre la trayectoria constante y negativa (-g).
De acuerdo con lo anterior, la ecuacin de la rapidez v y de la posicin e en cualquier ins-
tante vendrn dadas respectivamente por:
g
(1) v = v0 - gt (2) e v0t t 2
2
En las ecuaciones anteriores hemos optado por colocar el signo correspondiente (de
acuerdo con el criterio arbitrario especificado al comienzo) antes de sustituir ningn valor
numrico.
La altura mxima coincidir en este caso con el valor de la posicin e durante la subida en
el preciso instante en que el objeto se pare (momentneamente) para volver a caer. Ese
instante puede calcularse haciendo v = 0 en la ecuacin (1) y despejando t.
Otra posibilidad para resolver el problema es mediante de consideraciones de trabajo y
energa. En este caso como no actan fuerzas no conservativas la aplicacin del teorema
fundamental, que relaciona el trabajo resultante con la variacin de energa cintica, a la
subida del objeto sometido exclusivamente a la accin de la fuerza peso, nos dara que:

Wres = Ec WP = Ec -Ep = Ec Ep0 + Ec0 = Ep + Ec = constante.


Por lo que podramos igualar la energa cintica ms la potencial gravitatoria del objeto
(situado en la superficie de la Tierra) a dicha suma en la posicin de altura mxima y des-
pejar el valor de la altura h correspondiente.

Mediante la primera estrategia, a partir de la ecuacin (1) hacemos v = 0 y despejamos t (que


coincidir entonces con el tiempo que tarda en subir), con lo que: 0 = v0 gt t = v0/g
Cinemtica 55

2
v g v
Sustituyendo ahora en la ecuacin (2) queda que: e v0 0 0 y simplificando:
g 2 g

v02
Obtenemos finalmente la expresin de la altura mxima e
2g

Mediante la segunda estrategia, bastara con tomar como nivel 0 de energa potencial gra-
vitatoria el suelo y tener en cuenta que al alcanzar la altura mxima h, el objeto se para
momentneamente con lo que la energa cintica en ese punto ser nula. As pues:
2
1 2 v0
Ep0 + Ec0 = Ep + Ec mv mghmax h max
2 0 2g

Como vemos mediante las dos estrategias propuestas hemos llegado al mismo resultado
literal, lo que refuerza su validez. Por otra parte, esta forma de proceder nos permite ana-
lizar dicho resultado y darnos cuenta si se cumplen o no las hiptesis de partida y los po-
sibles casos lmite considerados. En primer lugar, la ecuacin es dimensionalmente
homognea (L en ambos lados). Adems cuanto mayor es la rapidez con que se lanza el
objeto mayor altura mxima alcanzar, pero ahora, es posible darse cuenta de una forma
ms precisa cmo influye esa variable que al estar elevada al cuadrado hace que a doble
rapidez inicial no se obtenga doble altura mxima sino cudruple.

C-2. Dnde se cruzarn dos vehculos R y A que circulan por la misma carretera y
al encuentro?
Qu pasa ah? Cmo me imagino la situacin?
Se trata de dos movimientos uniformes en los que los mviles se desplazan por una tra-
yectoria fija y conocida, en sentidos opuestos, de modo que llegar un momento en el que
se cruzarn y queremos saber en qu punto ocurre eso. Es un problema que puede tener su
inters en el control del trfico de vehculos (trenes, autobuses, etc.).

Para resolver el problema es preciso en primer lugar elegir un sistema de referencia ade-
cuado, as como un criterio de signos. Escogeremos como origen de espacios la posicin
inicial del mvil rojo y valores de la posicin positivos hacia la derecha de ese punto.
Como origen de tiempos (t0 = 0) escogeremos tambin el instante inicial, cuando ambos
estn separados por la distancia D. Dado que cuando se cruzan ambas posiciones son igua-
les, el problema se puede operativizar como: A qu distancia "d" del origen de espacios
O, se cumple que eR = eA?
De qu factores depender d? Proponed posibles hiptesis argumentando cmo cabe espe-
rar que influya cada uno de ellos en el valor de d, considerando tambin algn caso lmite.
Cinemtica 56

En principio, podemos pensar que d depender de la rapidez con que se mueva cada uno y
de la distancia inicial D que los separe:
d = f (vR, vA, D) donde vR = rapidez del mvil rojo y vA = rapidez del mvil amarillo
Es ms, cabe esperar que si a igualdad de los restantes factores:
vR aumenta d aumentar
vA aumenta d disminuir
D aumenta d aumentar
Tambin podemos pensar en algunos casos lmite evidentes como, por ejemplo:
Si vR = 0 d = 0; si vA = 0 d = D; si vR = vA d = D/2
Proponed una posible estrategia de resolucin
Se trata de dos movimientos uniformes, podemos saber dnde estarn los mviles rojo y
amarillo en cualquier instante. Hallar d equivale a calcular e R (o eA) en el preciso instante
"t" en que ambas posiciones coinciden. Por tanto, una posible estrategia ser:
Escribir las ecuaciones eR = f(t) y eA = f(t) y hallar el instante "t" en que e R = eA y de ah
finalmente d.
Proceded a la resolucin propiamente dicha
La ecuacin general de la posicin de un movimiento uniforme es: e = e 0 + v(t - t0)
Para el mvil rojo, tenemos t0 = 0, e0 = 0, v = vR , e = eR , con lo que la ecuacin ser:
eR = vRt
Para el mvil amarillo, tenemos t0 = 0, e0 = D, v = -vA , e = eA , con lo que la ecuacin ser:
eA = -vAt + D
Obsrvese que al desplazarse sentido negativo, la rapidez de A es negativa. En el instante
en que eR = eA, se tendr que cumplir que:
D
vRt = -vAt + D de donde: t
(v R v A )

Ahora basta sustituir este valor de "t" en la ecuacin de la posicin de cualquiera de los
dos mviles, para tener la distancia "d" buscada. Si lo hacemos as, obtenemos:

vR 1
d D o lo que es equivalente: d D
(v R v A ) v
1 A
vR

Proceded a analizar el resultando obtenido, comprobando si se cumplen o no las hipte-


sis de partida y los casos lmite considerados.
Si nos detenemos en analizar el resultado anterior, podemos comprobar fcilmente cmo
se cumplen todas las hiptesis de partida. Es fcil ver, por ejemplo, que cuando ambos
mviles se desplazan con la misma rapidez, se encontrarn justo a mitad de camino (D/2),
o que si, por ejemplo, vA = 0 ocurre que d = D, etc.
3. DINMICA DEL PUNTO

1. Se lanza un cuerpo verticalmente hacia arriba. Considerando nulo el rozamiento,


sealad cul de los siguientes esquemas representa correctamente las fuerzas que
actan sobre dicho cuerpo poco antes de que alcance la mxima altura.

Algunas personas piensan que al lanzar un cuerpo hacia arriba se le comunica una fuerza
mayor que su peso y que dicha fuerza, de alguna manera, queda impresa en l y le impulsa
hacia arriba de forma que, poco a poco, se va "gastando" (disminuye) conforme el cuerpo
va subiendo.

Explicad qu equivocaciones se cometen en el razonamiento anterior

En primer lugar, es cierto que al lanzar un cuerpo hacia arriba ejercemos durante el lan-
zamiento una fuerza en sentido opuesto al peso y mayor que esta. En consecuencia, el
cuerpo saldr de la mano con una cierta velocidad vertical hacia arriba. Sin embargo es
preciso tener en cuenta que dicha fuerza no queda impresa en el cuerpo. Por el contrario,
las fuerzas son debidas a interacciones y cuando el cuerpo se halla en el aire ya no inter-
acciona con quien le lanz sino tan solo con la Tierra (interaccin de naturaleza gravitato-
ria). As pues, desde el preciso instante en que el cuerpo abandona la mano que lo lanza
hasta que choca contra el suelo, la nica fuerza que acta sobre el mismo (si se desprecia
el rozamiento con el aire) es la fuerza peso, de manera que asciende cada vez ms lenta-
mente porque la fuerza peso tira de l hacia abajo. Si no fuese por el peso, se movera
indefinidamente con la misma rapidez con que se lanz y en la misma direccin y sentido
(en ausencia de otras interacciones). Por tanto, la respuesta correcta a la cuestin es la c).

En conclusin: cuando tengamos que representar las fuerzas que estn actuando sobre
cualquier objeto, lo que hemos de hacer es preguntarnos con qu est interaccionando, no
con qu interaccion, y tener presente que las fuerzas hay que relacionarlas con la acele-
racin (cambios de velocidad) y no con la velocidad.
Dinmica 58

2. Dibujad todas las fuerzas y la fuerza resultante que en cada una de las situaciones
representadas actan sobre la bola. (La flecha indica el sentido del movimiento en el
instante representado y el rozamiento se considera nulo).

sol: En el pndulo simple solo actan el peso y la tensin del hilo. En el plano horizontal,
el peso y la fuerza normal que ejerce el plano hacia arriba. En el tiro oblicuo, el peso. En
el primero la fuerza resultante en ese instante es vertical y hacia arriba (ser la fuerza

normal necesaria para que v cambie en direccin) en el segundo es nula y en el tercero
coincide con el peso de la bola.

3. Un pndulo se encuentra suspendido del techo de un vagn de tren. Imagnate que


te encuentras parado(a) en el andn pero que puedes ver lo que ocurre dentro del
vagn. Dibuja la posicin correcta del pndulo en los distintos casos, as como las
fuerzas que actan sobre la bola del mismo. (Las flechas representan la velocidad del
vagn).
Dinmica 59

Para dibujar las fuerzas que actan sobre la bolita del pndulo y su resultante, hemos de
analizar con qu est interaccionando dicha bola y hacia dnde va la aceleracin con la
que se mueve ya que, como sabemos, la fuerza resultante que acta sobre una masa y la
aceleracin (no la velocidad) siempre tienen la misma direccin y sentido.

En los tres casos representados, la bolita del pndulo interacciona con el hilo (enganchado
al techo)
y con la Tierra. Como consecuencia de dichas interacciones el hilo ejerce una
fuerza T (tensin) sobre labola y la Tierra una fuerza P (peso). La suma de las dos fuerzas
ser la fuerza resultante Fres T P que siempre deber de tener la misma direccin y
sentido
que el vector aceleracin ya que segn la ecuacin fundamental de la dinmica:

Fres m a . Por lo tanto, para resolver la cuestin, nos convendr analizar previamente
cul ser la direccin y sentido del vector aceleracin en cada caso.


a) En este caso no hay aceleracin y por tanto T + P = 0
por lo que ambas fuerzas se anulan y como el peso tiene
la direccin vertical, tambin la deber tener la tensin
T (la bolita se ha colgado estando el vehculo con mo-
vimiento rectilneo y uniforme y, por tanto, no oscila).

b) En este caso el vector aceleracin va hacia la izquier-


da (en sentido contrario
a la velocidad) y por tanto para
obtener el vector Fres en esa misma direccin y sentido,

el vector T deber estar inclinado segn se indica en la
figura.

c) En este caso la aceleracin tiene la misma direccin y


sentido que la velocidad y, por tanto, la bolita se encon-
trar desplazada a la izquierda
de la vertical, de manera
que la suma de T y P nos de una fuerza resultante con
la misma direccin y sentido que la aceleracin.

En la resolucin de este problema hemos tenido en cuenta que la direccin y sentido del

vector aceleracin no vienen determinados por el vector velocidad sino por el vector d v .
Dinmica 60

4. Sobre un mvil de 20 kg de masa actan si-


multneamente las fuerzas representadas en la Z
figura adjunta (se considerar incluido el peso).

Determinad la posicin del mvil en funcin del


tiempo sabiendo que: F1 = 20 N, F2 = 40 N y F3 = F3
10 N, y que, al comenzar a actuar dichas fuer- F2 Y
zas, el mvil se encontraba en el punto (0,1,0) m
desplazndose con una rapidez de 2 m/s segn el X F1
eje OY (+).

Como con la informacin de que disponemos no podemos prever la trayectoria del mvil,
ser necesario trabajar con magnitudes vectoriales, lo que significa que la posicin del

mvil vendr dada por el vector de posicin r . Dicho vector se puede obtener, como ya
dr
sabemos, integrando en v . Para ello necesitamos conocer previamente la funcin
dt
dv
v v (t ), que, a su vez, se puede determinar integrando a .
dt

Lo primero que tendremos que hallar, pues, ser el vector aceleracin.



Aplicando la ecuacin fundamental de la dinmica Fres m a podemos obtener la acele-
racin. Para ello hemos de tener en cuenta el carcter vectorial de las fuerzas:

F1 (20, 0, 0) N

F2 (0, 40, 0) N

F3 (0, 0, 10) N

Fres (20, 40, 10) N



Fres (20, 40, 10) m
a ( 1, 2, 0'5 ) 2
m 20 s

dv
Utilizaremos ahora este resultado para calcular v v (t ) integrando en a , teniendo
dt
en cuenta que para t0 = 0, v0 = (0, 2, 0) m/s:


dv a dt v v
v t t
0 (1, 2, 0'5) dt (t , 2t , 0'5t ) v (0, 2, 0) (t , 2t , 0'5t )
v0 0 0


de donde obtenemos finalmente que v = (t, 2t+2, 05 t).
Dinmica 61


dr
Podemos ahora calcular la expresin r r (t ) , integrando en v teniendo en cuenta
dt
que en el instante t0 = 0, el vector de posicin era r0 = (0, 1, 0) m:


t t2 2 t2
r0dr 0v dt r r0 0(t , 2t 2, 0'5t ) dt ( 2 , t 2t , 4 )
r t

t2 t2 t2 t2
de donde r (0, 1, 0) ( , t 2 2t , ) ( , t 2 2t 1, )
2 4 2 4

5. Sobre una masa puntual de 5 kg actan las fuerzas representadas en la figura ad-
junta (se considerar que el peso ya est incluido en estas fuerzas). Determinad su
posicin a los dos segundos de comenzar a actuar las fuerzas, sabiendo que el cuerpo
se encontraba inicialmente en reposo, en la situacin de la figura:

Y
F2 =10 N
F1 =15 N

X
0
60
F3 =20 N


sol: r = (-092 , 0, 0) m

6. Sobre una masa de 50 kg que se desplaza a 40 m/s por un plano horizontal sin ro-
zamiento, con movimiento rectilneo y uniforme, comienza a actuar una fuerza de 500
N, en sentido contrario a su movimiento. Hallad la distancia recorrida a los 8 s de
comenzar a actuar dicha fuerza.

La masa se mueve inicialmente con un movimiento rectilneo y uniforme sobre una super-
ficie horizontal. Podemos suponer, pues, que nicamente interacciona con la superficie
del plano y con la Tierra de manera que la fuerza
normal R que ejerce el plano sobre di-
cha masa, ha de anularse con la fuerza peso P o fuerza con
que la Tierra
la atrae.
Si ahora
comienza a actuar otra fuerza F , podemos escribir que Fres R P F F de manera
que, a partir de este momento, al actuar una fuerza resultante en sentido contrario al vec-
tor velocidad, nicamente se modificar su mdulo y la trayectoria seguir siendo rectil-
nea.

En este tipo de problemas, de trayectoria conocida, nos convendr expresar la ecuacin


fundamental de la dinmica en componentes intrnsecas (ved figura al final de la pgina
siguiente):
Dinmica 62


Fres m a Fres t Fres n n mat man n

y descomponindolo en dos ecuaciones escalares:

Fres t = mat - F = mat at = - F/m = -500/50 = - 10 m/s2


Fres n = man 0 = man an = 0 (trayectoria rectilnea).

Describid el movimiento del cuerpo con el mayor detalle posible

El movimiento de la masa m queda descrito como el de un mvil que se desplaza con tra-
yectoria rectilnea de forma que en el instante inicial t 0 = 0 se encuentra en la posicin e0
= 0 (tomamos origen de espacios en dicha posicin) movindose con una rapidez v 0 = 40
m/s en el sentido que hemos tomado como positivo y sometido a una aceleracin sobre la
trayectoria de -10 m/s2 que har que su rapidez vaya decreciendo regularmente en 10 m/s
cada segundo, de modo que llegar un instante en que v valdr 0 y a partir de l la rapidez
seguir disminuyendo, es decir, tomando valores negativos cada vez ms altos, por lo que
la masa retroceder cada vez ms aprisa.

Las ecuaciones que describen ste movimiento, por ser uniformemente acelerado, son:

e = v0 t +1/2 at2 = 40t -5t2


v = v0 + at = 40 -10t

En dichas ecuaciones hemos sustituido at por a (ya que la componente normal an es 0)

Cmo podremos calcular la distancia total recorrida a los 8 segundos?

Lo primero que tendremos que saber es si en el instante t = 8 s el cuerpo est todava mo-
vindose en el sentido inicial o por el contrario ya se detuvo y est regresando. Si en la
ecuacin: v = 40 -10t, hacemos v = 0, podemos saber en qu instante se detiene y sustitu-
yendo ese valor de t en la ecuacin e = 40t - 5t2, la posicin que ocupa en dicho instante:

0 = 40 - 10t t = 4 s
e4 = 40 4 - 5 16 = 80 m

R v
n
F

O P (+)

Luego a los 8 s el cuerpo se encuentra ya de regreso despus de haber recorrido 80 m en el


sentido que hemos tomado como positivo. Para calcular la distancia recorrida en total,
bastar hallar la posicin del mvil en el instante 8 s y proceder a sumar la distancia reco-
rrida en cada sentido. Tratad de prever el resultado que se obtendr antes de realizar nin-
guna operacin.
Dinmica 63

e8 = 40 8 - 5 64 = 0 m.
v8 = 40 - 10 8 = - 40 m/s

8s IDA
0s 4s
(+)

O v= 0
e = 80 m
VUELTA

En el instante 8 s, se encuentra en el origen de espacios movindose en sentido negativo


con una rapidez de -40 m/s y por tanto la distancia total recorrida en ese tiempo habr sido
de 160 m (mientras que el cambio de posicin, e, ser 0). Este resultado era previsible,
ya que, al tratarse de un movimiento uniformemente acelerado el cambio de rapidez expe-
rimentado en los 4 segundos que tarda en pararse tendr que coincidir con el que experi-
menta en los 4 segundos siguientes. Un ejercicio interesante para comprender este movi-
miento sera construir e interpretar las grficas: e = e (t) y v = v (t).

7. Desde el suelo se lanza, hacia arriba, un cuerpo de 2 kg de masa con una rapidez de
20 m/s. Al mismo tiempo sopla un viento lateral que le origina una fuerza constante
horizontal de 8 N. Determinad la ecuacin de la trayectoria que describe el cuerpo
hasta llegar al suelo.

sol: x = 2t2, y = -5t2 +20t , z = 0

8. Un proyectil de 2 g de masa sale de la boca de un arma con una rapidez de 300 m/s.
Sabiendo que la fuerza resultante que actu sobre l a lo largo del can del arma
4 105 t
fue: F 400 (F en N para t en s). Calculad:
3
a) Tiempo para que el proyectil recorra el can.
b) Longitud del can.
sol: a) t = 0003 s. b) L = 06 m

9. En el interior de un ascensor hay una persona de 80 kg de


masa. Determinad la fuerza que ejercer sobre el suelo del
ascensor cuando este:

a) Sube con una aceleracin de 2 m/s2


b) Baja con una aceleracin de 2 m/s2
c) Sube con una rapidez constante de 3 m/s
d) Cae libremente al romperse el cable
Dinmica 64

La persona interacciona con el suelo


del ascensor y con la Tierra. Se trata de dos interac-
ciones diferentes.
Llamaremos P a la fuerza de atraccin que la Tierra ejerce sobre la
persona, P a la fuerza con que la persona atrae a la Tierra, R a la fuerza normal y hacia
arriba que el suelo del ascensor ejerce sobre la persona y N a la fuerza normal y hacia
abajo que la persona hace sobre el suelo del ascensor (ved figura siguiente). Aplicando el
tercer principio de la dinmica a cada una de ellas, podemos escribir que:

Interaccin Tierra-persona: P = - P
Interaccin suelo del ascensor-persona: R = - N

Conviene no confundir P con N ya que se trata de dos fuerzas distintas: la primera, tiene
naturaleza gravitatoria, es ejercida por la Tierra sobre la persona y no depende del tipo de
movimiento que lleve el ascensor; la segunda, no es gravitatoria, es ejercida por la persona
sobre el suelo y su valor depende, como veremos, del tipo de movimiento que lleve el as-
censor

La fuerza resultante que actuar sobre la persona vendr dada pues por la suma vectorial

Fres R P , como R = - N en todo momento, para obtener N lo nico que tendremos

que hacer ser despejar - R de la anterior ecuacin, con lo que nos quedar que:

N P Fres en todos los casos.

Para resolver el problema necesitamos definir un


sistema de coordenadas con el que poder expresar las
fuerzas vectorialmente. Podemos escoger, por ejem-
plo, un sistema de coordenadas cartesianas con ori-
gen en la persona, como el que se indica en la figura
de la derecha:


En la figura no hemos incluido la fuerza P que estara localizada
en el centro de la Tierra y,
para poder diferenciarla de las dems, hemos desplazado N de su lnea de accin (hacia la
izquierda). Calculemos N en las distintas situaciones que se demandan en el enunciado (en
todas ellas, para simplificar, supondremos un valor absoluto para g de 10 m/s2):

a) Asciende con una aceleracin de 2 m/s2



P = (0, -mg)
Fres = (0, ma)

N = P - Fres = (0, -mg) - (0, ma) = (0, - mg - ma)

N = (0, -800-160) = (0, -960) N N = 960 N


Dinmica 65

En este caso, la persona ejerce sobre el suelo una fuerza mayor que su peso. Todos hemos
podido notar este efecto cuando un ascensor inicia la subida (acelera hacia arriba). En
esos momentos notamos como presionamos sobre el suelo con ms intensidad de la habi-
tual (como consecuencia de que estamos ejerciendo mayor fuerza).

b) Desciende con una aceleracin de 2 m/s2



P = (0, -mg)

Fres = (0, -ma)

N = P - Fres = (0, -mg) - (0, -ma) = (0, -mg + ma)

N = (0, -640) N N = 640 N

Ahora, la persona ejerce sobre el suelo una fuerza menor de lo que pesa. Todos hemos
podido notar este efecto cuando un ascensor inicia el descenso (acelera hacia abajo). En
esos momentos notamos como presionamos sobre el suelo con menos intensidad de la
habitual

c) Asciende con una rapidez constante de 3 m/s



P = (0, -mg)

Fres = 0

N = P = (0, -mg) = (0, -800) N N = 800 N

Como el ascensor lleva un movimiento rectilneo y uniforme, el valor de la fuerza que


hace la persona sobre el suelo del ascensor coincide numricamente con el valor de la
fuerza peso, aunque, insistimos, no se trata de la misma fuerza.

d) Cae libremente al romperse el cable



Podemos utilizar el resultado del apartado b, segn el cual N =
(0, - mg + ma) sin ms que sustituir a por g, con lo que nos

queda que: N = 0, es decir, en este caso la persona no hara
ninguna fuerza sobre el suelo del ascensor, flotara sobre el
suelo de la cabina, aunque, naturalmente,
sigue siendo atrada
por la Tierra con la misma fuerza P . (Se encontrar en una
situacin de cada libre). As pues, en este caso:

N=0
Dinmica 66

10. En la figura adjunta se muestran dos cuerpos que se encuentran sobre una super-
ficie plana sin rozamiento. Ambos se hallan inicialmente en reposo el uno junto al
otro. Se ejerce entonces una fuerza F sobre el objeto A tal y como se muestra en la
figura. Explicad a continuacin cul de las siguientes propuestas es correcta:

F A B

a) La fuerza que actuar sobre B ser menor que F


b) La fuerza que actuar sobre B tendr el mismo valor que F
c) La fuerza que actuar sobre B ser mayor que F

Una respuesta precipitada podra llevarnos a sealar la propuesta b) como correcta, pen-
sando que si el bloque A es bastante rgido, lo nico que har ser transmitir la fuerza que
se haga sobre l al bloque B que est justo a su lado. Sin embargo, si reflexionamos un
poco nos daremos cuenta de lo incorrecto de esta apreciacin.

Pensad que ocurrira con la aceleracin con que se movera cada bloque si la fuerza que
actuase sobre B tuviese el mismo valor que la que acta sobre A.

En efecto, si la misma fuerza que acta sobre el A actuase tambin sobre el B, como las
masas de ambos son distintas, tambin lo seran las aceleraciones, lo que hara que stos
se separasen y dejasen de estar en contacto, en cuyo caso A ya no podra transmitir la

fuerza F a B.

De acuerdo con el razonamiento anterior, conviene que analicemos lo que ocurre con ms
detalle:

Consideremos, en primer lugar, el efecto que tendra F sobre A en el supuesto de que no
existiese B.

En ese caso, las fuerzas actuantes sobre el cuerpo A seran la fuerza F , el peso PA , y la

fuerza normal R que le ejerce la superficie, de modo que la resultante sera la suma de
esas tres fuerzas:

RA

F

Fres = F + PA + R A = F (ya que PA y R A se anulan entre si)
PA
Dinmica 67


es decir, a todos los efectos sera como si sobre A solo actuase la fuerza F . Como conse-

cuencia este experimentara una aceleracin: a F/m , que le producira una velocidad v

en la misma direccin y sentido que F y cuyo mdulo ira aumentando.

Veamos ahora lo que suceder si, tal y como se especifica en el enunciado, al lado de A y
en contacto con l existe otro bloque B.

La accin de F se ejerce nicamente sobre A, pero en cuanto A comience a desplazarse se

encontrar con B y le ejercer una fuerza que llamaremos FBA (fuerza que sobre B hace
A). De acuerdo con el principio de accin-reaccin, al mismo tiempo B har otra fuerza

de igual mdulo pero sentido contrario sobre A, que llamaremos FAB , y por tanto en la

interaccin entre ambos bloques se cumplir que FAB = - FBA . El efecto total, ser que los
dos cuerpos se desplazan hacia la derecha mantenindose en todo instante juntos (con la
misma aceleracin). En este caso, las fuerzas que actan sobre cada uno son:

RA RB
FAB F FBA

PA PB

y la resultante sobre cada uno de ellos ser:


Sobre A: Fres A F FAB PA R A F FAB

Sobre B: Fres B FBA PB R B FBA

F FAB FBA
por lo que la aceleracin vendr dada por: a A y aB
mA mB

y, como sabemos, estas dos aceleraciones han de ser iguales, ya que ambos bloques se
movern conjuntamente hacia la derecha (no existe rozamiento).

Analizad los resultados obtenidos hasta aqu y extraer conclusiones respecto a las propo-
siciones que se plantean en el enunciado.

Podemos resumir los razonamientos anteriores en las siguientes conclusiones:

1) El cuerpo A se desplaza con una aceleracin menor de lo que lo hara si no estuviese


B. Dinmicamente
la existencia de B se manifiesta en A por la existencia de una nueva
fuerza FAB opuesta a F , que hace que la aceleracin de A sea menor.
Dinmica 68

2) El cuerpo B se desplaza con la misma aceleracin que A, por la actuacin sobre l , no



de la fuerza F (que solo acta sobre A), sino de la fuerza FBA , que aunque sea consecuen-

cia de haber actuado F sobre A, no es la misma fuerza (sus agentes son distintos y ni si-

quiera tienen el mismo valor). De hecho, si FBA fuese igual a F , implicara que:

Fres A F FAB = F FBA = 0

Es decir, que A no acelerara y, como se encuentra inicialmente en reposo, no se movera,



cosa que no es cierta. Por otra parte, si FBA fuese mayor que F ello implicara que la

fuerza resultante sobre A, Fres A F FAB , ira hacia la izquierda (porque son dos fuerzas
en sentido contrario y FAB sera mayor que F), con lo que A comenzara a moverse en ese
sentido, cosa que, evidentemente tampoco es cierta.

3) La fuerza que acta sobre B es menor que la fuerza aplicada a A, como podemos ver si
igualamos las aceleraciones y operamos:

F FAB FBA F FBA FBA mB
mB F mA FBA mB FBA FBA F
mA mB mA mB mB mA

11. Sobre un cuerpo de 2 kg de masa, inicialmente en reposo, acta una fuerza resul-
tante (incluyendo el peso) variable con el tiempo: F=10-2t, segn el eje OX (+). Calcu-
lad su cantidad de movimiento.

Sabemos que la fuerza resultante est relacionada con la cantidad de movimiento median-
te la expresin:


dp
Fres
dt

Para resolver este ejercicio bastar con utilizar dicha expresin teniendo en cuenta los
datos presentes en el enunciado.

dp
t
dp t Fres dt
p
Fres dp Fres dt
p0
dt 0

Como el movimiento transcurre en el eje X y sabemos que en t 0 = 0 el cuerpo se hallaba


en reposo (con lo que la cantidad de movimiento inicial es nula), podemos trabajar solo en
dicho eje, con lo que la expresin anterior queda como:

dp 0(10 2t ) dt px = (10t-t ) o bien: p (10t-t , 0 , 0)
px t 2 2
0
Dinmica 69

12. Un bloque de 450 kg de masa se encuentra


R F1
en reposo sobre un plano horizontal, cuando F2
30
comienzan a actuar sobre l las fuerzas F1 y

F2 de mdulos 7000 N y 4000 N respectivamen- P
te, tal y como se indica en la figura. Suponiendo
el rozamiento despreciable, se pide:
a) Distancia que habr recorrido al cabo de 5 s de actuar dichas fuerzas.
b) Valor de la fuerza normal que el bloque ejerce sobre el suelo.

En este problema tenemos un cuerpo, que est inicialmente en reposo y se le somete a la ac-
cin de diversas fuerzas constantes, por lo que ir aumentando su velocidad en la direccin y
sentido de la fuerza resultante, segn una trayectoria rectilnea.

En aquellos casos en los que, como ste, la trayectoria sea conocida de antemano (sea o no
rectilnea) es ms cmodo trabajar utilizando las componentes intrnsecas de las fuerzas que
actan sobre el cuerpo. Para ello conviene seguir los siguientes pasos:
1) Realizar un esquema detallando la trayectoria y todas las fuerzas que actan sobre el cuerpo.
2) Tomar un origen O en un punto cualquiera de la trayectoria y escoger (arbitrariamente)
un sentido como positivo.

3) Dibujar los vectores unitarios tangencial y normal n . Recordemos que dichos vectores

estn centrados en el cuerpo (aunque los dibujemos desplazados por claridad visual) y que

siempre ha de ser tangente a la trayectoria y en sentido positivo, mientras que n siempre ha
de ser perpendicular a la trayectoria y sentido hacia el centro de la curva (si es recta da igual el
sentido en que se dibuje).

4) Descomponer la ecuacin fundamental de la dinmica Fres m a en dos ecuaciones
escalares segn las componentes intrnsecas, es decir: Fres t = mat y Fres n = man y trabajar
despus con dichas ecuaciones.
En el problema que nos ocupa un posible esquema podra ser el siguiente:

R F1 n
F2
30
+

O N P

Dado que todas las fuerzas que actan a lo largo de la trayectoria son constantes, el tipo de
movimiento del cuerpo ser uniformemente acelerado (si la componente tangencial de la
fuerza resultante es constante, la aceleracin tangencial tambin lo ser). Por tanto, para cal-
cular el desplazamiento que nos piden, bastar hallar primero la aceleracin tangencial y a
continuacin aplicar las ecuaciones del movimiento uniformemente acelerado. En cuanto a la
aceleracin normal, al tratarse de una trayectoria rectilnea, vale 0.

Fres t = mat F1t + F2t + Rt + Pt = F1cos - F2 = mat (1)


Fres n = man F1n+ F2n+ Rn+ Pn = F1sen + R P = 0 (2)
Dinmica 70

F1 cos F2
a) Despejando at de la primera ecuacin obtenemos que: a t = 46 m/s2
m
1
Sustituyendo en e at t 2 obtenemos finalmente e = 575 m
2

b) En cuanto al valor de la fuerza normal N que ejercer el bloque sobre el suelo, segn el
principio de accin y reaccin, el mdulo de dicha fuerza deber coincidir con el de la fuerza

normal R que el suelo hace sobre l, por tanto, de (2):

R = P - F1sen y sustituyendo: R = 450 98 7000 05 = 910 N N = 910 N

13. Se quiere determinar el coeficiente de rozamiento entre una caja y un tabln, ele-
vando poco a poco el tabln y observando cundo comienza a deslizar la caja. Al rea-
lizar la experiencia, se observa que la caja empieza a deslizar cuando la inclinacin
del tabln es de 28. Qu valor presenta el coeficiente de friccin?

Veamos, en primer lugar, las fuerzas que actan sobre la caja. sta interacciona con el
plano y con la Tierra, por lo que
sobre ella se ejercen las siguientes fuerzas:
fuerza de
rozamiento Froz , fuerza peso P , y la fuerza normal que ejerce el plano R . Sumando las
tres, obtendremos la fuerza resultante sobre la caja:

Fres Froz P R

Qu le puede suceder a la caja al dejarla en reposo sobre el plano?



a) Que permanezca en reposo, en cuyo caso Fres = 0.
b)
Que descienda siguiendo una trayectoria rectilnea a lo largo del plano, en cuyo caso
Fres tendr la direccin del plano y sentido descendente.

En caso de que la caja descienda, se trata de un movimiento de trayectoria conocida, por


lo que conviene estudiarlo escalarmente utilizando las componentes intrnsecas de las
fuerzas que actan, tal y como se propone a continuacin:

Expresad todas las fuerzas que actan sobre la caja tomando como sistema de referencia
el que se indica en la figura
Dinmica 71


P = P cos (90- ) + P cos (180 - ) n = P sen - P cos n

R = R cos 90 + R cos 0 n = R n

Froz = Froz cos 180 + Froz cos 90 n = - Froz

Sumando los vectores anteriores obtenemos que:



Fres= Fres t + Fres n n = (P sen - Froz) + (R- P cos ) n = m a = m (at + an n )

y descomponiendo la ecuacin anterior en dos segn las componentes escalares:

(1) Fres t = P sen - Froz = mat (2) Fres n = R - P cos = man

En lo sucesivo, obviaremos todos estos pasos y cuando vayamos a resolver un problema


mediante tratamiento escalar (en general aquellos en los que la trayectoria sea conocida),
procederemos (despus de haber especificado el sentido que se toma como positivo) a
escribir directamente las ecuaciones correspondientes a las componentes escalares tan-
gencial y normal de la fuerza resultante.

Qu informacin podemos extraer de las ecuaciones (1) y (2) ?

a) Al tratarse de una trayectoria rectilnea, la velocidad no cambia de direccin y, por tan-


to, la componente normal an del vector aceleracin, es nula. Introduciendo esta condicin
en la ecuacin (2), nos queda que: R - P cos = 0, de donde: R = P cos .

Una consecuencia directa de esto es que la fuerza de rozamiento mxima cuyo valor es N, se
podr expresar como: Fr max = mgcos, ya que R y N tienen el mismo valor absoluto.

b) Si analizamos la expresin (1) P sen - Froz = mat nos damos cuenta que:

Si el ngulo es muy pequeo, puede ocurrir que P sen sea menor que Fr max . En este
caso el valor de Froz ser inferior al valor mximo N y valdr lo mismo que P sen , de
forma que el cuerpo permanecer en reposo.

Si vamos aumentando el ngulo , llegar un momento en que tomar un valor C tal que para
dicho valor se cumplir que P sen C = Fr max = N. En este caso, Fres t seguir siendo nula y la
caja permanecer en reposo, pero bastara un ligero impulso hacia abajo para que se deslizara
con movimiento uniforme. Del enunciado del problema podemos considerar que C es preci-
samente 28, con lo que sustituyendo en la expresin anterior podremos hallar fcilmente el
coeficiente de friccin: P sen C = N = P cos C = tg C = tg 28 = 053.

Si el ngulo se hace mayor que C ( C ) tendremos que P sen N, es decir, existir


una fuerza resultante tangencial en sentido descendente, que har que la caja se mueva
hacia abajo deslizando por la superficie del plano con movimiento rectilneo uniforme-
mente acelerado. La aceleracin se podra calcular con la expresin (1):

P sen - Fr max = mat

y como Fr max = N y la fuerza normal N = P cos = mg cos , nos quedara que:

at = g (sen - cos ), expresin que solo podremos aplicar cuando C.


Dinmica 72

Y si en esta ltima expresin introducimos la condicin de que el ngulo sea de 90?.

Se tendra que la caja no ejercera ninguna fuerza sobre la superficie y caera libremente
con la aceleracin de la gravedad (at = g).

14. Si dejamos en libertad un cuerpo de 4 kg de masa sobre un plano inclinado de 30 0


y a una altura de 5 m, llega a la base del plano con una rapidez de 8 m/s. Determinad
el coeficiente de friccin y el valor F de la fuerza que deberamos hacer sobre dicho
cuerpo, en direccin perpendicular al plano para que llegase a la base con una rapi-
dez de 2 m/s.

En este ejercicio hay dos movimientos a diferenciar:

a) Cuando dejamos el cuerpo sobre el plano y desciende (el ngulo es superior al crtico).
En este caso, como ya hemos razonado en el ejercicio anterior, el cuerpo descender con
movimiento uniformemente acelerado de aceleracin: a t = g (sen - cos ). Vemos que
el valor de la aceleracin depende del valor del coeficiente de friccin. Si, como sucede
en este caso, nos piden el valor de dicho coeficiente, bastar conocer el valor a t y sustituir-
lo en:

at
tg
g cos
Cmo podramos calcular at?

Al tratarse de un movimiento uniformemente acelerado del que conocemos que, partiendo


del reposo, el mvil alcanza una rapidez de 8 m/s tras desplazarse 10 m (distancia hasta la
base), podemos obtener sus ecuaciones y con ellas tratar de calcular a t:

En efecto, si consideramos que t0 =


0, v0 = 0 y e0 = 0 (segn el sistema
referencial de la figura adjunta),
tenemos que:

v = att
e = att2/2

v2
De la primera ecuacin obtenemos: t = v/at y sustituyendo ahora en la segunda: a t
2e

Imponiendo ahora la condicin de que en cierto instante t, el mvil tiene una rapidez de 8
m/s y ocupa una posicin tal que su espacio es de 10 m, nos queda:

v2 64 m
at = 32
' 2 como valor de la aceleracin tangencial con la que desciende.
2e 2 10 s
Dinmica 73

Si ahora sustituimos este valor en la expresin de , obtenemos:

3'2
tg30 = 021
10 cos 30

b) Cuando dejamos el cuerpo sobre el plano pero presionamos sobre l con una fuerza F
perpendicular al plano y desciende pero llegando a la base con una rapidez menor que
anteriormente.

Cul puede ser el efecto de esta fuerza? Por qu hace que el cuerpo llegue abajo con
menos rapidez que antes?

En primer lugar, hemos de tener en cuenta que esta fuerza no altera el valor del coeficien-
te de friccin (que seguir siendo, por tanto, 021, ya que ste solo depende de la naturale-
za de las superficies puestas en contacto). Por otra parte, la fuerza se ejerce en direccin

normal al plano, por lo que al actuar segn n no podr influir directamente en el valor de
la aceleracin tangencial, pero, sin embargo, s que aumentar el valor mximo de la fuer-
za de rozamiento por deslizamiento que acta sobre el cuerpo (porque har que aumente

la fuerza normal N que el cuerpo ejerce sobre la superficie del plano) y, como consecuen-
cia, este efecto har que disminuya la aceleracin tangencial con que desciende el cuerpo.
De hecho, si el coeficiente de friccin pudiera ser nulo, la fuerza que estamos consideran-
do no tendra ningn efecto sobre la aceleracin tangencial (ya que por grande que fuese
N, no habra fuerza de rozamiento).

Cmo podramos calcular el valor de F que nos piden?

Para hacer un estudio cuantitativo de


la situacin, podemos considerar las
fuerzas que actan en este caso (ver
figura adjunta) y expresar la ecua-
cin fundamental de la dinmica en
componentes intrnsecas:

Fres t = P sen - Fr = mat (1)


Fres n = R - P cos - F = man (2)

Al ser la trayectoria rectilnea, a n = 0, y como el cuerpo desliza, el valor de la fuerza de


rozamiento ser el mximo, de manera que F r = N = R. Sustituyendo en las ecuacio-
nes (1) y (2), obtenemos:

mg sen - R = mat (3)


R = mg cos + F (4)

La ecuacin (3) nos da la aceleracin at y sustituyendo en ella el valor de R dado por (4)
y despejando at :
Dinmica 74

at = g (sen - cos ) - F/m at = at - F/m

Determinando at de forma anloga a como obtuvimos at, obtenemos que at = 02 m/s2


(que, como vemos es menor que el valor anterior) y sustituyendo en la ecuacin anterior,
podemos calcular fcilmente el valor de F que se nos pide, resultando ser F = 568 N.

Si nos detenemos en analizar la ltima expresin obtenida, nos podemos dar cuenta de
que adems de ser dimensionalmente homognea, el valor de a t es constante y depende
de la fuerza F que se ejerza, de modo que cuanto mayor sea F con menor aceleracin des-
cender el cuerpo. Adems contempla algunos casos evidentes, como que si F valiese 0, se
obtendra at = at, y tambin, que si F fuera 0 y = 90 (cada libre) at = g, etc.

15. Un cuerpo de 2 kg de masa se lanza con una rapidez de 6 m/s desde la base de un
plano inclinado de 5 m de longitud y 3 m de altura. Sabiendo que el coeficiente de
friccin es 06, se pide:

a) Altura mxima que alcanzar


b) Razonad si bajar o no.
c) En caso de que baje, calculad cunto tardara en llegar a la base.

sol: a) h max = 1m ; b) Si, porque ; c) t = 167 s

16. Un camin va cargado con cajas llenas de huevos. El coeficiente de rozamiento


entre las cajas y el suelo del camin es 03. Suponiendo que el camin se mueve a 72
km/h, calculad la distancia mnima en que puede detenerse, frenando de manera uni-
forme, para que las cajas no deslicen.

Supongamos que un observador en reposo situado fuera del camin, pudiera observar lo
que le ocurre a una de las cajas.

Qu es lo que dira respecto a las fuerzas que actan sobre la caja mientras el camin (y
la caja) se alejan de l con velocidad constante?
Dinmica 75

Si la caja se aleja del observador con velocidad constante (la misma que la del
camin),
es
porque sobre ella la fuerza resultante es nula y como sobre la caja actan P , R y, quizs,

la Froz , deber cumplirse: Fres = P + R + Froz = 0. Como Froz es perpendicular a P y R , la

nica posibilidad es que P y R se anulen y Froz = 0. En esta situacin, si el coeficiente de
friccin entre la caja y la superficie sobre la que se encuentra fuese nulo, y el camin fre-
nase, el observador externo vera que dicha caja continuara con la misma velocidad que
llevaba, solo que como el camin ira cada vez ms despacio, la caja deslizara por la su-
perficie hasta chocar finalmente con la cabina. No importar lo suave de la frenada,
porque la velocidad de la caja no disminuira nada.

No obstante, en la realidad, sabemos que si el camin frena con la suavidad suficiente, es


posible que la caja no se mueva respecto a la superficie donde se halla (no deslice).

Cmo se explicara este hecho?


Que la caja no se mueva respecto al suelo del camin cuando ste frena, quiere decir que
llevar la misma aceleracin que el camin, es decir, que su velocidad respecto del obser-
vador exterior ir cambiando de la misma forma que lo hace la del camin. Ahora bien: si
la velocidad de la caja disminuye de valor, tiene que haber una fuerza resultante en senti-
do contrario al movimiento. Dicha fuerza no puede ser otra que la fuerza de rozamiento
entre la caja y el suelo del camin, que impide que sta contine movindose hacia la de-
recha a 72 km/h cuando el camin frena (como hara si no hubiese friccin). As pues,
cuando el camin frena, segn
elobservador exterior, sobre la caja actuarn tres fuerzas
de modo que: Fres = P + R + Froz . Como la trayectoria es rectilnea, no existe acelera-

cin normal con lo que: P + R = 0, y la fuerza resultante sobre la caja es la fuerza de ro-
zamiento. Podemos tratar el problema de forma escalar y escogiendo como sentido positi-
vo el del movimiento, escribir:

(1) Fres t = mat - Froz = mat


(2) Fres n = man R- P = man = 0

Cul ser la mxima aceleracin que podr llevar la caja?


En principio, supondremos que el camin puede frenar con cualquier aceleracin, pero la
de la caja vendr limitada por el mximo valor que puede presentar la fuerza de rozamien-
to entre ella y la superficie donde se encuentra, de manera que:

amax = - Fr max /m = - N/m


Cmo podemos calcular N?
De la ecuacin (2) despejamos R y nos queda R = P. Como R = N (principio de accin y
reaccin) obtenemos que N = P = mg, con lo que:

amax = - N/m = - mg/m = - g

Razonad que le ocurrira a la caja si la aceleracin con que frenase el camin (en va-
lor absoluto) fuese: a) mayor que g; b) menor que g; c) igual a g
Dinmica 76

a) Si el camin frena con una aceleracin a t g la rapidez de la caja no podr disminuir


tan aprisa como la del camin y, por tanto, la caja se ver desplazada deslizando hacia
adelante. (Nosotros mismos notamos este efecto cuando estamos en un autobs y ste
disminuye bruscamente de velocidad). La fuerza de rozamiento sobre la caja tomara su
valor mximo, sin sobrepasarlo por grande que fuese la aceleracin con que frenase el
camin.

b) Si el camin frena con una aceleracin a t g , la caja para a la vez que el camin y
no deslizar. La fuerza de rozamiento toma un valor inferior al mximo N .

c) Si el camin frena con una aceleracin a t = g, la caja tampoco desliza pero se encuen-
tra en una situacin lmite ya que se trata de la mxima aceleracin que puede llevar. Si el
camin aumenta ms su aceleracin de frenado, la de la caja seguir valiendo g y, en
consecuencia deslizar adelantndose (caso a).

Cmo podemos calcular la menor distancia posible que precisar el camin para parar-
se sin que deslicen las cajas en su interior?

Cuanto mayor sea (en valor absoluto) la aceleracin con que frena el camin, menos dis-
tancia recorrer hasta pararse, luego la mnima distancia que necesitar para asegurarse de
que las cajas no deslizan, vendr determinada por la distancia correspondiente a una ace-
leracin de frenado at = g. Podemos saber cunto vale dicha distancia si consideramos
que se trata de un movimiento uniformemente acelerado (a t es constante) y manejamos las
ecuaciones correspondientes. Para ello podemos considerar como origen de espacios y
tiempos el lugar e instante en que comienza a frenar y sentido positivo (como ya hemos
dicho) el del movimiento, con lo que:

v = v0 + a (t-t0 ) v = v0 - g t
e = e0 + v0 (t - t0 ) + 1/2 a (t - t0 )2 e = v0 t - 1/2 g t2

En las ecuaciones anteriores, como an = 0, hemos hecho at = a. La distancia recorrida co-


incidir, en este caso, con el valor de e en el instante en que se pare el camin. Dicho ins-
tante lo podemos obtener haciendo v = 0 en la primera de las ecuaciones anteriores, de
modo que:

0 = v0 - g t t = v0 / g . Sustituyendo en la segunda ecuacin nos queda:

v0 1 v02 v02
e = v0 g e e = 202 / 20310 = 400/6 = 667 m.
g 2 ( g ) 2
2 g
Dinmica 77

Analizad el resultado literal anterior

En primer lugar, observemos que se trata de una ecuacin dimensionalmente homognea


(L en ambos lados del signo igual).
En segundo lugar nos podemos dar cuenta de que cuanto mayor sea la rapidez inicial del
camin, a igualdad de los restantes factores, ms distancia necesitar para pararse, pero
adems, el resultado obtenido muestra que como v0 est elevada al cuadrado, cuando la
rapidez inicial se duplique, la distancia necesaria para parar se cuadruplicar. Es por ello
que resulta de la mayor importancia guardar una distancia mnima de seguridad entre los
vehculos. Por otra parte, tambin se puede comprobar que si el rozamiento entre la caja y
el camin disminuye la distancia mnima necesaria para pararse sin que la caja deslice
aumentar y que en el caso hipottico de que no hubiese ningn rozamiento dicha distan-
cia mnima sera infinita, o lo que es lo mismo: en ese caso ideal, por muy lento que fre-
nase, no podra evitar que las cajas deslizaran.

17. Apoyamos, sobre la pizarra, un borrador de 200 g y con un dedo aplicamos una
fuerza horizontal (en sentido de la pizarra). Determinad el mnimo valor de la fuerza
que permitira que el borrador no cayera. Qu sucedera si aplicsemos una fuerza
doble de la evaluada? (Coeficiente de friccin pizarra-borrador = 12 ).

De acuerdo
con el principio de accin y reaccin, la fuerza que sobre el borrador hace el
dedo Fbd tendr el mismo mdulo y sentido contrario que la fuerza Fdb que sobre el dedo

hace el borrador Fbd = - Fdb . Lo mismo ocurrir con la fuerza normal N que el borrador

ejerce sobre la pizarra y la fuerza normal R que la pizarra hace sobre el borrador.
Las
fuerzas actuantes sobre el borrador sern, en principio, Fbd , R y la fuerza peso P . Como
el nico movimiento posible para el borrador es que deslice hacia abajo, no podr existir
aceleracin horizontal, deber cumplirse que la resultante segn la horizontal sea nula: Fbd

+ R = 0, y por tanto que: Fbd = R, o, lo que es lo mismo: Fbd = N. En estas condiciones, si
no existiera ningn rozamiento entre borrador y pizarra (es decir, si valiese 0), sera
imposible evitar que ste cayese debido a la fuerza peso. Sin embargo, sabemos que al
apretar el borrador contra la pizarra existir un cierto rozamiento entre ambos objetos,
que puede ser suficiente para evitar que el borrador caiga.

Cmo podemos interpretar que el borrador no caiga?


Dinmica 78

La nica explicacin es que sobre el borrador acte una fuerza vertical y ascendente que
equilibre a la fuerza peso. Dicha fuerza no puede ser otra que la fuerza de rozamiento. Si
P es mayor que el valor mximo de la fuerza de rozamiento (Fr max), el borrador cae, pero
si no supera dicho valor entonces el borrador no cae y la fuerza de rozamiento toma jus-
tamente el valor de P (Froz = P).
Cul es el valor mximo de la fuerza de rozamiento?
En el caso de rozamiento por deslizamiento, sabemos que el valor mximo de la fuerza de
rozamiento viene dado por la expresin Fr max = N = R. Como en este caso, segn
hemos visto, R = Fbd se concluye que Fr max = Fbd . Por tanto, cuanto mayor sea la fuer-
za que sobre el borrador hace el dedo, mayor ser el valor lmite de la fuerza de rozamien-
to y podremos aguantar un borrador ms pesado.

Cmo podremos hallar el valor mnimo de Fbd que permitir que el borrador no caiga?
Cuando el borrador se mueve (cae) es porque P supera el valor mximo de la fuerza de
rozamiento. Como dicho valor viene expresado por: Fr max = Fbd , igualando esta expre-
sin a la fuerza peso, obtendremos el valor que, como mnimo, debe tener la fuerza que
hemos de realizar sobre el borrador con el dedo, para que no caiga:

Fbd min = mg Fbd min = mg/ = 0210 /12 = 167 N.


Analizad el resultado literal anterior.
El resultado obtenido es dimensionalmente homogneo (las dimensiones de una fuerza en
ambos lados del signo igual).
Podemos ver que cuanto mayor sea el peso mg del borrador, mayor tendr que ser la fuer-
za mnima necesaria para aguantarlo, de manera que si el borrador no pesara, no necesitar-
amos hacer ninguna fuerza para mantenerlo en reposo sobre la pizarra. El coeficiente de
friccin influye de manera inversa ya que cuanto mayor sea su valor, menor ser la fuerza.
Naturalmente, podemos ejercer ms fuerza con el dedo sobre el borrador. En este caso no
vemos que el borrador se mueva hacia arriba Cmo se explicara este hecho?
Porque al hacer sobre el borrador una fuerza Fbd mayor que la mnima, el valor de la fuer-
za de rozamiento no aumenta. Lo que aumenta es el valor mximo que puede tomar la
fuerza de rozamiento (aquel para el cual el objeto comienza a deslizarse), de forma que el
borrador seguira en reposo. Lo que s que conseguiramos sera aguantar un borrador de
mayor peso sin que deslizase. En este sentido podemos plantearnos la siguiente cuestin:
Hasta que valor podra aumentar la masa del borrador para que siguiese donde est sin
caer, si duplicamos la fuerza mnima calculada anteriormente?

El valor mximo de la fuerza de rozamiento sera ahora Fr max = 2 Fbd . Conforme fue-
se aumentando el peso del borrador, la fuerza de rozamiento requerida ira tambin au-
mentando (recordemos que la fuerza de rozamiento no tiene un solo valor sino que puede
tomar infinitos valores entre 0 y un valor mximo). En cuanto el peso superase el valor
mximo que puede tomar dicha fuerza de rozamiento (2 Fbd ) el borrador comenzara a
caer. Cuando esto ocurra se cumplir que:
Dinmica 79

2 Fbd = m g m = 2 Fbd / g = 216712 / 10 = 04 kg (doble que antes).

18. Sobre un cuerpo de 5 kg de masa, que se encuentra encima de un plano inclinado


de 30, acta una fuerza F en la direccin de la figura. Sabiendo que el coeficiente de
friccin es 03, determinad F para que el cuerpo:

a) Ascienda con movimiento uniforme.


b) Descienda con movimiento uniforme.
c) Descienda con aceleracin de 1 m/s2

sol: a) 531 N. b) 118 N. c) 6'9 N


19. Sobre un bloque A de 10 kg de masa acta una fuerza F tal y como se aprecia en
la figura. Si le adosamos un cuerpo B de 2 kg de masa, con el que presenta
un coefi-
ciente de friccin de 04, determinad el mnimo valor que puede tomar F para que B
no caiga (el coeficiente de friccin entre A y el suelo vale 02 ).

B
F
A

Las interacciones que nos interesa considerar (ved esquema siguiente) son:

Entre los bloques y la Tierra. La Tierra atrae a los bloques con fuerzas PA y PB y es
atrada por los bloques con otras fuerzas iguales y de sentido contrario con origen en su
centro (que no dibujaremos).

Entre el bloque A y la superficie. El bloque A ejerce una fuerza sobre la superficie y esta
a su vez otra fuerza igual y de sentido contrario sobre el bloque. Esta interaccin se suele
descomponer en dos segn la tangente y la normal a la superficie. Hablamos as por una

parte, de la fuerza normal N SA que el bloque A hace sobre la superficie y su pareja RAS o
fuerza normal que la superficie hace sobre el bloque A, y, por otra parte, de la fuerza
de
rozamiento FrAS que la superficie ejerce sobre A y su correspondiente pareja FrSA del
mismo mdulo pero sentido contrario que acta sobre la superficie.

Entre ambos bloques: Si el bloque B no cae, podemos, al igual que se explic en el


prrafo anterior,
descomponer la interaccin entre A y B en dos. De esta forma nos referi-

remos a R BA como la fuerza segn la horizontal que sobre B hace A y a su pareja N AB
como la fuerza segn la horizontal que sobre A hace B. Anlogamente, las fuerzas que
actuaran segn la vertical seran FrBA o fuerza de rozamiento que sobre B ejerce la super-

ficie del bloque A y FrAB . Ambas parejas son del mismo mdulo y sentidos contrarios
(principio de accin y reaccin).
Dinmica 80


Adems de las fuerzas anteriormente descritas, est la fuerza F que desde el exterior al
sistema se ejerce sobre el bloque A. Podemos representar la situacin mediante el si-
guiente esquema:

Para que el bloque B no caiga, debe haber una fuerza que compense su peso. Esa fuerza
no puede ser otra que la fuerza de rozamiento que sobre B hace A. El valor lmite o
mximo que puede tomar dicha fuerza es, como sabemos, Fr BA = AB NAB = AB RBA , de
modo que si PB es mayor que este valor mximo el bloque B caer, y si PB es inferior a ese
valor mximo, entonces la fuerza de rozamiento sobre B vale justamente lo mismo que el
peso, y el bloque no cae.

En el problema se nos pide cunto ha de valer F como mnimo para que el bloque B no
caiga. Ello sugiere buscar una relacin entre F y Fr BA, partiendo de la hiptesis de que al
aumentar F tambin aumentara la fuerza normal N AB que el bloque B ejerce sobre el A y,
en consecuencia, la fuerza de rozamiento FrBA sobre el bloque B.

Cmo podemos relacionar FrBA con la fuerza exterior F?

Podemos aplicar la ecuacin fundamental de la dinmica a cada uno de los bloques, te-
niendo en cuenta que ambos se movern con la misma aceleracin tangencial. Para ello
expresaremos las fuerzas que intervienen en funcin de sus componentes intrnsecas, to-
mando tal y como se indica en la figura, como sentido positivo el del movimiento. En es-
tas condiciones:

Sobre el bloque A: Sobre el bloque B:

(1) F - NAB - FrAS = mAat (3) RBA = mBat


(2) RAS - PA - FrAB = mAanA (4) FrBA - PB = mBanB
Dinmica 81

El bloque A, que se mueve sobre la superficie y en lnea recta, no posee aceleracin nor-
mal y el bloque B, si no cae y permanece siempre a la misma altura, tampoco.
B deslizar cuando su peso supere el valor mximo que pueda tomar la fuerza de roza-
miento (FrBA = AB NAB ). Introduciendo esta condicin en la ecuacin (4) obtenemos
que: AB NAB = PB NAB = PB / AB . Este ser el mnimo valor que puede tener N AB ( o
RBA) para que B no caiga. Si conseguimos, pues, poner F en funcin de N AB, tendremos
resuelto el problema.
Como RBA = NAB (principio de accin y reaccin) podemos escribir (3) como: at = NAB/mB,
sustituir en la ecuacin (1) y despejar F:

F - NAB - FrAS = mA NAB/ mB F = (mA NAB/ mB ) + NAB + FrAS

Para calcular FrAS , dado que el bloque A se desliza, aplicamos: FrAS = AS NSA.
Al ser NSA = RAS (principio de accin y reaccin), podemos utilizar la ecuacin (2) y obte-
ner haciendo anA = 0 que NSA = RAS = PA + FrAB . Como FrAB vale lo mismo que su pareja
FrBA = AB NAB , tenemos que: NSA = PA + AB NAB y finalmente que:
FrAS = AS(PA + ABNAB).
Sustituyendo esta ltima expresin en la que nos proporciona F, obtenemos:

F = (mA NAB/ mB ) + NAB + FrAS = (mA NAB/ mB ) + NAB + AS (PA + AB NAB).

Introduciendo ahora en esta ltima ecuacin la expresin del valor mnimo que puede
tomar NAB para que B no caiga (PB /AB), obtendremos el valor mnimo de F que se nos
pide en el enunciado:

mA m B g m B g m g
Fmin = AS m A g AS AB B y simplificando nos queda:
AB m B AB AB

1
Fmin (m A m B ) g ( AS ) Fmin = (10 + 2)10 (1/04 + 02) = 324 N
AB

Analizad el resultado literal anterior


En primer lugar, hemos de comprobar si es o no dimensionalmente homogneo. Podemos
ver que en ambos lados del signo igual las dimensiones son MLT -2 .
En segundo lugar, podemos detenernos en analizar la influencia de algunas de las magni-
tudes en el resultado final (siempre suponiendo constantes las dems). As, por ejemplo,
cuanto mayores sean las masas de A y B, ms grande ser el valor de la fuerza horizontal
exterior que como mnimo hay que realizar para que B no caiga. Lo mismo ocurre con el
valor del coeficiente de rozamiento entre A y la superficie. Sin embargo la influencia del
coeficiente de rozamiento entre A y B, como es lgico, es al contrario y podemos ver que
cuanto ms pequeo sea su valor, ms grande tendr que ser Fmin , de manera que cuando
dicho coeficiente tiende a 0, la fuerza mnima tiende a infinito. Otro factor importante es
la gravedad g. El resultado obtenido nos muestra que en ausencia de gravedad (g = 0) no
se precisara ninguna fuerza para mantener a B sin caer.
Dinmica 82

20. A una cierta altura sobre la base del plano se abandonan simultneamente dos
bloques en contacto. Sabiendo que mB = 2 mA y que los coeficientes de friccin son
1
respectivamente A = y B = A/2, se pide:
4 3

a) Aceleracin con que desciende cada bloque.


b) Fuerza que el bloque B ejerce sobre el A.

sol: a = 41 m/s2 ; FAB = mAg/24

21. Calculad la aceleracin del sistema de la figura y la tensin de la cuerda, cuando


el valor de F sea de 400 N. Coeficiente de friccin con el suelo = 04; mA =20 kg y
mB= 30 kg.

F
B A

sol: at = 4 m/s2 (tomando como sentido positivo el del movimiento); T = 240 N.

22. Resolved el ejercicio anterior suponiendo que la cuerda tiene una masa de 5 kg.

sol: at = 364 m/s2 ; TA = 2472 N ; TB = 229 N .

23. Dos cuerpos (A y B) de masas mA = 3 kg y mB =1 kg cuelgan de los extremos de


una cuerda, que pasa por la garganta de una polea. Despreciando la masa de la cuer-
da y de la polea, se pide:

a) Calculad la aceleracin con que se mueve el conjunto y la tensin de la cuerda.


b) Si eliminamos la masa de 3 kg y tiramos de la cuerda hacia abajo con una fuerza
equivalente al peso de la masa de 3 kg cunto valdr entonces la aceleracin?

El sistema formado por las dos masas y la cuerda es un sistema que se mueve solidaria-
mente con la misma aceleracin. El siguiente esquema representa las fuerzas que actan
sobre las masas A y B. Dichas fuerzas se deben a la interaccin de cada una de las masas
con la cuerda y con la Tierra.
Dinmica 83


Como podemos observar, el peso PA tira de la masa A en el sentido que, arbitrariamente,

hemos escogido como positivo, mientras que el peso PB lo hace en el contrario.

De acuerdo con el tercer principio de la dinmica, si la cuerda tira del cuerpo A hacia
arriba con una fuerza TA , este ejercer sobre la cuerda otra fuerza igual y de sentido con-

trario T ' A . Anlogamente podemos razonar para el cuerpo B el cual tirar de la cuerda con

una fuerza T' B TB . (En la figura izquierda se han representado las fuerzas que actan
sobre los cuerpos y en la figura derecha las que actan sobre la cuerda).
Dado que conocemos la trayectoria de cada cuerpo podemos trabajar escalarmente, expre-
sando la fuerza resultante en componentes intrnsecas, y como adems, en este caso, todas
las fuerzas tienen la direccin de la trayectoria, solo habr componente tangencial.
En primer lugar, hemos de tener en cuenta que si aplicamos la ecuacin fundamental de la
dinmica a la masa de la cuerda m C , tenemos que: TA - TB = mC at = 0 ya que segn se
ha dicho, la masa de la cuerda se considera despreciable. Por tanto T A = TB = T y (de
acuerdo con el principio de accin y reaccin) TA = TB = T. (En adelante, cuando se repita
esta situacin, obviaremos todo este razonamiento).
Podemos ahora aplicar la ecuacin fundamental de la dinmica a los cuerpos A y B con el
fin de obtener la aceleracin con que se mover el sistema. Como la aceleracin tangen-
cial con que se mueve cualquiera de las masas es la misma y no hay aceleracin normal
para ninguna de ellas, podemos escribir que: a tA = atB = a.

(1) Para el cuerpo A: PA - T = mA a


(2) Para el cuerpo B: T - PB = mB a
PA PB m mB
Sumando ambas ecuaciones obtenemos: a = a A g
mA m B mA mB

31
Finalmente si sustituimos los datos numricos nos sale: a = 10 5 m/s2. Es decir,
31
que la rapidez con que se mueve A o B aumentar regularmente en 5m/s cada segundo.
Dinmica 84

A continuacin nos detendremos en analizar brevemente el resultado literal obtenido:

Como podemos comprobar, es dimensionalmente homogneo ya que en ambos miembros


del signo igual quedan las dimensiones correspondientes a la aceleracin (LT -2).

El resultado tambin contempla algunos casos lmite evidentes, as por ejemplo cunto
debera de valer la aceleracin si la masa de B fuese 0?; est claro que en ese caso, A
caera libremente con la aceleracin de la gravedad y eso es precisamente lo que obtene-
mos si en el resultado hacemos m B = 0. Otro caso lmite evidente es lo que ocurrira si no
hubiese gravedad; en ese supuesto, la aceleracin sera nula aunque las masas fuesen dife-
rentes (tal y como se obtiene haciendo g = 0). El resultado tambin nos muestra que cuan-
to mayor sea la diferencia mA - mB mayor ser el valor de la aceleracin a con que se mo-
vern las masas, etc.

Cmo podemos calcular el valor T de la tensin de la cuerda?

Para ello basta con que consideremos cualquiera de las ecuaciones (1) o (2) y despejemos
T. As, por ejemplo de la expresin (2) obtenemos que T = PB + mBa = mB (g+a) y susti-
tuyendo los datos numricos nos queda T = 1 (10+5) = 15 N.

Otra cuestin que se plantea en el enunciado del problema es qu pasara si en lugar de


colgar un peso de 30 N de uno de los extremos de la cuerda, tiramos de l con la mano
haciendo una fuerza de 30 N. Obtendramos el mismo resultado para la aceleracin?

La respuesta es que no, ya que ahora dicha fuerza nicamente acelerara a la masa m B de 1
kg, mientras que anteriormente lo haca con una masa de 4 kg.

F PB F m B g
En efecto, la aceleracin valdra ahora: a = 20/1 = 20 m/s2
mB mB

Quedan abiertas algunas interrogantes como que pasara si no considersemos la masa de


la cuerda despreciable? y la de la polea?, que sern tratadas en prximos captulos.

24. Dado el dispositivo esquematizado en la


figura adjunta, sabiendo que el coeficiente
de friccin es 015 y que cada bloque tiene A
B
una masa de 20 kg, determinad el tiempo
necesario para que el sistema se desplace
1m, partiendo de una situacin inicial de
reposo.
sol: t = 104 s.

25. Un bloque de 5 kg de masa est sujeto a una cuerda de masa despreciable y es


arrastrado hacia arriba mediante ella con una aceleracin de 2 m/s 2. Calculad la ten-
sin de la cuerda.
sol: T = 60 N
Dinmica 85

26. Un pndulo de 200 g de masa cuelga suspendido del techo de un vehculo. Sabien-
do que forma un ngulo de 20 con la vertical, determinad la aceleracin del vehculo
y la tensin del hilo.

sol: a = 364 m/s2 ; T = 213 N

27. En el sistema de la figura adjunta las masas de los bloques son mA = 2 kg y mB = 6


kg. Considerando las masas de la polea y de la cuerda despreciables, se pide:

a) Aceleracin con que se mover el sistema y tensin de la cuerda, suponiendo roza-


miento nulo entre B y la superficie.
b) Que le ocurrira a la fuerza que tira de B si el peso de A se duplicase?
c) La aceleracin, suponiendo que entre el bloque B y la superficie la friccin no es
despreciable y el coeficiente de rozamiento vale = 02.

Si se supone que no hay rozamiento, las fuerzas que actuarn sobre las masas y la superfi-
cie de la figura sern:


PA = fuerza con que la Tierra atrae a mA

PB = fuerza con que la Tierra atrae a mB

R = fuerza normal que el plano hace sobre B.
N = fuerza normal que B hace al plano
TA = fuerza con que la cuerda tira de A.

TB = fuerza con que la cuerda tira de B

Como la trayectoria es conocida, podemos calcular el valor de la aceleracin mediante un


tratamiento escalar. Tomaremos como sentido positivo el del movimiento y aplicaremos
la ecuacin fundamental de la dinmica a cada una de las masas, expresndola en compo-
nentes intrnsecas, teniendo en cuenta que al tener la cuerda una masa despreciable: T A =
TB = T, y que al moverse todo el sistema solidariamente, la aceleracin tangencial de cada
una de las masas tendr el mismo valor, que es, precisamente, lo que hay que calcular.
Dinmica 86

Para la masa A: Fres t = mA at PA - T = mA at

Para la masa B: Fres t = mB at T= mB at


Fres n = mB an = 0 R - P = 0 R = P

En el caso de A no existe ninguna fuerza perpendicular a la trayectoria. En el caso de B


existen dos (su peso y la fuerza normal ejercida por la superficie), pero estas se anulan
entre s, ya que la velocidad de B no cambia de direccin (se mueve segn una trayectoria
rectilnea sobre la superficie).
Sumando las componentes tangenciales de las fuerzas tenemos que:

PA mA 2
PA-T+T= (mA + mB) at at = g = 10 2'5 m/s2
(m A m B ) (m A m B ) 2+6

Analizad el resultado literal obtenido.

El resultado es dimensionalmente homogneo (dimensiones de aceleracin en ambos


miembros) y se cumplen casos lmites evidentes, como por ejemplo, que si m B fuese nula,
mA caera con la aceleracin de la gravedad, o que si m A fuese nula, no habra aceleracin.

Qu pasara si mA fuese mucho ms pequea que mB ?

Al no existir fuerza de rozamiento, el resultado obtenido nos muestra que por pequea que
fuese la masa de A frente a la de B, el sistema tendra que acelerar (ya que a t no sera 0),
cuestionando as la idea equivocada que tienen algunas personas de que para que B se
moviese partiendo del reposo hara falta colgar de la cuerda una masa mayor que m B.

Cmo podramos calcular la tensin de la cuerda?

Bastara utilizar cualquiera de las ecuaciones en las que figura T. As, por ejemplo, en la
ecuacin T= mB at si sustituimos los datos numricos obtenemos T = 6 25 = 15 N. Al
mismo resultado habramos llegado con la otra ecuacin : P A - T = mA at despejando T
con lo que T = PA - mA at = mA (g - at) = 2 (10-25) = 15 N.

Qu ocurrira con la fuerza con que la cuerda tira de B si se duplicase el peso de A?

Una respuesta precipitada a esta cuestin llevara a afirmar que el valor de dicha fuerza
tambin se duplicara, pero si reflexionamos un poco, nos daremos cuenta de que no es
as, ya que, si
bien es cierto que el peso de A se hara el doble, hay que tener en cuenta
que, no es PA sino TB la fuerza que tira de mB . Como TB = T = mB at concluimos que,
de acuerdo con esta expresin, solo si a t se hiciese doble, la fuerza que se ejercera sobre
B se hara tambin el doble. Ahora bien: que le ocurre a la aceleracin cuando P A se du-
plica?

PA 2PA
Sabemos que at = = 25 m/s2 . Si PA se hace el doble at = = 4 m/s2
(m A m B ) 2m A m B
Dinmica 87

Como vemos at no es el doble de at y, por tanto, TB no se duplicara, sino que valdra:

TB = mB at = 6 4 = 24 N

Cmo cambiara el problema si admitimos que entre B y la superficie el rozamiento no


es nulo y el sistema se halla inicialmente en reposo?

En ese caso debemos incluir una fuerza de


rozamiento horizontal que tira de B hacia la
izquierda y comenzar por analizar con los
datos del problema si la fuerza que hace la
cuerda sobre B supera al valor mximo que
puede tomar dicha fuerza de rozamiento (en
cuyo caso B se deslizara hacia la derecha) o
no lo supera (en cuyo caso la fuerza de roza-
miento sera igual a TB y el sistema seguira
en reposo).

Sabemos que el valor mximo que puede tomar la fuerza de rozamiento viene dado por: Fr
max = N, como N = R (principio de accin y reaccin) y que R = P B = mB g (ya que an =
0), por lo que: Fr max = mg y sustituyendo Fr max = 02 6 10 = 12 N. Como el peso de
A es de 20 N y la tensin de la cuerda es la misma en todos sus puntos, nos queda que la
fuerza tangencial resultante sobre el sistema tendra sentido positivo y valdra: P A -TA + TB
- Fr = PA - Fr = 20 - 12 = 8 N, con lo que concluimos que el sistema comenzara a moverse
hacia la derecha.

Para hallar la aceleracin sobre la trayectoria, podemos partir de la expresin que nos da
la fuerza tangencial resultante que acta sobre el sistema:

PA Fr
Frest = (mA + mB)at PA +T-T- Fr = (mA + mB) at at = .
(m A m B )

(m A m B )g
Que podemos expresar como: at
mA mB

y sustituyendo los valores numricos at = 1 m/s2

Si analizamos el resultado obtenido, nos podemos dar cuenta de que, como es lgico, el
valor de la aceleracin sale menor que cuando no haba rozamiento y de que la expresin
literal obtenida se transforma en la anterior cuando introducimos en ella la condicin de
que no exista rozamiento ( = 0). El hecho de que la aceleracin tangencial nos salga po-

sitiva, quiere decir que el vector aceleracin ( a ) tiene el sentido que hemos tomado como
positivo.
Dinmica 88

Otro problema que podemos plantearnos es averiguar el valor mnimo necesario que de-
bera de tener mA para que el sistema comenzase a moverse, ya que como hemos visto, si
hay rozamiento, las condiciones cambian y ya no se puede afirmar que m B se mover por
pequea que sea la masa de A.

28. Dado el dispositivo de la figura adjunta (mA = 4 kg, mB = 6 kg , mC = 10 kg), con-


siderad el rozamiento y las masas de la cuerda y polea despreciables y calculad:

a) Aceleracin del sistema


b) Tensin de las cuerdas
c) Cules seran los resultados anteriores si hubiera rozamiento entre los bloques y
la superficie con un coeficiente = 04?

sol: a) at = 5 m/s2 ; b) T1 =50 N ; T2 =20 N c) at = 3m/s2 , T1 = 70 N, T2 = 28 N

29. Determinad la aceleracin con que


desciende el cuerpo A y la tensin de la
cuerda que sujeta al cuerpo B de la fi-
gura (mA = 20 kg y mB = 1 kg) en los
siguientes casos:

a) No existe rozamiento
b) El coeficiente de rozamiento de A y
el plano es 02 y el de B con A es 05.

sol: a) a = 5 m/s2 ; T = 5 N ; b) a = 296 m/s2; T = 935 N


Dinmica 89

30. Calculad la tensin de la cuerda


que sujeta al cuerpo A y la acelera-
cin del cuerpo B sabiendo que el
cuerpo A tiene una masa de 5 kg, el B
de 15 kg y el C de 10 kg. El coeficien-
te de rozamiento entre los cuerpos A
y B vale 03 y entre B y el plano 01.

sol: TA = 147 N; at = 255 m/s2

31. En el sistema de la figura las dos poleas se suponen sin masa y sin rozamiento.
Desde una situacin inicial de reposo, se abandonan simultneamente las masas mA y
mB, observndose que ambas descienden. En el instante despus de abandonarlas se
pide:

a) Valor de la tensin del hilo del que pende A.


b) Aceleracin con que desciende B.
c) Fuerza total que ejerce C sobre D.

Datos: mA = 1 kg; mB = 2 kg; mC = 05 kg; mD = 1 kg; CD = 1/3; DS = 2/9

El sistema formado por A y C se mueve solidariamente hacia la izquierda y el formado


por B y D hacia la derecha. Si, arbitrariamente, elegimos como sentido positivo el del mo-
vimiento, podemos representar las fuerzas que actan en cada uno de dichos sistemas me-
diante los esquemas siguientes.
Dinmica 90

En el esquema de la izquierda se han incluido las siguientes fuerzas:



-Sobre el bloque A: T1 = fuerza que hace el hilo; PA = fuerza que hace la Tierra.

-Sobre el bloque C: T'1 =fuerza que hace el hilo; PC =fuerza que hace la Tierra;

R CD = fuerza normal que sobre C hace D; FrCD = fuerza de rozamiento que sobre C hace D.

En el esquema de la derecha se han incluido las siguientes fuerzas:



-Sobre el bloque B: T2 = fuerza que hace el hilo; PB = fuerza que hace la Tierra.

-Sobre el bloque D: T'2 = fuerza que hace el hilo; PD = fuerza que hace la Tierra;

R DS = fuerza normal que sobre D hace la superficie de la mesa; FrDS = fuerza de rozamien-

to que sobre D hace la mesa; FrDC = fuerza de rozamiento que sobre D hace el bloque C.

-Sobre la mesa: NSD = fuerza normal que sobre la superficie de la mesa hace D;

FrSD = fuerza de rozamiento que sobre la superficie de la mesa hace D. (Con objeto de no
complicar excesivamente la figura, estas dos fuerzas se han resaltado aparte mediante una
zona sombreada).

De acuerdo con el principio de accin y reaccin, sabemos que:


T1 = T1; RCD = NDC; RDS = NSD; FrCD = FrDC; FrDS = FrSD; T2 = T2

Cmo podramos calcular la tensin del hilo del que pende el bloque A?
Dado que las trayectorias son conocidas, podemos utilizar un tratamiento escalar y aplicar
la ecuacin fundamental de la dinmica al bloque A y al C para tratar de obtener T1:
Como sobre el bloque A no actan fuerzas normales a la trayectoria, la nica ecuacin
ser: Fres t = mA at1 PA - T1 = mA at1 (1)

Y para el bloque C: Fres t = mC at1 T1 - FrCD = mC at1 (2)


Fres n = mC anC RCD - PC = mC anC (3)
Dinmica 91

Para calcular la tensin del hilo del que pende m A podemos utilizar tanto la ecuacin (1)
como la (2), pero necesitamos saber primero cul es la aceleracin sobre la trayectoria.
Para ello bastar sumar las ecuaciones (1) y (2) y despejar a t1:

PA - T1 = mA at1
T1 - FrCD = mC at1
PA FrCD
Sumando: PA - FrCD = (mA + mC) at1 , y despejando: a t 1
mA mC

Para poder hallar at1 necesitamos determinar la fuerza de rozamiento que sobre C hace D.
Como los bloques se estn deslizando, el valor de esa fuerza coincidir con el valor
mximo, es decir: FrCD = CD NDC = CD RCD

Cmo podemos hallar RCD ?

Para calcular RCD hemos de aplicar la ecuacin (3) y tener en cuenta que al no cambiar la
direccin de su velocidad la aceleracin normal es 0. Por tanto:

RCD - PC = mC an = 0 RCD = PC con lo que FrCD = CD RCD = CD PC

PA FrCD P CD PC
As pues a t 1 = A
mA mC mA mC

Observemos que segn esta expresin, el valor de at1 podra ser negativo, para ello bastar-
a que PA fuese menor que la fuerza de rozamiento entre C y D (es decir, CD PC). Que
querra decir esto? Que la masa A subira en lugar de bajar, aunque no es este el caso, ya
que en el enunciado se nos informa que tanto A como B descienden.

Sustituyendo ahora esta expresin en la de T 1, nos queda:

PA - CD PC
T1 = PA - mA at1 T1 = PA - m A = 44 N
mA + mC

Si la masa de A valiera 0, la tensin del hilo del que pende tendra que ser nula Contem-
pla este hecho el resultado que acabamos de obtener? Puedes realizar algn otro razo-
namiento que permita analizar la validez de dicho resultado?

Podemos referirnos, por ejemplo, a que si la masa de C fuese nula tampoco debera haber
tensin en el hilo, como efectivamente se contempla en la expresin obtenida ya que en
ese caso nos quedara: T1 = PA - mA g = 0. Tambin a la igualdad en cuanto a las dimen-
siones, a lo que pasara si el rozamiento fuese nulo, si el peso de C aumentase, etc.

Cmo podemos calcular la aceleracin con que descender B?

En principio, podemos seguir la misma estrategia que utilizamos para calcular at1 , es de-
cir, aplicar la ecuacin fundamental de la dinmica a los bloques B y D y sumar para eli-
minar las tensiones.
Dinmica 92

Sobre el bloque B: PB - T2 = mB at2 (4)


Sobre el bloque D: T2 - FrDS - FrDC = mD at2 (5)
RDS - PD - NDC = mD anD (6)

Sumando las ecuaciones (4) y (5), nos queda : PB - FrDS - FrDC = (mB + mD) at2

PB FrDS - FrDC
y despejando: a t 2
mB mD

Para poder hallar at2 necesitamos determinar la fuerza de rozamiento que sobre D hace el
suelo y la que hace C. Como los bloques se estn deslizando, los valores de esas fuerzas
coincidirn con los valores mximos: FrDC = CD RCD y FrDS = DS NSD. El primero de
ellos ya vimos anteriormente que viene dado por: Fr DC = CD PC ; en cuanto al segundo,
para calcularlo, necesitamos saber NSD .

Podemos calcular NSD si tenemos en cuenta que de acuerdo con el principio de accin y
reaccin, NSD = RDS y la ecuacin (6) en la que la aceleracin normal es nula. Nos queda
as: RDS - PD - NDC = mD anD = 0, con lo que: RDS = PD +NDC. Como ya hemos visto: NDC =
PC, por tanto podemos escribir finalmente que: RDS = PD + PC y tambin que:
FrDS = DS (PD + PC).

Introduciendo estas expresiones de las fuerzas de rozamiento en la ecuacin que nos da a t2:

PB FrDS - FrDC P DS (PD PC ) DC PC


at2 at 2 B = 5 m/s2
mB mD mB mD

Podemos ahora analizar el resultado obtenido, comprobando en primer lugar que la ecua-
cin es dimensionalmente homognea y que contempla algunos casos evidentes, como
que si la masa de D y la de C fuesen nulas, B caera con la aceleracin de la gravedad o
cmo si eliminamos el bloque C, el resultado se transforma en otro ya conocido (ved pro-
blema 27).

Para terminar, calcularemos el valor de la fuerza total que el bloque C ejerce sobre el D.

Analizando la interaccin existente entre los dos bloques y centrndonos


en la fuerza que
el C ejerce sobre el D, concluimos que tanto la fuerza de rozamiento FrDC como la fuerza

normal N DC responden a la misma interaccin, de modo que sumadas nos darn la fuerza
total que sobre D ejerce C:

C
FrDC D FrDC

NDC FDC NDC


FDC FrDC N DC FDC (FrDC ) 2 N 2DC = 53 N
Dinmica 93

32. Un bloque de hierro ha sido lanzado hacia la derecha por una superficie horizon-
tal contra un muelle elstico tal y como se representa en la figura adjunta:

Al chocar, el bloque no se para inmediatamente sino que sigue movindose hacia la


derecha durante un tiempo y mientras esto ocurre ir empujando al muelle (razonad
cul es la proposicin correcta):
a) Cada vez con ms fuerza
b) Siempre con la misma fuerza
c) Cada vez con menos fuerza

Cuando el bloque interacciona con el muelle, segn el principio de accin y reaccin, la


fuerza que sobre el muelle hace el bloque FMB ha de ser del mismo mdulo pero sentido

contrario a la fuerza que sobre el bloque hace el muelle FBM , es decir: FMB = - FBM o bien
en mdulos FMB = FBM y esto ha de ser as mientras dure dicha interaccin.

Al tratarse de un muelle elstico, el mdulo de la fuerza que ejerce el muelle ha de ser


directamente proporcional al alargamiento o compresin de ste respecto a su longitud
inicial (ley de Hooke), es decir, conforme se vaya aumentando la compresin del muelle
ir aumentando tambin FBM y en consecuencia tambin lo har la fuerza que sobre el
muelle har el bloque FMB ya que, como hemos dicho, ambas tienen el mismo mdulo. Por
tanto, la proposicin correcta es la a).

El hecho de que muchas personas escojan como correcta la tercera propuesta suele deber-
se a que relacionan la fuerza con la velocidad en lugar de con la aceleracin y eso les lleva
a pensar que como el bloque va cada vez ms lento ha de hacer cada vez menos fuerza. No
tienen en cuenta que aunque el bloque vaya cada vez ms despacio el mdulo de la acele-
racin va aumentando (cada vez se para ms deprisa) y que alcanzar su valor mximo
justo en el momento en que el bloque se pare (mxima compresin del muelle) cuando la
fuerza tome tambin su valor mximo, a pesar de que en ese preciso instante la rapidez
sea nula. Finalmente, pensemos, que si en lugar del bloque fuese nuestra mano la que
comprime el resorte, no dudaramos en afirmar que para seguir comprimindolo tendra-
mos que hacer cada vez ms fuerza.

33. Si el cuerpo del ejercicio 25 se sujeta mediante un resorte de constante elstica 50


N/m. Determinad el alargamiento que sufrir el resorte.

34. El cuerpo de la figura tiene una masa de 5 kg.


Sabiendo que la constante elstica del resorte
vale 400 N/m, determinad la deformacin del
muelle en el equilibrio. (Se supone que no hay
rozamiento).
sol: x = 6/5 m
Dinmica 94

Qu fuerzas actuarn sobre el cuerpo en la posicin de equilibrio?



En la posicin de equilibrio, sobre el cuerpo actan las fuerzas P , R y Fe correspondien-
tes al peso del cuerpo, fuerza normal que ejerce el plano y fuerza elstica que ejerce el
muelle, y como se trata de una situacin de equilibrio, la aceleracin
ser nula y, en con-

secuencia, tambin deber serlo la fuerza resultante: Fres = P + R + Fe = 0

Cmo podramos hallar la deformacin del muelle en la posicin de equilibrio?

Sabemos que la fuerza elstica ejercida por el muelle ha de ser directamente proporcional
a la elongacin (ley de Hooke) segn la expresin: Fe = Kl, donde K es la constante
elstica del muelle e l la deformacin sufrida por el mismo (en valor absoluto). Para cal-
cular la deformacin, podemos hallar el valor de la fuerza elstica en primer lugar y susti-
tuir en la expresin anterior.

Para determinar el valor de la fuerza elstica ejercida por el muelle sobre el cuerpo, apli-
caremos la ecuacin anterior, expresando las fuerzas en un sistema de referencia carte-
siano (ya que se trata de un equilibrio), como el que se indica en el esquema adjunto:


P = P cos (90 + ) i + P cos (180 - ) j = -P sen i - P cos j

R = R cos 90 i + R cos 0 j = R j

Fe = Fe cos 0 i + Fe cos 90 j = Fe i

Con lo que: Fres = P + R + Fe Fres = (Fe -P sen ) i + (R- P cos ) j = 0

Descomponiendo la ecuacin vectorial anterior en dos escalares nos queda:

(1) Fe - P sen = 0
(2) R- P cos = 0
Dinmica 95

De la primera obtenemos que Fe = P sen y como el mdulo de la fuerza elstica tambin


puede obtenerse como Fe = Kl (poniendo l en valor absoluto), podemos igualar
P sen mg sen
y despejar l, con lo que nos queda: l = = 00625 m
K K
La expresin anterior puede analizarse comprobando, como hacemos habitualmente, que
es dimensionalmente homognea (L en ambos miembros) y verificando que contempla
algunos casos particulares evidentes, como, por ejemplo, que la comprensin del muelle
en la situacin de equilibrio ha de ser tanto mayor cuanto mayor sea la componente tan-
gencial de la fuerza peso y menor sea la constante elstica del muelle .

Conviene tener en cuenta que Psen y Fe no son una pareja de accin y reaccin (corres-
ponden a dos interacciones distintas) y que solo coinciden (en valor absoluto) justo en el
punto de equilibrio, ya que P sen , vale siempre lo mismo mientras que Fe va cambiando
segn lo comprimido o alargado que est el resorte (Fe = Kl).
Qu le ocurrira al cuerpo si, en las condiciones explicitadas en el enunciado, se aban-
donase sobre el plano, sujeto al extremo libre del muelle?
El objeto descendera porque inicialmente la fuerza elstica del muelle sera menor que la
componente tangencial del peso P sen pero como Fe ira aumentando llegara un mo-
mento (posicin de equilibrio) que se igualaran ambas fuerzas. En ese punto el objeto
estara descendiendo con su mxima velocidad (siendo nula la fuerza resultante sobre el
mismo), por lo que no se parara inmediatamente sino que continuara descendiendo. A
partir de que se sobrepasa la posicin de equilibrio, la fuerza elstica superara cada vez
ms a la componente tangencial del peso y el cuerpo ira frenando hasta que en un instante
dado se parara (rapidez cero y Fe mxima) para comenzar a ascender empujado por el
muelle hasta la situacin inicial. El ciclo se repetira indefinidamente (rozamiento nulo) y
se puede demostrar que correspondera a un movimiento armnico simple.

35. Demostrad que dos resortes unidos de constantes K1 y K2 equivalen a uno solo de
constante K1 K2 / (K1 + K2 ).

36. De un resorte de 50 cm de longitud, sujeto al techo de un autobs en reposo, se


suspende un cuerpo de 4 kg que le produce un alargamiento de 10 cm. El autobs
arranca con una aceleracin de 4 m/s2 y en lnea recta. Qu ngulo formar el re-
sorte con la vertical y cul ser su longitud en esa nueva situacin?
Conviene distinguir entre la situacin inicial con el autobs en reposo y la situacin poste-
rior, con el autobs movindose en lnea recta y con aceleracin constante.

La primera (autobs en reposo), corresponde a


una situacin de equilibrio en la que la fuerza
elstica que ejerce el muelle sobre el cuerpo
hacia arriba se ver compensada por la fuerza
peso que ejerce la Tierra sobre el cuerpo hacia
abajo, segn se indica en el esquema adjunto:
Dinmica 96


En la posicin de equilibrio queda claro que : Fe1 P 0 , y por tanto que: Fe1 = P, es
decir: K l1 = mg, de donde podemos obtener el valor de la constante elstica del muelle
haciendo K = mg/l1 = 400 N/m

En la segunda situacin (autobs acelerando), el cuerpo que pende del resorte, para un
observador externo no se encuentra en equilibrio sino que se est moviendo con la misma
aceleracin que el autobs. A qu fuerzas cabe atribuir dicha aceleracin?

Las
fuerzas que actan sobre el cuerpo sern el peso

P y la fuerza elstica del muelle Fe 2 (distinta de la
correspondiente a la situacin anterior), de manera
que la resultante de ambas tendr la misma direccin
y sentido que la aceleracin con que se mueve el
cuerpo:
n

Fres = P + Fe 2 = m a


Expresando las fuerzas en componentes intrnsecas, segn los vectores y n de la figura
y sustituyendo, nos queda:

Fres t = mat Fe2 sen = mat


Fres n = man Fe2 cos - P = man = 0

A continuacin mediante las dos ecuaciones anteriores podemos obtener el valor del
ngulo y el de la fuerza elstica Fe2

Despejando sen y cos de las ecuaciones anteriores y dividiendo:

m at
sen Fe2 a a 4
tg t arctg t arctg 21'8
cos P g g 10
Fe2

Sustituyendo este valor en cualquiera de las ecuaciones anteriores y operando, obtenemos


que Fe2 = 43'1 N.

Para saber el alargamiento experimentado por el muelle, aplicamos Fe 2 = Kl y despejamos


l:

l = Fe2 /K = 43'1/400 = 0108 m = 10'8 cm

A la luz de lo hecho en este problema, proponed una forma de calcular la aceleracin de


un avin durante la maniobra de despegue (o aterrizaje) utilizando un pndulo simple y
un transportador de ngulos.
Dinmica 97

37. Un cuerpo de 2 kg de masa se desplaza por un plano horizontal describiendo una


circunferencia de 5 m de radio, sometido a la accin de una fuerza tangencial de
mdulo 4/(1+t)2 ( en N para t en s). Determinad: a) su rapidez a los 3 s de comenzar a
actuar la fuerza, sabiendo que inicialmente se encontraba en reposo; b) valor de la
fuerza total que deber estar actuando sobre el cuerpo a los tres segundos.

Si el cuerpo describe una trayectoria circular, su velocidad va cambiando continuamente


de direccin, luego adems de la fuerza
tangencial deber de actuar tambin otra fuerza

normal, de manera que Fres Ft Fn Ft Fn n . Si establecemos como sentido po-
sitivo el mismo que el del vector fuerza tangencial, podemos expresar las componentes
intrnsecas del vector fuerza resultante como:

(1) Ft = mat 4/1+t2 = mat


(2) Fn = man = m v2/5

Cmo podemos obtener ahora la ecuacin que nos da la rapidez en cualquier instante?

Sabemos que la rapidez est relacionada con la aceleracin sobre la trayectoria mediante
la ecuacin at = dv/dt. Integrando a partir de esta ecuacin, podemos hallar v = v (t). Lue-
go, bastar sustituir t por el valor correspondiente para calcular v en cualquier instante.

v t v t 2 2 2t
v0
dv a t dt dv
t0 0 0 (1 t) 2
dt v
(1 t)
2v
(t 1)

Basta ahora sustituir t por 3 s para obtener v = 15 m/s.

Cmo podramos calcular la expresin de la fuerza total que acta sobre el cuerpo en
cualquier instante?


De acuerdo con la expresin Fres Ft Fn Ft Fn n tendremos que obtener en
primer lugar Ft y Fn .

Ft = mat = 2 2/(1+t)2 = 4/(1+t)2

Fn = man = mv2/R= 2v2/5

4 2 2
Con lo que Fres v n
(1 t ) 2 5


En el instante t = 3 s, nos queda: Fres 0'25 0'9 n N
Dinmica 98

38. Sobre una plataforma plana, capaz de girar en torno a un eje perpendicular, se
deposita un cuerpo que presenta con la misma un coeficiente de rozamiento de 08.
Encontrad la mxima distancia a que puede encontrarse el cuerpo del eje de giro, sin
ser lanzado hacia el exterior, si hacemos girar la plataforma a razn de una vuelta
por segundo.

Si resolviramos de forma experimental este ejercicio, veramos que existe una distancia
mxima al eje tal que para valores superiores a la misma, el cuerpo deslizara por la super-
ficie desplazndose hacia el exterior, mientras que para valores inferiores, el cuerpo per-
manece en el lugar en que se le site, describiendo, al igual que cualquier punto del disco,
un movimiento circular uniforme. Para que esto ltimo suceda, ser necesario que el
cuerpo se encuentre sometido a la accin de una fuerza resultante segn la direccin del
radio y dirigida hacia el centro de la plataforma, es decir, en el mismo sentido que el vec-

tor unitario n . A qu se debe esa fuerza?

Consideremos el objeto en un punto cualquiera de su trayectoria con un movimiento cir-



cular y uniforme alrededor del eje de giro. En dicho punto tendr una velocidad v tangen-
te a la trayectoria. Si no contina movindose con dicha velocidad en la misma direccin
y sentido, es porque ha de haber una fuerza resultante perpendicular a la trayectoria que
obligue a cambiar constantemente la direccin del vector velocidad.

v
n

Las fuerzas que actan sobre el objeto cuando est describiendo


un movimiento circular
uniforme solidariamente a la plataforma,
son el peso P , la reaccin del plano R y la fuer-
za de rozamiento con la superficie Froz (si no hubiese rozamiento sera imposible que el

objeto pudiera girar estando sobre la plataforma) Fres P R Froz . Por otra parte, al tra-
tarse de un movimiento circular uniforme, la fuerza resultante deber de tener la direccin

y sentido de n .
Dinmica 99

R
Froz
n

Expresando
en componentes intrnsecas la ecuacin fundamental
de la dinmica:


Fres m a 0 + m a n n , y sustituyendo nos queda: Fres P R Froz = m a n n .
Si analizamos la ltima ecuacin, nos daremos cuenta de que la nica posibilidad de que

la fuerza resultante la mismadireccin y sentido que n , es que P + R = 0, con lo
tenga
que tendremos: Fres = Froz = m a n n .

Todo sucede, pues, como si la nica fuerza actuante fuera la de rozamiento que, adems,
est dirigida hacia el centro de la circunferencia descrita, de forma que si trabajamos esca-
larmente con la componente normal:

Fres n = Froz = m an = m v2 /r y como v = w r nos queda que Froz = m w2 r

La expresin obtenida nos dice que cuanto mayor sea r (a igualdad de los restantes facto-
res) mayor ser la fuerza de rozamiento necesaria para que el objeto siga girando sobre la
plataforma alrededor del eje con movimiento circular y uniforme.

Cmo podramos calcular el mximo valor de r posible?


El mayor radio posible ser aquel que corresponda al valor mximo que puede tomar la
fuerza de rozamiento entre el cuerpo y la plataforma. Dicho valor mximo viene dado por:

Fr max = N = R = P = mg. Sustituyendo en Froz = m w2 r podemos escribir que:

g
mg = m w2 rmax y despejando: rmax rmax = 02 m
w2

Podemos analizar la expresin literal del resultado comprobando que es dimensionalmen-


te homognea (L = L) y que contempla algunos casos evidentes como, por ejemplo, que
cuanto mayor sea el coeficiente de rozamiento mayor ser el radio mximo y que al au-
mentar la rapidez angular de la plataforma disminuira el radio mximo. En este ltimo
caso, es posible darse cuenta adems de que al estar w elevada al cuadrado, influye ms
que los restantes factores, de forma que cuando w se duplica el radio mximo no se hace
la mitad sino cuatro veces ms pequeo.

39. El cuerpo del ejercicio anterior tiene una masa de 5 kg y se encuentra sujeto por
una cuerda de 05 m desde el centro de giro. Determinad a qu rapidez angular se
romper la cuerda si soporta como mximo una tensin de 60 N.
Dinmica 100

En este caso la fuerza resultante que se requiere para que el cuerpo gire con movimiento
circular uniforme ser la suma de la fuerza de rozamiento y de la fuerza que realice la
cuerda sobre el cuerpo (que como mximo puede valer 60 N). Dicha fuerza tendr la di-
reccin de la normal y estar dirigida hacia el centro de la circunferencia descrita.

Si el cuerpo gira con una rapidez angular w tal que la fuerza de rozamiento Froz es suficien-
te para suministrarle la fuerza normal requerida, la cuerda no har ninguna fuerza (la ten-
sin de la cuerda ser nula). Pero si la w es tan elevada que el valor mximo de la fuerza
de rozamiento Fr max es inferior al de dicha fuerza normal, la diferencia ser aportada por
la cuerda al ponerse en tensin (aunque como mximo solo pueda suministrar 60 N).

El planteamiento del ejercicio ser similar al anterior (ved all figura correspondiente),
trabajando con las componentes intrnsecas de las fuerzas presentes.

Fres P R Frmax T

Dado que P y R se anulan, podemos plantear directamente que: Fres Frmax T
y como se trata de un movimiento de trayectoria conocida nos convendr expresar todas
las fuerzas en componentes intrnsecas:

Fres t ma t 0 = at (como corresponde a un movimiento uniforme)


Fres n ma n Fr max + T = man = m w2 r

Fr max T
Despejando nos queda: w
m r

La mayor rapidez angular con que la masa m podr girar con movimiento circular unifor-
me alrededor del eje y a una distancia r del mismo, la podemos obtener a partir de la ex-
presin anterior, simplemente sustituyendo la tensin por su valor mximo y teniendo en
cuenta que Fr max = N = R = P = mg :

mg Tmax
w max = 63 rad/s
m r

40. Un pndulo cnico es un dispositivo forma-


do por una masa m que cuelga de un hilo de
longitud L, describiendo un crculo horizontal
con cierto radio r. Calculad las expresiones que
proporcionan la rapidez del cuerpo y la tensin
del hilo en funcin de la masa m, la longitud L
y el ngulo que forma el hilo con la vertical.
Aplicadlo al caso en que la masa sea de 200 g,
el radio de 50 cm y el ngulo de 300.
Dinmica 101

Para comenzar conviene que nos preguntemos por las fuerzas que estn actuando sobre la
masa que pende del hilo. Dichas fuerzas son la que ejerce el hilo
T y
el peso P (para sim-
plificar, suponemos que no hay rozamiento), de manera que Fres = T + P .

El problema presenta la particularidad de que ninguna de las fuerzas que actan sobre el
cuerpo, se encuentra en el plano que contiene a la trayectoria. Por otra parte, dichas fuer-
zas no se anulan entre s ya que en ese caso el movimiento no podra ser circular, como de
hecho es, sino rectilneo y uniforme.

Para resolver el problema podemos aplicar la ecuacin fundamental de la dinmica para


saber con qu aceleracin se mueve el cuerpo. Luego, a partir, de la aceleracin podemos
obtener la rapidez.

En primer lugar, veamos una forma sencilla de obtener la fuerza resultante si considera-
mos que el vector aceleracin ha de estar contenido en el plano que contiene a la circunfe-
rencia
descrita por el mvil, la fuerza resultante tambin deber estar en ese mismo plano

( Fres = m a) . Para comprender mejor la situacin, podemos descomponer (ver figura
siguiente) la tensin del hilo en dos fuerzas; una, en el plano de la trayectoria (dirigida
hacia el centro O), y otra, perpendicular a ella (dirigida hacia arriba). Con ello tendremos
tres fuerzas, que al sumarlas tienen que dar una fuerza resultante situada en el plano de la
trayectoria y para que esto ocurra, la nica posibilidad es que la componente vertical de la
tensin y el peso se anulen, es decir, que T cos = P.

De acuerdo con lo anterior, la fuerza resultante sobre la bolita coincidir con la compo-
nente horizontal de la tensin (dirigida siempre hacia el centro de la circunferencia descri-
ta) y valdr T sen . Si expresamos pues la ecuacin fundamental de la dinmica en com-
ponentes intrnsecas, tendremos:

(1) Fres t = m at = 0
(2) Fres n = m an T sen = m an

Si en la ecuacin (2) sustituimos la componente normal de la aceleracin por la expresin


v2/r y despejamos v, nos queda:
Dinmica 102

r T sen
v
m

As pues para hallar v necesitamos saber T Y cmo podemos hallar T?

Recordemos que al no existir aceleracin en la vertical, se debe de cumplir que la fuerza


vertical con que el hilo tira de la bolita hacia arriba se debe de anular con la fuerza peso y
por tanto que: T cos = P = mg. Podemos, pues, utilizar esta expresin para obtener T,
como: T = mg/cos = 23 N. Introduciendo esta expresin en la de v obtenida anterior-
mente, nos queda que:

r T sen r mg sen
v = v rg tg = 17 m/s
m m cos

Podemos analizar el resultado obtenido comprobando en primer lugar que es dimensio-


nalmente homogneo, ya que a la izquierda del signo igual tenemos LT -1 y a la derecha
L2 T 2 L T 1 . Por otra parte, es fcil darse cuenta de cmo dicho resultado contem-
pla algn caso particular evidente, como por ejemplo, el hecho de que si el radio r es nu-
lo, la rapidez sera 0 (no girara), o que cuanto mayor sea r mayor ser v.

Otra forma, ms rigurosa, de resolver este tipo de problemas en los que actan fuerzas

que no estn contenidas en el plano definido por y n, es mediante la utilizacin de los

vectores unitarios tangencial , normal n y binormal b , tal y como se indican en el es-
quema adjunto:

Si expresamos las fuerzas actuantes en estas componentes:



T T cos 90 T cos (90 ) n T cos b T cos b T sen n

P = P cos 90 + P cos 90 n + P cos 180 b = -P b

Sumando obtenemos: Fres = T sen n + (T cos - P) b
Dinmica 103

La fuerza resultante expresada en componentes intrnsecas debe ser:

Fres t = m at
Fres n = m an
Fres b = 0

ya que como sabemos, tanto el vector aceleracin como la fuerza resultante tienen que

estar siempre contenidos en el plano formado por los vectores unitarios y n (recorde-
mos que la aceleracin puede tener solo dos componentes intrnsecas de modo que

a a t a n ) y que esto obliga a que la componente binormal se anule.

Igualando ahora las componentes nos queda:

0 = m at
T sen = m an
T cos - P = 0

De la primera de ellas, queda claro que a t = 0, de la segunda podemos despejar a n como:


an = Tsen/m y de la tercera T = P /cos y a continuacin podemos proseguir como
hicimos anteriormente para hallar v y T.

41. Un muelle de 1 m de longitud y constante elstica K = 103 N/m tiene un extremo


fijo y en el otro una masa de 1 kg, encontrndose ambos sobre un plano horizontal y
sin rozamiento. Si se hace girar la masa con rapidez angular constante de 10 rad/s
cul ser la deformacin que sufrir el muelle?

sol: l =1/9 m

42. Desde la parte inferior de un rizo se lanza


una bola de masa m. Determinad la mnima ra-
pidez con que deber llegar a la parte superior
del rizo para conseguir pasarlo sin caer. Apli-
cadlo al caso en que el radio sea de 10 m.

Para comenzar, conviene hacer un anlisis cualitativo previo de la situacin: la bolita lleva
en cada instante una cierta velocidad y se encuentra con que la curvatura de la trayectoria
la obliga a cambiar constantemente la direccin de dicha velocidad. Dicho de otro modo,
la bolita est chocando con la superficie y como consecuencia de esa interaccin se
ejercern fuerzas iguales y contrarias entre s (principio de accin y reaccin). La fuerza
que sobre la bolita haga la superficie sumada con el peso, nos proporcionar en todo mo-
mento la fuerza resultante que acta sobre la misma. (Ntese que en el momento en que la
bolita cae, no existe interaccin entre ella y la superficie).
Dinmica 104

Otra consideracin de inters es que a lo largo de la primera cuarta parte del recorrido, la
bolita no puede caer (se lo impide la superficie), sino solo retroceder (en caso de que no se
lanzase con la suficiente rapidez inicial). En principio, podemos pensar que el punto ms
crtico ser el ms alto, donde "intuimos" que la interaccin entre la bolita y la superfi-
cie ser menor.

Como la bolita se encuentra describiendo un movimiento de trayectoria previamente co-


nocida, convendr expresar las fuerzas mediante sus componentes intrnsecas. Por otra
parte, podemos descomponer el trayecto circular en dos tramos AB y BC analizando qu
es lo que le ocurre a la bolita en cada uno. Para ello utilizaremos los siguientes esquemas:


En el trayecto horizontal actan el peso P y la fuerza normal R que hace el plano. Puesto
que la bolita, en ese tramo, lleva movimiento rectilneo y uniforme, la resultante de ambas
fuerzas es nula. No ocurre as en el trayecto circular, puesto que en l, la velocidad cambia
tanto en mdulo (disminuye conforme va subiendo la bolita) como de direccin. De
acuerdo con el principio de accin y reaccin la fuerza normal que el plano ejerce sobre la
bolita R ha de ser en todo instante de igual mdulo y sentido contrario a la que ejerce la

bolita sobre dicho plano ( N ), que no hemos incluido en la figura.

Tomando como sentido positivo el del movimiento, nos queda que para el tramo circular
la fuerza resultante tiene las siguientes componentes intrnsecas:

Fres t = P cos (90 + ) = -P sen


Fres n = P cos (180 - ) + R cos0 = R - P cos

Aplicando ahora la ecuacin fundamental de la dinmica, nos queda:

(1) m at = -P sen
(2) m an = R - P cos

De la ecuacin (1), comprobamos que existe aceleracin tangencial y que es negativa, por
lo que la rapidez ir decreciendo a medida que la bolita asciende.

La interpretacin de la ecuacin (2), ser ms clara si analizamos el movimiento de la


bola por separado en los tramos AB y BC:
Dinmica 105


En el tramo AB, no puede caer porque la componente normal Pn del peso provoca que la
bolita presione a la superficie. Dicha componente normal (P cos ) va disminuyendo des-
de un valor mximo P en el punto A ( = 0) hasta un valor 0 en el punto B ( = 90 ). La
superficie haceuna fuerza R sobre la bolita dirigida hacia el centro
de la trayectoria circu-
lar. La fuerza R no solo compensa en todo momento a la fuerza Pn sino que es mayor que
ella, de manera que siempre habr una fuerza normal resultante dirigida hacia el centro de
la circunferencia (necesaria para el giro). En efecto, la ecuacin (2) puede ponerse como
R = m an + P cos , en la que los dos trminos de la derecha son positivos ( estar
comprendido entre 0 y 90), por lo que en AB siempre existir R , y por tanto, en el tramo
AB siempre habr interaccin entre la bolita y la superficie.

En el tramo BC la componente normal del peso Pn va hacia el centro de la trayectoria (cos

es negativo), al igual que R . Para que la bolita gire con un radio r (igual al de la circun-
ferencia) sabemos que ha de haber una fuerza normal resultante lo bastante grande como
para obligarla a ello. En este tramo, Pn va aumentando desde 0 (en B) hasta P (en C),
mientras que R va disminuyendo (la bolita va cada vez ms lenta y presiona con menos
fuerza a la superficie). Sabemos que cuanto mayor sea la rapidez de la bolita mayor tendr
que ser la fuerza normal necesaria para obligarla a girar (Fres n = mv2/r), por tanto, si en un
punto dado de este tramo, la velocidad de la bolita es tan pequea que solo P n ya resulta
mayor que la fuerza normal necesaria, r disminuir, es decir, la bolita describir una curva
con un radio menor, por lo que se despegar y caer. Por el contrario, si la rapidez es tal
que con solo Pn sera insuficiente para mantenerla en un giro de ese radio, la bolita tien-
de a irse por la tangente (para aumentar
r), pero la
superficie (curva) no se lo permite
ejerciendo sobre ella una fuerza R que sumada con Pn nos dan la fuerza normal necesaria.
A estas mismas conclusiones se pueden llegar manejando la ecuacin (2) en la forma:

R = m v2/r + P cos

En dicha ecuacin, P cos es negativo, por lo que mientras que la rapidez de la bolita se
mantenga lo bastante grande como para que m v2/r sea igual o mayor que el valor abso-
luto de P cos , existir R ( y N) y no caer (estar interaccionando con la superficie).
Pero si en algn momento m v2/r nos da menor que P cos (en valor absoluto), tendr-
amos que la R debera ser negativa, lo cual es imposible ya que ello querra decir (segn
el sistema de referencia adoptado) que la superficie tirara de la bolita hacia s, lo cual es
absurdo.

El punto ms crtico, como caba esperar, es el C, ya que es all donde P n presenta el ma-
yor valor posible (P). En ese punto la ecuacin queda como R = m v2/r - P y la rapidez
mnima con que se podr pasar ser aquella que R = 0, es decir:

v 2min
0=m mg v min gr = 10 m/s.
r

Cul ser la expresin de la rapidez de la bolita en cualquier punto de la trayectoria?

Podemos utilizar la ecuacin (2) sustituyendo en ella la aceleracin normal por v2/r con lo
que nos queda que:
Dinmica 106

( R P cos ) r
R- P cos = m v2 /r de donde podemos despejar v y obtener: v =
m

En el punto ms alto de la trayectoria = 180 con lo que la expresin de v para ese pun-
to en particular ser:

( R P) r ( R mg ) r
v=
m m

Analizad el resultado obtenido

Podemos ver que si la rapidez aumenta, la fuerza R (y por tanto N) tiene que aumentar (ya
que un aumento de v no provoca variacin alguna en P ni en el radio), pero si v disminu-
ye, llegar un momento en que R (y por tanto N) valdr 0, se ser el mnimo valor de v
necesario, por debajo del mismo, la bolita no podr pasar el punto C.

Qu ocurrira si en el punto C hiciramos un agujero del tamao de la bola?

Si la rapidez con que la bola llega a ese punto fuese la mnima necesaria no ocurrira nada
( N = 0), pero si fuese mayor (N 0), se colara. Esto debe interpretarse como que en ese
caso la componente normal del peso sera insuficiente para proporcionar la fuerza normal
necesaria para girar con lo que el radio de su trayectoria aumentara. Este aumento se ve
impedido por la superficie, que suministra la fuerza normal que falta, pero, si no hay su-
perficie, nada se opone al aumento de r y la bola se escapara por el agujero.

43. Un vehculo de 2000 Kg toma una


curva de 20 m de radio y 30 de peralte.
Suponiendo que el rozamiento es des-
preciable, determinad la nica rapidez
con que podra tomar la curva (con di-
cho radio). Razonad lo que sucedera si
el vehculo tomase la curva con otra ra-
pidez.

Si queremos conocer la rapidez constante a la que el vehculo podr tomar la curva, de-
bemos considerar que dicho vehculo describe un movimiento circular uniforme, y, en
consecuencia, la fuerza resultante que acta sobre l debe tener solo componente normal
(direccin perpendicular a la trayectoria ydirigida hacia su centro O ). Las nicas fuerzas
que actan sobre el vehculo son el peso P y la fuerza R que le ejerce la superficie y dan:

Fres P R

Conviene sealar que, el mvil no desliza a lo largo del plano que forma la curva peralta-
da sino que se mueve describiendo una circunferencia y manteniendo siempre la misma
Dinmica 107

altura sobre la base del plano inclinado que se forma al realizar un corte transversal de la
curva, tal y como se muestra en el esquema siguiente:

El vehculo, al igual que en la bolita del problema anterior, se encuentra permanentemente


chocando con la superficie debido, en este caso, a la curva peraltada, lo que le impide
seguir una trayectoria rectilnea. Como el vector aceleracin solo tiene componente nor-
mal y dirigida hacia O, hemos
de concluir que la fuerza peso tendr que anularse con la
componente vertical de R , es decir: P = R cos y que el valor de la fuerza resultante que
produce el movimiento circular uniforme del vehculo, vendr dado por Fres n = Rsen
con lo que aplicando la ecuacin fundamental de la dinmica Fres n = man tenemos Rsen
= man = mv2 /r, de donde podemos despejar v:

r R sen
v= Para obtener v, necesitamos conocer R
m

En este caso R se obtiene a partir de la expresin ya vista antes P = R cos , de la que


R = P/ cos = mg/cos, y sustituyendo este valor en la expresin anterior nos queda:

v g r tg = 108 m/s

El anlisis del resultado obtenido, nos revela en primer lugar que es dimensionalmente
homogneo. Adems da cuenta de las siguientes situaciones:

Si el radio de la curva aumenta, tambin aumentar la rapidez a la que se puede tomar. Sin
embargo, cuanto menor sea el peralte menor ser la rapidez permitida, de manera que si
no hay peralte (ni tampoco rozamiento), no ser posible tomar la curva. Finalmente, si
ponemos la expresin obtenida en la forma v2 /r = g tg , nos damos cuenta que para un
ngulo de peralte dado solo hay una rapidez posible a la que puede tomarse cada curva ya
que si se aumenta (disminuye) v, como el cociente es constante (g tg ), el radio tambin
ha de aumentar (disminuir) y entonces el vehculo se puede salir de la carretera.

Cunto vale la fuerza N que el vehculo ejerce sobre la carretera?


Dinmica 108

Dicha fuerza valdr en todo momento lo mismo que la fuerza R que la carretera hace so-
bre el vehculo, que ya hemos calculado como mg/cos. Conviene darse cuenta que el
valor obtenido es mayor que el del peso mg (ya que cos ser inferior a 1). Este resultado
es coherente con la idea ya expuesta, de que la curva peraltada al impedir que el mvil
siga una trayectoria rectilnea, hace que este realice sobre el suelo una fuerza mayor de la
que hace cuando no hay peralte.

Otra forma, ms rigurosa, de resolver este mismo problema, es mediante la utilizacin de



los vectores unitarios tangencial , normal n y binormal b , tal y como se indican en el
esquema adjunto:


En el esquema anterior, los vectores unitarios n y se encuentran en un mismo plano (pa-
ralelo a la base del plano inclinado) y el vector b es perpendicular a ambos vectores. Po-
demos expresar los vectores fuerzas en funcin de dichos vectores unitarios:

P = P cos 90 + P cos 90 n + P cos 180 b = - P b

R = R cos 90 + R cos (90- ) n + R cos b = R sen n + R cos b

Sumando obtenemos el vector fuerza resultante, que deber estar dirigido hacia O (el au-
tomvil lleva movimiento circular y uniforme)

Fres = P + R = R sen n + (R cos - P) b

Como la fuerza resultante no puede tener componente en la direccin del vector binormal,
podemos descomponer la ecuacin vectorial anterior en las siguientes escalares:

(1) Fres t = 0 = mat at = 0


(2) Fres n = R sen = man R sen = m v2/r
(3) Fres b = R cos - P = 0 R cos = P

Una vez que hemos llegado a estas ecuaciones se procedera igual que hicimos anterior-
mente para calcular la rapidez a la que se podra tomar la curva.
Dinmica 109

44. Determinad el ngulo mnimo con que habra que peraltar una curva de 25 m de
radio para que un vehculo de 500 Kg pudiese tomarla, sin deslizar, con rapidez de 72
km/h, sabiendo que el coeficiente de friccin es 08.

En primer lugar, conviene constatar que nicamente con la fuerza debida al rozamiento
(sin peralte), el vehculo no podra tomar una curva de ese radio con la rapidez que se
nos indica:

En efecto,
sobre una curva sin peraltar actuaran sobre el vehculo las fuerzas P , R y Fr
siendo Fr la fuerza de rozamiento.

Para que el vehculo describa la curva con movimiento circular uniforme, la fuerza resul-
tante ha de tener la direccin del radio y sentido hacia el centro O de la circunferencia. La
fuerza resultante viene dada por:

Fres = P + R + Fr

Como la aceleracin no tiene ninguna componente segn la vertical, P y R se tienen que
anular (P = R = N) y la fuerza resultante coincidir con la de rozamiento.

De acuerdo con la ecuacin fundamental de la dinmica:

v2
Fres n = m an = m
r

con lo que sustituyendo obtenemos el valor que debera de tener la fuerza resultante para
que el vehculo pudiera dar la curva en las condiciones que se nos impone en el enuncia-
do del problema: Fres n = 8000 N.

Qu valor mximo puede tener la fuerza de rozamiento?

Si lo calculamos veremos que Fr max = N = P = mg = 4000 N, por tanto, insuficiente


para que el vehculo pueda dar una curva de ese radio con esa rapidez, por lo que ser
necesario peraltarla para que R contribuya tambin a obtener la fuerza resultante necesa-
ria (ver figuras siguientes).
Dinmica 110


En las figura adjuntas se puede ver que Fres = P + R + Fr pero ahora, P + R no se anulan
sino que sumadas con la fuerza de rozamiento, proporcionan la fuerza resultante necesaria
para que el vehculo de la curva con movimiento circular y uniforme.

Cmo podemos hallar el peralte que ha de tener la curva para que el vehculo pueda
tomarla sin derrapar en las condiciones que se dan en el enunciado?

De acuerdo con el esquema anterior podemos expresar


las fuerzas en funcin de los vecto-

res unitarios tangencial , normal n y binormal b y sumar para obtener la fuerza resultan-
te que acta sobre el vehculo (en cuya expresin ha de figurar el ngulo de peralte). Lue-
go aplicaremos la ecuacin fundamental de la dinmica y a partir de la misma trataremos
de obtener el valor de necesario para dar la curva con movimiento circular y uniforme,

P = -P b

R = R cos (90- ) n + R cos b = R sen n + R cos b

Fr = Fr cos n + Fr cos (90 + ) b = Fr cos n - Fr sen b

Fres = P + R + Fr = (R sen + Fr cos ) n + (R cos - P- Fr sen ) b

Teniendo en cuenta ahora que la fuerza resultante solo tiene componente normal (el veh-
culo lleva movimiento circular uniforme ), podemos descomponer la ecuacin vectorial
anterior en las siguientes ecuaciones escalares:

Fres t = 0 = mat at = 0
Fres n = R sen + Fr cos = man R sen + Fr cos = m v2/r
Fres b = R cos - P - Fr sen = 0 R cos - Fr sen = P

Sustituyendo la fuerza de rozamiento por su valor mximo: Fr = N = R, nos queda:

R sen + R cos = m v2/r


R cos - R sen = P
sen cos v 2
Dividiendo miembro a miembro nos queda:
cos sen rg
Dinmica 111

v 2 gr
A partir de la expresin anterior obtenemos que: tg = 035
gr v 2
As pues: = arctg 035 = 193

v2
Qu ocurrira si no existiese rozamiento? El resultado quedara como: tg , que
gr
es precisamente el que obtuvimos en el problema anterior (rozamiento nulo).

A qu velocidad mxima podramos tomar la curva considerada en este problema? Si


despejamos v en el resultado obtenido, tendremos el valor mximo a que se puede tomar
la curva (tengamos en cuenta que hemos sustituido la fuerza de rozamiento por su valor
mximo):

gr(tg )
vmax = = 20 m/s
(1 tg )

Analizad el resultado anterior. Adems de comprobar que la expresin obtenida para la


rapidez mxima es dimensionalmente homognea, podemos ver que:

a) Si el ngulo se hiciera 0 (curva sin peralte), la rapidez mxima a la que el vehculo


podra tomar la curva sera: vmax = gr = 141 m/s

b) Si conservando el peralte, no hubiese rozamiento, la expresin sera: v = gr tg = 93


m/s y representara la nica rapidez a que se podra tomar dicha curva, de manera que
todo aumento en la misma supondra un aumento de r (el vehculo se ira hacia afuera de
la curva) y viceversa.

c) Si + tg = 0 (lo que ocurrir cuando = arctg (-), es decir para un cierto peralte
negativo o curva con pendiente al revs), la rapidez mxima ser 0 y, por tanto, la curva
no se podra tomar ya que a cualquier rapidez se producira derrape.

Quiere esto decir que no se puede circular por ninguna curva cuya inclinacin est
hacia el lado contrario al que debera? No necesariamente, ya que la expresin obtenida
no prohbe que se puedan dar curvas con peralte negativo, siempre y cuando + tg 0.
Sin embargo, este tipo de curvas son muy peligrosas y se puede derrapar a velocidades
muy pequeas.

45. Si el vehculo del problema anterior entrara en la misma curva peraltada con una
rapidez de 15 m/s, qu valores presentaran la fuerza de rozamiento y la reaccin del
plano?

Si el vehculo entra en la curva peraltada de = 193, seguro que podr tomarla ya que,
como hemos visto en el problema anterior, la friccin es tal que podra entrar hasta con
una rapidez de 20 m/s.
Dinmica 112

Sabemos tambin que la rapidez con la que podra tomarse la curva sin necesidad de ro-
zamiento, sera: v = gr tg = 10 25 tg 19'3 = 93 m/s.

Qu sucedera con un vehculo que entrase en la curva con una rapidez menor, igual o
mayor de 93 m/s?

a) Si la rapidez fuese inferior al valor hallado de 93 m/s, el vehculo, adems de girar,


tendera a deslizar hacia abajo por el plano inclinado que forma la carretera y como
consecuencia se producira una fuerza de rozamiento en sentido contrario (hacia arriba del
plano)
que lo impedira tomando para ello el valor idneo para que al sumarla con el peso
P y la fuerza normal R que ejerce la superficie nos de la fuerza resultante necesaria para
que se produzca un movimiento circular uniforme (fuerza resultante en la direccin de la
normal y hacia el centro de la circunferencia descrita).

b) Si la rapidez fuese precisamente 93 m/s, la suma de P y R ya nos da la fuerza resul-
tante necesaria y, por tanto, la fuerza de rozamiento por deslizamiento que se dara sera
nula. Si el vehculo tomase la curva peraltada con esa velocidad, no importara que en la
carretera se hubiesen formado placas de hielo.

c) Si la rapidez fuese mayor de 93 m/s, el vehculo, adems de girar, tendera a deslizar


hacia arriba por el plano inclinado que forma la carretera, con lo que se producira una
fuerza de rozamiento hacia abajo que intentara
impedirlo, tomando
para ello el valor
idneo para que al sumarla con el peso P y la fuerza normal R que ejerce la superficie
nos de la fuerza resultante necesaria para que se produzca un movimiento circular unifor-
me, aunque, como ya sabemos, si la rapidez fuese mayor de 20 m/s esto no se podra con-
seguir ya que en ese caso el valor mximo que puede tomar la fuerza de rozamiento ya no
sera suficiente para conseguir que la resultante tuviera la direccin de la normal.

En este problema, la fuerza de rozamiento actuara hacia abajo (como en el anterior) pero
su valor no ser el mximo (puesto que entra a menos de 20 m/s). Al tener el mismo sen-
tido, las ecuaciones a utilizar sern las mismas, solo que ahora el dato es el ngulo y la
incgnita la fuerza de rozamiento Fr.

R sen + Frcos = mv2/r


R cos - Frsen = P

P Fr sen
Despejando R de la segunda ecuacin obtenemos: R = , y sustituyendo en la
cos
primera, podemos despejar Fr, con lo que nos queda:

mv 2
Fr cos Psen = 25945 N.
r

Sustituyendo ahora este valor en la expresin de R, nos queda: R = 62063 N


Dinmica 113

46. En algunos parques de atracciones existe un


rotor o cilindro hueco que se pone en rotacin
alrededor de un eje vertical que pasa por el centro
del mismo. Cuando una persona se sita dentro de
este aparato pegada a la pared, el cilindro va au-
mentado su rapidez de giro progresivamente hasta
que al llegar a un valor predeterminado el suelo
baja y, sin embargo, la persona queda pegada a la
pared sin caer.

Suponiendo un rotor que tenga un radio de 25 m y


que el coeficiente de friccin con la pared del mismo
sea 04, determinad cul debera de ser la rapidez
angular mnima para que una persona pegada a la
pared del mismo permaneciese sin caer.

sol : 10 rad/s


47. Un mvil de 2 Kg de masa se desplaza segn la trayectoria: r = (0, 2t, 3) (m si t en
s). Determinad su momento cintico respeto del origen de coordenadas y comentad el
resultado.

El momento
cintico respecto al origen de coordenadas viene dado por el producto vecto-

rial L rx p . Como conocemos r podremos calcular v y por tanto la cantidad de movi-

miento p .

v =d r /dt = (0, 2, 0) m/s. p = m v = (0, 4, 0) kgm/s

i j k

L 0 2t 3 = -12 i kgm2/s
0 4 0

Cmo puede interpretarse el resultado obtenido?

Podemos ver que el momento cintico es constante y dirigido segn OX. Se trata de un
resultado que cabra esperar si analizamos las implicaciones que tiene el hecho de que la
velocidad sea constante. En efecto, si la velocidad es constante quiere decir que no habr

aceleracin y que, por tanto, la fuerza resultante Fres m a y el momento de dicha fuerza

dL
respecto al origen de coordenadas M r x Fres sern nulos. Ahora bien, como M , si
dt
M = 0, quiere decir que L tiene que ser constante, tal y como hemos obtenido anterior-

mente. Por otra parte, al estar los vectores r y p contenidos en el plano YOZ, su producto
vectorial deber dar en la direccin OX.
Dinmica 114

48. Calculad la expresin del momento cintico respecto del origen de coordenadas
para un mvil que describe un movimiento circular y uniforme con centro en dicho
origen.

Sol: L m w r 2 k

49. Calculad el momento cintico de una masa puntual m que se mueve con rapidez
constante v a lo largo del eje OX+, respecto del punto A (0, y, 0 ).

sol: L = (0, 0, mvy)


50. Un cuerpo de 0'2 kg se desplaza con velocidad v = (2t, t, 1) m (si t en s). Sabiendo
que en el instante inicial se encontraba en el origen, determinad el momento de la
fuerza actuante respecto del origen.

Cmo puede calcularse la expresin del momento de una fuerza respecto a un punto?

la ecuacin M r x F . En este caso no conocemos ni el vector de
Una posibilidad es manejar

posicin r ni la fuerza F , pero podemos calcularlos a partir de los datos del enunciado:

dr r t t t2
v dr v dt r (2t , t , 1) dt (t 2 , , t )
dt 0 0 0 2

dv
Si ahora tenemos en cuenta que a (2, 1, 0) m/s2 y que F = m a = (04, 02, 0) N
dt

podemos ya calcular el momento M r x F sin ms que efectuar el producto vectorial:


i j k

M t 2
0'5 t 2 t = (-02t, 04t, 0)
0'4 0'2 0

dL
Otra posibilidad de calcular el momento habra sido partir de la expresin M
dt

A continuacin comprobaremos que mediante este otro mtodo obtenemos el mismo resul-
tado al que acabamos de llegar.


i j k
dL
L rx p t 2 0'5t 2 t (01
' t 2 , 0'2t 2 , 0) M (0'2t , 0'4t , 0)
dt
0'4t 0'2t 0'2
4. TRABAJO Y ENERGA

1. Una partcula que se mueve en el Y


plano XY describe bajo la accin de B (2 , 3)
2 A m
la fuerza F = (y , -xy) el contorno
triangular cerrado de la figura ad-
junta. Determinad el trabajo realiza-
do por dicha fuerza a lo largo de todo
el ciclo. (Se supone todo en unidades
del sistema internacional).
O X

El trabajo realizado a lo largo del ciclo lo podemos expresar como:

A B O
WF OO O F dr A F dr B F dr . Se trata, pues, de resolver cada integral y sumar.

Desde O hasta A, x = 0 (nos movemos sobre el eje Y). Lo que cambia es y, que lo hace
desde 0 en el punto O hasta 3 en el punto A. Por tanto dx = 0, y dy 0.
A 2
O dr O (y , xy) (dx,dy) 0 y dx 0 xydy 00 0
A 0 2 3
F

Desde A hasta B, lo que cambia es x (y = 3 en todo este trozo de trayectoria), que lo hace
desde x = 0 en el punto A, hasta x = 2 en el B.

F dr (y 2 , xy) (dx, dy) y 2 dx xydy 9dx 0 9x0 18 J
B B 2 3 2

2
A A 0 3 0

Desde B hasta O, cambian x e y, de forma que x vara entre 2 y 0 y la y lo hace entre 3 y 0,


todo ello a lo largo de una trayectoria recta.

F dr (y , xy) (dx, dy) B y 2 dx B xy dy .
O O O O
2
B B

Como podemos ver, ninguna de estas dos integrales puede resolverse si no obtenemos
previamente la funcin que relaciona a las variables x e y.

Encontrar dicha funcin es obtener la ecuacin de la trayectoria, que, en este caso, se trata
de una lnea recta que pasa por el origen de coordenadas, por lo que y = mx, siendo m la
pendiente de la recta. Particularizando para el punto B, podemos hallar el valor de dicha
pendiente: 3 = m2 con lo que m = 3/2 y la ecuacin de la trayectoria y = 3x/2. Diferen-
Trabajo y energa 116

ciando nos queda que dy = 3dx/2 y sustituyendo en las dos integrales anteriores ya pode-
mos resolverlas:

2 y3 2 y3
0 0

y dx B xy dy 3 y dy 3 y 2 dy = 6 6 0 J
O
2
O 0 2 2 02

B 3 3 3 3 3 3 3 2

Al calcular estas integrales, hemos tenido en cuenta que el signo de dx y de dy viene


implcito en los lmites de la integral y, por tanto, no hay que ponerles ningn signo, aun-
que sepamos que tanto x como y disminuyen conforme nos movemos de B hacia O.

As, pues, el trabajo realizado por la fuerza a lo largo de todo el ciclo ser:

WF OO = 0 + 18 + 0 = 18 J

2. Se deja caer un cuerpo de 2 kg desde una altura de 45 m al tiempo que sopla un


viento lateral que le ejerce una fuerza horizontal de valor F = 40 -12 t (unidades in-
ternacionales). Determinad el trabajo realizado por la fuerza resultante que acta
sobre dicho cuerpo desde que se suelta hasta que llega al suelo.


Sobre el cuerpo actan
dos fuerzas: el peso P , que se considera constante, vertical y hacia
abajo, y la fuerza F que le ejerce el viento, que es variable y perpendicular al peso. La
trayectoria que sigue el cuerpo desde que se suelta en A hasta que llega al suelo en B es,
en principio, desconocida, por lo que ser necesario manejar magnitudes vectoriales.

Y (m)

45 A
F

r P

B X (m)

El trabajo resultante puede calcularse mediante la expresin:

B
Fres x dx y Fres y dy z Fres z dz
xB yB zB
Wres A A Fres d r =
B
xA A A

Para ello hemos de obtener en primer lugar la expresin de la fuerza resultante. Utilizando
el sistema de coordenadas de la figura anterior, podemos escribir:

Fres F P (40 1' 2t,0) (0, 20) (40 1' 2t, 20), con lo que el trabajo resultante
entre los puntos A y B ser:
Trabajo y energa 117

B
Wres AB A F dr x (40 1' 2t) dx y 20 dy
xB yB

A A

De las dos integrales anteriores, la primera no podemos resolverla sin hacer un cambio de
variable (ni x ni t son constantes), mientras que la segunda no ofrece ninguna dificultad:

20 dy 20y 45 = 20 45 = 900 J.
0 0

45

Para resolver la primera integral necesitamos conocer la relacin funcional exis-


tente entre x y t. Esto podemos hacerlo hallando la ecuacin de la trayectoria en funcin
de t.

Fres (40 1' 2t, 20)
a (20 0' 6t, 10).
m 2

Teniendo en cuenta que t0 = 0 y v 0 = 0, podemos integrar para obtener el vector velocidad
en cualquier instante:

vdv t a dt 0dv 0(20 0' 6t, 10) dt v (20t 0' 3t2 , 10t) .

v t v t

0 0


Considerando que el vector de posicin del cuerpo a los 0 s es r0 (0, 45) m, podemos

integrar de nuevo para obtener r r (t ) :

dr v
dt 0 (20t 0' 3t2 , 10t) dt r r0 (10t 0' 1t3 , 5t 2 )
r t t

r0 0


r (0, 45) (10t 0' 1t3 , 5t 2 ) r (10t 0' 1t3 , 5t 2 45) , con lo que la ecuacin de
la trayectoria ser:

x 10t 2 0' 1t3 dx (20t 0' 3t2 ) dt


y diferenciando:
y 5t 2 45 dy 10t

Tenemos pues la funcin que nos relaciona la coordenada x con la variable t, con lo que
ya podemos hacer el cambio de variable y resolver la integral:

(40 1' 2t) dx t (40 1' 2t) (20t - 0' 3t2 ) dt .


xB tB

xA A

Sabemos que tA = 0, pero no conocemos tB o instante en que el cuerpo llega al suelo. Po-
demos calcularlo si tenemos en cuenta que en ese momento la componente "y" del vector
de posicin ser 0 y, por tanto: -5t2 + 45 = 0, de forma que tB = 3 s. Conociendo los lmi-
tes, podemos resolver ahora la integral anterior
3
0'36 4 36 3 800 2
0 (40 1'2t) (20t - 0'3t ) dt 0 (800t 36t o'36t ) dt 4 t 3 t 2 t =
3 2 3 2 3

32833 J
Trabajo y energa 118

Con lo que el trabajo resultante desde A hasta B ser: Wres = 900 + 32833 = 41833 J


3. Calculad el trabajo realizado por la fuerza F = (x, yz, 2) cuando acta sobre un
cuerpo de masa m que se desplaza a lo largo de la curva: x=2t; y=t 3 ; z=3t2 desde el
punto A(0, 0, 0) al punto B(2, 1, 3).

sol: W = 9125 J.

4. Resolved el ejercicio anterior en el caso en el que el desplazamiento sea a lo largo


de la recta que une los puntos A y B.

sol: W = 9 J.

5.Comprobad
que en el caso de los 2 ejercicios anteriores, si la fuerza viniese dada
por F =(1, 3, 2) N, el trabajo sera independiente de la trayectoria.

6. Un esquiador de 80 kg se desliza por


una pista plana y con una inclinacin de
40. Suponiendo que el coeficiente de ro-
zamiento valga 01 y que el esquiador re-
corra una longitud de 500 m, determinad
el trabajo realizado por cada una de las
fuerzas que actan sobre el mismo.

Podemos considerar
que a lo largo del desplazamiento, sobre el esquiador actan tres
fuerzas:
el peso P , la fuerza normal R que ejerce la superficie y la fuerza de rozamiento
Fr . Todas ellas tienen mdulo constante y forman un ngulo tambin constante con el
desplazamiento. Por tanto, para calcular el trabajo podremos utilizar la expresin:

W = F r = Fr cos

en la que, el trabajo se calcula como el producto escalar de los vectores fuerza y despla-
zamiento. En este caso, el vector desplazamiento ser tangente al plano y dirigido hacia
abajo, de manera que su mdulo coincidir con la distancia recorrida por el esquiador. Por
otra parte, como la trayectoria se supone rectilnea, la velocidad solo puede cambiar de
mdulo y la fuerza resultante (si no es nula) deber ser tangente a la trayectoria y en el
mismo sentido que el desplazamiento.
Trabajo y energa 119

Aplicando la expresin anterior a cada una de las fuerzas actuantes tendremos:



WP BA = P r = P r cos 50 = 800500cos 50 = 257.115 J

WR BA = R r = R r cos 90 = 0 J

WFr BA = Fr r = Fr r cos 180 = - Fr r, donde Fr = N

Para determinar el valor de N, hemos de tener en cuenta que N = R (principio de accin y


reaccin) y que, como la componente normal de la aceleracin es nula (la trayectoria es rec-
tilnea), R ha de valer lo mismo que Pn = mg cos 40, con lo que Fr = mg cos 40.

WFr BA = - Fr r = - mg cos 40 r = -30.6418 J

Cmo podemos hallar el trabajo realizado por la fuerza resultante?

Una posibilidad es determinar primero la fuerza resultante y despus calcular el trabajo


que realiza a lo largo del desplazamiento fijado:

Fres P R Fr (P cos50 , P cos140 ) (R cos90 , R cos0 ) (Fr cos180 , Fr cos90 )

Fres (mg cos50 Fr , R mg cos40 ) = (mg cos 50 - mg cos 40, R- mg cos 40)

Wres BA = Fres r = (mg cos 50 - Fr, R- mg cos 40)(r, 0)

Wres BA = (mg cos 50 - mg cos 40) r

y sustituyendo nos queda: Wres BA = 226.4732 J.


Trabajo y energa 120

Otra posibilidad es sumar cada uno de los trabajos correspondientes a las distintas fuerzas
actuantes (ya que, en casos como el que nos ocupa, el trabajo total coincide con el trabajo
realizado por la fuerza resultante):

Wres BA = WP BA + WR BA + WFr BA = 257.115 -30.6418 = 226.4732 J.

El problema tambin puede resolverse si en lugar


de calcular el trabajo como el producto
escalar de la fuerza y el desplazamiento W = F r = Fr cos, utilizamos la expresin
equivalente:
W = Ft e

en la que Ft es la componente escalar tangencial del vector fuerza,


que ser positiva o ne-

gativa segn lo sea el coseno del ngulo que forme el vector F con el vector unitario
(siempre tangente a la trayectoria y en el sentido tomado como positivo), y el e es el
desplazamiento espacial, que ser positivo o negativo dependiendo de que el espacio e
aumente o disminuya.

Aplicando la expresin anterior a cada una de las fuerzas tendremos:

WP BA = Pt e = P cos 50 e = mg e cos50 = 800500cos 50 = 257.115 J

WR BA = Rt e = R cos 90 = 0 J

WFr BA = Frt e = Fr cos 180 e = - Fr e = - Ne = - mg cos 40 e = -30.6418 J

Para calcular el trabajo realizado por la fuerza resultante se procedera de forma anloga a
como hicimos anteriormente.

7. Una persona arrastra por el suelo un trineo mediante una cuerda, que forma un
ngulo de 30 con la horizontal, recorriendo una distancia de 5 m. Si la tensin de la
cuerda es de 200 N, qu trabajo realizar?
Trabajo y energa 121

sol: 870 J

8. Admitiendo que la Luna tiene una masa 81 veces


menor que la de la Tierra y que gira alrededor de
nuestro planeta con un movimiento circular unifor-
me de 384.000 km de radio dando una vuelta comple-
ta cada 273 das. Calculad el trabajo realizado por
la fuerza con que la Tierra la atrae, cuando la Luna
haya recorrido la mitad de su rbita (El esquema no
est a escala).

sol: 0 J.


9. Sobre un cuerpo acta una fuerza F = (- 400x, 0, 0) N (si x en m). Calculad el tra-
bajo realizado por dicha fuerza cuando el cuerpo se desplaza desde el punto A (5, 0,
0) cm al punto B (0, 0, 0) cm. Ser necesario conocer la trayectoria?.

Dado que tenemos el vector fuerza expresado segn sus componentes cartesianas y que no
se trata de una fuerza constante, utilizaremos para calcular el trabajo la expresin:

WF BA x Fx dx y Fy dy z Fz dz
xB yB zB

A A A

donde Fx, Fy y Fz son las componentes cartesianas escalares (positivas o negativas) del
vector fuerza F . Conviene recordar que en estas integrales a dx, dy y dz no se les pone
nunca signo, ya que ste se halla implcito en los lmites de la integral.

0
Sustituyendo los datos nos queda: WF A - 400x dx = 05 J
B
0' 05

Como vemos, las integrales anteriores se pueden resolver sin necesidad de conocer la
forma que tiene la trayectoria, por lo tanto, sea cual sea la forma de la trayectoria seguida
por el cuerpo cuando se desplaza desde A hasta B, el trabajo siempre valdr 05 J.
Trabajo y energa 122

10. Dad el coste del consumo energtico mensual de una plancha de 1200 W que fun-
ciona una media de 0'7 horas al da. (Suponed que 1 kWh cuesta 0'125 y un mes de
30 das).

sol: 3'15

11. El motor de un ascensor dispone de una potencia mxima de 10 kW. Sabiendo que
la caja del mismo tiene una masa de 300 kg, determinad la mxima rapidez con que
podra ascender.

sol: v = 333 m/s

12. una persona de 80 kg sube por una


cuesta de 30 con rapidez constante de 36
km/h. Calculad la potencia que desarrolla.
(Se supone nulo el rozamiento con el aire).

Para que la persona pueda subir la cuesta con velocidad constante,


la fuerza resultante que
acta sobre ella ha de ser nula. Llamaremos fuerza motriz Fm a la fuerza paralela al plano
que la hace ascender.

Como las fuerzas perpendiculares al desplazamiento no realizan ningn trabajo (cos 90 =


0), solo nos interesan las componentes tangenciales de todas las fuerzas que actan. En
este caso, se desplaza con movimiento uniforme, luego a t = 0.

As pues, la componente tangencial de la fuerza resultante es nula: Fres t = 0. Como Fres t =


Fm - Pt , nos queda que para que la persona ascienda con movimiento uniforme debe exis-
tir una fuerza motriz de mdulo Fm = Pt y en sentido ascendente.

Cmo podemos calcular la potencia que desarrolla?

La potencia que desarrolle la persona vendr dada por el trabajo que realice la fuerza mo-
triz, por unidad de tiempo. Como en este caso las fuerzas son constantes y el movimiento
rectilneo y uniforme, podemos calcular el valor de dicha potencia mediante:
Trabajo y energa 123


W Fm r
P= Fm v Fm v cos0 Pt v = mg sen30 v = 400 W
t t

Analizad la expresin literal obtenida

La expresin obtenida para la potencia nos relaciona la misma con el valor de la fuerza
motriz y de la velocidad, de manera que, cuando se mantenga constante una potencia de-
terminada (por ejemplo la mxima posible), si se aumenta la fuerza motriz la velocidad
disminuir y viceversa. La persona puede subir una cuesta de mayor pendiente con la
misma potencia, pero para ello ha de disminuir la velocidad con que asciende.

En general, muchos vehculos tienen distintas marchas que para una misma potencia, son
capaces de desarrollar ms o menos velocidad. Las marchas cortas pueden hacer que la
fuerza motriz sea muy grande, pero en cambio la velocidad es ms pequea, mientras que
con las largas sucede lo contrario. Por este motivo, cuando se precisa realizar un adelan-
tamiento rpido se recomienda cambiar a una marcha ms corta (para conseguir mayor
fuerza motriz y, por tanto, mayor aceleracin).

Para terminar. Quin ejerce la fuerza motriz que acta sobre la persona?(recordad lo
que dice el principio de accin-reaccin).

13. La persona del ejercicio anterior asciende con una aceleracin de 0'1 m/s2, par-
tiendo con rapidez nula desde la base. Qu potencia instantnea desarrolla a los 3 s?
Cul ser la potencia media desarrollada en estos 3 s?

Para que suba la cuesta con movimiento rectilneo uniformemente acelerado, ser necesa-
rio que sobre la persona acte una fuerza resultante tangente al plano, dirigida hacia arriba
y de mdulo constante. Si, al igual que en el ejercicio anterior, tomamos como sentido
positivo el del movimiento, podemos escribir:

Fres t = m at Fm - Pt = m at Fm = m at + Pt = m at + mgsen30

Cmo puede calcularse la potencia que se est desarrollando en un instante dado?

La potencia instantnea es una magnitud que indica lo aprisa que se est realizando tra-
bajo en un momento dado y se calcula derivando el trabajo respecto al tiempo:

dW Fm dr
Pi = Fm v
dt dt

Teniendo en cuenta que en este caso, la fuerza motriz y la velocidad tienen la misma di-
reccin y sentido, y que se trata de un movimiento uniformemente acelerado, nos queda:

Pi = Fmvcos0 = Fmv = (mat + mg sen30 )(v0 + at t)

Sustituyendo, obtenemos Pi = 122'4 W.


Trabajo y energa 124

La expresin obtenida, nos permite comprobar que la potencia desarrollada, en las condi-
ciones que se fijan en el enunciado del problema, depende del tiempo, de forma que su
valor ir aumentando regularmente con el tiempo.

Si la potencia va cambiando con el tiempo cmo podemos determinar su valor medio


durante los tres primeros segundos?

Como en este caso la potencia vara linealmente con el tiempo, su valor medio entre dos
instantes cualesquiera, se podr determinar mediante la semisuma de los valores de la
potencia inicial y final. No obstante, es mejor aplicar el procedimiento general, que con-
siste en calcular el trabajo realizado y dividirlo por el intervalo de tiempo correspondiente:

dW = Pi dt W = 0Pi dt , con lo que:


t

at 2
(m at mg sen 30 ) (v0 t )
W 0 Pi dt =
t

Pm 2 y sustituyendo: Pm = 61'2 W
t t 3

14. Un ciclista de 65 kg de masa utilizando una bicicleta


de 10 kg, desciende por una carretera cuya pendiente es
del 6% con una rapidez constante de 72 km/h. Determi-
nad la fuerza de rozamiento que le est haciendo el aire,
sabiendo que para mantener constante esa rapidez el ci-
clista ha de estar suministrando continuamente una po-
tencia de 500 W. (La pendiente de una carretera nos da
los metros de desnivel por cada 100 que se recorren).

Hablamos de potencia suministrada porque se mide nicamente la energa que por uni-
dad de tiempo recibe el conjunto
ciclista-bicicleta. Las fuerzas que actan sobre el sistema
ciclista-bicicleta son F, P, R, y Fr (fuerza motriz, peso, fuerza normal y fuerza de roza-
miento con el viento, respectivamente), segn se aprecia en la figura adjunta:


Trabajo y energa 125

Como es un movimiento de trayectoria conocida podemos trabajar escalarmente, conside-


rando las componentes escalares tangencial y normal de las distintas fuerzas:

Fres t = mat F + Pt - Fr = mat. Por ser un movimiento uniforme at = 0 Fr = F + Pt (1)


Fres n = man Pn - R = man . Por ser un movimiento rectilneo an = 0 R = Pn (2)

La fuerza Fr debida a la friccin con el aire, podemos calcularla a partir de la ecuacin (1)
si determinamos primero F y Pt. La fuerza F se puede obtener mediante la relacin que
liga la potencia con la velocidad, que en este caso,
como en los dos ejercicios anteriores,

viene dada por P = Fv (porque el ngulo entre F y v es 0), con lo que F= P/v. La com-
ponente tangencial del peso vale Pt = mg sen. Sustituyendo en (1):

Fr = F+Pt = P/v + mg sen = 25 + 45 = 70 N.

15. Determinad la fuerza de frenado (por efecto del aire) que sufre un vehculo de
1200 kg sabiendo que, para mantener constante una rapidez de 72 km/h al ascender
por una carretera que tiene una pendiente del 13 %, debe desarrollar una potencia
constante de 60 CV. ( 1 CV = 735 W).

sol: 645 N

16. Sobre un cuerpo de 2 kg de masa, inicialmente en reposo sobre una superficie


horizontal, se ejerce una fuerza de 20 N paralela a la superficie. Sabiendo que el co-
eficiente de rozamiento entre el cuerpo y la superficie es de 02, determinad el incre-
mento de energa cintica cuando dicho cuerpo se haya desplazado 3 m.

sol: Ec = 48 J.

17. Se eleva un cuerpo de 15 kg de masa, mediante una fuerza igual a su peso. Calcu-
lad la variacin de energa cintica que sufrir cuando haya recorrido 10 m.

sol: Ec = 0 J.

18. Probad, aplicando el teorema del trabajo-variacin de energa cintica, que si un


cuerpo describe un movimiento circular y uniforme, la fuerza resultante sobre l ha
de ser, en todo momento, perpendicular a la trayectoria.

19. Sea un cuerpo de 4 kg de masa sobre el que actan varias fuerzas. Determinad de
tantas formas como sea posible, el trabajo realizado por la fuerza resultante entre los
instantes 1 y 2 segundos, sabiendo que su vector de posicin viene dado por la expre-
t
3

sin r = i 2 j (t 1) 2 k m (si t en s).
3

Si llamamos estados A y B a las situaciones en los instantes 1 s y 2 s, el trabajo realizado


por la fuerza resultante se podr evaluar aplicando:
B
Wres BA A Fres dr

Trabajo y energa 126

Cmo podramos determinar la fuerza resultante?



Podemos obtenerla a partir de la aceleracin ya que Fres = m a y la aceleracin se obtiene

dr
a partir de v
dt
dv
t 2 , 0, 2(t 1) a
dt
(2t, 0, 2) m/s2, con lo que nos queda:

Fres = (8t, 0, 8).


B
Sustituyendo este valor en la expresin del trabajo Wres BA A Fres dr tenemos:

Wres AB A 8t dx +A 8 dz . Cmo podemos resolver estas integrales?


B B

En la primera de las integrales anteriores existen dos variables, por lo que para poder re-
solverla, ser necesario un cambio de variable para reducirlas a una sola. La relacin fun-
cional entre las variables la determina la ecuacin de la trayectoria.

x = t3/3 dx = t2 dt
y=2
z = (t-1)2 dz = (2t-2)dt

Wres AB 1 8t t 2 dt +1 8 (2t - 2) dt = 30 + 24 - 16 = 38 J.
2 2

De qu otra forma podramos haber resuelto el problema?

Otro de mtodo de resolucin consistira en utilizar la expresin que relaciona directa-


mente el trabajo resultante con la variacin de energa cintica (antiguamente conocido
como teorema de las fuerzas vivas), aplicndola entre los estados A y B:

m v 2B m v 2A
Ec B Ec A
B
Wres A
2 2

Tanto vB como vA se pueden obtener a partir de la expresin del vector velocidad


v t 2 , 0, 2(t 1) sustituyendo t por los valores correspondientes:

v A = (1, 0, 0) m/s vA = 1 m/s

v B = (4, 0, 2) m/s vB = 20 m/s

Sustituyendo ahora los valores numricos en la expresin del trabajo, nos queda:

m v 2B m v 2A
Wres AB Ec B Ec A 2 20 2 38 J
2 2

20. Deducid la expresin de la energa potencial gravitatoria del sistema formado por
un cuerpo de masa m (situado a una pequea altura sobre el suelo) y la Tierra.

sol: Ep = mg0h + C (siendo C una constante, m la masa del cuerpo y g0 la intensidad del
campo gravitatorio terrestre en la superficie del planeta).
Trabajo y energa 127

21. Calculad la expresin de la energa potencial elstica de un resorte de constante


elstica K.

En el sistema de la figura cuando el muelle tiene su longitud normal (relajado) su extremo


de la derecha se encuentra en x = 0. Vamos a analizar lo que ocurre cuando al extremo
libre del muelle sujetamos un bloque, estiramos hasta llevarlo a una posicin a la derecha
de A y luego lo dejamos en libertad. Los valores de x representan la elongacin del muelle
y son positivos a la derecha del punto 0 y negativos a su izquierda. Cuando el bloque
de
masa m se desplaza hacia la izquierda sometido a la accin de la fuerza elstica Fe y nos
centramos en lo que pasa entre las posiciones A y B, podemos decir que dicha fuerza rea-
liza un trabajo positivo (la fuerza tiene el mismo sentido que el desplazamiento), cuyo
valor se puede determinar como:

xB xB xB Kx 2 Kx2A
WFe A F dr Fx dx Kx dx B
B
xA xA xA 2 2

Analizando el resultado obtenido podemos ver que el trabajo realizado por la fuerza els-
tica entre las posiciones A y B, se expresa como la diferencia entre lo que vale la funcin
Kx2/2 + C (donde C es una constante) en el primer punto A y lo que vale esta misma fun-
cin en el segundo punto B. Dicha funcin, para un muelle dado, solo depende del valor
de x, lo que nos muestra que la fuerza elstica es una fuerza conservativa (el trabajo que
realiza entre dos puntos dados, no depende de la trayectoria seguida). Designaremos, pues,
a esa funcin como energa potencial elstica Ep.

Conviene darse cuenta de que la energa potencial es una funcin indeterminada (no sa-
bemos lo que vale la constante C). Sin embargo esto lo nico que quiere decir es que no
tiene sentido hablar de valores absolutos de la energa. La principal utilidad de la funcin
Ep es que al ser una funcin de estado (depende de la posicin), permite calcular fcil-
mente el trabajo realizado por la fuerza conservativa asociada a dicha energa potencial,
mediante la expresin general:

WcAB EpAB ( Ep B EpA ) EpA Ep B

As pues, lo que medimos siempre, son variaciones de energa y no valores absolutos de la


misma. No obstante, si adoptamos el criterio de asignar el valor 0 a la energa potencial
elstica del muelle cuando este se encuentra relajado (es decir, cuando x = 0), tenemos
que Ep0 = 0 + C C = 0, es decir la constante C queda determinada y podemos referirnos
a la energa potencial elstica del muelle en cualquier posicin como:
Trabajo y energa 128

Kx 2
. Aunque lo que en realidad significa esta expresin es que si el muelle pasara de
2
encontrarse distendido (extremo libre del mismo en la posicin x = 0) a otra situacin de
compresin o alargamiento (extremo libre en una posicin cualquiera x distinta de 0) la
Kx 2
energa potencial elstica aumentara justamente en .
2

22. Se lanza un cuerpo de 8 kg de masa, verticalmente hacia arriba, con rapidez de 7


m/s. Determinad la altura que alcanzar en los siguientes casos: a) no existe roza-
miento con el aire; b) durante el ascenso pierde una energa de 80 J por el rozamien-
to. Cmo sera la energa cintica con la que llegara al suelo en cada uno de los ca-
sos anteriores?

Si despreciamos el efecto del rozamiento con el aire, podemos suponer que la energa
cintica inicial del cuerpo va disminuyendo conforme
este asciende debido al trabajo ne-
gativo que realiza sobre l la fuerza peso ( P forma un ngulo de 180 con el desplaza-
miento) hasta llegar a un punto en el que dicha energa cintica ser nula, habiendo alcan-
zado el cuerpo su mxima altura
respecto al punto de lanzamiento. Como el cuerpo sigue
sometido a la misma fuerza P , esta le hace descender realizando ahora el mismo trabajo
que antes pero positivo (formar un ngulo de 0 con el desplazamiento), lo que causar
un aumento de la energa cintica, de forma que, cabe pensar que el cuerpo llegue de vuel-
ta al punto de lanzamiento con la misma energa cintica con que parti.
Para resolver el ejercicio mediante consideraciones de trabajo y energa, hemos de esta-
blecer (a conveniencia) dos estados A y B entre los cuales aplicaremos la expresin:

(1) Wres BA Ec

Seleccionaremos como estado A, el correspondiente a cuando el cuer-


po sale (porque de l tenemos informacin) y como estado B el que se
encuentra el cuerpo cuando alcanza la mxima altura (porque de l nos
piden informacin). La ecuacin anterior puede escribirse como:

(2) Wres BA Wc BA Wnc BA Ec

en la que Wc BA representa el trabajo realizado por las fuerzas conservativas que actan
sobre el cuerpo desde la posicin A a la B y Wnc BA el que realizan las fuerzas no conserva-
tivas. Como en nuestro caso, la nica fuerza que acta es el peso y esta es conservativa,
nos queda que:
(3) Wc BA WP BA = Ec

Esta ecuacin puede servirnos para calcular la altura h B alcanzada por el cuerpo si conse-
guimos expresar el trabajo realizado por la fuerza peso en funcin de dicha altura.
Trabajo y energa 129

Cmo podemos calcular el trabajo que realiza la fuerza peso?

Como siempre que la fuerza sea conservativa, habr dos formas:

a) Aplicando la expresin del trabajo realizado por una fuerza (en este caso constante):

WP AB P r P r cos180 mgh B

b) En funcin de la variacin de la energa potencial a la que est asociada dicha fuerza.:

WP BA Ep BA (Ep B Ep A ) mgh B

En este ejercicio no tiene mayor importancia realizar el clculo de una forma u otra, pero
en otros casos el clculo del trabajo realizado puede ser mucho ms complejo (por ejem-
plo, si la trayectoria seguida no es rectilnea) y, si la fuerza es conservativa, el hecho de
que el trabajo pueda calcularse simplemente restando dos valores de la energa potencial,
supone una gran ventaja.

Sustituyendo ahora en la ecuacin (3) y teniendo en cuenta que vB = 0, nos queda:

mv 2A mv 2A
- mghB = 0 o lo que es lo mismo: mgh B
2 2

Cmo puede interpretarse la ecuacin que acabamos de obtener?

Toda la energa cintica inicial del lanzamiento, se ha transformado en energa potencial


gravitatoria cuando el cuerpo alcanza la mxima altura

Obtened la altura mxima alcanzada y analizad el resultado obtenido

v A2
Despejando hB de la expresin anterior, obtenemos que: hB hB = 245 m
2g

En primer lugar, vemos que el resultado literal obtenido es dimensionalmente homogneo


(L en ambos miembros). Por otra parte, nos damos cuenta de que contempla algunos casos
evidentes, como por ejemplo, que si vA = 0, la altura ser 0; que si no hubiese gravedad la
altura alcanzada por el cuerpo sera infinita (no habra nada que le frenase); si v A aumenta,
la altura mxima aumentar, etc. Adems, podemos comprobar que en este caso, la masa
del cuerpo no influye, es decir, un guijarro o un gran peasco, alcanzarn la misma altura
siempre que se lancen hacia arriba con la misma velocidad y en ausencia de rozamiento.

Comprobad que la energa cintica del cuerpo cuando llega de vuelta al punto de partida
es la misma con que sali.
2
Basta con aplicar (ahora desde B hasta A): Wres B = Ec AB = EcA - EcB = EcA = mv A
A

2
Trabajo y energa 130

Teniendo en cuenta de nuevo que la nica fuerza es el peso y que se trata de una fuerza
A
conservativa: Wres B = WP AB Ep AB mgh B
mv 2A
Con lo que finalmente obtenemos que: mgh B
2
Es decir, toda la energa potencial gravitatoria correspondiente a la situacin B, se ha
transformado en energa cintica en el instante en que el cuerpo vuelve al punto desde
donde se lanz.

Cmo cambia el problema si no hacemos la simplificacin de considerar la friccin con


el aire despreciable?

En ese caso, ya no puede pensarse en que la energa cintica con que sale el cuerpo sea la
misma que con la que vuelve, ya que la fuerza de rozamiento es no conservativa. El traba-
jo realizado por este tipo de fuerzas a lo largo de una trayectoria cerrada es siempre nega-
tivo y esto hace que la energa mecnica con que vuelve el cuerpo al punto de partida sea
siempre menor que la energa con la que sali.

Ahora en el ascenso actan dos fuerzas, el peso P y la fuerza de rozamiento Fr . Ambas
tienen sentido contrario al desplazamiento, por lo que realizan un trabajo negativo que
producir una disminucin de la energa y, en consecuencia, podemos pensar que el cuer-
po se quedar sin energa cintica a menos altura que antes (cuando no haba rozamiento).
Aplicando de nuevo la ecuacin (2):

Wres BA Wc BA Wnc BA Ec WP AB WFr AB Ec AB Ec AB Ep AB WFr AB

La ecuacin anterior puede ponerse como: (EcB + EpB ) = (EcA + EpA) + WFr BA

Interpretad la expresin obtenida

Como el trabajo realizado por la fuerza de rozamiento es negativo, la suma de la energa


cintica y potencial en la situacin final ser menor que en la inicial. (En este caso 80 J
menor porque, de acuerdo con el enunciado: WFr BA = -80 J).

Conviene recordar que, aunque habitualmente se hable de energa perdida, ello no quiere
decir que esa energa haya desaparecido. En realidad sigue estando, pero se encuentra
repartida entre muchas de las partculas que forman el cuerpo y el aire.

Teniendo en cuenta que vB = 0 y que hA = 0, esta ltima ecuacin puede ponerse como

mv A2
mghB - = WFr BA .
2
mv 2A
WFr A
B

Despejando hB obtenemos: hB 2 hB = 145 m


mg
Trabajo y energa 131

Analizad la expresin obtenida

Podemos resaltar, por ejemplo, cmo, ahora s que influye la masa que tenga el cuerpo y
v A2
que esta expresin se convertira en la anterior ( h B = ), si no hubiese rozamiento.
2g

23. Demostrad, por aplicacin del teorema


del trabajo-variacin de energa cintica, que
la fuerza F que debemos aplicar para subir el
cuerpo de la figura con movimiento uniforme F
es mg/2. (Suponed masas de poleas y cuerda
despreciables).
m

24 . El cuerpo de la figura adjunta tiene una masa de 3 kg y se encontraba inicialmen-


te en reposo
en la base del plano inclinado (origen de espacios) cuando se le aplic una
fuerza F en la direccin del plano y sentido ascendente. El mdulo de la fuerza, que
depende de la posicin "e", viene dado por: F = e2 +25 (N para "e" en m) y el coefi-
ciente de rozamiento entre el cuerpo y el plano vale 05. Cul ser la rapidez del
cuerpo cuando haya recorrido 10 m sobre el plano?

Designaremos como A la posicin inicial del cuerpo y como B la posicin que ocupa tras
recorrer los 10 m sobre el plano inclinado siguiendo la trayectoria rectilnea que se indica

en la figura.
Sobre dicho cuerpo actan
tres fuerzas que realizan trabajo: el peso P , la
fuerza F y la fuerza de rozamiento Fr . La rapidez con que se mueva el cuerpo al llegar a
B, depender del trabajo realizado por la fuerza resultante a lo largo del trayecto desde A
hasta B:

Wres BA Ec B Ec A
Trabajo y energa 132

El trabajo resultante puede calcularse en este caso como: Wres BA WP BA WF BA WFr BA


A continuacin trataremos de obtener cada uno de esos tres trabajos, para despus calcular
el trabajo resultante sumndolos y poder hallar la rapidez que nos piden:

Al ser el peso una fuerza conservativa: WP AB Ep AB ( Ep B Ep A ) mg (h B h A )


Como hA = 0 y sen = hB/L (siendo L = 10 m la longitud de plano entre A y B), nos que-
da:

WP AB mg h B mgLsen = -1805 J

En cuanto a la fuerza que tira del cuerpo hacia arriba, podemos hallar el trabajo como:

L3

B B L
WFAB F de
A t
F de (e2 25) de 25L = 5833 J
A 0 3

Finalmente, el trabajo que realiza la fuerza de rozamiento, vendr dado por:

WFr AB Frt e = -Fr L= -NL

N, es el mdulo de la fuerza normal que el cuerpo hace sobre el plano y, de acuerdo con el
principio de accin y reaccin N = R. Como, por otra parte, la componente normal de la
aceleracin es nula, tiene que cumplirse que R = Pn = mgcos. Por tanto Fr = -mg cos.

WFr AB - mg cosL = -1198 J

Sumando los tres trabajos tenemos: Wres AB = -1805 + 5833 -1198 = 283 J.

As pues, la energa cintica se ver incrementada en 283 J. Como el cuerpo estaba ini-
cialmente en reposo podemos escribir que Ec B = 283 J y por tanto mv2/2 = 283 de donde
podemos obtener la rapidez del cuerpo como:

2 283
v = 137 m/s
3

Resolved de nuevo el problema calculando antes la fuerza resultante que acta sobre el
cuerpo, para despus calcular el trabajo realizado por la misma y, finalmente, la rapidez
que nos piden mediante la aplicacin del teorema trabajo resultante-variacin de energa
cintica.

25. Un bloque de 1 kg choca contra un resorte, de masa despreciable y constante els-


tica K = 2 N/m, segn se aprecia en la figura. Sabiendo que el resorte sufre una com-
presin mxima de 4m y que el coeficiente de friccin es 025, determinad la rapidez
con la que choc el cuerpo con el resorte.
Trabajo y energa 133

El bloque llegar con cierta rapidez vA al resorte (que se encuentra inicialmente distendi-
do) y comenzar a interaccionar con l comprimindolo. Las fuerzas que actuarn
sobre
el
bloque durante la compresin del resorte sern, la fuerza
de rozamiento Fr , el peso P , la
fuerza normal R que hace el plano y la fuerza elstica Fe que ejerce el resorte.

Qu caractersticas tiene cada una de dichas fuerzas?



La fuerza de rozamiento es constante, vale N siendo N la fuerza
normal que el bloque
hace sobre el plano (del mismo mdulo y sentido contrario que R ), tiene sentido contrario
a la velocidad del bloque y es no conservativa. La fuerza elstica, tambin tiene sentido
contrario a la velocidad del bloque, pero es conservativa y no es constante, sino que va
aumentando conforme aumenta la compresin x que sufre el muelle, de manera que su
mdulo vale Fe = Kx, siendo K la constante elstica del muelle y x lo que ha variado su
longitud (en valor absoluto). El peso y la fuerza normal que hace el plano sobre el bloque
tienen sentido contrario y han de anularse (P = R) ya que la aceleracin no tiene compo-
nente normal (al ser una trayectoria rectilnea la velocidad no cambia de direccin). El
bloque se mover, pues, cada vez ms lento y la fuerza elstica que acta sobre el mismo
ir aumentando hasta alcanzar un valor mximo en el instante en que se pare (x = 4 m).

Cmo podemos hallar la rapidez que llevaba el bloque al chocar contra el muelle?
Una posibilidad es mediante consideraciones de trabajo y energa, aplicando la ecuacin
Wres = Ec a los estados A (cuando el bloque choca) y B (cuando el muelle se halla com-
primido al mximo y el bloque parado). Si distinguimos entre las fuerzas conservativas y
no conservativas que actan sobre el bloque, podemos escribir la ecuacin anterior como:

Wres BA Wc BA Wnc BA Ec .

Si tenemos en cuenta que Wc AB Ep AB y reagrupamos los trminos, la expresin ante-


rior puede escribirse como:

(1) Wnc BA Ec A Ep A Ec B Ep B en la que:


Trabajo y energa 134


Wnc AB WFr AB Fr r Fr x B cos180 N x B mg x B

mv 2A Kx 2B
EcA = , EpA = 0, Ep B , EcB = 0.
2 2
Kx 2B 2 mgx B
Sustituyendo en (1) y despejando, obtenemos v A = 72 m/s
m
Un anlisis del resultado literal obtenido, nos permite mostrar en primer lugar que es di-
mensionalmente homogneo (L/T en ambos miembros) y al mismo tiempo que contempla
algunos casos evidentes como es el hecho de que si x B valiese 0 (el muelle no se compri-
me) la rapidez sera nula, o que cuanto mayor resulte x B (siempre a igualdad de los restan-
tes factores) con ms rapidez habr chocado el bloque contra el muelle, etc.

26. Sobre el plato de una balanza de muelle se deja suavemente, acompandola con
la mano, una masa de 200 g de forma que se alcanza la posicin de equilibrio que co-
rresponde a una compresin de 1 cm. Si apretando se la hace descender 2 cm ms, y
se suelta el plato, se efectan oscilaciones. Cul es la energa cintica de la masa
cuando el muelle tiene su longitud natural?

sol: Ec = 310-2 J

27. Sobre un resorte de masa despreciable, dispuesto verticalmente, se deja caer un


cuerpo de 25 kg. Determinad la constante elstica del resorte sabiendo que sufre una
compresin mxima de 15 cm al soltar el cuerpo desde una altura de 90 cm sobre el
extremo superior del resorte.
sol: K = 2.333 N/m.

28. Un cuerpo de masa m se desliza


sobre un plano inclinado terminado
en una circunferencia de radio r.
Admitiendo nulos los rozamientos,
desde qu altura mnima debemos
soltar el cuerpo para que pase el rizo?

Como ya vimos en dinmica (ejercicio 42), para que el cuerpo pase el rizo, es necesario
que en la parte superior disponga de la rapidez mnima necesaria para describir una curva
de igual radio que la circunferencia. Ello exige que exista una fuerza normal tal que Fn =
mv2/r, donde Fn = P + R, siendo P el mdulo de la fuerza peso y R el de la fuerza normal
que hace la superficie sobre el cuerpo, de manera que para ese punto podemos escribir:

(mg R) r
mg + R = mv2/r y despejar v con lo que nos queda: v
m
Trabajo y energa 135

de donde concluimos que el valor mnimo que podr tomar v en el punto considerado,
corresponder al caso en el que R sea 0, es decir, que toda la fuerza normal requerida sea
suministrada por la fuerza peso, con lo que vmin = gr . Si llegase con una rapidez infe-
rior, la fuerza peso sera mayor que la fuerza normal necesaria y r disminuira por lo que
el cuerpo caera. Si llega con una rapidez mayor, el radio tendera a aumentar porque el
peso sera insuficiente para mantener al cuerpo en una trayectoria con esa curvatura, pero
la superficie se lo impide suministrando la fuerza R necesaria.
La rapidez con que llegue el cuerpo a la parte superior del rizo, depender de la altura
desde la que se suelte. Cmo podemos encontrar esta relacin?
En el sistema formado por el bloque, las superficies y la Tierra, al no haber rozamientos
no hay ninguna fuerza no conservativa que realice trabajo y, por tanto, la energa mecni-
ca ha de conservarse, es decir, la energa mecnica en A ha de valer lo mismo que la
energa mecnica en B.

EcA + EpA = EcB + EpB

Teniendo en cuenta que en A la energa cintica es 0, obtenemos: Ep A = EcB + EpB

Tomando como nivel 0 de energa potencial gravitatoria la superficie horizontal:

v 2B
gh B 5r
1 2
mgh A mv B mgh B , de donde: hA 2 y sustituyendo vB h A
2 g 2

29. Dado el esquema de la figura, determinad (por aplicacin del principio de conser-
vacin de la energa) la mxima compresin que se producir en el resorte cuando
tras soltar el cuerpo impacte en el mismo. La altura hA vale 1 m, el cuerpo se aban-
dona en el punto A, su masa es de 1 kg, la constante elstica del muelle vale K = 20
N/m, el coeficiente de rozamiento en el tramo AB vale 02 y en el tramo horizontal se
considera despreciable.

El cuerpo de la figura descender por el plano inclinado AB (el valor mximo de la fuerza
de rozamiento resulta inferior a la componente tangencial del peso) y mientras descienda
el trabajo realizado por la fuerza resultante har que aumente su energa cintica. Luego se
desplazar por el tramo horizontal BO sin que se realice trabajo sobre l (la fuerza resul-
tante es 0) por lo que la energa cintica se mantendr constante. Al llegar al resorte cho-
Trabajo y energa 136

car contra el comprimindolo, de manera que la fuerza elstica que el resorte ejerce so-
bre el cuerpo lo ir frenando haciendo que la energa cintica vaya disminuyendo hasta
valer 0 en el punto D (instante en que la compresin del resorte es mxima y el cuerpo se
encuentra momentneamente en reposo).
Para resolver el problema, podemos aplicar la expresin que relaciona el trabajo resultante
con la variacin de energa cintica (en la cual se halla implcito el principio de conserva-
cin de la energa) en cada uno de los tramos, teniendo en cuenta que entre A y B solo
realizan trabajo el peso y la fuerza de rozamiento, entre B y C no se realiza trabajo y fi-
nalmente que entre O y D es la fuerza que el muelle ejerce sobre el bloque la nica que
realiza trabajo.

Wres A Ec B Ec A
B

Wres B Ec O Ec B
O

Wres O Ec D Ec O
D

Sumando nos queda que: Wres BA + Wres OB + Wres DO Ec D Ec A o lo que es lo mismo:

Wres DA Ec D Ec A 0
Wres DA WP BA WFr BA WFe DO 0

Cmo podemos determinar la compresin mxima que sufrir el muelle?


El trabajo que realiza la fuerza elstica est relacionado con la variacin de la energa
potencial elstica Kx2/2, en la que x representa la variacin de longitud del resorte y K su
constante elstica. Podemos hallar el trabajo realizado por cada una de las fuerzas y susti-
tuir en la ecuacin anterior para obtener la x correspondiente a la mxima compresin.

Como la fuerza peso es conservativa WP BA (Ep B Ep A ) Ep A Ep B

y tomando el suelo como nivel 0 de energa potencial gravitatoria, nos queda que:

WP BA Ep A Ep B mg h A

El trabajo de la fuerza de rozamiento (no conservativa) lo podemos hallar como:


Trabajo y energa 137

hA
WFr AB Fr r Fr r cos180 N
sen30

En la expresin anterior, N es la fuerza normal que el cuerpo hace sobre el plano. Como
en el tramo AB la velocidad no cambia de direccin, la componente normal del vector
aceleracin es 0, lo que implica que R Pn 0 y por tanto que R = Pn = mg cos30 con lo
que N = mg cos30 y el trabajo de rozamiento a lo largo del plano inclinado puede expre-
sarse como:
hA
WFr AB mg cos30
sen30
La fuerza elstica es una fuerza conservativa, de manera que teniendo en cuenta que el
punto O corresponde a la posicin del extremo del muelle cuando est distendido (x = 0) y
que el punto D (situado a x m de O) es el que corresponde a la situacin de mxima com-
presin del muelle, podemos escribir que:

Kx 2
(Ep D Ep O ) Ep O Ep D 0
D
W Fe O
2
Sumando cada uno de los trabajos anteriores e igualando a 0:

- Kx = 0
hA 2
mghA - mgcos30
sen30 2

En la ecuacin anterior x corresponde a la compresin mxima experimentada por el mue-


lle. Despejando, obtenemos que:

2mg h A
x 1 = 081 m
K tg30

Podemos ahora analizar el resultado obtenido:

Conviene comprobar en primer lugar que es dimensionalmente homogneo. A continua-


cin podemos ver que el resultado contempla el hecho de que cuanto mayor sea la altura
hA desde la que se deje el cuerpo, mayor sea su peso y menor sea la constante elstica
(muelle ms blando), mayor ser la compresin del muelle. Cuanto menor sea el ngu-
lo de inclinacin del plano, menor ser la compresin. El resultado tambin nos permite
predecir cul ser la compresin mxima en el caso de que no hubiera rozamiento (man-
teniendo constantes los dems factores):

2mg h A
x=
K

Adems, podemos ver que si cambiamos el coeficiente de rozamiento y lo aumentamos


hasta que llegue a valer tg30 = 058, el muelle no se comprimir (x = 0). Ello es lgico,
puesto que, como ya se vio en dinmica, cuando vale lo mismo que tg es porque la
fuerza de rozamiento mxima (mgcos), toma el mismo valor que la componente tan-
gencial del peso (mg sen) y, en consecuencia, el cuerpo permanecera en su situacin
Trabajo y energa 138

inicial de reposo. Este mismo efecto se conseguira si, para un coeficiente de rozamiento
dado vamos variando el ngulo de inclinacin del plano hasta que su tangente tome el
valor de dicho coeficiente.

30. Disponemos de un bloque de 10 kg unido a un resorte, de masa despreciable y cu-


ya constante elstica vale 40 N/m, segn se aprecia en la figura. Calculad la mxima
compresin que sufrir el resorte al aplicar la fuerza constante F de 5 N.


Al aplicar la fuerza F se provocar una aceleracin al bloque el cual ir comprimiendo el
resorte ejerciendo sobre ste una cierta fuerza, que, en todo momento ser del mismo
mdulo y sentido contrario a la fuerza que har el resorte sobre el bloque (principio de
accin y reaccin). As pues,
sobre el bloque actuarn dos fuerzas paralelas al plano: la
que nosotros
le aplicamos F , que es constante, y otra de sentido contrario que le ejerce el
resorte Fe cuyo mdulo es directamente proporcional a la variacin de longitud que sufre
el muelle (es decir, que va aumentando conforme el resorte se comprime).

Sobre
el bloque tambin actan otras dos fuerzas, que son el peso P y la fuerza normal
R que ejerce la superficie , pero no las tendremos en cuenta ya que se anulan entre si y no
realizan ningn trabajo.

Como al principio la fuerza aplicada es mucho mayor que la fuerza elstica que ejerce el
muelle, el bloque acelera hacia la derecha, pero como F permanece constante mientras
que Fe va aumentando, la aceleracin ser cada vez ms pequea y llegar un momento en
que valdr 0 porque los mdulos de ambas fuerzas sern iguales F = Fe (posicin de equi-
librio). En un principio, podra pensarse que esta es la situacin de mxima compresin
del resorte, pero no es as, ya que en ese instante, aunque la fuerza resultante sea nula, el
bloque no est parado sino que se est moviendo con una cierta rapidez (alcanzada a lo
largo del trayecto) y se seguir moviendo comprimiendo al resorte, si bien a partir de la
posicin de equilibrio, la fuerza elstica superar a la fuerza aplicada F y, en consecuen-
cia, habr una fuerza resultante hacia la izquierda, cuyo mdulo ser cada vez mayor, que
provocar una aceleracin que har que la velocidad del bloque vaya disminuyendo hasta
que se detenga. En ese punto, la velocidad del bloque pasa por el valor 0 y la fuerza resul-
tante sobre l ir hacia la izquierda, haciendo que el bloque contine movindose en ese
sentido, contrario al inicial.

Para resolver el problema tomaremos como origen de espacios la posicin inicial del blo-
que y como sentido positivo el del movimiento durante la compresin del muelle.

Cmo podramos determinar la compresin mxima que sufrir el resorte?


Trabajo y energa 139

Podemos tratar de resolver el problema aplicando la expresin que relaciona el trabajo


resultante con la variacin de energa cintica, ya que en ella interviene el trabajo realiza-
do por la fuerza elstica, que est relacionado, como sabemos, con la variacin de energa
potencial elstica, y esta con la posicin.

Tomaremos como estado inicial A el correspondiente al instante en que comienza a actuar


la fuerza F y como estado final B, el del instante en que la rapidez del bloque es 0.

Wres BA Ec BA Ec B Ec A 0

Wres BA WF BA WFe BA

WF A Ft x Fcos 0 x B F x B
B

Kx 2B
WFe A Ep (Ep B Ep A )
B

Sustituyendo las expresiones obtenidas en la del trabajo resultante nos queda:

K x 2B Kx 2B
Wres A F x B
B
= 0, o lo que es equivalente: FxB =
2 2

Esta ltima expresin podemos interpretarla diciendo que todo el trabajo realizado por F
entre A y B, se ha empleado en aumentar la energa potencial elstica. A partir de dicha
expresin es fcil obtener la compresin mxima del muelle como:

2F
xB = 025 m
K

El anlisis del resultado literal obtenido nos lleva a comprobar en primer lugar que es
dimensionalmente homogneo y a constatar que contempla algunos casos particulares
evidentes como, por ejemplo, que si F = 0, no habr compresin, que si F aumenta la
compresin mxima conseguida ser mayor y que si K aumenta (muelle mas duro) di-
cha compresin ser menor.

Otra posibilidad es resolver el problema utilizando un tratamiento cinemtico-dinmico


en lugar de utilizar consideraciones de trabajo y energa. Esto puede hacerse en muchos
problemas de mecnica, lo que constituye una muestra la coherencia de este campo de
conocimientos y de la validez de las expresiones que en l se manejan. No obstante, la
resolucin de problemas a partir de las ecuaciones de trabajo y energa suele ser, ms cor-
ta y sencilla que mediante cinemtica y dinmica, como se puede comprobar en este mis-
mo ejemplo.

A partir de la ecuacin fundamental de la dinmica, podemos hallar la aceleracin tan-


gencial del bloque y con ella la expresin de la rapidez. Luego, hemos de buscar una rela-
cin entre la rapidez y la posicin del bloque, para hallar dnde estar cuando se detenga.
Trabajo y energa 140

FKx dv F K x
Frest = mat F-Kx = mat at = . Como at = dv/dt, nos queda:
m dt m

Para poder integrar, multiplicaremos por dx en ambos miembros, con lo que nos queda:

dv FKx (F K x) v x (F K x
dx dx v dv dx v dv dx
dt m m 0 0 m

Resolviendo estas integrales obtenemos una expresin que nos relaciona v con x:

2F x K x 2 2F
v2 y haciendo v = 0 y x = xB, se obtiene fcilmente que: x B
m K

31. Dado el dispositivo de la figura ad-


junta determinad la rapidez con que
llegar al extremo del plano un cuerpo
de 2 kg de masa que se apoya sobre el
resorte, cuando tras comprimir a ste
ltimo 50 cm, se deje en libertad.

Datos: Longitud del resorte distendido


1m, constante elstica del resorte 400
N/m y coeficiente de rozamiento 02.

sol: v = 54 m/s

32. Disponemos de un pndulo constituido por un hilo, de masa despreciable y un


cuerpo esfrico de 1 kg. Sabiendo que el hilo tiene una longitud de 20 cm y que sopor-
ta como mximo una tensin de 15 N, determinad el mnimo valor de la amplitud an-
gular "" de partida, para que al dejar libre el pndulo se rompa el hilo.

Si el pndulo permanece en reposo en la posicin = 0 lafuerza resultante sobre la bola


ha de ser nula y por tanto, el peso P y la tensin del hilo T han de ser de igual mdulo y
tener sentidos contrarios.

Al soltar el pndulo desde una cierta posicin correspondiente a 0, la fuerza resultante



sobre la bola ser perpendicular a la tensin del hilo en ese instante ( "T0 " ) y la bola des-
cribir un movimiento de trayectoria circular, a lo largo del cual irn variando tanto la
tensin como la fuerza resultante. Cuando se encuentre en la posicin = 0, tendr cierta
rapidez (tanto mayor cuanto mayor sea la altura desde la que se solt). En este punto la
resultante no ser nula, ya que la bola no est en reposo sino describiendo un movimiento
circular (de radio igual a la longitud L del hilo), por lo que en dicha posicin tendr que
haber una fuerza resultante en la direccin de la normal (recordemos que en el movimien-
to circular la direccin del vector velocidad est cambiando constantemente).
Trabajo y energa 141

Cmo podemos calcular el valor de la mxima rapidez con que podr llegar la bola al
punto considerado?

Aplicando la ecuacin fundamental de la dinmica a la bolita en el instante en que pasa


por la posicin ms baja, podemos obtener el valor de la tensin del hilo en ese punto:

Fres n m a n v2
T P m an T P m
Fres n T P L

Como P, m y L son valores fijos, vemos que la tensin de la cuerda depende de la rapidez
con que la bola pase por la posicin ms baja, cuanto mayor sea dicha rapidez, mayor ser
la tensin del hilo. Sin embargo la mxima tensin que puede aguantar el hilo es de 15 N,
por tanto, para calcular la rapidez mxima que podr llevar la bola, tendremos que susti-
tuir T por su valor mximo y despejar v, con lo que nos queda:

(Tmax P) L
v max = 1 m/s
m

Esta rapidez depender del ngulo desde el que se haya soltado la bolita.
Cmo podemos determinar la relacin existente entre ambas magnitudes?

Podemos aplicar la ecuacin Wres BA Ec BA entre los estados A (cuando soltamos la bolita
desde un ngulo ) y B (cuando pasa por la posicin ms baja):

mv 2B
Wres BA Ec B Ec A Ec B
2
mv 2B
W B
res A WB
PA W B
TA W B
PA W B
TA
2

WP BA Ep BA Ep A Ep B Ep A mgh A

(tomando como Ep = 0 la del punto ms bajo).


WT BA 0 ya que en todo momento la fuerza T es perpendicular a la trayectoria.
Trabajo y energa 142

mv 2B v B2
Sustituyendo en W B
res A obtenemos que: mg hA = hA
2 2g

Si ahora sustituimos vB por su valor mximo obtendremos el valor de la altura lmite des-
de la que podemos soltar la bolita sin que el hilo se rompa: h max = 005 m.

Lh
Como cos = , basta con sustituir h por el valor obtenido para poder hallar al ngu-
L
lo que nos piden, resultando cos = 075 = 414. Soltando la bolita del pndulo
desde este ngulo, la tensin del hilo en la posicin ms baja ser de 15 N. Para ngulos
mayores la tensin ser mayor y, por tanto, el hilo se romper.

33. El sistema de la figura representa una


masa puntual colgada de un hilo sin masa
que se abandona desde una altura h0.
Cuando llega a la vertical, el hilo se en-
cuentra con un punto A que hace que se
doble dicho hilo. A qu altura h ascen-
der la masa?

sol: h = ho

34. Desde lo alto de una esfera de radio r = 24 m, se desliza, sin rozamiento, un cuer-
po de masa m. Determinad cul ser su rapidez en el momento en que se separe de la
esfera.


Las nicas fuerzas que actan sobre el cuerpo son el peso P y la fuerza normal R que ejerce
la superficie. Cuando la chica se encuentra en la posicin ms alta y en reposo, ambas fuerzas
se ejercen en la misma direccin y se anulan. Sin embargo, en cuanto se separe ligeramente de
Trabajo y energa 143

esa
posicin, comenzar a deslizar
ya que las dos fuerzas dejan de estar en la misma direccin
( P no cambia, pero s lo hace R que en todo momento ha de ser perpendicular a la superficie)
y no se anulan. Conforme vaya deslizando, la velocidad ir aumentando de valor y cambiando
de direccin, por lo que el vector aceleracin tendr componentes tangencial y normal.

Podemos pensar que al ir deslizando por la superficie de la semiesfera, debido a la curvatura


de
sta, la fuerza normal N que la chica hace sobre la superficie (y por tanto, la fuerza normal
R que la superficie hace sobre ella) va disminuyendo, hasta que al llegar a una cierta posicin
B, llega a valer 0. A partir de ese punto ya no hay interaccin entre la chica y la superficie.

Por otra parte, como no actan fuerzas no conservativas, la suma de la energa potencial gravi-
tatoria y cintica ha de permanecer constante, de manera que, si bien la energa potencial va
disminuyendo conforme la chica se mueve hacia el suelo, la energa cintica debe ir aumen-
tando justo en la misma proporcin, para que la suma de ambas valga siempre lo mismo.

Como la trayectoria desde el punto ms alto hasta donde se despega, es conocida, para ese
recorrido podemos aplicar un tratamiento escalar, con lo que:

Fres t = mat P cos (90-) = mat at = g sen (1)


mg cos R
Fres n = man P cos- R = man an = (2)
m

Mediante la ecuacin (1), podemos comprobar que la componente tangencial de la acele-


racin no es nula (va aumentando) y por tanto la rapidez ser cada vez mayor.

En cuanto a la aceleracin normal, sabemos que esta viene dada por a n = v2/r siendo r el
radio de curvatura. Como v va aumentando y entre A y B el radio es constante, queda cla-
ro que la aceleracin normal tambin tendr que aumentar y que para que ello ocurra,
segn la expresin (2), la fuerza R tiene que disminuir de forma que la diferencia P n - R
sea cada vez mayor.

Lo que ocurre entre A y B se puede interpretar admitiendo que como la rapidez va aumen-
tando cada vez es mayor la fuerza normal que se necesita para mantener a la chica descri-
biendo una trayectoria del mismo radio (Fn = mv2/r). El valor de esa fuerza normal viene
dado por la diferencia Pn - R en donde Pn va disminuyendo conforme el ngulo aumenta
Trabajo y energa 144

(recordemos que Pn = mg cos), luego R ha de disminuir ms aprisa todava para que la


diferencia de ambos trminos vaya aumentando. En el punto B, R ha llegado a valer 0 y
toda la fuerza normal es suministrada por Pn, pero, como la rapidez sigue aumentando
tambin lo har la fuerza normal requerida para que siguiese describiendo una trayecto-
ria de igual radio r. Dicha fuerza normal, a partir de ese punto, ser superior a Pn por lo
que r aumentar (ya que Pn ser insuficiente para mantener el mismo radio). A esta misma
conclusin se llega mediante la expresin P n = mv2/r (vlida a partir de B) en la que po-
demos ver que si v aumenta y Pn disminuye, r tiene que aumentar y, en consecuencia, la
chica se despega de la superficie.

Cmo podramos hallar la rapidez con que se mueve la chica en el preciso instante en
que deja de tener contacto con la superficie?

Una posibilidad sera partir de at = g sen y como at = dv/dt, tratar de integrar con el fin
de obtener v en funcin del ngulo y luego particularizar (en el punto B) introduciendo
la condicin de que R valga 0. Otro procedimiento, ms sencillo, es mediante considera-
ciones de trabajo y energa, relacionando el trabajo resultante entre A y B con la variacin
de energa cintica producida:

Wres BA Ec BA

Wres BA WP BA WR BA WP BA ( R no realiza trabajo por ser perpendicular a la trayectoria).

Como la fuerza peso es conservativa, podemos hacer WP BA Ep BA

As pues obtenemos que -Ep AB Ec AB es decir, la disminucin de energa potencial gra-


vitatoria que se produce cuando la chica pasa desde A hasta B, es igual al aumento de
energa cintica.
mv 2B
Desarrollando esta ltima expresin obtenemos que: mg h A - mg hB = , con lo que:
2

v 2B 2g (h A h B ) 2g (r - rcos B )

Vemos que, como habamos supuesto, la rapidez va aumentando a medida que baja (au-
menta ). Para conocer la rapidez en el punto B, necesitamos saber el valor del ngulo.

Recordemos que mediante la ecuacin (2) podemos relacionar el ngulo con la rapidez.

2
mg cos R v
En efecto, segn dicha ecuacin an = y como a n y en B la fuerza R es nula,
m r
obtenemos para el punto B, que:

v 2B v 2B
g cos B , con lo que cos B = y sustituyendo en la expresin anterior nos queda:
r gr
Trabajo y energa 145

v 2B
v = 2g (r - r cos B ) = 2g (r - r
2
B )
gr
y de aqu podemos simplificar y despejar vB con lo que:

2gr
vB = 4 m/s
3
Cmo podramos calcular B ?
v 2B
Bastara introducir el resultado obtenido en la ecuacin cos B = y despejar, con lo
gr
que se obtiene cos B = 2/3 (independiente del radio) y B = 48.

35. De un resorte de 1 m de longitud y constante elstica 200 N/m se suspende un


cuerpo de 2 kg. Se le acompaa hasta la posicin de equilibrio y una vez en sta se le
aplica una fuerza vertical y hacia abajo de 10 N. Determinad la longitud mxima que
alcanzar el resorte.

sol: L = 12 m

36. El bloque a de la figura tiene una masa de 10 kg y el b de 2 kg. Determinad la ra-


pidez con que se desplazarn cuando se les deje en libertad y hayan recorrido 4 m
cada uno de ellos. (El coeficiente de friccin entre el bloque a y el plano vale 01).

En este problema tenemos dos cuerpos (a y b) que se desplazan bajo la accin de unas
fuerzas, de manera que el trabajo de la fuerza resultante que acta sobre cada uno produ-
cir una variacin en su energa cintica. Como los dos bloques se hallan unidos mediante
una cuerda que se supone inextensible, la rapidez de ambos en cada instante deber de
coincidir.

La trayectoria que siguen los dos cuerpos es nica y conocida. Tomaremos como origen
de espacios la posicin inicial del bloque a y como sentido positivo el del movimiento.
Supondremos que ambos bloques se encuentran inicialmente en reposo, tomando como
origen de tiempos el instante en que se dejan en libertad.

Qu podemos hacer para calcular la rapidez cuando se hayan desplazado 4 m?


Trabajo y energa 146

Una posibilidad es aplicar la ecuacin Wres AB Ec AB Ec B Ec A tomando como estado


A el correspondiente al instante inicial o momento en que se dejan en libertad (v A = 0) y
como final el que corresponde a cuando se han desplazado 4 m sobre la trayectoria.

Para el bloque a: WT AB WFr AB Eca B Eca A Te - Fra e = ma v2/2


Para el bloque b: WT AB WPb B Ecb B Ecb A -Te + mb g e = mb v2/2
A

Sumando las dos ecuaciones anteriores nos queda:

mb g e - Fra e = (ma + mb) v2/2

La fuerza de rozamiento vale N (N es la fuerza normal que el bloque a ejerce sobre la


superficie). Sabemos que, de acuerdo con el principio de accin y reaccin N = R (fuerza
normal que la superficie hace sobre el bloque). Por otra parte, la componente normal del
vector aceleracin es nula (el bloque a se mueve en lnea recta) y, por tanto, R = P, con lo
que concluimos que en este caso N = P = m a g y Fra = ma g. Sustituyendo en la expre-
sin anterior y despejando v, queda:
2ge (m b m a )
v = 258 m/s
ma mb
Analizad el resultado literal obtenido

Se trata de un resultado dimensionalmente homogneo (L/T en ambos miembros).


Adems contempla algunos casos evidentes, como, por ejemplo, que si la gravedad o el
desplazamiento son nulos, la rapidez ser 0 (no se movern). Lo mismo ocurrir cuando
ma valga lo mismo que mb (o, lo que es equivalente, cuando la fuerza de rozamiento
sobre a y el peso de b sean iguales).

Utilizad el resultado del problema para averiguar con qu rapidez se movera el bloque b
a los 4 m si la masa del a valiese 0.

Lo nico que hemos de hacer es sustituir m a por 0 en la expresin obtenida, con lo que nos
quedara:
2g e m b
v 2 g e = 8'9 m/s
mb

(Como era de esperar, mayor que la anterior).

A los mismos resultados podramos haber llegado si en lugar de aplicar la ecuacin


Wres AB Ec AB a cada uno de los bloques por separado y luego sumar, hubisemos plan-
teado directamente la suma como:

WPb AB WFr AB Eca Ecb Epb WFr AB Eca B Ecb B

Otra posibilidad es utilizar un tratamiento de cinemtica y dinmica:


Trabajo y energa 147

Resolved el problema mediante un tratamiento cinemtico-dinmico y comprobad que se


obtiene el mismo resultado.

37. Los bloques a y b de la figura (de 4 kg y 3 kg de masa respectivamente), se en-


cuentran inicialmente a la misma altura. En el momento de dejarlos en libertad co-
mienza a actuar sobre b una fuerza vertical y hacia abajo, de 15 N. Determinad la
rapidez de los cuerpos cuando la distancia entre ellos sea de 6 m.


Como el peso de a es mayor que el de b, si no existiera la fuerza F que tira del b hacia
abajo, al dejar el sistema en libertad a descendera y b subira, pero como Pb + F Pa, ocu-
rre lo contrario y es b quien desciende y a quien sube.

Cmo podemos calcular la rapidez cuando la diferencia de alturas sea de 6 m?

Podemos aplicar la expresin Wres AB Ec AB Ec B Ec A a los estados A (cuando solta-


mos los cuerpos) y B (cuando a haya subido 3 m y b haya descendido otros 3 m). Toma-
remos como sentido positivo el del movimiento y como origen de espacios la posicin
inicial del bloque a, con lo que nos queda:

Wres AB WPa AB WPb AB WTa AB WTb AB WF AB (Eca B Eca A ) (Ecb B Ecb A ) .

Como el desplazamiento sobre la trayectoria es el mismo para los dos cuerpos, las fuerzas
son constantes y la fuerza que ejerce la cuerda sobre cada cuerpo tiene el mismo mdulo
(se considera que la masa de la cuerda es despreciable), podemos expresar cada uno de los
trabajos como el producto de la componente escalar tangencial de cada fuerza y el despla-
zamiento espacial (cada factor con su signo correspondiente), como se indica a continua-
cin:

WPa AB Pa e; WPb AB Pb e; WTa AB T e; WTb AB T e; WF AB F e

As pues: Wres AB ma g e mb g e F e Eca B Ecb B = (ma + mb) v2/2


Trabajo y energa 148

2ge (m b m a ) 2 Fe
Finalmente, despejamos v y obtenemos: v v = 207 m/s
ma m b
Analizad el resultado literal obtenido

Despus de comprobar que es dimensionalmente homogneo (L/T en ambos lados del


signo igual), podemos ver, por ejemplo, que si el desplazamiento es 0, tambin lo ser v
(no se mueven los cuerpos) y que lo mismo ocurrira si las masas fuesen iguales y F = 0.
Por otra parte, si ma fuese 0 y F tambin, obtendramos v = 2ge , que coincide, preci-
samente con la rapidez de un objeto que se deja caer libremente, en el instante en que ha
descendido e m.

38. Calculad, aplicando el principio de conservacin de la energa, la rapidez que ad-


quirirn los cuerpos de la figura, cuando se hayan desplazado 1m. (El coeficiente de
rozamiento es 02, ma = 20 kg y mb = 5 kg).

sol: v = 31 m/s.

39. Dado el esquema adjunto, determinad la mxima compresin que sufrir el resor-
te al dejar en libertad el sistema. (La masa de A es de 4 kg, la de B es de 1 kg , la cons-
tante elstica del resorte vale 20 N/m y se supone que no hay rozamiento).

sol: 1 m

40. Sobre un resorte de masa despreciable y dispuesto verticalmente, se deja caer un


cuerpo de 2'5 kg de masa. Determinad la constante elctrica del resorte sabiendo que
este sufre una compresin mxima de 15 cm cuando el cuerpo se suelta desde 90 cm
altura respecto del extremo superior de dicho resorte. sol: 233N/m
5. SISTEMAS DE PARTCULAS

1. Obtened las coordenadas del centro de masas del sistema formado por las tres
partculas de la figura (m1 = 1g , m2 = 1g , m3 = 2 g) en cada uno de los sistemas de
coordenadas cartesianas representados (O y O).

Tal y como se define el centro de masas (en adelante, cdm) de un sistema de partculas, es
evidente que sus coordenadas dependern del sistema de referencia elegido, pero su loca-
lizacin fsica (su situacin respecto a las partculas que forman el sistema) solo depen-
der de las masas dichas partculas y de sus posiciones relativas, por tanto, sea cual sea el
sistema de referencia que escojamos, el centro de masas ha de ocupar el mismo lugar,
mientras no cambie la disposicin relativa de las partculas que lo forman.

El ejercicio se puede resolver sin ms que aplicar las expresiones correspondientes a las
coordenadas del cdm en cada uno de los sistemas de referencia.

En el sistema de referencia O, las coordenadas de cada partcula son:

Para m1: x1 = 2 cm, y1 = 1 cm


Para m2: x2 = 3 cm, y2 = 2 cm
Para m3: x3 = 4 cm, y3 = 0 cm

En el sistema de referencia O, las coordenadas de cada partcula son:

Para m1: x1 = 1 cm, y1 = 1 cm


Para m2: x2 = 2 cm, y2 = 2 cm
Para m3: x3 = 3 cm, y3 = 0 cm
Sistemas de partculas 150

El vector de posicin del cdm segn el sistema de referencia O, viene dado por:

m
n
r
i i
rC
1

M

en donde mi es la masa de una de las n partculas de que consta el sistema, ri su vector de
posicin y M, la masa total (suma de las masas de todas las partculas). As pues:

m
n
i i

r 1(2, 1) 1(3, 2) 2 (4, 0) 13 3
rC , cm
1
=
M 4 4 4

El vector de posicin del cdm segn el sistema de referencia O', viene dado por:


m
n
i r 'i 1 (1, 1) 1(2, 2) 2 (3, 0) 9 3
r' C 1
, cm
M 4 4 4

Analizando los resultados obtenidos, es fcil darse cuenta de que la coordenada y es la


misma en ambos sistemas de referencia (debido a que al coincidir el eje X con el X las
ordenadas de las tres partculas son las mismas en los dos sistemas). Adems, si seala-
mos la posicin del cdm, en ambos sistemas de referencia obtenemos el mismo punto, tal
y como supusimos al principio.


2. Determinad el vector de posicin rC del c.d.m. del sistema de partculas represen-
tado en la figura adjunta (m1 = 2 kg, m2 = 3 kg y m3 = 5 kg).

Z(m)
m3

m Y(m)

sol: rc = (1'5, 1'8, 2'5) m X(m 1 m2
)

3. Determinad la posicin del c.d.m. del sistema del ejercicio anterior, en el supuesto
de que el origen del sistema referencial se trasladara hasta ocupar la misma posicin
que la partcula de masa m2.

Sol: rC = (-1'5, -4'2, 2'5) m
Sistemas de partculas 151

4. Dado el sistema de partculas de la figura,


las cuales se encuentran inicialmente en reposo
y sabiendo que estn unidas por una varilla Y(m
rgida y de masa despreciable, se pide: ) F1
m1
a) El vector de posicin del c.d.m. un instante t
F2
despus de comenzar a actuar las dos fuerzas
exteriores representadas.
b) Caso de no existir la varilla Afectara eso al m2
movimiento de cada una? Y al del c.d.m.?
X(m)
(m1 = 02 kg; m2 = 03 kg; F1 = 3N; F2 = 4 N)

El sistema est formado por dos partculas sobre las que actan, respectivamente, las

fuerzas exteriores Fext 1 = (3, 0) N y Fext 2 = (0, 4) N, as como dos fuerzas interiores F12

(fuerza que sobre m1 hace m2) y F21 (fuerza que sobre m2 hace m1) que son desconocidas
aunque
sabemos
que, por tratarse de una pareja de accin y reaccin se debe de cumplir
que F12 = - F21

Cmo podemos obtener la posicin del cdm en cualquier instante?


m
n
r
i i
Una posibilidad sera, en principio, intentar aplicar la expresin rC
1

M

Sin embargo, para ello, necesitaramos conocer la ecuacin de movimiento r r (t ) de
cada una de las partculas. Dicha ecuacin se podra obtener por integracin si supira-
mos la expresin de la aceleracin con que se mueve cada partcula, pero como no sabe-
mos cunto valen las fuerzas interiores, no podemos tener ese dato y, por tanto, no es po-
sible resolver el problema de esta forma.

Otro procedimiento, consiste en calcular la aceleracin del centro de masas y despus,


por integracin, llegar a obtener el vector de posicin.

El clculo de la aceleracin correspondiente al cdm puede realizarse teniendo en cuenta


que la fuerza resultante que acta sobre un sistema de partculas es la suma de todas las
fuerzas exteriores y de todas las interiores. Dicha suma puede expresarse como:

Fres Fext i Fint ij Fext i Fext 1 mi a i
n

en la que hemos tenido en cuenta que la suma de las fuerzas interiores entre cada dos
partculas ij es siempre 0 ya que se trata de parejas accin-reaccin. Por otra parte, del
concepto mismo del cdm, sabemos que si multiplicamos la masa total del sistema por la
aceleracin del cdm, deberemos de obtener tambin la fuerza resultante que acta sobre
el sistema, por tanto:
Fres M a C
Sistemas de partculas 152

Igualando las dos expresiones anteriores, podemos obtener la aceleracin del cdm como

F ext i Fext (3, 0) (0, 4) (3, 4)
aC = = (6, 8) m/s2
M M 0' 2 0' 3 0' 5

La expresin literal anterior nos dice que solo mediante una fuerza exterior resultante es
posible cambiar el movimiento del cdm de un sistema, es decir, las fuerzas internas de un
sistema no pueden modificar la velocidad del cdm, que permanecer constante mientras
no haya una fuerza exterior resultante actuando sobre el sistema.

dv C
Expresando ahora la aceleracin como a C y teniendo en cuenta que el sistema se
dt
encuentra inicialmente en reposo podemos integrar para obtener la expresin de la velo-
cidad del cdm en cualquier instante:

dv C 0 a C dt v C a C t = (6t, 8t) m/s

vC t
0


drC
La velocidad del cdm se puede expresar como v C con lo que integrando de nuevo
dt
podemos obtener el vector de posicin del cdm:

drC 0 v C dt rC r0C 0 a C t dt rC r0C a C t 2

r t t 1

r0 2

El vector de posicin inicial rCO del cdm se puede calcular fcilmente puesto que sabe-
mos las posiciones iniciales de cada una de las partculas, de modo que:


rCO
m r
i i

0' 2 (0, 3) 0' 3(4, 0) (1' 2, 0' 6)
(2' 4, 1' 2) m
M 0' 5 0' 5

Sustituyendo este resultado en la expresin obtenida para rC nos queda finalmente:

1 1
rC r0C a C t 2 (2' 4, 1' 2) (6, 8) t 2 rC (3t 2 2' 4, 4t 2 1' 2)
2 2

Si las partculas no estuviesen unidas por la varilla, el movimiento del cdm sera el mis-
mo, porque lo nico que se eliminaran seran las fuerzas interiores y estas, como ya
hemos visto, no afectan a la velocidad del cdm. No obstante, lo que s se vera afectado
sera el movimiento de cada partcula, ya que la fuerza resultante sobre cada una de ellas
sera distinta. La varilla impide tambin que cambie la distancia entre las partculas, for-
mando as lo que se conoce como un slido rgido.
Sistemas de partculas 153

5. Determinad la posicin del dentro de masas de una varilla longitudinal homognea


de masa m y longitud L

Siempre que tengamos un objeto extenso homogneo y de geometra regular, podremos


calcular las coordenadas de su cdm considerndolo como un sistema formado por un con-
junto de infinitos elementos de masas infinitesimales, tan pequeos como queramos. En
el caso que nos ocupa se trata de un objeto longitudinal, es decir, un objeto en el que una
de sus dimensiones es mucho mayor que las otras dos y podemos descomponerlo en toda
una serie de elementos de longitud dL a cada uno de los cuales corresponder una masa
dm, tal y como se indica en la figura adjunta (en donde ya hemos escogido un sistema de
referencia, respecto al que obtendremos las coordenadas del cdm).

Y
dm
-L/2 Z dx L/2 X

Si tomamos el sistema de referencia de la figura, podemos ver que dL = dx y que los ele-
mentos infinitesimales se distribuyen de forma continua desde que x = -L/2 (extremo iz-
quierdo de la varilla) hasta que x = L/2 (extremo derecho de la varilla. Se trata de una
suma de infinitos sumandos infinitesimales, lo que nos conduce a expresar las coordena-
das del centro de masas como:


L/2

L/2
dm x
xC ; yC 0; zC 0 .
dm
m

En el numerador de la expresin de xC figura una integral que, de entrada, no podemos


resolver porque en ella existen dos variables (m y x). Una forma de solucionar este incon-
veniente es buscar una relacin entre dichas variables.

Como la varilla es homognea la densidad de la misma ser constante, esto significa que
ser la misma sea cual sea la longitud del trozo de varilla que se considere. Por otra parte,
al tratarse de un objeto longitudinal, podemos calcular el valor de su densidad dividiendo
la masa entre la longitud, de manera que, si designamos por a la densidad:

= m/L y tambin para una masa infinitesimal: = dm/dL= dm/dx, con lo que: dm = dx

Sustituyendo ahora dm en la expresin de xC obtenemos que:

L/ 2
x2 L2 L2
L /2

dm x L /2 x dx 2 M

xC m L/ 2

8 8
0

dm
0
M M L M
Sistemas de partculas 154

Es decir, el cdm de la varilla estara situado en el centro de la misma, coincidiendo con su


eje de simetra. Este resultado no es, por supuesto, fruto de la casualidad sino que obede-
ce a que, debido a la simetra de la figura, siempre que consideremos un elemento de ma-
sa dm situado fuera del eje de simetra a una cierta distancia x i del mismo, le podemos
hacer corresponder otro igual situado en la posicin -xi, de forma que la posicin del cdm
de ambos coincidir con la del centro de simetra y esto lo podemos hacer para todas las
parejas de elementos infinitesimales que componen la varilla:

Y
dm dm
-xi xi X

Este resultado se puede generalizar para cualquier otro objeto que tenga centro de simetr-
a y sea homogneo, ya que podra considerarse como compuesto de una serie de varillas
de masa infinitesimal dispuestas de forma que sus centros coincidan con el centro de si-
metra, de manera que, siempre que se cumplan las condiciones anteriores (homogenei-
dad y existencia de centro de simetra) el cdm de un objeto estar localizado en el mismo
centro de simetra.

6. Un tringulo rectngulo est constituido por tres varillas homogneas de longitud


13 cm, 12 cm y 5 cm, cuyas masas son, respectivamente: 100 g, 80 g y 20 g. Obtened
la situacin del c.d.m. correspondiente a dicho tringulo.

De acuerdo con lo tratado en el ejercicio anterior, para localizar el cdm de este sistema
cada una de las varillas puede considerarse como una masa puntual situada en su centro
de simetra y de valor igual a la masa total de la varilla, de manera que el problema se
reduce as a determinar la posicin del cdm de un sistema formado por tres masas puntua-
les, dispuestas de cierta forma. Para ello, hemos de escoger en primer lugar un sistema de
referencia que nos resulte cmodo, como, por ejemplo el que se indica en la figura si-
guiente:
Sistemas de partculas 155

Y
C
A

B X

Los datos correspondientes a cada una de las masas sern pues:



mA = 20 g; r = (0, 25) cm
CA
mB = 80 g; rCB = (6, 0) cm

mC = 100g; rCC = (6, 25) cm

y el vector de posicin del cdm del sistema vendr dado por:


m A rCA m B rCB m C rCC 20 (0, 2' 5) 80 (6, 0) 100 (6, 2' 5)
rC = (54, 15) cm
mA mB mC 200

El cdm de las tres varillas se encuentra pues, en el interior del tringulo formado por las
mismas y situado, como es lgico, un poco a la izquierda de la recta que une los centros
de las varillas C y B. Por otra, parte podemos comprobar que el cdm puede, como ocurre
en este caso, estar situado en un punto donde en realidad no exista masa alguna.

7. Hallad las coordenadas del c.d.m. del sistema formado por un cilindro homogneo
de 2 kg cuyo centro de masas se encuentra en el punto A (0, 1, 0) m y una varilla
homognea de 4 kg cuyo centro se encuentra en el punto B (2, 0, 0) m.

sol: (4/3, 1/3, 0) m

8. Determinad la posicin del c.d.m. de la figu- Y(m) L


ra adjunta. (Se considera que la figura es
homognea y que solo tiene las dos dimensio- L/2
nes indicadas en el dibujo).
L/4 2L/3

sol: rc = (0, 0'68L) m
L/2 X(m)
Sistemas de partculas 156

9. Determinad el c.d.m. de un cuadrado homogneo de


lado L, en el que se ha practicado un orificio, segn se
aprecia en la figura adjunta.

En este caso, debido al agujero, la figura no presenta centro de simetra. Se trata pues de
obtener la posicin del cdm correspondiente a la parte sombreada del cuadrado, que es la
nica que tiene masa. Cmo podramos hacerlo?

Lo ms sencillo es suponer que la figura estuviese completa y que la podamos descompo-



ner en dos: una, que llamaremos A, que es la parte sombreada (cuyo rCA es la incgnita
del problema) y otra, que llamaremos B, que ser el crculo. El cdm del cuadrado comple-
to se podr determinar sustituyendo cada una de las figuras A y B por un punto, (su pro-
pio cdm), en el que est concentrada toda su masa:

m A rCA m B rCB
rC
mA mB

En la figura adjunta hemos situado el cuadrado en Y


un sistema de referencia de tal forma que la posi-

cin de su cdm ser rC = (L/2, L/2) y corresponder A
al cdm de dos masas puntuales mA, cuya posicin
L/2
rCA queremos hallar, y mB situada segn el vector
B
de posicin rCB = (L/4, L/4). Aplicando la expre-

sin de rC obtenemos que:
L/2 X


m A rCA m B rCB (mA mB )rC mB rCB m rC mB rCB
rC de donde rCA =
mA mB mA m mB

Dado que no conocemos las masas de A ni de B, hemos de buscar algn modo de poner
dichas masas en funcin de los datos que se nos dan (longitud del lado del cuadrado).

Ello puede conseguirse mediante la utilizacin del concepto de densidad (en este caso
densidad superficial , puesto que se trata de un objeto bidimensional).

Para el cuadrado completo: m = L2


Para el crculo: mB = L2/16
Sistemas de partculas 157

Para la figura A: mA = L2 - L2/16 = L2 (1-/16)

Sustituyendo las expresiones anteriores nos queda que:



m rC m B rCB L2 (L/2, L/2) L2 /16 (L/4, L/4) (L/2, L/2) (L/64, L/64)
rCA
m mB L2 (1 / 16) (1 /16)

y finalmente: rCA = (056L, 056L)

Otra forma de resolver este tipo de problemas, consiste en suponer el objeto real como un
cuadrado completo al que se le ha superpuesto en la posicin indicada un crculo imagi-
nario de igual densidad pero negativa. Se trata de un procedimiento matemtico ya que
obviamente no existen objetos con masa negativa, pero que puede hacer que los clculos
resulten bastante ms cmodos. Aplicadlo en este caso y comprobad que se obtiene el
mismo resultado.

10. Determinad el c.d.m. del cuerpo formado al unir dos esferas; una de 10 kg masa y
2 m de radio y la otra de 3 kg de masa y 1 m radio, mediante una varilla de 2 kg y 10
m de longitud, segn se aprecia en la figura.


sol: rc = (-3'46, 0) m

11. La superficie rectangular de la figura


adjunta tiene doble densidad en la zona
ms oscura que en la ms clara. Determi-
nad el radio del orificio circular, con cen-
tro en el punto P (0, 3) cm, que deberemos
practicar para que el c.d.m. de la figura
resultante quede situado en el punto (0, -1)
cm.

Si la figura tuviese toda ella la misma densidad, la posicin del cdm coincidira con la del
centro de simetra (situado en el origen de coordenadas). Sin embargo, como la densidad
de la parte superior es mayor que la de la parte inferior, el cdm estar sobre el eje Y pero
Sistemas de partculas 158

situado en la parte positiva. Si realizamos un orificio circular con centro en el punto de


coordenadas (0, 3) cm, el cdm de la figura resultante se habr desplazado hacia abajo
(pues desaparece masa de la parte superior). Como es lgico, dicho desplazamiento ser
tanto mayor cuanto mayor sea el radio del orificio. Esto quiere decir que ha de existir una
relacin entre la posicin del cdm y el radio del agujero que hagamos, de forma que si
fijamos un valor para la posicin del cdm, ello determinar el radio que deba de tener
dicho agujero.

Cmo podramos hallar el radio orificio?

Podemos considerar el rectngulo como el conjunto de dos objetos:

Cuadrado superior A, de lado L, masa mA, con un agujero circular de radio R en su centro
y con su cdm situado en la posicin xCA = 0, yCA = 3 cm, es decir, coincidiendo con el
centro del agujero. Densidad = 2.

Cuadrado inferior B, de lado L, masa mB, y cdm en xCB = 0, yCB = -3 cm. Densidad = .

La masa del cuadrado agujereado A, se puede considerar como la masa del cuadrado
completo al que le hemos quitado un trozo de forma circular, es decir: mA = 2L2 -
2R2. El radio del orificio se podra calcular entonces a partir de la expresin del cdm
del conjunto formado por dos masas puntuales m A y mB situadas respectivamente en el
cdm correspondiente a cada uno de los cuadrados, ya que el cuadrado B no cambia y al
superior se le ha practicado el agujero en el centro del mismo. Por otra parte, debido a la
simetra de la figura y al sistema de referencia escogido, queda claro que x C = 0, por lo
que solo calcularemos yC.

L3
LR 2
m A y CA m B y CB (2 L2 2R 2 )L/2 L2 ( L/2)
yC 22
mA m B (2 L 2R ) L
2 2 2
3L 2R 2

L3 6L2
Sustituyendo yC por -1 cm y despejando el radio R, queda R =293 cm.
2 (2 + L)

12. Determinad la posicin del centro de masas de un semianillo de radio R.

En este caso no existe un centro de simetra y si queremos determinar la posicin del cdm
tendremos que proceder descomponiendo el anillo en elementos de longitud infinitesimal
dl (a cada uno de los cuales corresponder una masa dm) distribuidos de forma continua a
lo largo del mismo. Tomaremos el sistema de referencia de la figura ya que al ser el eje
OY de simetra, el cdm estar situado en uno de sus puntos y, en consecuencia, x C valdr
0, quedando reducido el problema a determinar yC.. Supondremos que el dimetro del
alambre del que est hecho el semianillo, es despreciable frente a su longitud (cuerpo
longitudinal).
Sistemas de partculas 159

La expresin correspondiente a yC se podr obtener como: y C


ydm
0

La integral anterior ha de abarcar a todo lo que es el semianillo. Podemos resolverla bus-


cando una relacin entre la variable y y la variable m, o bien, una relacin de ambas
con una tercera. Para ello podemos utilizar la densidad del anillo.

En efecto, sabemos que: m = L = R. Como es homogneo, tambin se cumplir


que dm = dL, de manera que:
m L

yC
ydm y dL
0 0

m m

En este caso dL no se puede identificar con dx (como suceda en la varilla recta) ni con
dy, aunque s que podemos relacionarlo con el ngulo ya que dL representa, precisamen-
te, la longitud del arco infinitesimal que abarca en ngulo d y, por tanto, dL = Rd, con
lo que sustituyendo en la expresin anterior tenemos:

ydm
L
y dL

yRd R0 yd

yC
0 0

m m m m

Sin embargo, todava no podemos resolver la integral, puesto que tenemos dos variables:
e y. Tendremos, pues, que buscar una relacin funcional entre ambas que nos permita
eliminar una de ellas.

Si nos fijamos en la figura, podemos darnos cuenta de que la ordenada "y" del elemento
dL es y = R sen , luego:

R0 Rsend R 2 0 send

R 2 cos 0

2R 2
yC
m m m m

2R
y si consideramos que m = R, nos queda finalmente que: yC = 0'64 R

Sistemas de partculas 160

13. Determinad la posicin del centro de masas de un semicrculo de radio R.

Se trata, en este caso, de una figura superficial que no presenta centro de simetra, aunque s
un eje de simetra en el cual debe encontrarse su cdm. Tomaremos el sistema de referencia,
segn se aprecia en el dibujo, de forma que nicamente necesitaremos calcular yC.

ydm
Considerando el objeto como un conjunto de masas infinitesimales: y C
m

Cada elemento dm se puede calcular como dm = dS = dx dy, siendo la densi-


dad superficial del objeto. Esto nos llevara a tener que resolver una integral doble que,
no siendo excesivamente complejo, queda fuera del nivel en que nos encontramos.

Podemos superar este inconveniente si en lugar de descomponer el semicrculo en cua-


drados infinitesimales lo hacemos en semianillos de superficie dS. Para ello, con centro
en O dibujamos semicircunferencias cuyos radios "r" vayan aumentando sucesivamente
en dr, de esta forma tendremos infinitas semicircunferencias que descomponen el semi-
crculo en infinitos semianillos de espesor dr. A cada semianillo le corresponde una masa
dm cuyo valor estar relacionado con su superficie mediante la expresin dm = dS

Cul sera la expresin de dS en funcin del radio r?

Dicha superficie infinitesimal puede evaluarse fcilmente si tenemos en cuenta que cada
uno de esos semianillos si lo desdoblsemos equivaldra a un rectngulo infinitesimal
de longitud r y altura dr (por lo que dS = r dr) o tambin, teniendo en cuenta que dS
es la superficie comprendida entre el semicrculo de radio r y el de radio r+dr, de forma
que restando obtendramos:
Sistemas de partculas 161

(r dr ) 2 r 2 r 2 2rdr (dr ) 2 r 2
dS =
2 2

y como (dr)2 se puede considerar despreciable frente al resto, nos queda que dS = r dr.
Otro procedimiento (ms riguroso) consiste en diferenciar la funcin S = r2/2 con lo que
resulta: dS = r dr.

Proceded ahora a determinar la posicin del cdm del semicrculo.

El cdm de cada uno de esos semianillos estara situado segn y = 2r/ (ved resultado del
ejercicio anterior). Por tanto la posicin del cdm correspondiente a todo el semicrculo se
podr calcular considerando a cada uno de esos semianillos como una masa puntual de
valor dm colocada sobre el eje OY en la ordenada y = 2r/, con lo que:


2
r dS 2/ 0 r r dr
R
ydm
2R 3
yC =
m m m 3m

y teniendo en cuenta que m = R2/2, nos queda finalmente que: yC = 4R/3 = 042R

14. Determinad la posicin del centro de masas correspondiente a un cono macizo, de


densidad constante, radio de la base R y altura h.

El cdm del cono deber de estar ubicado en el eje de simetra que presenta el cuerpo. Si
tomamos el sistema referencial del dibujo, solo ser necesario calcular la coordenada x
del centro de masas.

Se trata ahora, de un objeto tridimensional y cuya masa est distribuida de forma continua
y homognea. La coordenada x del cdm correspondiente viene dada por:

xdm
xC
m
Sistemas de partculas 162

En qu tipo de elementos de masa dm nos convendra descomponer el cono para poder


resolver la integral anterior de forma sencilla?

Una posibilidad es trazar planos paralelos a la base y distanciados dx unos de otros, de


esta forma el cono quedara descompuesto en infinitos discos de volumen dV, correspon-
diendo a cada uno de ellos una masa dm. El radio r de cada uno de esos discos ira cam-
biando desde 0 (vrtice del cono) hasta R (base del cono) y su volumen dV se podra eva-
luar considerando que corresponde a un cilindro infinitesimal de radio r y altura dx, con
lo que nos queda dV = r2 dx. Cada disco tiene su cdm en su centro de simetra (sobre el
eje OX) y su masa se puede expresar en funcin de la densidad como dm = dV.

La posicin del cdm de todo el cono se podr determinar sustituyendo cada uno de los
discos por una masa puntual de valor dm situada sobre el eje OX en la coordenada x que
corresponda a dicho disco.

xdm x dV x r
2
dx
xC =
m m m

Para resolver la integral anterior ser necesario que quede una nica variable.

Si nos fijamos en el dibujo, nos podemos dar cuenta de la relacin existente entre "r" y
"x", ya que ambos son los catetos de una tringulo rectngulo que es semejante a otro
mayor cuyos catetos son R y h, por lo que por semejanza de tringulos podemos escribir
r/x = R/h y despejar r, con lo que r = Rx/h. Podemos ahora sustituir r por esta expresin
en la integral anterior y resolverla:

x r
h h
dx x (R /h) 2 x 2 dx (R /h) 2 x 3 dx
2
(R /h) 2 (h 4 /4)
xc 0
0

m m m m

y considerando ahora que m = V = R2h/3 nos queda finalmente: xC = 3h/4


Sistemas de partculas 163

Por tanto, el cdm de un cono macizo y homogneo de altura h, est situado a una distan-
cia de 3h/4 de su vrtice (o lo que es lo mismo, a h/4 del centro de su base). El hecho de
que est ms cerca de la base que del vrtice es lgico, si tenemos en cuenta cmo se dis-
tribuye la masa en la figura (ms masa cuanto ms cerca de la base).

15. Determinad la posicin del centro de masas de un tringulo equiltero de lado L.

sol: Est situado a 2h/3 del vrtice superior, siendo h la altura del tringulo.

16. Determinad la posicin del centro de masas de una varilla de longitud L cuya
densidad aumenta linealmente con la distancia a uno de sus extremos.

En este caso, aunque la densidad no sea constante podemos conocer su valor en cada pun-
to (se halla determinada por una cierta funcin) lo que nos permite que tambin podamos
considerar la varilla como un conjunto de masas infinitesimales y, despus de escoger un
sistema de referencia apropiado, proceder a integrar la expresin correspondiente para
determinar la posicin del cdm.

En la figura anterior hemos representado una varilla lineal, de forma que las coordenadas
yC y zC sern 0, con lo que el problema se reduce a determinar x C correspondiente a una
varilla cuya densidad aumenta de un extremo al otro de forma lineal, es decir: = Kx
donde K es una constante y x la distancia al extremo hacia el cual la densidad se va
haciendo ms pequea, extremo que, en la figura, hemos situado en el origen de coorde-
nadas.

Hacia dnde estar situado el centro de masas de la varilla?

Es evidente que en este caso no estar en la mitad (como ocurra con la varilla homog-
nea) sino que se hallar desplazado hacia la derecha del punto medio, ya que la densidad
de la varilla va aumentando hacia ese lado y por tanto los elementos de masa dm en que
hemos considerado descompuesta la varilla, tendrn cada vez ms influencia en cuanto
a su contribucin a la situacin del centro de masas (dm = dx).

Determinad la coordenada xC correspondiente al centro de masas de la varilla

Para ello podemos utilizar la expresin correspondiente a x C para una distribucin conti-
nua de masa, teniendo en cuenta que la densidad no es constante y que no conocemos el
valor de la masa total de la varilla.
Sistemas de partculas 164

dm x x dx Kx dx K x /3
L L
2 3
2L

0 0
xC = 067 L
dm dx Kx dx K x /2
0
L 2 L
0
3

Como podemos comprobar, el centro de masas se halla desplazado del centro de la varilla
hacia la zona en donde la densidad va aumentando tal y como habamos supuesto ante-
riormente.

17. Una granada cae verticalmente y cuando se encuentra a 200 m del suelo y con
rapidez de 80 m/s, estalla rompindose en dos fragmentos de igual masa. Sabiendo
que uno de los fragmentos sale verticalmente y hacia abajo a 100 m/s, determinad:

a) La rapidez con la que sale el otro fragmento.


b) La posicin de cada fragmento 2 s despus de la explosin y, a partir de las mis-
mas, la del c.d.m. en ese mismo instante.
c) La posicin de la granada a los 2 segundos en caso de que no hubiese estallado.

La granada est cayendo hacia el suelo y en un instante determinado estalla rompindose


en dos fragmentos. Considerando la granada como un sistema de dos partculas, podemos
decir que durante el breve intervalo de tiempo que dura la explosin, actan unas fuerzas
internas de gran intensidad debido a los gases en expansin, que hacen cambiar sensible-
mente la velocidad con que se mueve cada uno de los fragmentos en relacin a la veloci-
dad que llevaban (la de la granada) al comenzar la explosin. Como en el problema nos
piden precisamente la velocidad con que sale despedido uno de esos fragmentos, una
cuestin importante que nos hemos de plantear en primer lugar es si la cantidad de mo-
vimiento del sistema se conserva o no durante el tiempo que dura la explosin.

El principio de conservacin de la cantidad de movimiento de un sistema, exige que sobre


dicho sistema no acte ninguna fuerza exterior (o que la suma de todas las que acten sea
0). En dichas condiciones sabemos que, debido a las fuerzas interiores, puede cambiar la
cantidad de movimiento de algunas de las partes del sistema, pero que ello se har de
forma que la cantidad total de movimiento del sistema se mantenga siempre constante.
En este caso, s que acta una fuerza exterior al sistema: la fuerza gravitatoria (peso) con
que, tanto la granada, como cada uno de los trozos en que se parte, son atrados por la
Tierra. Esta fuerza es la responsable de que la cantidad de movimiento de la granada no
se conserve sino que vaya aumentando conforme va cayendo hacia el suelo.

No obstante, si consideramos nicamente el intervalo de tiempo que dura la explosin,


podemos ignorar el efecto de la fuerza peso en el sistema ya que sta es muchsimo me-
nor que las fuerzas interiores y durante un tiempo tan sumamente corto apenas puede in-
fluir en la cantidad de movimiento de cada uno de los fragmentos. Por el contrario las
fuerzas interiores, aunque acten muy poco tiempo s que cambian sensiblemente la velo-
cidad de cada trozo, ya que son mucho ms intensas que el peso, pero como se trata de
fuerzas internas no influyen en la cantidad de movimiento del sistema. En definitiva pues,
podemos considerar que durante el brevsimo tiempo que dura la explosin, la cantidad
de movimiento del sistema se conserva y por tanto ser prcticamente la misma al co-

menzar y al finalizar la explosin. Si llamamos p a la cantidad de movimiento del siste-
Sistemas de partculas 165


ma al iniciarse la explosin, p'1 y p'2 a las cantidades de movimiento respectivas que los
dos fragmentos 1 y 2 en que se rompe la granada tienen justo al finalizar la explosin,
podemos escribir, pues, que :

p p'1 p' 2 o lo que es lo mismo: mv m1 v' 1 m 2 v' 2 .

Podemos representar la situacin mediante el siguiente esquema:

En el dibujo de la izquierda se ha escogido


un sistema de coordenadas cartesianas con
origen en el suelo y cuyo eje de ordenadas
coincide con la lnea vertical de cada de la

granada. En dicho sistema de referencia v es
la velocidad que lleva la granada al comen-

zar la explosin y vale v = (0, -80) m/s. As

mismo, v'1 es la velocidad del fragmento 1

justo al finalizar la explosin y vale v'1 = (0,

-100) m/s y v'2 es lo que hemos de calcular.

Cmo podemos determinar la velocidad del


segundo fragmento inmediatamente despus
de la explosin?

Dicha velocidad puede calcularse a partir de la ecuacin anterior correspondiente al prin-



cipio de conservacin de la cantidad de movimiento, sin ms que despejar de ella v'2

Hacia donde debera de moverse el fragmento 2 inmediatamente despus de la explo-


sin?

Si antes de explotar la cantidad de movimiento vena representada por un vector que solo
tena componente y, como el fragmento 1 sigue movindose en el eje Y, concluimos
que el 2 tambin ha de hacerlo para que la cantidad de movimiento del sistema sea igual
(mdulo, direccin y sentido) inmediatamente antes y despus de la explosin.

De la expresin anterior, resultante de aplicar el principio de conservacin de la cantidad


de movimiento, obtenemos la velocidad con que saldr despedido el fragmento 2 como:

mv m1v'1
v'2
m2

Dado que la granada se parte en dos trozos iguales, se cumplir que m 1 = m2 = m/2 con lo
que sustituyendo en la expresin anterior nos queda finalmente que:

mv' 1
mv -
mv m1 v' 1 2 v' 1
v' 2 2 (v - ) = (0, -60) m/s.
m2 m/2 2
Sistemas de partculas 166

Cmo podramos determinar ahora la posicin de cada uno de los fragmentos 2 s des-
pus de la explosin?

Dado que las velocidades tras la explosin son vectores en la direccin vertical y que la
fuerza peso tambin acta en la misma direccin, el movimiento posterior a la explosin
ser de trayectoria conocida (rectilnea) y eso nos permite aplicar un tratamiento escalar:

Si consideramos como instante inicial el momento en que se produce la explosin y ori-


gen de espacios el suelo tal y como se representa en los esquemas anteriores, podemos
obtener las ecuaciones de la rapidez v = v (t) y de la posicin e = e (t) de cada uno de los
fragmentos. Para ello podemos tener en cuenta que la aceleracin con que se mueven
ambos es constante (aceleracin de la gravedad) y aplicar directamente las ecuaciones
correspondientes al movimiento uniformemente acelerado:

v = v0 + a (t-t0)
e = e0 + v0 (t-t0) + a (t-t0)2/2

Para el fragmento 1, a = -10 m/s; v10 = -100 m/s, e10 = 300 m; t 10 = 0 s, con lo que:

v1 = -100 - 10t
e1 = 300 - 100t -5t2

Para el fragmento 2, a = -10 m/s; v 2 0 = -60 m/s; e 2 0 = 300 m; t 2 0 = 0 s, con lo que:

v2 = -60 -10t
e2 = 300 -60t -5t2

Las ecuaciones anteriores nos proporcionan la rapidez y posicin de cada uno de los
fragmentos desde la explosin hasta el instante en que llegan al suelo. As si queremos
saber el valor de dichas magnitudes en el instante 2 s, sustituimos t por 2 y nos queda:
Sistemas de partculas 167

v1 = -100 -20 = -120 m/s; e1 = 300 - 200 - 20 = 80 m


v2 = -60 -20 = -80 m/s; e2 = 300 -120 -20 = 160 m

Cmo podemos calcular la posicin del centro de masas?

Como conocemos la posicin de cada uno de los fragmentos en cualquier instante y la


componente x es nula (el movimiento transcurre en el eje Y), bastar con determinar y C
mediante la expresin correspondiente al instante t = 2 s.

m m
y1 y 2
m y m2 y 2 y y 2 80 160
yC = 1 1 2 2 1 120 m
m m 2 2

Dnde se encontrara la granada si no hubiese estallado?

No es necesario realizar ningn clculo para admitir que como la nica fuerza exterior
que acta es el peso y las fuerzas internas (en este caso las de la explosin) no pueden
modificar la velocidad del cdm, la posicin de la granada en cualquier instante coincidir
con la de su cdm independientemente de que estalle o no. Para comprobarlo no tenemos
mas que calcular la posicin de la granada en el instante t = 2 s (suponiendo que no esta-
lla) y ver que nos sale tambin 120 m. En efecto:

Si la granada no estalla, a = -10 m/s2 ; t0 = 0s; v0 = -80 m/s; e0 = 300 m, de modo que:
e = 300 -80t -5t2 y sustituyendo t por 2 s nos queda que e = 120 m

18. Escribid la ecuacin vectorial correspondiente al Principio de Conservacin de la


Cantidad de Movimiento en la colisin representada en la figura adjunta.

v2
Y
m1 v1 m2
X

v1

v1
19. Una granada que se encuentra des-
plazndose horizontalmente a razn de 8 Y
km/h, estalla rompindose en tres frag- v2
v 45
mentos de igual masa segn se aprecia en
la figura. Determinad las rapideces de los m 45 X
fragmentos 1 y 3, si el fragmento 2 tiene
una rapidez de 16 km/h. v3

sol: 4 2 km/h.
Sistemas de partculas 168

20. Un ncleo, inicialmente en reposo, decae radiactivamente emitiendo un electrn


de cantidad de movimiento 922 10-21 kgm/s y, formando un ngulo recto con el
electrn, un neutrino de cantidad de movimiento 533 10 -21 kgm/s. Cul ser la
cantidad de movimiento del ncleo residual?

sol: p 'r = (-5'33 10-21, -9'22 10-21 ) kgm/s

21. Una persona de 80 kg se encuentra sobre una lmina de 20 kg y 4 m de longitud.


Sabiendo que la lmina va provista de unas ruedas de masa despreciable (lo que nos
permite ignorar las fuerzas de friccin), determinad el desplazamiento de la misma
cuando la persona se haya desplazado de un extremo al otro.


Para desplazarse, la persona (A) ejerce una fuerza FBA sobre la lmina (B) hacia la dere-
cha de la figura y, de acuerdo con el principio de accin y reaccin, la lmina ejerce so-

bre ella otra fuerza FAB del mismo valor y sentido contrario, de manera que, de acuerdo
con las condiciones expuestas en el enunciado, persona y lmina se movern en sentidos
contrarios.

Si consideramos como sistema el conjunto de los cuerpos A y B, la pareja de fuerzas an-


teriormente sealada, corresponde a la interaccin entre la persona y la lmina. La fuerza

normal R AB que la lmina hace sobre la persona y la fuerza normal N BA que la persona
hace sobre la lmina, tambin responden a esa misma interaccin interna. Adems actan

tambin las fuerzas exteriores PA y PB debidas a la interaccin gravitatoria con la Tierra.

Al ser un movimiento rectilneo, la componente normal de la aceleracin es nula y, por



tanto, las nicas fuerzas a considerar sern FAB y FBA . Dichas fuerzas por ser interiores
no modifican movimiento del cdm del sistema. Esto mismo viene expresado en la ecua-
cin:

Fext m a C

segn la cual la fuerza exterior resultante sobre el sistema (de masa m) es la que produce
un cambio en la velocidad del centro de masas. Como en nuestro caso dicha fuerza exte-
rior es nula, ello implica que la velocidad del cdm no cambiar y, por tanto, como antes
de que comenzase a andar la persona sobre la lmina, el cdm se encontraba en reposo,
deber de continuar en reposo aunque persona y lmina se desplacen por la accin de
fuerzas interiores.
Sistemas de partculas 169

Qu podramos hacer para conocer el desplazamiento experimentado por la lmina


cuando la persona se haya movido de un extremo al otro de la misma?

Como el cdm tiene que permanecer durante todo el proceso en su posicin inicial ( rC0 ),
bastar con fijar dicha posicin y exigir que sea la misma cuando la persona haya llegado
al otro extremo de la lmina.

Y
cdm
B A
*
X
Para ello podemos tomar el sistema de referencia mostrado en la figura y sustituir cada
cuerpo por una masa puntual ubicada en su cdm. Adems, como la componente Y no
sufre modificacin alguna, solo consideraremos la componente x del cdm con lo que:

L
mA L mB
m x mB x B 2 80 4 20 2 = 36 m
x CO A A
mA mB m 100

En la expresin literal anterior podemos ver que conforme disminuye el valor de la masa
de la persona la posicin del cdm se va acercando a L/2. Anlogamente, si es la masa de
la tabla la que va disminuyendo, la posicin del cdm se acerca a L.

Cuando la persona llegue al otro extremo de la lmina la situacin ser la representada en


la figura siguiente:

Y
cdm
A B
d *
X

La posicin del cdm ser la misma que antes, aunque las posiciones de m A y mB habrn
cambiado. Podemos escribir pues que:

m A x' A m B x' B m A d m B (L/2 d)


xC = xC0
mA mB m

En la expresin anterior L es la longitud de la lmina y d la distancia a la que se encuen-


tra su extremo izquierdo del eje de ordenadas en el instante en que la persona llega al
mismo. Despejando d obtenemos:
L
m x CO m B
d= 2 = 32 m
m
Sistemas de partculas 170

22. Dos cuerpos de masas m1 = 20 g y m2 = 100 g se encuentran en reposo y separa-


dos 9 cm sobre una superficie horizontal y sin rozamiento. Se cargan elctricamente
con el mismo signo de forma que se repelen. Determinad el desplazamiento sufrido
por el de masa m2 cuando el cuerpo de masa m1 se haya desplazado 10 cm.

sol: 2 cm

23. Dado el sistema de partculas de la Y(m)


figura adjunta, determinad: 2 m/s
a) Posicin y velocidad del c.d.m. en el m3 = 1kg
m1 = 1kg
instante representado en la figura.
kg 2 m/s
b) Comprobad que la cantidad de movi-
miento del sistema respecto del c.d.m. es 1 m/s
0.
X(m)
sol: rC = (15 , 1) m ; v C = (1, 05) m/s. m2 = 2 kg

24. Sea un sistema de tres masas cuyas posiciones en el momento de poner el cron-
metro en marcha son las de la figura. En dicho instante, las masas m1 y m3 se en-
+
cuentran en reposo, sobre m3 acta una fuerza exterior de 24 N segn el OX y m2
tiene una velocidad de 4 m/s en la direccin representada. Se pide:

Z(m)
m3 = 6 kg
kg
m1 =1 kg
O
m2 =2 kg Y(m)
kg
X(m)

a) Posicin del c.d.m. en cualquier instante.


b) Cantidad de movimiento del sistema a los 3 s.
c) Momento cintico del sistema respecto del origen de coordenadas, a los 3 s.

1'2t 2 6 8t 4 4
sol: rC , , ; p C = (72, 8, 0) kg.m/s; L = (0, 14'4 , 24) kg.m2/s
9 9 3
Sistemas de partculas 171

25. Las masas de la figura se encuentran


Z(m
en reposo en el instante que comienzan a
) m3 = 3kg
actuar las fuerzas exteriores represen-
tadas. Determinad:

a) Vector de posicin del centro de masas, F2 = 6 N


cantidad de movimiento y energa cintica
del sistema formado por las tres masas.

b) Momento cintico del sistema respecto F1 =12 t m2 =2 kg Y(m)


del origen de coordenadas, y comprobad m1 =1 kg
que su derivada temporal coincide con el X(m)
momento de las fuerzas exteriores.

El vector de posicin del centro de masas correspondiente a un sistema de partculas co-


mo el que se describe en el enunciado de este problema, se puede calcular mediante la
expresin:


n
mi ri
rC 1

M

en la que M es la masa total del sistema, m i la de una de sus n partculas y ri el vector de
posicin correspondiente a esa partcula. Para determinar pues la posicin del cdm en
cualquier instante mediante la expresin anterior, necesitamos conocer el vector de posi-
cin de cada una de las partculas de que consta el sistema.

Como conocemos las fuerzas que actan sobre cada una de las partculas, podemos hallar
su aceleracin y despus integrar convenientemente (teniendo en cuenta las condiciones
iniciales) para calcular el vector de posicin correspondiente:

F1 (12t, 0, 0) F2 (0, 0, 6)
a1 (12t, 0, 0) ; a 2 (0, 0, 3) m/s2 ; a 3 0
m1 1 m2 2

Integrando ahora sucesivamente podemos determinar la velocidad y la posicin de cada


partcula en cualquier instante t.

Sabiendo que para la partcula 1, t0 = 0, v10 = 0 y r10 = (3, 0, 0) m:

dv v1 t t
a1 1 dv1 a1 dt dv1 a1 dt v1 (12t, 0, 0) dt (6t 2 , 0, 0)
dt 0 0 0

dr r1 t t
v1 A dr1 v1dt dr1 v1dt r1 r10 (6t 2 , 0, 0) dt (2t 3 , 0 , 0)
dt r10 0 0

r1 (3, 0, 0) (2t , 0, 0) (3 2t , 0, 0)
3 3


Para la partcula 2 se procede igual, sabiendo que v 20 0 y r20 = (0, 3, 0) m:
Sistemas de partculas 172


dv v2 t t
a 2 2 dv 2 a 2 dt dv 2 a 2 dt v 2 (0, 0, 3) dt (0, 0, 3t)
dt 0 0 0

dr r2 t t 3t 2
v2 2 dr2 v2dt dr2 v2dt r2 r20 (0, 0, 3t) dt (0, 0 , )
dt r20 0 0 2
3t 2 3t 2
r2 (0, 3, 0) (0, 0, ) ( 0, 3, )
2 2

Como la partcula 3 se encuentra en reposo y no est sometida a fuerza alguna, su vector



de posicin no cambiar y sabiendo que para t 0 = 0, v 30 = 0 y r30 = (0, 0, 4) m, tendre-
mos que:

r3 = (0, 0, 4) m en cualquier instante.

Sustituyendo los vectores de posicin obtenidos para cada partcula en la expresin que
nos da el cdm del sistema obtenemos finalmente:
3t 2
) 3(0, 0, 4)
1 mi ri m1 r1 m2 r2 m3 r3
n 1(3 2t 3
,0, 0) 2(0, 3,
rC 2
M m1 m2 m3 6

rC = (t3/3 + 1/2, 1, t2/2 + 2)

De qu otra forma podramos haber calculado el vector de posicin del centro de masas
en cualquier instante?

Otra estrategia para resolver el problema consiste en calcular en primer lugar la acelera-
cin del cdm, junto con su posicin inicial y despus mediante integracin obtener la ve-
locidad del cdm en cualquier instante y su vector de posicin:

aC
F ext i

FA FD

(12t, 0, 0) (0 0, 6)
= (2t, 0, 1)
M mA mB mD 6

Para poder integrar ser necesario conocer la velocidad y posicin del cdm en el instante 0 s:

m A rA0 m B rB0 rD0 1(3, 0, 0) 2 (0, 3, 0) 3(0, 0, 4)
rC0 = (05, 1, 2) m
mA mB mD 6

v CO = 0

dv
Integrando ahora en a C C obtenemos la velocidad del cdm en cualquier instante:
dt

v C = (t2, 0, t)

dr
Volviendo a integrar en: v C C , obtenemos el vector de posicin:
dt

rC = (t3/3 + 1/2, 1, t2/2 + 2)

Cmo podramos calcular ahora la cantidad de movimiento del sistema?


Sistemas de partculas 173

Una posibilidad es sumar las cantidades de movimiento de cada una de las partculas que
lo forman. En este caso al haber solo tres partculas este procedimiento no llevara mucho
tiempo, pero si el sistema tuviera muchas partculas los clculos seran muy largos. Otro
procedimiento alternativo, que se puede utilizar cuando conocemos la velocidad del cdm,

consiste en tener en cuenta que la cantidad de movimiento p de un sistema se puede cal-
cular siempre como la cantidad de movimiento de su cdm o ms propiamente, como la
cantidad de movimiento de una partcula de la misma masa que el sistema y que se mo-
viese con la velocidad del cdm, es decir:

p M v C en donde M es la masa total y v C la velocidad del centro de masas.

Sustituyendo pues los datos nos queda: p M v C = 6 (t2, 0, t) = (6t2, o, 6t)

Cmo podramos calcular la energa cintica del sistema? Dado que conocemos la ve-
locidad correspondiente a cada partcula respecto al sistema de referencia indicado en la
figura, se puede calcular la energa cintica del sistema como la suma de las energas
cinticas de las partculas:

1 m v 2 m 2 v 22 m 3 v 32 36t 4 18t 2
Ec m i v 2i 1 1 18t 4 9t 2
2 2 2

Si calculamos la energa cintica correspondiente al cdm, obtenemos un resultado diferente:

1
EcC = MvC2 = 3 (t4 + t2) = 3t4 + 3t2
2

Por qu no coincide la energa cintica del cdm con la energa cintica del sistema? En
general la energa cintica con que se traslada un sistema de partculas (en un determina-
do sistema de referencia), no coincide con la energa cintica correspondiente a su cdm.
tal y como viene reflejado en la expresin:

1 1
Ec MvC2 mi v'i2
2 2

En esta expresin el segundo sumando permite calcular la energa cintica del sistema

respecto al cdm siendo v 'i la velocidad con que cada partcula se mueve respecto de dicho
punto, de manera que si quisiramos utilizarla para calcular la Ec, primero tendramos
que calcular cada uno de los dos sumandos de que consta. Solo cuando las partculas del
sistema no se desplazan respecto del cdm, ocurre que la energa cintica del sistema coin-
cide con la que le correspondera al cdm de dicho sistema.
1
Obtened la energa cintica del sistema respecto al cdm mediante: 2m v'i
2
i y com-
probad que si la sumamos a la energa cintica del cdm, nos resulta la energa cintica
del sistema Ec anteriormente calculada.

Respecto al clculo del momento cintico L del sistema, podemos proceder de forma
similar a como hicimos con la energa cintica y obtener:
Sistemas de partculas 174


L ri mi vi r1 m1v1 r2 m2 v2 r3 m3v3 (18t, 0, 0)

Calculad el momento cintico del centro de masas y comprobad que no coincide con el
que acabamos de hallar. A qu se debe? En qu caso particular coincidiran?

En cuanto al momento de las fuerzas exteriores, ste se puede calcular como:



M ext ri Fext i r1 F1 r2 F2 . Sustituyendo se obtiene:

M ext = (3+2t3, 0, 0) x (12t, 0, 0) + (0, 3, 3t2/2) x (0, 0, 6) = (18, 0, 0) N m

dL
Si derivamos la expresin del momento cintico nos queda = (18, 0, 0) N m
dt
que, como vemos coincide con el valor del M ext

26. Sobre dos masas m1 =10 kg y m2 = 6 kg que Y F1 = 6 N


se encuentran en reposo en la situacin de la
figura, comienzan a actuar las fuerzas exterio-
res representadas. Se pide:

a) Posicin del c.d.m. en el instante inicial.


b) Posicin del c.d.m. en funcin del tiempo. F2 = 8
c) Momento cintico del sistema. N

d) Comprobad que se verifica que M ext dL / dt
X

sol: b) rC = (0187 t2 +15 , 025 t2 +187); c) L = (0, 0, 14t)

27. Una lancha de 1000 kg de masa, se desplaza por el agua con rapidez de 18 km/h
cuando dispara un proyectil de 2 kg en sentido contrario a su movimiento y con rapi-
dez de 300 m/s, respecto del agua. Calculad el cambio que se produce en la rapidez de
la lancha suponiendo despreciable el rozamiento. Qu variacin de energa cintica
sufre el sistema al producirse el disparo?

Durante el disparo se produce una interaccin entre el proyectil de masa m 1 y el resto de


la lancha (de masa m2 ). Si consideramos el conjunto como un sistema de dos partculas
(una de las cuales es el proyectil ), estas fuerzas sern interiores y, como consecuencia, la
cantidad de movimiento del sistema no se ver afectada por su actuacin y ser la misma

antes que inmediatamente despus del disparo, es decir: p p ' donde :
Sistemas de partculas 175


p p 1 p 2 m1 v 1 m 2 v 2

p p '1 p ' 2 m1 v '1 m 2 v' 2

Tomaremos como sistema de referencia el especificado en el esquema siguiente:

Y Y
v v2 ?
v1
m X m1 X
m2

Podemos ahora expresar los vectores velocidad en funcin de sus componentes escalares
de acuerdo con el sistema de referencia especificado.

Cantidad de movimiento del sistema en el momento que comienza el disparo:



p p 1 p 2 m1 v 1 m 2 v 2 m (v, 0)

Cantidad de movimiento del sistema justo despus del disparo:



p p'1 p'2 m1v'1 m2 v' 2 m2 (-v'1 , 0) m2 v2'

Como puede verse, en las ecuaciones anteriores hemos tenido en cuenta que antes del
disparo la velocidad de las dos partculas que hemos considerado que forman el siste-

ma, es la misma ( v ) y que se trata de un vector que, de acuerdo con el sistema de refe-
rencia escogido, solo tiene componente escalar segn el eje x y esta es positiva.

Determinar la velocidad que llevar la lancha justo despus del disparo.

Igualando las expresiones correspondientes a las cantidades de movimiento del sistema


en el momento del disparo y justo despus del mismo tenemos que:

m (v, 0) = m1 (-v1 , 0) + m2 v'2

Despejando, queda: v2' (mv m1 v'1 , 0)


m2


Y operando: v'2 = (561, 0) m/s, es decir, aumenta en 061 m/s.
Analizad la expresin literal obtenida.

En primer lugar, constatamos que es dimensionalmente homognea (L/T en ambos lados


de la igualdad). Adems podemos comprobar tambin que contempla algunos casos parti-
culares, como, por ejemplo, que si la masa del proyectil (o la velocidad con que sale) fue-
se nula, la velocidad del resto del sistema sera la misma que inicialmente, que cuanto
mayor sea la velocidad con que sale el proyectil, ms aumentar la velocidad del resto
despus del disparo y que cuanto mayor sea la masa de la lancha (sin el proyectil) menor
Sistemas de partculas 176

ser el aumento de velocidad que experimenta al disparar un proyectil (todo ello, como
siempre, a igualdad de los restantes factores).
Por otra parte, hemos de sealar que al tener lugar todo el movimiento en una sola direc-
cin, podramos evitar el tratamiento vectorial si manejamos, ya de entrada, nicamente
las componentes escalares de los vectores segn el eje considerado.

Calculad la variacin de energa cintica que experimenta el sistema.

La variacin de energa cintica producida se puede calcular como: Ec = Ec - Ec.

m1 v'12 m 2 v' 22
Ec = mv2/2 = 12.525 J; Ec = = 90.000 + 15.736 = 105.736 J
2 2

Restando los valores anteriores nos queda finalmente que Ec = Ec - Ec = 93.211 J

Cmo podemos explicar que la actuacin de las fuerzas interiores no aumente la canti-
dad de movimiento de un sistema y s en cambio su energa cintica?

Para contestar a la cuestin anterior, es necesario recordar que la energa cintica de un


sistema de partculas se puede expresar como:

mi vi2 mv C2
Ec E C'
2 2

en la que el primer sumando representa la energa cintica con que se traslada el sistema
(m es la masa total y vC la rapidez del cdm) y el segundo la energa con que se mueven las
partculas del sistema respecto del cdm (energa cintica interna). Las fuerzas interiores
no pueden modificar la velocidad con que se mueve el cdm pero s las de las partculas
del sistema y por tanto, podrn alterar la energa cintica interna y, en consecuencia Ec.

28. Una patinadora de 60 kg de masa y un patinador de 70 kg que se deslizan en la


misma direccin y sentidos contrarios con rapideces de 8 m/s y 10 m/s respectivamen-
te, chocan frontalmente permaneciendo unidos tras la colisin. Suponiendo el roza-
miento despreciable, determinad la rapidez con que se desplazarn y la variacin de
energa cintica que se habr producido.

sol: v = 169 m/s; Ec = -52344 J


Sistemas de partculas 177

29. Entre dos cuerpos de masas m1 = 2 kg y m2 = 8 kg, que se encuentran sobre un


plano horizontal y sin rozamiento, se sita un resorte de constante elstica K=20
N/m, apoyando sus extremos en los cuerpos. A continuacin, se empujan dichos cuer-
pos hasta conseguir comprimir el resorte 40 cm. Hallad la rapidez con que ser ex-
pulsado cada uno de los cuerpos al dejar el sistema en libertad.

m1 m2

En este caso tenemos un sistema formado por tres partes (los dos cuerpos y el resorte). No
obstante, como nos dicen que la masa del resorte se considera despreciable, podemos
interpretar la situacin como si el sistema estuviera formado solo por dos partculas 1 y 2
que entre ellas se ejercen fuerzas,
con lo que el resorte se limitara
a hacer el papel de
mero transmisor de la fuerza F21 que sobre 2 hace 1 y de la fuerza F12 que sobre 1 hace 2,
siendo ambas fuerzas una pareja de accin y reaccin. Junto con las fuerzas anteriores
actuaran adems el peso y la componente normal de la fuerza ejercida por la superficie
sobre cada uno de los cuerpos, sin embargo, como la componente normal de la acelera-
cin es nula, dichas fuerzas se anulan en cada uno de los cuerpos, quedando solo F21 y

F12 (siempre iguales y de sentido contrario), cuyo valor vendr dado en cada instante por
la compresin del resorte, es decir F12 = F21 = KL siendo K la constante elstica del
resorte e L su compresin (en valor absoluto) respecto a su longitud original L0.

Inicialmente la energa del sistema se puede determinar como la energa potencial elsti-
ca correspondiente a la compresin del muelle en ese instante. Al dejar el sistema en li-
bertad, el valor de dicha energa potencial disminuye y aumenta la energa cintica, dicha
energa cintica ir aumentando hasta que el muelle alcance su longitud L0. A partir de
ese momento el muelle deja de empujar a los cuerpos (la fuerza elstica cambia de senti-
do) y stos continan movindose con la velocidad alcanzada.

El problema consiste, pues, en calcular la rapidez con que cada cuerpo llega a la situa-
cin en la que el muelle recupera su longitud normal, que es cuando el resorte deja de
empujarles y estos se separan del mismo.

Para calcular la rapidez que se nos pide, podemos aplicar la expresin Wres = Ec, entre
los estados A (cuando dejamos el sistema en libertad) y B (cuando los cuerpos se separan
del resorte), con lo que:
Wres A Wext A Wint A
B B B

Como el trabajo realizado por las fuerzas exteriores es 0 (tanto el peso como la que ejerce
la superficie son perpendiculares al desplazamiento) el trabajo resultante coincidir con
el trabajo realizado por las fuerzas internas, que globalmente coincidir con la disminu-
cin de la energa potencial elstica que se produce cuando el sistema pasa de la situacin
A a la situacin B. Por tanto:

WresA Wint A Ec Ec B
B B
1 1 1
EpA Ec B KL m1v1B m2 v2B
2 2 2

Wint A Ep EpA
B
2 2 2
Sistemas de partculas 178

La expresin obtenida refleja el hecho de que, al no haber fuerzas no conservativas, la


energa potencial elstica inicial (estado A) se transforma ntegramente en energa cinti-
ca en el estado B. No obstante, al tratarse de una ecuacin con dos incgnitas hemos de
buscar una nueva relacin para poder calcular v 1 y v2 en el momento en que ambos
cuerpos dejan de tener contacto con el muelle.

Como la fuerza exterior resultante que acta sobre el sistema es 0, se deber conservar la

cantidad de movimiento del sistema entre los estados A y B, es decir: p A p B y, dado
que el movimiento tiene lugar en una sola direccin, podemos hacer un planteamiento
escalar utilizando directamente las componentes escalares de los vectores cantidad de
movimiento en esa direccin.

pA = pB 0 = m1 v1B + m2 v2B de manera que si juntamos esta ecuacin con la anterior,


nos queda un sistema de dos ecuaciones con dos incgnitas:

1 1 1
KL2 m1v1B 2
m2 v22B
2 2 2 Sustituyendo los valores numricos nos queda:
0 m1v1B m2 v2B

1 1 2 1 2
20 0' 42 2v1B 8v2B
2 2 2 Resolviendo obtenemos: v1B = -113 m/s y v2B = 028 m/s
0 2v1B 8v2B

Qu sucedera si solo existiera el cuerpo 2 y el resorte fuese de masa despreciable?

Al no existir el cuerpo 1 la interaccin se dara entre el cuerpo 2 y el resorte, pero si


la masa del resorte fuese nula, el cdm del sistema coincidira con la posicin de 2. Dado
que sobre el sistema no acta ninguna fuerza exterior resultante, la velocidad del cdm ha
mantenerse constante, por tanto, como el cuerpo de masa m 2 estaba inicialmente en repo-
so debe continuar en el mismo sitio, es decir, si quitamos uno de los cuerpos, el resorte no
puede ejercer ninguna accin sobre el otro. Se trata, naturalmente, de un resultado vlido
nicamente en el caso ideal expuesto (resorte de masa despreciable). Esto no es lo que
ocurre en la realidad en la que, por supuesto, cualquier resorte tendr una cierta masa,
pero realizar simplificaciones de este tipo para hacer los problemas ms fcilmente abor-
dables es una de las caractersticas del trabajo cientfico.

30. Un bloque de masa m1 = 2 kg, que se desliza con rapidez v1 =10 m/s a lo largo de
una superficie lisa choca frontalmente con otro bloque de masa m2 = 8 kg, que se en-
cuentra en reposo sobre la misma superficie. Sabiendo que el choque es perfectamen-
te elstico, determinad la rapidez de cada uno de los bloques despus del choque.

sol: a) v1 = - 6 m/s; v2 = 4 m/s (sentido positivo el del movimiento de m1 antes del choque).
Sistemas de partculas 179

31. Disponemos de dos pndulos, cuyas masas son


m1 = 2 kg y m2 = 3 kg, dispuestos como se indica
en la figura adjunta. Al dejar en libertad el
pndulo de masa m1, se producir un choque els-
tico con la masa m2 que cuelga verticalmente y se
encuentra en reposo. Determinad la rapidez de
cada una de las masas justo despus del choque.
Qu sucedera si las masas fueran iguales?

Al dejar en libertad la esfera de masa m 1, describir una trayectoria circular de radio igual
a la longitud del pndulo a lo largo de la cual su velocidad ir aumentando. Cuando al-
cance la vertical chocar frontalmente con la otra esfera de forma elstica. Si nos limita-
mos al intervalo de tiempo que dura el choque, podemos afirmar que las velocidades justo
al comienzo y al final del mismo solo tienen una componente en la direccin horizontal.

Cmo podemos saber la rapidez de cada bola despus del choque?

Si consideramos el sistema formado por ambas esferas en el breve intervalo de tiempo


que dura el choque entre ellas, se cumplir que la cantidad de movimiento ser la misma
en el momento del impacto que inmediatamente despus del mismo, de manera que apli-
cando un tratamiento escalar (tomando como sentido positivo hacia la derecha) podemos
escribir que:

p = p m1 v1 = m1 v1 + m2 v2

Por otra parte, al tratarse de un choque elstico, la energa cintica tambin debe ser igual:

1 1 1
Ec E' c m1 v 12 m1 v'12 m 2 v' 22
2 2 2

En ambas ecuaciones, hemos tenido en cuenta el hecho de que la esfera de masa m 2 se


encuentra inicialmente en reposo (v2 = 0).

Obtened las expresiones de v1 y v2 y analizad los resultados.

Las ecuaciones anteriores pueden escribirse como:

m1 (v1 - v 1) = m2 v2 (1)

m1 (v 12 v'12 ) m2 v' 22 (2)

Dividiendo (2) entre (1) obtenemos una tercera ecuacin: v1 + v1 = v2 (3)

(m1 m 2 ) v 1
Sustituyendo v2 en la expresin (1) nos queda: v'1
m1 m 2
Sistemas de partculas 180

2m1 v 1
Si ahora hacemos igual con v1 despejndola de (3) y sustituyendo en (1): v' 2
m1 m 2

Los resultados anteriores, son dimensionalmente homogneos (L/T en ambos lados de la


igualdad). Por otra parte si nos fijamos en el primero de ellos, podemos ver que si la masa m2
fuese nula, la rapidez de la esfera que choca, como es lgico, no cambiara. Tambin nos
muestran que si m2 fuese mucho mayor que m1 , sta retrocedera con la misma rapidez con
que choc, es decir v1 -v1 y la otra quedara prcticamente en reposo (v2 0).

Qu ocurrira en el caso de que las dos masas fuesen iguales?

En ese caso, analizando los resultados anteriores podemos comprobar que la esfera 1
quedara parada y la 2 saldra con la misma rapidez con que le impacto la 1. Este fen-
meno es muy conocido entre los aficionados al juego del billar en donde a veces se pro-
duce un choque frontal de una bola contra otra igual que se encuentra en reposo.

Para poder calcular la rapidez de cada una de las


esferas justo despus del choque necesitamos conocer
la rapidez v1 que lleva la 1 en el momento del impac-
to. Ello puede conseguirse si aplicamos la expresin
Wres = Ec a la esfera 1, desde el momento en que la
soltamos (situacin A) hasta el instante en que impac- A
ta contra la otra esfera (situacin B). 1
1 B

Wres A EcAB WP A EcAB EpAB EcAB


B B

De acuerdo con la ecuacin anterior, la nica fuerza que realiza trabajo entre A y B es el
peso de la bola de masa m1 (la tensin del hilo es perpendicular a la trayectoria descrita) y
la disminucin experimentada por la energa potencial del sistema conlleva un aumento
igual de energa cintica, de modo que basta sustituir por las correspondientes expresio-
nes de la energa para obtener v1 :

1
mgh = mv 12 v 1 2gh = 10 m/s
2

Como hemos especificado hacia la derecha positivo, escogeremos v 1 = 10 m/s (positivo).


Sustituyendo este valor en las expresiones anteriores podemos obtener finalmente las ra-
pideces de las esferas justo despus del choque:

(m1 m 2 ) v 1 2m1 v 1
v'1 = -2 m/s . v' 2 = 8 m/s
m1 m 2 m1 m 2

El signo menos de v1 nos informa, de acuerdo con el sistema de referencia especificado


anteriormente, de que la esfera 1 retrocede a consecuencia de su choque con la 2, que sale
despedida hacia la derecha con una rapidez de 8 m/s.
Sistemas de partculas 181

32. Una molcula de gas que lleva una rapidez v A = 300 m/s choca elsticamente con
otra molcula de la misma masa inicialmente en reposo (v B = 0). Tras el choque, la
primera se mueve formando un ngulo de 30 con su direccin inicial. Calculad la
rapidez con que se mover cada molcula tras la colisin.

Consideraremos el sistema formado por las dos molculas y lo que le ocurre en el corto
intervalo de tiempo que corresponde al choque entre ambas. De acuerdo con las condi-
ciones que se especifican en el enunciado, se trata de un choque elstico entre dos mol-
culas de un gas, por tanto la energa cintica del sistema no variar. Lo mismo podremos
decir respecto a la cantidad de movimiento del sistema que ser la misma al comienzo
que al final del choque.

En el problema se nos pide la rapidez de cada molcula despus de la colisin. Podemos


tratar de calcularlas haciendo uso de la conservacin de la cantidad de movimiento y de

la energa cintica, es decir, desarrollando las ecuaciones p p ' y Ec = Ec, donde p y
Ec representan la cantidad de movimiento y la energa cintica del sistema en el momen-

to en que se inicia el choque, y p ' y Ec lo mismo pero justo despus del choque.

Y Y VA
VA 30

B X X
VB?


Desarrollando p p ' tenemos que :

mA v A mA v' A mB v' B . Expresando ahora los vectores velocidad analticamente:

mA (vA , 0) mA (v'Ax , v'Ay ) mB (v'Bx , v'By ) .

Podemos ahora descomponer esta ecuacin en otras dos escalares, con lo que nos queda:

En el eje X: mA vA = mA vAx + mB vBx

En el eje Y: 0 = mA vAy + mB vBy

1 1 1
Desarrollando Ec = Ec tenemos que: m A v 2A m A v' 2A m B v' 2B
2 2 2

Como en este caso las masas son iguales, las ecuaciones anteriores se simplifican:

vA = vAx + vBx
0 = vAy + vBy
v 2A v' 2A v' 2B
Sistemas de partculas 182

Cmo podemos reducir el nmero de incgnitas presentes en las ecuaciones anteriores?

Podemos tener en cuenta que vAx = vA cos 30 y que, anlogamente vAy = vA cos 60.
Por otra parte sabemos que v'2B se puede expresar en funcin de sus componentes como:
v' 2B v' 2B x v' 2B y . Incorporando estas expresiones a las ecuaciones anteriores nos queda
finalmente un sistema de tres ecuaciones con tres incgnitas, que ya podemos resolver:

300 v' A cos 30 v' B x



0 v' A cos 60 v' B y

300 2 v' 2A v' 2B x v' 2B y

De la primera de las ecuaciones obtenemos vBx = 300 - vA cos30


De la segunda de las ecuaciones obtenemos vBy = - vA cos60

Sustituyendo en la tercera ecuacin y operando, hallamos que: v A = 260 m/s y sustitu-


yendo este dato en la primera y segunda expresiones anteriores obtenemos que v Bx =
748 m/s y que vBy = -130 m/s respectivamente, lo que nos indica que la molcula blanco
-
a consecuencia del impacto sale por el 4 cuadrante (XOY ), con una rapidez VB = 149
m/s.

Hallad la direccin en que se mueve la molcula blanco.



Para ello podemos calcular los ngulos directores del vector v' B

+
cos = vBx /vB = 748/149 = 05, con lo que forma un ngulo de = 60 con OX
+
cos = vBy /vB = -130/149 = -087, con lo que forma un ngulo = 150 con OY

33. Sobre un cuerpo de masa m1 = 8 kg inicialmente en reposo, interacciona "direc-


tamente" un segundo cuerpo de masa m2 = 2 kg que se desplaza con una rapidez v2 =
10 m/s. Sabiendo que tras la interaccin el primer cuerpo se mueve con una rapidez
v1 = 3 m/s, determinad la rapidez v2 del segundo justo despus del choque y la va-
riacin de energa cintica experimentada por el sistema (formado por ambos cuer-
pos) en la interaccin.

sol: v2 = -2m/s; Ec= - 60 J.

34. Una partcula de 0'2 kg de masa, que se desplaza a 0'4 m/s, choca con otra de 0'3 kg
que se encuentra en reposo. Despus de la colisin, la primera se mueve a 0'2 m/s en una
direccin y sentido que forma un ngulo de 40 con la inicial. Obtened la velocidad de la
segunda partcula y la variacin de energa cintica producida en la colisin.

sol: v ' B (0' 17, 0' 07) m/s; Ec = - 00071 J
Sistemas de partculas 183

35. Se dispara horizontalmente una bala de masa m1 =15 g, sobre un bloque de made-
ra de masa m2 = 3 kg que est suspendido de unos hilos. La bala queda incrustada y
el conjunto oscila hasta alcanzar una altura de 10 cm por encima de su posicin ini-
cial. Determinad la rapidez v1 con que incide la bala en el bloque.

Este problema hace referencia a un pndulo balstico que consiste en un procedimiento


para medir la velocidad de las balas. Un pndulo de este tipo suele consistir en un gran
bloque de madera suspendido de dos cuerdas, como se indica en la figura:


La bala realiza un choque directo contra el bloque con una velocidad v1 quedando empo-
trada en l. A consecuencia del choque, el conjunto formado por la bala y el bloque oscila
ascendiendo hasta una cierta altura h respecto al nivel inicial.

Cmo podramos calcular la rapidez con que incide la bala sobre el bloque?

Si consideramos el sistema formado por la bala y el bloque, podemos admitir que la cantidad de
movimiento segn la horizontal es la misma al comienzo del choque que justo al finalizar el mis-
mo, con lo que (tomando como sentido positivo el del movimiento de la bala) podemos escribir:

m1 v1 = (m1 + m2) v

siendo v la rapidez con que sale el conjunto bloque-bala despus del choque. Dicha rapidez
es, en principio desconocida, por lo que no podremos hallar v1 mientras no la determinemos.

Cmo podemos hallar la rapidez v del conjunto bala-bloque justo despus del choque?

Considerando el conjunto formado por el bloque y la bala,


podemos hallar el trabajo resultante sobre el mismo desde
justo despus del impacto (situacin A) hasta que alcanza la
altura mxima h (situacin B). A lo largo de ese trayecto, B
actan la fuerza peso y la tensin del hilo. No obstante,
como la tensin es en todo momento perpendicular a la
trayectoria, no realiza ningn trabajo y podemos expresar el h
v
trabajo resultante como: A
Sistemas de partculas 184

Wres A WP A EcAB . Como la fuerza peso es conservativa WP = -Ep, de modo que:


B B

1
EcAB EpAB o lo que es lo mismo: (m1 m2 )v'2 = (m1 + m2 ) g h
2

es decir, toda la energa cintica con que sale el conjunto bloque-bala, se halla en forma de
energa potencial gravitatoria en el instante en que alcanza la mxima altura h (en el cual la
rapidez es momentneamente nula). De la ecuacin anterior podemos obtener v como:

v' 2gh

Finalmente, sustituyendo esta expresin en la ecuacin que expresa la conservacin de la


cantidad de movimiento y despejando v1 nos queda que:

m1 m 2
v1 2gh v1 = 2842 m/s
m1

Analizando el resultado literal obtenido, vemos que adems de ser dimensionalmente


homogneo, contempla algunos casos evidentes como, por ejemplo, que cuanto mayor
haya resultado ser la altura mxima alcanzada, mayor ser la rapidez con que incidi la
bala y que para una rapidez v1 dada, cuanto mayor sea la masa m2 del bloque, menor altu-
ra mxima alcanzar el conjunto, etc.

Cunta energa cintica se pierde a causa del choque?

Para calcular la variacin de energa cintica que sufre el sistema formado por el bloque y
la bala a consecuencia del choque, basta con restar la energa cintica inicial de la bala a
la energa cintica del sistema inmediatamente despus del choque, con lo que:

1 1
Ec = (m1 m 2 )v' 2 - m1 v 12 = 3015 - 6062 = - 60319 J
2 2

Naturalmente dicha energa no ha desaparecido sino que se halla ahora como energa in-
terna asociada a las partculas submicroscpicas que forman la bala y el bloque. Conviene
resaltar el gran porcentaje de la energa cintica inicial del sistema que sufre esta trans-
formacin (del orden del 995 %) o lo que es lo mismo: solo el 05 % de la energa cin-
tica con que la bala incide en el bloque, se convierte en energa potencial gravitatoria del
conjunto formado por el bloque y la bala cuando este alcanza la mxima altura h. Este
resultado permite comprender lo incorrecto que sera resolver este problema igualando la
energa cintica de la bala a la potencial gravitatoria del conjunto.
Sistemas de partculas 185

36. Dos vehculos 1 (de 600 kg de masa) y 2 (de 800 kg de masa) chocan cuando se
desplazaban a 20 m/s y 10 m/s respectivamente tal y como se indica en la figura. Sa-
biendo que tras el choque permanecen unidos, obtened su velocidad tras la colisin,
as como la variacin de energa cintica experimentada.


sol: v' = (85 , 57) m/s; Ec= - 86682 J.

37. El cuerpo de masa m1 = 4 kg es lanzado al dejar en libertad el resorte que se en-


contraba comprimido 60 cm. Si no existe ningn tipo de rozamiento y choca de forma
elstica y directa con un segundo cuerpo de masa m2 = 2 kg, que se encuentra en re-
poso. Determinad la constante elstica del resorte sabiendo que dicho segundo cuerpo
asciende por el plano inclinado hasta una altura de 5 m.

En la situacin inicial (A), el resorte se encuentra comprimido al mximo y la energa


Kx 2A
potencial elstica del sistema viene dada por: Ep A = en donde K es la constante
2
elstica del muelle y xA lo que se ha comprimido respecto a su longitud original (60 cm),
de manera que si pudisemos evaluar dicha energa potencial, podramos determinar el
valor de K. Cmo podramos hallar la energa potencial elstica en la situacin A?

Dado que no hay rozamiento, dicha energa, cuando se deje el sistema en libertad, ir
disminuyendo conforme el muelle se vaya alargando, hasta que en el momento en que
ste recupere su longitud original valdr 0 (x = 0) y se habr transformado toda ella en
energa cintica, dada por :
m v2
Ec = 1 1
2

A partir de que el muelle recupera su longitud original la fuerza elstica deja de actuar
sobre m1, que sigue movindose con velocidad constante v1 hasta que choca con m2.
Sistemas de partculas 186

En efecto, si aplicamos la expresin que nos relaciona el trabajo resultante con la varia-
cin de la energa cintica desde la situacin inicial A hasta la situacin B, correspon-
diente al instante en que se produce el choque tendremos, que como la nica fuerza que
realiza trabajo mientras que acta sobre m 1 es la fuerza elstica, podemos escribir:
1
WresAB EcAB WFe EcAB Ep EcAB EpA m1v12
2
Como vemos no podemos calcular la energa potencial elstica correspondiente a la si-
tuacin inicial si no conocemos la rapidez v1 con que m1 impacta contra m2 . Cmo
podramos determinar dicha rapidez?

Como entre m1 y m2 se produce un choque elstico y directo, la cantidad de movimiento


y la energa cintica se conservan y podremos emplear un tratamiento escalar (arbitraria-
mente, tomaremos como sentido positivo el del movimiento inicial de m 1).

Como el segundo cuerpo se halla inicialmente en reposo (v2 = 0) se cumplir que:

p = p m1 v1 = m1 v1 + m2 v2
1 1 1
Ec E' c m1 v 12 m1 v'12 m 2 v' 22
2 2 2
Tal y como ya hemos visto en un ejercicio anterior (ved problema 31) de las ecuaciones
anteriores se deduce que:

(m1 m 2 ) v 1 2m1 v 1
v'1 y v' 2
m1 m 2 m1 m 2

Sin embargo con las expresiones anteriores no podemos averiguar v1 ya que solo tenemos
dos ecuaciones y 3 incgnitas (v1 , v1 y v2). Para conocer v1 tendramos que saber v1 o
bien v2 , de esa forma podramos despejar v1 de alguna de las dos ecuaciones. Cmo
podramos conocer la rapidez v2 con que la masa m2 sale al finalizar el choque?

Podemos aplicar la expresin que relaciona el trabajo resultante con la variacin de


energa cintica, entre los estados C (cuando el cuerpo de masa m 2 llega a la base del pla-
no inclinado) y D (cuando alcanza la mxima altura). Como en dicho trayecto la nica
fuerza que realiza trabajo es el peso y sta es conservativa, tendremos:

WresCD EcCD WP A EcCD EpCD EcCD - (EpD - EpC ) = (EcD - EcC )


B

donde Ep se refiere a la energa potencial gravitatoria. Si tomamos la superficie horizontal


como nivel 0 para dicha energa potencial, llamamos h a la altura mxima alcanzada por m2
(situacin D) podemos desarrollar la ecuacin anterior como: EcC = EpD que nos dice que
toda la energa cintica con que m2 llega a la base del plano inclinado, se ha transformado en
energa potencial gravitatoria en el instante en que alcanza la mxima altura sobre el mismo.

1
As pues: m 2 v' 2 m 2 gh
2
Sistemas de partculas 187

Proceded a obtener ahora la expresin que nos da el valor de la Energa potencial els-
tica en la situacin inicial A.

Despejando v2 de la ecuacin anterior obtenemos v' 2 2 gh = 10 m/s. Dado que sale


2m1 v 1
en el sentido escogido como positivo, ser v'2 = 10 m/s. Sustituyendo en v' 2
m1 m 2
podremos hallar la rapidez v1 que llevaba m1 al comienzo del mismo:

(m1 m 2 ) v' 2 (m1 m 2 )


v1 2gh = 75 m/s. Introduciendo esta expresin en la
2m1 2m1
1 1 1 m1 v 12
ecuacin inicial Ep A m1 v 1 nos queda: Kx m1 v 1 K
2 2 2
= 625 N/m
2 2 2 x2

38. Un proyectil de masa m1 = 40 g se lanza contra un cuerpo de masa m2 = 360 g que


se encuentra en reposo y en el que se incrusta. A continuacin el conjunto se desplaza
sobre una superficie horizontal en la que choca de forma elstica y directa con un
tercer cuerpo de masa m3 = 600 g que se haba soltado sobre un plano inclinado de
300 y desde 5 m de altura y que se halla tambin movindose sobre la misma superfi-
cie. Suponiendo rozamiento nulo, determinad:

a) Rapidez con la que impact el proyectil sabiendo que tras la colisin el cuerpo que
se solt en el plano inclinado retrocede con rapidez de 2 m/s.
b) Qu variacin sufre la energa cintica del sistema en cada una de las colisiones?

sol: a) 50 m/s; b) - 45 J y 0

39. Un tomo de hidrgeno movindose a 384 m/s choca contra un tomo de yodo que
se mueve a 128 m/s perpendicularmente respecto al primero. A causa del choque
ambos quedan unidos formando una molcula que se mueve en una direccin que
forma un ngulo de 767 con la direccin inicial del tomo de hidrgeno. Calculad
cuntas veces es mayor la masa del tomo de yodo que la del de hidrgeno.

Se trata de una colisin entre dos partculas que podemos considerar como masas puntua-
les. De acuerdo con lo que sabemos, la cantidad de movimiento del sistema formado por
ambas en el momento del choque ser la misma que justo despus del mismo. Para resol-
Sistemas de partculas 188

ver el problema escogeremos un sistema de coordenadas adecuado en el que representa-


remos el choque, aplicaremos el principio de conservacin y, a partir de la ecuacin resul-
tante, trataremos de obtener la relacin entre las masas que se nos pide.

Y Y
v
m

m1 X

m2 X

En la figura anterior hemos representado el sistema antes del choque y justo despus del
mismo, designando por m1 la masa del hidrgeno, m2 la masa del tomo de yodo y m la
masa de la molcula resultante. Aplicando el principio de conservacin de la cantidad de
movimiento:

p p' m1 v1 m2 v2 m v ' m1 (v1 , 0) + m2 (0 , v2) = m (vcos , vsen)

En la ecuacin anterior, v1, v2 y v representan mdulos de velocidades. Podemos


descomponerla en dos ecuaciones escalares (equivalente a aplicar la conservacin de la
cantidad de movimiento segn cada uno de los ejes):

En el eje X: m1 v1 = m vcos
En el eje Y: m2 v2 = m vsen
m2 v2
Dividiendo ahora la segunda ecuacin por la primera obtenemos: tg
m1 v1
v1
Despejando, nos queda: m2 tg m1 y sustituyendo resulta: m2 = 1269 m1
v2

As pues, la masa del tomo de yodo es 1269 veces la del de hidrgeno.

40. Los cuerpos de la figura tienen 500 g de


masa cada uno y los hilos son de la misma
longitud y masa despreciable. Si soltamos el
cuerpo 1 desde la posicin indicada en la
figura, determinad la altura que alcanzarn
sabiendo que permanecen unidos. Cul
ser el trabajo interior realizado durante la
colisin?

sol: 20 cm; -2 J
6. SLIDO RGIDO

1. Analizad la siguiente proposicin argumentando si se est o no de acuerdo con lo


que en ella se afirma:

Cualquier fuerza exterior aplicada a un slido rgido ligado a un eje fijo, se convier-
te siempre en un par de fuerzas ya que, por el hecho de estar el eje fijo, sobre los pun-
tos del cuerpo en contacto con l aparece una fuerza igual y de sentido contrario ejer-
cida por el eje

Si la proposicin anterior fuese vlida para todos los casos, tendra que ocurrir que siem-
pre que se ejerciese una fuerza en las condiciones que se indican en dicha proposicin, el
centro de masas (cdm) del slido no cambiase de velocidad, ya que el slido rgido puede
considerarse como un sistema de partculas con lo que la aceleracin del centro de masas
viene dada por:


aC
F ext i
=
Fext
m m

y si la proposicin
fuese cierta la fuerza exterior total sobre el sistema debera de ser nula

Fext = F (F) = 0, de modo que la aceleracin del centro de masas a C tambin tendra
que ser nula y, por tanto, si dicho centro de masas estaba en reposo, debera de continuar
estndolo.

Es eso lo que sucede siempre?

Al actuar una fuerza exterior, aplicada sobre un cuerpo slido rgido ligado a un eje de
rotacin, intentar desplazar al cuerpo, pero como ste se encuentra atravesado por un
eje que se mantiene fijo, dicho eje har sobre el cuerpo la fuerza necesaria para
seguir
mantenindose fijo. As pues, sobre el cuerpo

siempre actuarn dos fuerzas: F o fuerza
exterior que se le aplica directamente y Fe o fuerza ejercida por el eje sobre el cuerpo.

Sometido a la accin de F y de Fe el nico movimiento posible del cuerpo (mientras el
eje sea fijo) ser girar alrededor del eje y podrn darse dos casos: que el eje pase por el
centro de masas o que no pase.

Qu le ocurrira a la velocidad del centro de masas en cada uno de esos dos casos?

a) En el caso de que el eje fijo pase por el centro de masas, dicho punto deber permane-
cer en reposo (puesto que se encuentra sobre el propio eje) y la aceleracin del centro de
masas, al no cambiar su velocidad, ser 0. Ello obliga a que la fuerza exterior resultante

que acte sobre el slido sea nula y por tanto a que Fe = - F . Cuando esto sucede, se dice
que las dos fuerzas forman un par de fuerzas".
Slido rgido 190

b) En caso de que el eje fijo no pase por el cdm del cuerpo, dicho punto girar alrededor
del eje, con lo que su velocidad cambiar y estar sometido a una aceleracin, lo que obli-

ga a que la resultante de las fuerzas exteriores no sea nula y, por tanto, a que Fe - F , de
modo que dichas fuerzas ya no se pueden considerar como un par.

As pues, hemos de concluir que la proposicin solamente es cierta en el caso de que el


eje que atraviesa al cuerpo pase por el centro de masas. Ello no invalida, por supuesto, el
principio de accin y reaccin, ya que la fuerza que hace el eje sobre el cuerpo s que ser
igual y de sentido contrario a la que hace el cuerpo sobre el eje, pero sta ltima no debe
confundirse con la fuerza que un agente exterior aplica sobre el cuerpo.
Slido rgido 191

2. De qu depende el efecto de giro (aceleracin angular) que una fuerza puede


provocar a un cuerpo capaz de girar libremente alrededor de un eje fijo?

Para concretar ms la cuestin consideremos el ejemplo de abrir o cerrar una puerta. En


ese caso, es conocido que el efecto de giro o aceleracin angular que experimenta la
puerta, no solo depende del valor de la fuerza exterior que se aplique sobre ella, sino tam-
bin de otros factores.

Cules son esos otros factores?

Cabe pensar que, adems del valor de la fuerza ejercida, influya tambin la distancia des-
de el punto de aplicacin de la misma al eje de giro y la direccin en la que acte, es de-
cir: = (F, r, ) siendo F el mdulo de la fuerza ejercida, r la distancia de su punto de

aplicacin al eje y el ngulo que forma el vector fuerza con el vector r (ved figura).

Cmo influir cada uno de los factores anteriores?

Consideremos el caso de una puerta a la que apliquemos la misma fuerza pero a diferentes
distancias del eje de giro. Podemos esperar que la puerta cierre o abra tanto ms fcilmen-
te (la aceleracin angular sea mayor) cuanto mayor sea la distancia r existente entre el
punto de aplicacin y el eje de giro y tambin cuanto mayor sea el valor F de la fuerza
aplicada.


Slido rgido 192

No obstante, es necesario matizar la apreciacin anterior ya que el efecto de giro que se


puede conseguir con fuerzas de igual valor y aplicadas en el mismo punto pero que se
ejercen en direcciones distintas, puede ser muy diferente.

En el esquema anterior cabe suponerque, a igualdad de los restantes factores, la acelera-



cin angular que se conseguir con F1 ( = 90) ser mxima, mientras que con F2 ser

menor y con F3 ( = 0 ) ser nula (tambin lo sera si valiese 180). Esta dependencia
lleva a pensar que la aceleracin angular depender del ngulo mediante la funcin sen,
ya que el valor de dicha funcin va aumentando desde 0 (para = 0) hasta su valor mxi-
mo 1 (para = 90), para luego ir disminuyendo hasta valer 0 de nuevo cuando = 180.

La validez de las hiptesis anteriores se puede comprobar experimentalmente (basta ma-


nipular una puerta de forma adecuada) y se pueden generalizar para cualquier slido rgi-
do que gire en torno a un eje de rotacin.

Convendr definir una nueva magnitud fsica como funcin de F, r, y sen y que ser de
quien dependa el efecto de giro de la fuerza aplicada a un slido rgido

capaz de girar li-
bremente alrededor de un eje fijo. Esa magnitud es el momento M de dicha fuerza res-
pecto al punto O, definido como:

M r F

y, como vamos a ver, contempla todas


las consideraciones anteriores. En efecto, si anali-
zamos dicha expresin vemos que M se trata de un vector siempre perpendicular al plano

de giro (que contiene a r y a F ) y que solo puede tener dos sentidos (uno para cada senti-
do de giro). Por otra parte, su mdulo viene dado por:

M = r F sen

Comprobad que la expresin anterior se hallan implcitas todas las hiptesis que hicimos
anteriormente.
Slido rgido 193

Analizando la expresin podemos ver, por ejemplo, que si r o F son nulas, M = 0; si r o F


aumentan, M aumenta; si vale 90, M toma su valor mximo; si vale 0 o 180, M es
nulo, etc. En definitiva, pues, podemos considerar que si en el caso de la traslacin la ace-
leracin producida a un cuerpo dado depende de la fuerza resultante que acte sobre l, en
el caso de la rotacin alrededor de un eje, la aceleracin angular depender del momento
resultante que acte sobre dicho cuerpo, de forma que cuanto mayor sea M, mayor ser el
valor de dicha aceleracin.

Ello viene expresado en la ecuacin M I (ecuacin fundamental de la dinmica de la
rotacin) en la que I es una constante de proporcionalidad caracterstica del cuerpo res-
pecto de un eje de giro dado. Se denomina momento de inercia y tiene un significado an-
logo a la masa inerte m que ya conocemos (en este caso mide la inercia del cuerpo a la
rotacin en torno a un eje dado, de forma que cuanto mayor sea I se precisar un momento
M mayor para conseguir una determinada aceleracin angular).

En adelante, pues, cuando una fuerza resultante acte sobre un slido rgido, ser el mo-
mento de dicha fuerza respecto del eje de giro la magnitud que nos interesar considerar
para evaluar la aceleracin angular con que gira.

3. Obtened una expresin vlida para calcular el momento resultante de un par de


fuerzas, respecto de un punto cualquiera, en funcin del valor de las fuerzas y de la
distancia existente entre sus lneas de accin.

Al tratarse de dos fuerzas iguales y de sentidos contrarios, la suma de ambas ser 0, lo que
implica que el centro de masas del cuerpo no sufrir aceleracin (permanecer en reposo
o con movimiento rectilneo y uniforme, respecto a un cierto sistema de referencia). Por
tanto, el nico efecto que puede provocar un par es una aceleracin angular que, como
sabemos, depender del momento resultante.


M O M1 M 2 r1 F r2 (F) (r1 r2 ) F

M O , ser un vector, perpendicular al plano del par de fuerzas y de mdulo:


M O r1 r2 F sen F d
Slido rgido 194

Como podemos ver, el momento resultante no depende de la situacin del par respecto del
eje de giro sino que nicamente depender del mdulo de una de las fuerzas (las dos tie-
nen el mismo) y de la distancia existente entre sus rectas de aplicacin (o lneas de ac-
cin), tambin llamada brazo del par.

4. Cul es la condicin para que un cuerpo extenso e indeformable se encuentre en


reposo o movindose con movimiento de traslacin rectilneo y uniforme?
En dinmica del punto se vio que para que un cuerpo, que pueda considerarse como una
masa puntual, se encuentre en reposo o con movimiento rectilneo y uniforme de trasla-
cin (es decir, para que su vector velocidad no cambie), se requera que la resultante de
todas las fuerzas que pudieran estar actuando sobre dicho cuerpo fuese nula:

Fres m a si Fres 0 a 0 v constante

Cuando se trate de un objeto que tengamos que considerar como extenso, podemos esta-
blecer de forma anloga que para que dicho objeto se encuentre en reposo o en movimien-
to rectilneo y uniforme (siempre respecto a un cierto sistema de referencia), ha de ocurrir

que todos sus puntos tengan velocidad constante (considerando v 0 como un caso parti-
cular de velocidad constante) y, por tanto, que el vector aceleracin de cualquiera de sus
puntos, sea 0.
Trataremos de relacionar ahora este hecho, con las magnitudes angulares. Si el vector ace-
leracin de cualquiera de los puntos de que consta el cuerpo ha de ser cero, cuando nos
fijemos en un punto dado del slido:

Cmo deber ser la rapidez angular del cuerpo respecto a dicho punto?
Es evidente que la rapidez angular del cuerpo (tengamos en cuenta que en un slido rgido
que gira todos los puntos lo hacen con la misma rapidez angular) deber de permanecer
nula, lo que implica que la aceleracin angular tambin lo ser (sin cambio de velocidad

no hay aceleracin), es decir: w = 0 y = 0. Si no fuera as, los puntos del slido que gi-
rasen respecto al punto considerado estaran cambiando su vector velocidad (al menos en
direccin) y, por tanto su aceleracin no podra ser nula, tal y como se ha exigido.
En principio, cualquier punto del slido se puede tomar como referencia para estudiar su
movimiento. No obstante, nos interesa escoger al centro de masas, ya que su movimiento
de traslacin est regido por Fext y puede ser conocido de antemano.

Tratad de plasmar las conclusiones anteriores tomando como punto de referencia el cen-
tro de masas de un slido rgido.
Si nos fijamos en el centro de masas deber cumplirse que, para que el vector aceleracin
correspondiente a dicho punto sea nulo, la fuerza exterior resultante sea tambin nula ya
que, como sabemos,
solo una fuerza exterior resultante puede modificar la velocidad del

centro de masas ( Fext m a C ). Adems, el momento exterior resultante respecto de dicho
punto (centro de masas ) tambin tendr que ser 0, ya que si la aceleracin
angular es 0,
no puede haber momento resultante, puesto que como sabemos M res M ext I (la
suma de los momentos de las fuerzas interiores en un sistema de partculas es nula).
Slido rgido 195

Cmo podemos expresar las conclusiones anteriores en el caso de que el slido rgido
considerado se encuentre atravesado por un eje fijo?

Es claro que si el cuerpo est atravesado por un eje fijo (pase o no por su centro de ma-
sas), no podr tener un movimiento de traslacin rectilneo y uniforme, sino tan solo de
rotacin o encontrarse en reposo. En este caso, la condicin para que se encuentre en re-
poso ser, obviamente, que su rapidez angular inicial sea nula y que el momento exterior
resultante respecto de dicho eje, sea tambin nulo:

w 0 0 y M ext 0

En estas condiciones se halla implcito que la fuerza exterior resultante sea nula ya que si
no lo fuese, el centro de masas tendra que cambiar de velocidad (en este caso girar, lo

que obligara a que existiera ) con lo que el momento exterior no sera 0. El que la rapi-
dez angular inicial

sea nula es necesario especificarlo ya que, de lo contrario podra darse
el caso de que M ext 0 pero que el cuerpo estuviese ya girando con rapidez angular cons-
tante.

Si, por el contrario, el cuerpo no se encuentra atravesado por ningn eje fijo, tendremos
que considerar que, para que se encuentre en reposo o con movimiento de traslacin rec-
tilneo y uniforme, es necesario que se cumpla:

Fext 0 ; M ext 0 y w 0 0

Mediante este ejercicio hemos podido comprobar cmo la misma ecuacin que nos sirvi
para estudiar los cambios de movimiento en el caso de una masa puntual, se puede utilizar
para derivar a partir de ella las ecuaciones necesarias para estudiar los posibles cambio de
movimiento de un slido rgido, mostrando as la coherencia y globalidad de las leyes de
la mecnica.

5. En las figuras siguientes se ha representado un volante en diversas situaciones. Ex-


plicad con el mayor detalle posible el movimiento de ese volante en cada caso, utili-
zando para ello las ecuaciones fundamentales
de la dinmica de la traslacin y de la

rotacin para un slido rgido ( Fext m a C y Mext I ).

a) Eje fijo que pasa b) Eje fijo que pasa c) Ningn eje y d) Eje fijo que no e) No hay ningn
por cdm. F es la por el cdm. Par de sometido a un par pasa por el cdm. F es eje y F es la fuerza
fuerza aplicada. fuerzas. de fuerzas. la fuerza aplicada aplicada.
Slido rgido 196

Para resolver este ejercicio hemos de analizar qu fuerza exterior resultante acta sobre el
centro de masas y cul es el momento resultante respecto del eje de giro en cada caso.

a) Sobre el volante actan dos fuerzas:
la fuerza F aplicada a su
periferia y la que hace el eje fijo Fe . Como el cdm (situado en el
centro del volante) est en el eje, no se mover y su aceleracin

ser a C = 0. Por tanto, la fuerza exterior resultante Fext = F + Fe

= m a C = 0, es decir: sobre el volante actuar, en este caso, un par
de fuerzas.

El movimiento de rotacin que sufrir el volante, se explica, precisamente, por el momen-


to resultante debido al par de fuerzas respecto al eje de giro: M = r F, siendo r el radio
del volante. Dicho momento es responsable de la aceleracin angular con que girar el
volante respecto al eje.

b) Sobre el volante, atravesado por el mismo eje fijo que en el caso



anterior,
acta
un par
de fuerzas, de modo que la fuerza exterior resultante ser nula: Fext = F + (- F ) = 0 y el
cdm seguir estando en reposo. El volante girar alrededor del eje debido al momento del
par de fuerzas, cuyo valor vendr dado por M = 2r F (doble que en el caso anterior).
Ntese que el eje no ejerce ninguna accin y si no estuviese todo sucedera igual.

c) En este caso podemos hacer las mismas consideraciones que en el anterior, ya que, tan-
to la fuerza exterior resultante sobre el sistema, como el momento del par de fuerzas res-
pecto del cdm, sern los mismos, pero ahora el eje de giro no es fijo y el volante adems
de girar podra estar trasladndose con movimiento rectilneo y uniforme.

d) El volante no puede trasladarse pero s girar alrededor del eje que en este caso no pasa

por el centro de masas. Como el cdm tambin gira alrededor del eje, su aceleracin a C no

es nula, por lo tanto la fuerza exterior resultante (suma de la fuerza F aplicada y la Fe que
hace el eje sobre el volante) tampoco lo ser.

El valor del momento resultante respecto del eje de giro vendr dado por M = M F ya que
MFe = 0 puesto que Fe est aplicada al eje de giro. Dicho momento es responsable de la
aceleracin angular con que girar el volante alrededor del eje indicado, tal y como se
muestra en la figura adjunta (en ella no hemos incluido Fe ).
Slido rgido 197

e) En este caso el volante no est sujeto por ningn eje fijo y la fuerza exterior sobre el
volante coincide con la fuerza
aplicada F por lo que el cdm se traslada con una cierta

aceleracin dada por a C = F /m (siendo m la masa del volante).

Adems el momento exterior respecto del cdm vale M = rF, lo que implica que el volante
tambin girar respecto al cdm con una cierta aceleracin angular, de modo que en este
caso se producir un movimiento combinado de traslacin y de giro.

6. Determinad el momento de inercia de una varilla de masa m y longitud L:

a) Respecto de un eje perpendicular por su centro.


b) Respecto de un eje perpendicular por su extremo.

Sabemos que el momento de inercia I de una masa que pueda considerarse como puntual,
respecto de un eje del que se encuentra a una distancia r, viene dado por la expresin I =
mr2. Sin embargo en este ejercicio la varilla no puede considerarse como una masa pun-
tual.

Cmo podramos calcular entonces su momento de inercia?

Podemos considerar a cualquier cuerpo extenso de masa m como un nmero infinito de


masas puntuales dm y su momento de inercia I, respecto de un eje dado, como la suma de
los momentos de inercia dI que, respecto de ese mismo eje, tienen cada uno de los ele-
mentos de masa dm en que hemos considerado descompuesto dicho cuerpo. Al tratarse de
una distribucin continua de masa dicha suma se ha de hacer integrando:

m
I dI dm r 2
0

En nuestro caso, descompondremos pues la varilla de masa m y longitud L, en infinitos


elementos de longitud dL a cada uno de los cuales le corresponder una masa dm y un
momento de inercia dI respecto del eje que se considere.

En la figura adjunta hemos dispuesto la varilla sobre el eje OX y vamos a calcular su mo-
mento de inercia respecto de un eje perpendicular que la atraviesa por su centro. Para
simplificar, supondremos que las dos dimensiones alto y ancho de la varilla son despre-
ciables frente a su longitud y en consecuencia se puede considerar lineal.

El momento de inercia de la masa dm representada (en color negro), ser dI = dmx2 y el


m
momento de inercia total vendr dado por I= 0
x 2 dm .
Slido rgido 198

Para poder integrar necesitamos relacionar las variables de la masa y de la distancia x al


eje de giro.

Para ello tendremos en cuenta (tal y como hacamos en la determinacin de la posicin


del centro de masas) que la densidad lineal de la varilla est relacionada con su masa m
y su longitud L mediante la expresin = m/L. Por otra parte, para un elemento infinite-
simal = dm/dL = dm/dx de donde dm = dx. Si la varilla es homognea, el valor de la
densidad correspondiente a cualquier trozo de la misma ser constante.

Proceded a calcular el momento de inercia I

Basta con resolver la integral correspondiente desde que x = -L/2 hasta que x = L/2 :

L
m
L
x3 2 L3 L3 L3
I dI x 2 dm 2L x 2 dx
0
2 3 L 3 8 8 12
2

Teniendo ahora en cuenta que = m/L nos queda finalmente que el momento de inercia
de una varilla respecto de un eje perpendicular que pasa por su centro vale:

I = mL2/12

Cmo sera el momento de inercia si el eje fuese paralelo al anterior pero pasando por
un extremo de la varilla?

Analizando la figura superior, es fcil darse cuenta de que el momento de inercia para los
puntos de la primera mitad de la varilla ser el mismo que en el caso anterior del otro eje,
pero que los dems puntos se encuentran ahora a mayor distancia del eje, por lo que su
momento de inercia debe ser mayor, lo que nos lleva a pensar que el momento de inercia
total de la varilla tambin tendr que ser ms grande que el de antes. Para comprobarlo, no
tenemos mas que integrar cambiando los lmites (que ahora sern entre 0 y L).

L
m L x3 3 mL2
I dI x dm
2
x dx L
2
0 0
3 0 3 3

Como vemos, el momento de inercia nos sale ahora cuatro veces mayor. Qu influencia
tiene este hecho en la rotacin de la varilla?

De acuerdo con el significado fsico de la magnitud momento de inercia, los resultados


obtenidos nos muestran que para conseguir un cambio determinado en la rapidez angular
Slido rgido 199

con que gira una varilla (siempre en un tiempo dado), precisaremos un momento resultan-

te M cuatro veces mayor en el caso de que gire respecto de un eje perpendicular que pasa
por uno de sus extremos que si lo hace respecto de otro paralelo al anterior y que pase por
su centro. As por ejemplo, si lo que queremos es parar (en un tiempo dado) una varilla

que se encuentra girando, precisaremos un momento M mayor en un caso que en otro.
Este tipo de consideraciones tienen una gran importancia en el diseo de mquinas que
tengan piezas que giren respecto de algn eje.

7. Determinad el momento de inercia de un disco de masa m y radio R, respecto de un


eje perpendicular que lo atraviesa por su centro.

Para resolver este ejercicio (y otros similares) conviene plantearse en primer lugar en qu
tipo de elementos de masa dm nos interesa descomponer el objeto para facilitar la resolu-
m
cin de la integral I dI dm r 2
0

En este caso, podemos considerar el disco como una serie de infinitos anillos concntricos
de superficie dS y masa dm, situados uno a continuacin del otro desde que r = 0, hasta
que r = R. Para ello basta con trazar circunferencias concntricas cuyo radio r vaya au-
mentando un dr al pasar de una a otra.

A cada uno de esos anillos le corresponder un momento de inercia dI = dm r2 siendo r el


radio del anillo y dm su masa que podremos evaluar como dm = dS ( es la densidad
m
superficial del disco, , que suponemos constante), de modo que:
4 R 2

dI = dm r2 = dS r2 y como dS = 2 r dr, nos queda finalmente: dI = 2 r3 dr

Proceded a integrar la expresin anterior para obtener el momento de inercia correspon-


diente a todo el disco respecto del eje considerado.

El momento de inercia del disco ser la suma de todos los momentos de inercia dI corres-
pondientes a cada uno de los anillos infinitesimales en que se puede considerar descom-
puesto, es decir, desde que r = 0 hasta que r = R, con lo que:
Slido rgido 200

R mR 2
I dI 2 r 3 dr
0 2

Qu sucedera si en lugar de un disco tuvisemos un cilindro macizo y quisiramos cal-


cular el momento de inercia del mismo respecto de su eje de simetra de revolucin?

Podramos descomponerlo en infinitos discos dibu-


jando para ello infinitos planos perpendiculares a la
generatriz desde la base hasta la cara superior del
cilindro. Cada uno de estos discos tendra un radio
R, una masa dm y un momento de inercia dado por
dI = dmR2/2, por lo que, para calcular el momento
de inercia del cilindro bastara con sumar los mo-
mentos de inercia de todos los discos que constitu-
yen el cilindro:

m 1 mR 2
I dI dm R 2
0 2 2

Para resolver la integral anterior hemos tenido en cuenta que R es constante y la nica
variable es, por tanto, la masa.

8. Determinad el momento de inercia de una esfera


maciza respecto de un eje que coincida con su di-
metro.

Como siempre ocurre en este tipo de problemas, la primera cuestin a plantearse es en


qu tipo de trozos infinitesimales nos puede interesar descomponer la esfera para poder
hallar el momento de inercia de dicha esfera respecto al eje que se nos indica.

Si dibujamos infinitos planos perpendiculares al eje, tendremos descompuesta la esfera en


infinitos discos cuyo momento de inercia ser, como ya sabemos, dI = dm r2/2, con lo que
el momento de inercia de la esfera ser la suma de todos ellos:

m dm r 2
I dI
0 2

Sin embargo, en este caso, a diferencia de lo que ocurra con el cilindro, el radio de los
discos va cambiando y vale 0 en la parte inferior de la esfera para ir aumentando hasta
coincidir con el radio R de la esfera cuando nos encontramos en la mitad y luego ir dismi-
nuyendo hasta hacerse nuevamente 0 en la parte superior de la esfera. Por tanto hemos de
buscar un cambio de variable adecuado que nos permita resolver la integral anterior.
Slido rgido 201

Si tomamos el sistema de referencia de la figura y tenemos en cuenta que


m m
la densidad es
V 4
R3
3

la masa del disco se podr evaluar como dm = dV y su volumen como dV = r2 dy


y el momento de inercia ser:

m dm r 2 1 R 2 R 4
I dI r r 2dy
2 R
r dy
0 2 2 R

No obstante, todava no podemos integrar porque seguimos teniendo ms de una variable.


Es necesario, pues, buscar alguna relacin entre el radio r y la y que nos permita re-
solver la integral.

Si nos fijamos en la figura veremos que utilizando el teorema de Pitgoras para el tringu-
lo que se forma (de hipotenusa R) podemos escribir que r2 = R2 - y2 , y como R es constan-
te, ya podemos integrar la expresin anterior.

R R 16R5 2
r dy (R 2R y y ) dy
4 4 2 2 4
I= mR 2
2 R 2 R 2 15 5

Cmo cambiara el momento de inercia si toda la masa estuviese repartida en la perife-


ria (es decir si se tratase de una esfera hueca de igual masa y del mismo radio)?

Cabe pensar que en este caso el momento de inercia debera de ser mayor puesto que, ex-
ceptuando los puntos que ya se encontraban en la periferia, el resto de ellos ha aumentado
su distancia respecto al eje de giro y, por tanto, su momento de inercia.

Operativamente, la diferencia estara en que en lugar de descomponer el cuerpo en discos,


lo haramos en anillos, de momento de inercia dI = dm r 2 y que al tratarse de un objeto
superficial su densidad ser = m/s = m/4R2 y la masa de uno de los anillos vendr dada
Slido rgido 202

por dm = dS = 2 r dy. Teniendo en cuenta estas consideraciones, ya se puede proce-


der a resolver la integral correspondiente.

2
I = Rdmr 2 R 2 r 3dy mR 2
R R

3
Como podemos ver el momento de inercia de una esfera hueca respecto a un eje que la
atraviese segn la direccin de uno de sus dimetros es 5/3 mayor que si la esfera fuese
maciza (de igual masa y radio).

Utilizad el momento de inercia de una esfera hueca como dato de partida para deducir a
partir del mismo el momento de inercia correspondiente a una esfera maciza.

9. Determinad, por aplicacin del teorema de Steiner, el momento de inercia de una


esfera maciza respecto de un eje tangente a la misma.

sol:7mR2/5

10. Dado un hexgono regular constituido por seis varillas homogneas de longitud L
y masa m, calculad su momento de inercia respecto de un eje perpendicular al plano
que lo contiene y que pasa por su centro.

En este problema hemos de calcular el momento de inercia I de un hexgono, cuyos lados


son varillas, respecto de un eje de giro perpendicular a su plano y que lo atraviesa por su
centro.

De qu puede depender dicho momento de inercia?

Dada la simetra de la figura, cabe esperar que el


momento de inercia dependa de la masa de las
varillas y de su distancia al centro del hexgono,
la cual depende de la longitud del lado.

Cmo podramos calcular el momento de inercia que nos piden?

Podemos calcular el momento de inercia Iv de una de las varillas respecto del eje a que se
refiere el problema y, dada la simetra de la figura, se cumplir que I = 6I v , siendo I el
momento de inercia que buscamos.

Sabemos que el momento de inercia de una varilla respecto de un eje perpendicular que la
atraviesa por su centro viene dado por IC = mL2/12. No obstante el eje respecto del cual
queremos calcular el momento de inercia no es este, sino otro eje paralelo. Ello nos per-
mite, pues, determinar dicho momento de inercia aplicando el teorema de Steiner, segn el
Slido rgido 203

cual I = IC + m d2 en donde IC es el momento de inercia respecto de un eje que pasa por


el centro de masas del objeto, I el momento de inercia respecto de otro eje paralelo al an-
terior y d la distancia entre ambos ejes. En las figuras que siguen hemos representado el
hexgono en perspectiva (izquierda) y visto desde arriba (derecha), con el fin de facilitar
la visualizacin de ejes y distancias.

En nuestro caso, el teorema de Steiner nos lleva a escribir I v = mL2/12 + md2 de donde
podemos obtener Iv si previamente calculamos la distancia d entre los ejes.

Como se trata de un hexgono regular (ver figura anterior derecha) se cumple que d2 = L2
- L2/4 = 3L2/4. Sustituyendo ahora en la expresin anterior nos queda:

Iv = mL2/12 + md2 = mL2/12 + m3L2/4 = 5mL2/6

De acuerdo con ello el momento de inercia total ser: I = 6 Iv = 5mL2

11. Dos esferas macizas de radio R y


masa m, se encuentran unidas mediante
una varilla de longitud 3R y masa m.
Determinad el radio de giro del cuerpo,
respecto de un eje perpendicular a la
varilla por su centro.

El radio de giro k de un objeto respecto de cierto eje, se define como la distancia a que
debera encontrarse del eje una masa puntual igual a la masa del objeto, para que tuviese
el mismo momento de inercia que el objeto. Como es lgico, a un mismo objeto le corres-
ponden tantos radios de giro como ejes se considere (tantos como momentos de inercia).
Qu podemos hacer, pues, para calcular el radio de giro?

Para obtener el radio de giro de un cuerpo respecto de un eje indicado bastar con deter-
minar su momento de inercia respecto de dicho eje y luego evaluar a qu distancia del
mismo debera encontrarse una masa puntual igual a la del cuerpo para que su momento
de inercia fuese el mismo. Si designamos por k el radio de giro y tenemos en cuenta que el
momento de inercia de una masa puntual se define como el producto de dicha masa y el
cuadrado de su distancia al eje de giro, podemos escribir:

I
I = mk2 k =
m
Slido rgido 204

Vemos pues, que el radio de giro depende del momento de inercia del objeto en cuestin y
de su masa. Lo primero que tendremos que hacer pues, es

obtener el momento de inercia del sistema que se nos indica en el enunciado.

Para calcular el momento de inercia I en este caso, podemos considerar que se trata de un
objeto compuesto por dos esferas iguales y una varilla, calcular el momento de inercia
correspondiente a cada uno de esos cuerpos y luego sumar.

Sabemos que el momento de inercia de una esfera maciza, de masa m y radio R, respecto
de un eje que la atraviesa por su centro viene dado por 2mR 2/5. Cmo podemos calcular
el momento de inercia respecto del eje que nos piden?

Podemos hacer uso del teorema de Steiner: I = IC + m d2 que, como ya sabemos, nos rela-
ciona el momento de inercia IC de un objeto de masa m, respecto de un eje de giro que
pase por su centro de masas, con el que dicho objeto presenta respecto de otro eje paralelo
a este y situado a una distancia d del mismo. En este caso, la aplicacin de dicho teore-
ma a cualquiera de las dos esferas, de masa m e , nos conduce a expresar su momento de
inercia respecto del eje que se indica en el enunciado como

Ie = 2me R2/5 + me (25R)2 = 665 me R2

El clculo del momento de inercia de la varilla Iv , no ofrece ninguna dificultad ya que


para ello basta con aplicar la expresin correspondiente (ya obtenida en un problema ante-
rior), con lo que:

Iv = mv L2/12 = mv (3R)2/12 = 075 mv R2

Sumando los momentos de inercia anteriores, teniendo en cuenta que la masa de cada
esfera me y la de la varilla mv son iguales (me = mv = m), podemos obtener el momento de
inercia completo como:

I = 2Ie + Iv = 2665 m R2 + 075 m R2 = 1405 m R2

Ahora ya podemos determinar el valor del radio de giro k, exigiendo que:

I
I = mT k2 siendo mT = 3m. De esta forma nos queda finalmente: k = = 216R
3m
Slido rgido 205

12. Calculad el radio de giro de los cuerpos a los que se refieren los enunciados de los
ejercicios 6, 7, 8, 9 y 10 respecto de los ejes que se indican en dichos enunciados.

sol: Varilla: L/ 12 , L/ 3 . Disco: R/ 2 . Esfera: ( 2 5 ) R; ( 7 / 5 ) R; Hexgono: 5 L

13. El cuerpo de la figura est girando


a 2 r.p.s sobre un plano horizontal con
el que no presenta rozamiento. Si tira-
mos del hilo y reducimos el radio de 20
cm a 10 cm Qu variacin sufrir la
rapidez angular del cuerpo?


Sobre el cuerpo actan tres fuerzas: el peso P , la fuerza normal R que ejerce el plano y

la tensin T del hilo. Dado que el movimiento es circular y uniforme, la resultante de las

tres fuerzas anteriores ha de ser precisamente igual a T , que ser la fuerza necesaria para
que el cuerpo se mantenga con ese movimiento, describiendo una trayectoria circular de
20 cm de radio. El valor de dicha fuerza resultante, se calcula como:

Fres n = m an T = m an = m v2/r = m w2 r, de
forma que si particularizamos la ecuacin anterior a
la situacin inicial (que designaremos como A)
tendremos:

TA m w 2A rA

Es evidente que la cuerda est tensa porque tiramos de ella, si ahora tiramos con ms
fuerza, la tensin aumentar (pasando de valer T A a valer TB) y que el radio de la curva
disminuir hasta alcanzar un nuevo valor r B (como se muestra en la figura anterior) tal que
en la nueva situacin se cumpla:

TB m w 2B rB

Qu le habr ocurrido a la rapidez angular?

Dado que el radio disminuye y que el valor de la tensin ha aumentado, como consecuen-
cia de tirar del hilo hacia abajo, concluimos que, de acuerdo con la ecuacin anterior, co-
mo la masa no cambia, la nueva rapidez angular wB deber ser mayor que la inicial wA.
Slido rgido 206

Cmo podemos calcular el valor de la nueva rapidez angular?

En el trayecto entre la situacin inicial A y la final B, no se cumple el principio


de la con-
servacin de la cantidad de movimiento al existir una fuerza resultante T que acta sobre
la bolita de masa m, en cambio s que se cumple el principio de conservacin del momen-

to cintico L (tambin llamado cantidad de movimiento angular), porque el momento

M de la fuerza resultante respecto del eje de giro es nulo ( T tiene siempre la misma direc-

cin que r ), de modo que:

dL
M res r T 0 L constante L A L B
dt

El vector L debe ser constante en mdulo, direccin y sentido. Como la trayectoria se
mantiene siempre en el mismo plano y el cuerpo gira siempre en el mismo sentido, la di-
reccin y sentido del momento cintico se mantiene constante y en cuanto a su mdulo se
cumplir que: LA = LB y por lo tanto que IAwA = IB wB . Sustituyendo los momentos de
inercia por sus expresiones correspondientes obtenemos que: m rA2 w A m rB2 w B

2
r
y despejando de ella obtener la rapidez angular wB, como w B A w A = 16 rad/s
rB

Si analizamos el resultado literal obtenido, podemos darnos cuenta en primer lugar que es
dimensionalmente homogneo (T-1 en ambos lados de la igualdad) y que contempla algu-
nos casos evidentes as como los razonamientos que hemos realizado anteriormente. Por
ejemplo, vemos que si el radio permanece igual, tampoco cambiara la rapidez (w B = wA) y
si el radio de giro disminuye la rapidez angular ha de aumentar. Sin embargo el resultado,
nos permite precisar adems, que dicho aumento, en la situacin que analizamos, no es
directamente proporcional a la disminucin que experimente el radio, ya que, como po-
demos comprobar, cuando el radio se reduce a la mitad (rB = rA/2) la rapidez angular no se
hace el doble, sino el cudruple wB = 4wA).

14. Una plataforma circular de 80 kg de masa,


gira a 12 r.p.m. Un muchacho de 40 kg de ma-
sa que se encontraba en el eje de giro, se tras-
lada hasta la periferia. Cul ser la nueva
rapidez angular?

sol: /5 rad/s
Slido rgido 207

15. La barra horizontal de la figura tiene un momento de inercia respecto al eje de


rotacin de 510-3 kg m2, y cada una de las bolas que pueden deslizar sobre ella tiene
una masa de 50 g y puede considerarse de dimensiones despreciables. El conjunto est
girando libremente alrededor del eje representado con las bolas dispuestas simtri-
camente respecto al mismo y sujetas por un hilo de 20 cm de longitud. Si se rompe el
hilo cuando el conjunto gira a 20 rad/s, determinad la nueva rapidez angular cuando
las bolas lleguen a los topes del extremo de la barra.

sol: 1066 rad/s.

16. Dos nios de 30 kg de masa cada uno, se encuentran sobre una barra horizontal,
de masa despreciable, que gira a razn de 2 rad/s, alrededor de un eje perpendicular
por su punto medio. Inicialmente, los nios se encuentran a 2m del eje de giro y avan-
zan hasta colocarse a 1m. Se pide:

a) Rapidez angular final.


b) Trabajo realizado por los nios.

sol: a) 8 rad/s; b) 1440 J

17. Disponemos de dos discos que se encuentran gi-


rando en torno a un mismo eje pero con sentidos
contrarios en la situacin que se aprecia en la figu-
ra. El disco 1 de 50 cm de radio y 6 kg de masa, gira
a 05 r.p.s mientras que el 2, de 80 cm de radio y 5
kg de masa, gira a 03 r.p.s. Si en determinado ins-
tante se deja caer el disco 1 sobre el 2, determinad:

a) Rapidez angular de giro del conjunto.


b) Variacin de energa cintica que se produce en
el proceso.

(La distancia del eje al centro del disco 2 es de 9 cm)

Si consideramos el sistema formado por ambos discos, podemos distinguir entre dos esta-
dos. El estado A, corresponde a la situacin en que comienza la interaccin entre los dos
discos. En dicha situacin cada uno de ellos se encuentra girando con una rapidez angular
distinta y con un determinado momento cintico (o cantidad de movimiento angular) res-
Slido rgido 208


pecto del eje de giro. Designaremos como w 1A a la velocidad angular del disco superior y

como L 1A a su cantidad de movimiento angular, L 1A = I1 w 1A . Anlogamente para el

disco inferior, w 2A representar su velocidad angular en la situacin A y L 2A su cantidad

de movimiento angular, de modo que L 2A = I2 w 2A . En la figura siguiente se ha repre-
sentado esquemticamente esta situacin:

Al caer el disco 1 sobre el 2, ocurre una interaccin entre ambos y al final los dos giran
con la misma velocidad angular. Este hecho ha de interpretarse admitiendo que sobre cada
disco acta un momento resultante que modifica su velocidad angular hasta que ambos
giren con la misma. Ese momento responsable del cambio de velocidad angular no pue-
de ser otro que el que se debe a la fuerza de rozamiento que acta sobre cada uno de los
discos mientras desliza el uno sobre el otro. Si designamos como estado B a la situacin

en que ambos giran ya con la misma velocidad angular w B , la cantidad de movimiento

angular de cada uno valdr ahora L 1B = I1 w B para el de arriba, mientras que para el de

abajo ser L 2B = I2 w B , donde w B (cuyo sentido y valor numrico desconocemos a
priori) es la velocidad angular que queremos hallar.

Para calcular w B hemos de relacionar los estados A y B Cmo podramos hacerlo?

Si consideramos a los dos discos como un nico sistema, las fuerzas que se ejercen entre
ellos sern todas interiores. Sabemos que los momentos de las fuerzas internas no pueden
modificar la cantidad de movimiento angular total de un sistema (aunque s la de las part-
culas que lo formen). Podemos hacer uso de este principio para tratar de resolver el pro-
blema. Para ello, consideraremos el sistema de referencia de la figura anterior, en el que:

L A L B L1A L 2A L1B L 2B (I 1 w 1A I 2 w 2A ) k (I 1 I 2 )w B k

En la ecuacin anterior wB es la componente escalar del vector velocidad angular w B la
cual podemos despejar, de manera que obtenemos:

I 1 w 1A I 2 w 2A
wB
I1 I 2

En esta expresin el momento de inercia del disco 1 corresponde al que presenta respecto
a un eje perpendicular al plano que lo contiene y que pasa por su centro. Este se puede
Slido rgido 209

calcular como mr2/2. Sin embargo para determinar I2, hemos de tener en cuenta que el eje
respecto del cual est girando no pasa por su centro de masas sino que se trata de un eje
paralelo situado a una distancia d = 9 cm del mismo. Para obtener I2 podemos aplicar el
teorema de Steiner. Teniendo todo esto en cuenta nos queda finalmente que:

m1 r12 m r2
w 1A 2 2 m 2 d 2 w 2A
I 1 w 1A I 2 w 2A 2 2 0'44 1'07 rad
wB 3'5
I1 I 2 mr m r
2 2
0'01 017
' s

1 1
m2 d 2
2 2

2 2

Como hay que interpretar que la rapidez final salga negativa?

De acuerdo con el sistema de referencia utilizado, la nica componente escalar del vector
velocidad angular del disco inferior 2, al comienzo de la interaccin es negativa (y as lo
hemos tenido en cuenta al proceder a la resolucin literal del problema). El hecho de que
la rapidez final, comn a ambos discos, nos salga negativa, deber interpretarse como que
el conjunto gira en el mismo sentido que el disco inferior.

Cmo podemos calcular la variacin de energa cintica que se ha producido?

En este caso se trata de energa cintica de rotacin. Sabemos que dicha energa cintica
se calcula mediante la expresin: Ec = Iw 2/2, luego para calcular la variacin de energa
cintica que se pueda haber producido, bastar con aplicar dicha expresin a las situacio-
nes A y B ya descritas y efectuar la resta correspondiente. Una cuestin previa es si esa
variacin resultar una cantidad positiva, nula o negativa.

Dado que durante la interaccin actan fuerzas de friccin que modifican el movimiento
de los discos frenndolos, cabe esperar que el trabajo realizado por stas se traduzca en
una disminucin de la energa cintica de rotacin del sistema, de forma que la energa
cintica en el estado B sea inferior a la que haba en el estado A. La diferencia (si el sis-
tema est aislado) se encontrar en forma de energa interna asociada a las partculas que
forman ambos discos (que se pone de manifiesto en el aumento que experimenta su tem-
peratura).

La energa cintica de rotacin en el estado A ser: Ec A = Ec1A + Ec2A , y sustituyendo:

1 1
Ec A I 1 w 12 I 2 w 22 = 967 + 336 = 1303 J
2 2

La energa cintica de rotacin en el estado B: Ec B = Ec1B + Ec2B , y sustituyendo:

1
Ec B (I 1 + I 2 ) w 2B = 11 J
2

La variacin de energa cintica producida ser pues: Ec = EcB - EcA = -1193 J


Slido rgido 210

18. Una plataforma circular de 12 m de radio y 80 kg de masa, es capaz de girar li-


bremente (sin rozamiento) alrededor de un eje perpendicular al plano que la contiene
y que pasa por su centro. Dicha plataforma, inicialmente en reposo, es empujada me-
diante una fuerza F hasta conseguir que gire a 02 rps. Se pide:

a) Trabajo realizado sobre la plataforma por la fuerza que la empuj.


b) Qu suceder si, cuando est girando a 02 rps, se sienta en su periferia un nio
de 40 kg que se encuentra en reposo?
c) Y si el nio, antes de sentarse, comienza a correr junto a la plataforma y se sienta
cuando su rapidez de giro sea igual que la de la plataforma?

Al encontrarse la plataforma ligada a un eje fijo no puede trasladarse y la fuerza que se


realiza sobre ella, hace que aumente su rapidez angular.

Cmo podramos calcular el trabajo


realizado?

Si consideramos como sistema a la plata-


forma y como estado A el que se encuen-
tra en el instante que la persona empieza a
empujar (reposo), y como estado B el que
tiene cuando ha alcanzado la rapidez an-
gular de 02 rps, podemos calcular el tra-
bajo realizado mediante la expresin que
relaciona el trabajo resultante sobre un
sistema con la variacin de energa cin-
tica de rotacin producida:

1 2 1 2
Wres A Ec rot A Iw B Iw A
B B

2 2

La plataforma se puede considerar como un disco macizo que gira respecto a un eje per-
pendicular al plano que forma y que pasa por su centro, con lo que su momento de inercia
ser I = mr2/2. Sustituyendo en la expresin anterior y teniendo en cuenta que wA = 0 y
que wB = 02 2 rad/s obtenemos:

1 2 1
WF A Ec rot A ' 2 (0'2 2 ) 2 = 4548 J
Iw B 80 12
B B

2 2

Qu ocurre cuando el nio, desde el reposo, se monta en la plataforma?

Teniendo en cuenta que el nio al contactar con la plataforma se encuentra en reposo,


podra pensarse que la rapidez angular no iba a cambiar. Sin embargo, la experiencia nos
dice que sta disminuye a otro valor menor tanto ms apreciablemente cuanto ms hacia
la periferia se site el nio. Cul puede ser la explicacin de este hecho?
Slido rgido 211

Consideraremos el conjunto nio-plataforma como un nico sistema, situacin inicial C la


que corresponde al nio en reposo y plataforma girando, y situacin final D cuando plata-
forma y nio se encuentran girando con la misma rapidez angular. Los momentos de las
fuerzas que actan entre ambas situaciones, al tratarse de fuerzas internas, no pueden
cambiar la cantidad de movimiento angular (o momento cintico) del sistema y, por tanto,

se debe cumplir que L C L D . Sin embargo, los momentos de las fuerzas internas s que
cambian la cantidad de movimiento angular de las partes del sistema sobre las que actan,
por eso, en nuestro caso cambiar la cantidad de movimiento angular de la plataforma
pero tambin la del nio de manera tal que su suma siga valiendo lo mismo.

Cmo podemos hallar la rapidez angular del conjunto cuando el nio ya ha subido?

Podemos obtener la expresin de la cantidad de movimiento angular del sistema formado


por
el nio y la plataforma en los estados C y D ya descritos e igualar (en este caso
M ext 0 ) con el fin de obtener la rapidez angular wD con que girar el conjunto. Si desig-
namos como 1 al nio, como 2 a la plataforma y como eje Z al propio eje de giro, ten-
dremos:

L1 C L2C L1 D L2D I 2 w 2C k (I 1 I 2 )w D k

En la obtencin de la ecuacin anterior hemos tenido en cuenta que la cantidad de movi-


miento angular del nio en la situacin A es nula (por encontrarse en reposo). A partir de
la misma podemos obtener la rapidez angular wD como:

I 2 w 2C w 2C w 2C
wD = = = 072 rad/s.
I1 I 2 I1 2m 1
1 1
I2 m2

(Inferior a la rapidez wC = 126 rad/s)

Analizad el resultado literal obtenido

El resultado literal anterior, adems de ser dimensionalmente homogneo (T -1 en ambos


miembros) nos muestra que cuanto mayor sea la inercia I2 a la rotacin de la plataforma (a
igualdad de los restantes factores) ms se aproxima w D a wC , es decir, menos se modifi-
car la rapidez angular inicial w2C de la plataforma por el hecho de que alguien se siente
en ella; cuanto ms pequea sea la masa m1 del nio menos cambiara la rapidez angular
de la plataforma, etc.

Que pasara si el nio, en lugar de colocarse en la periferia, hubiese saltado desde el


suelo hasta el eje de giro?

En este caso su momento de inercia I1 respecto al eje sera nulo y wD = w2C, es decir, la
rapidez angular de la plataforma no cambiara.
Slido rgido 212

19. Un disco de 40 cm de radio gira a 80 r.p.m. alrededor de un eje vertical, por su


centro, y que se mantiene fijo. Su momento de inercia respecto de este eje es 20 kg m2.
Se presiona con un taco en su periferia, el cual le ejerce una fuerza de rozamiento
tangencial y constante Fr , de forma que al cabo de 60 s el disco se detiene. Calculad:

a) Aceleracin angular de frenado.


b) Valor de la fuerza aplicada.
c) Nmero de vueltas que dar hasta detenerse.

Como se trata de un movimiento de rotacin pura alrededor de un eje fijo, nicamente nos
interesar considerar aquellas fuerzas actuantes sobre el cuerpo que tengan momento
respecto del eje de giro ya que es el momento resultante de dichas fuerzas el causante de
la aceleracin angular y, por tanto, de la modificacin del estado de rotacin del disco,
que girar cada vez ms despacio hasta pararse. Para determinar la aceleracin angular del
disco trataremos, pues, de obtener la expresin del momento resultante que acta sobre el
mismo.

Analizad qu fuerzas se ejercen sobre el disco y obtened la expresin de la ecuacin fun-


damental de la dinmica de la rotacin aplicada al mismo.

Sobre el disco actan la fuerza peso P , la fuerza Fe ejercida por el eje y la fuerza de roza-

miento Fr ejercida por el taco en la periferia. Como, tanto P como Fe estn aplicadas en el
eje
de giro, su momento respecto del mismo ser nulo y solo tendr efecto el momento de
Fr de manera que ser la fuerza de rozamiento la que tendremos que tener en cuenta para
calcular el momento resultante que acta sobre el disco.

En qu cambiara el razonamiento anterior si el disco hubiese estado atravesado por un


eje que no pasara por su centro de masas?

Aunque el disco no estuviese atravesado por su centro de


masas, seguiramos sin considerar el efecto de la fuerza
peso ya que aunque el momento de P intentara hacer cdm
girar el cuerpo, dicho giro se efectuara en un plano no per-
mitido por el eje de modo que en los puntos de sujecin del
disco al eje se produciran unas fuerzas cuyo momento
compensara al ejercido por la fuerza peso.
Slido rgido 213

Aplicando la ecuacin fundamental de la dinmica de la rotacin tenemos que:



M I y como el plano de giro permanece constante, tanto M como solo podrn
tener una direccin (perpendicular al plano de rotacin), aunque con dos sentidos posi-
bles, dependiendo del sentido en que se produzca el giro. Este hecho nos permite trabajar

solo con una componente escalar de los vectores M , y w , que podr ser positiva o
negativa segn el sistema de referencia escogido.

En lo que sigue llamaremos eje Z al eje en el


que se realice el giro y adoptaremos el crite-
rio de considerar como sentido positivo el de
la rotacin, lo que implica que el vector axial

w tendr que dibujarse sobre el eje Z y en el

mismo sentido que el vector unitario k , tal y
como se muestra en la figura de la derecha


De acuerdo con lo anterior M = I se podr expresar como Mz = I z . Podemos ob-

viar expresar que se trata de la componente z porque los vectores como M , , w se
encontrarn todos ellos sobre el eje Z, de manera que, en lo sucesivo, siempre que sea
posible utilizaremos un tratamiento escalar. Ello implica que en la ecuacin fundamental
de la dinmica de la rotacin M = I el momento resultante M y la aceleracin angular

no representarn, en general, los mdulos de los vectores M y , sino sus componen-
tes escalares (positivas o negativas segn el sentido que tengan los vectores correspon-
dientes).

En nuestro caso, si nos fijamos en el producto vectorial r Fr vemos que el vector M
tiene el sentido considerado como negativo, tal y como se muestra en la figura siguiente,
por tanto, su nica componente escalar ser negativa.

La ecuacin fundamental de la dinmica de la rotacin expresada escalarmente ser pues:


- M = I o lo que es lo mismo - (Fr r) = I siendo la incgnita a determinar, por lo
que no le pondremos signo, aunque sepamos que ha de resultar negativa al ser la acelera-
cin angular un vector siempre en la misma direccin y sentido que M .
Slido rgido 214

Cmo podramos calcular la aceleracin angular de frenado?

De la ecuacin anterior nos queda que = - Fr r /I pero esto no nos sirve para determinar
porque no conocemos la fuerza de friccin. Otra posibilidad es utilizar las ecuaciones del mo-
vimiento circular uniformemente acelerado (vlidas para cualquier punto del cuerpo):

w = w0 + (t-t0)
= 0 + w0 (t-t0) + (t-t0)2/2

que en nuestro caso, tomando como instante inicial 0 s cuando se comienza a frenar la
rueda, quedan como:

(1) w = w0 + t
(2) = w0 t + t2
w w0
De la ecuacin (1) obtenemos que .
t

Como sabemos la rapidez en el instante inicial t 0 = 0 s (w0 = 80 rpm = 8/3 rad /s) y en el
instante final t = 60 s ( w = 0), basta sustituir en la expresin anterior para obtener = -
w0/t = -014 rad/s2, comprobando como, en efecto, su valor nos sale negativo.

Podemos ahora calcular tambin el valor Fr de la fuerza de frenado.

A partir de la ecuacin -(Fr r) = I podemos despejar el mdulo de la fuerza de friccin


Fr con lo que:

I 20 (0' 14)
Fr = 7N
r 0' 4

Clculo del nmero de vueltas que dar hasta detenerse:

Podemos obtener dicho nmero N utilizando la ecuacin (2) anterior para calcular el
ngulo descrito (en radianes) en el intervalo de tiempo considerado (desde t 0 = 0 hasta t =
60 s). Despus, teniendo en cuenta que cada vuelta equivale a 2 radianes, basta dividir
por 2 para hallar N:

N = /2 = (w0 t + t2/2)/2 = 25065/2 = 399 vueltas

20. Un disco de 3 m de dimetro cuya masa es de 300 kg, gira a razn de 180 rev/min.
Calculad:
a) La energa cintica del disco.
b) Cuntas vueltas dar desde que se le aplica "un par" de frenado de 800 Nm.
c) Tiempo que tardar en pararse, en el caso anterior.

sol: Ec =60 000 J; N = 12 vueltas; t = 8 s


Slido rgido 215

21. Sobre una rueda de 20 cm de radio capaz de girar en torno a un eje fijo perpendi-
cular que pasa por su centro y 5 Kgm2 de momento de inercia, acta una fuerza tan-
gencial dada por F = 5 t2 +2. Cunto valdrn la aceleracin angular y la rapidez angu-
lar a los 2 segundos de comenzar a actuar dicha fuerza, si la rueda se encontraba ini-
cialmente en reposo?


Sobre la rueda acta la fuerza peso P , la fuerza Fe ejercida por el eje y la fuerza tangencial

F aplicada en la periferia, pero, al igual que en el ejercicio anterior, como el cuerpo solo
puede girar
alrededor de un eje fijo, nos interesar nicamente el efecto del momento de
la fuerza F respecto al eje de giro, como causante de la aceleracin angular.

Aplicando la ecuacin fundamental de la dinmica de la rotacin en forma escalar (to-


mando como sentido positivo el sentido en que se realiza la rotacin) y teniendo en cuenta
que, en este caso, M = r F sen 90= rF, nos queda que:

M = I r F = I de donde podemos despejar la aceleracin angular:

= r F/I y sustituyendo F nos queda = r (5t2+2)/I = 02 (5t2+2)/5 = 02t2 + 008

Como vemos, se trata de una aceleracin variable, es decir, cualquier punto del cuerpo
realiza un movimiento circular y variado, de modo que a los 2 s la aceleracin angular de
cualquier punto de la rueda valdr:

= 02t2 + 008 = 02 4 + 008 = 088 rad/s2

Cmo podemos calcular la rapidez angular a los 2 s?

Dado que el movimiento es variado, no podemos aplicar las ecuaciones correspondientes


a otro movimiento distinto, como sera el uniformemente acelerado, sino que hemos de
proceder integrando a partir de la expresin: = dw/dt, considerando que en el instante t 0
= 0 la rapidez angular vale w0 = 0.

w t t3
dw = dt dw (0' 2t 0' 08)dt w 0' 2 0' 08t
2
0 0 3
Slido rgido 216

La expresin anterior nos da el valor de la rapidez angular w en cualquier instante t, de


modo que sustituyendo t obtendremos el valor de w que se nos pide: w = 069 rad/s

22. Tenemos un volante de 80 cm de dimetro y 50 kg de masa, capaz de girar en tor-


no a un eje perpendicular por su centro. Se desea saber:
a) Su aceleracin angular si partiendo del reposo, tira de l una cuerda arrollada a su
periferia, con una fuerza constante de 10 N.
b) La rapidez angular del volante y la de traslacin de un punto de la cuerda al cabo
de un tiempo de 10 s.
c) Longitud de cuerda desenrollada en ese tiempo.

sol: a) = 049 rad/s2 . b) w = 49 rad/s; v = 196 m/s. c) L = 98 m.

23. En la periferia de un disco de 5 kg de masa y 40 cm de ra-


dio, capaz de girar en torno a un eje que pasa por su centro, se
enrolla una cuerda de la que pende un cuerpo de 1 kg.
Determinad la aceleracin con que descender el cuerpo al
dejarlo en libertad y el nmero de vueltas dado por el disco a
los 10 s.

El dispositivo de la figura puede ser considerado como un sistema constituido por dos
cuerpos: La polea (1) y el cuerpo suspendido (2). Al dejar dicho sistema en libertad, la
polea comenzar a girar alrededor de un eje fijo y el cuerpo que pende de ella descender
siguiendo una trayectoria rectilnea.

Analizad las fuerzas que actan sobre cada uno de los dos cuerpos considerados y descri-
bid su movimiento.

a) Movimiento
de la polea. Sobre la polea actan tres fuerzas: la
fuerza Fe que ejerce sobre el centro de la polea y hacia arriba el

hilo que la sujeta al techo, el peso P1 de la polea y la tensin T1 o
fuerza que hace la cuerda. Como el cdm de la polea no puede
moverse (est sujeto por el eje)

la resultante de estas tres fuerzas
es nula, y por tanto: Fe ( P1 T1 ) .

El nico movimiento posible de la polea ser el giro, y en conse-


cuencia, de las fuerzas anteriores solo nos interesarn aquellas
cuyo momento respecto del eje de giro de la polea no sea nulo.
En este caso nicamente el momento de T1 no es 0 (las otras dos
fuerzas estn aplicadas directamente sobre el eje), de modo que
Slido rgido 217

si consideramos como sentido positivo el de la rotacin de la polea, podemos aplicar la


ecuacin fundamental de la rotacin para describir el cambio de movimiento de la misma,
como: M = I siendo I el momento de inercia de la polea respecto al eje considerado y
la aceleracin angular de la polea.

Como r y T1 son perpendiculares, podemos expresar M como T 1 r con lo que nos queda
T1r = I, y dado que M es constante, la polea girar en torno a su eje con aceleracin
angular constante.

b) Movimiento del cuerpo suspendido. El cuerpo que pende de la polea describe un mo-
vimiento de trayectoria conocida (rectilnea) y para estudiar su movimiento podemos apli-
car un tratamiento escalar. Las fuerzas que actan sobre dicho cuerpo son: la tensin T2

que ejerce la cuerda hacia arriba y el peso P2 o fuerza con que la Tierra lo atrae. Si elegi-
mos como sentido positivo el del movimiento, podemos expresar la ecuacin fundamental
de la dinmica como:

Fres t = m2 a2t . Como an = 0, escribiremos a2t = a2 y sustituyendo Fres t por su valor (suma
de las componentes escalares tangenciales de las fuerzas actuantes, con el signo que les
corresponda), nos queda:

P2 - T2 = m2 a2

Como las fuerzas P2 y T2 son constantes, el cuerpo descender con un movimiento uni-
formemente acelerado, siendo la aceleracin a 2 lo que hemos de determinar.

De qu factores cabe esperar que dependa la aceleracin con que desciende el cuerpo
que cuelga de la polea?

Parece claro que cuanto mayor fuese la masa m 2 del cuerpo que desciende y menor el
momento de inercia de la polea (o su masa ya que I = m 1 r2/2), mayor debera de resultar la
aceleracin a2. Por otra parte, es evidente que si la masa de la polea fuese despreciable
dicha aceleracin coincidira con la de la gravedad (el cuerpo caera libremente).

Cmo podramos calcular la aceleracin con que desciende?

Las ecuaciones que disponemos son, en principio, las siguientes:

Rotacin de la polea: T1 r = I
Traslacin del cuerpo: P2 -T2 = m2 a2

En las ecuaciones anteriores tenemos 4 incgnitas. Sin embargo, si consideramos a la ma-


sa de la cuerda como despreciable, sabemos que T 1 = T2 = T. Por otra parte, la aceleracin
angular de la polea y la aceleracin con que desciende la masa m 2, no son independientes,
ya que la cuerda toca punto a punto a la periferia de la polea y, en consecuencia, tanto
como avance un punto Q cualquiera de dicha periferia lo har el cuerpo, con la misma
rapidez y aceleracin tangencial.
Slido rgido 218

En efecto, para el punto Q se ha de cumplir (ya que describe un


movimiento circular) que eQ = r; vQ = w r y que atQ = r.
Por otra parte, si tomamos un origen de tiempos, al cabo de un
tiempo t, el punto Q (tal y como se puede observar en la figura
adjunta) se habr desplazado un espacio e Q (lnea roja punteada),
que deber coincidir con el que se haya desplazado el cuerpo 2 en
ese mismo tiempo. Es decir: eQ = e2, de modo que derivando en
esta igualdad nos queda que: vQ = v2 y que: atQ = at2 = a2.
De acuerdo con lo anterior, la aceleracin con que desciende el
cuerpo tambin ser: a2 = r, disponiendo ahora de tres ecuacio-
nes para resolver el problema:

T r = I
P2 -T = m2 a2
a2 = r

Teniendo en cuenta que el momento de inercia de la polea (considerada como un disco


macizo) vale I = m1 r2/2, podemos obtener finalmente la expresin correspondiente a la
aceleracin a2 con que descender el cuerpo.

Para ello sustituimos I = m1 r2/2 y = a2/r en la primera ecuacin con lo que T = m 1a2/2
Introduciendo ahora la expresin de T obtenida, en la segunda de las ecuaciones y despe-
jando la aceleracin a2, nos queda finalmente:

m2
a2 g a2 = 286 m/s2
m1
m2
2
Analizad el resultado literal obtenido
Si analizamos el resultado anterior podemos darnos cuenta de que, adems de ser dimen-
sionalmente homogneo (dimensiones de la aceleracin en ambos lados de la igualdad),
contempla las hiptesis de partida, como por ejemplo, que la aceleracin ser tanto mayor
cuanto menor sea la masa m1 de la polea y que cuando dicha masa sea despreciable frente
a m2, el cuerpo descender con aceleracin g (la de la gravedad). Por otra parte, el resulta-
do nos muestra tambin que la aceleracin a 2 no depende del radio que tenga la polea.

Sugerid y llevar a cabo un procedimiento para determinar el nmero de vueltas dado por
la polea en 10 s.

Ya hemos visto que la polea gira con aceleracin angular constante = a2/r. Ello nos
permite aplicar las ecuaciones del movimiento circular uniformemente acelerado para
calcular el ngulo girado por cualquier punto de su periferia en los 10 s. Como cada
vuelta equivale a 2 radianes, bastar despus dividir el desplazamiento angular obtenido
entre 2 para obtener el nmero N de vueltas correspondiente.
Slido rgido 219

1
En efecto, considerando que en t0 = 0, w0 = 0, podemos escribir: t2
2
a2
y teniendo en cuenta que , nos queda que: = a2t2/2r
r

a t2
Sustituyendo ahora en N , nos queda finalmente que: N = 2 = 569 vueltas
2 4 r

En el resultado anterior podemos ver que, cuanto mayor sea la aceleracin con que cae el
cuerpo, menor el radio de la polea y mayor el tiempo considerado, ms vueltas dar la
polea.

24. Dado el dispositivo de Atwood representado en la


figura adjunta, en el que la polea puede ser considera-
da como un disco de 20 cm de radio y 1 kg de masa,
determinad la aceleracin de las masas 1 y 2 al dejarlo
en libertad as como la tensin de la cuerda (m1 = 7 kg
y m2 = 4 kg).

En este problema tenemos un sistema formado por tres cuerpos que al dejarlos en libertad
se movern. La polea al estar sujeta como se indica en la figura solo puede girar en torno a
su eje. El peso del cuerpo 1 provocar un momento respecto del eje de giro de la polea
que hara girar a sta hacia nuestra derecha, mientras que el momento del peso del cuerpo
2 la hara girar hacia nuestra izquierda. Como el peso de 1 es mayor que el de 2 (y r es el
mismo en ambos casos e igual al radio de la polea) el momento resultante har girar la
polea hacia nuestra derecha, de modo que la masa m 1 descender y la m2 subir.

Describid con el mayor detalle posible el movimiento de cada uno de los cuerpos.

Las fuerzas que actan sobre cada uno de los cuerpos determinan su movimiento, aunque,
al estar sujetos por medio de una cuerda inextensible, se cumplir que la aceleracin con
que se desplazarn los cuerpos 1 y 2 ser la misma para ambos y tambin coincidir con la
aceleracin sobre la trayectoria de cualquier punto de la periferia de la polea, ya que se
supone que dicha periferia y la cuerda se tocarn punto a punto (es decir, la cuerda no
resbala). Si tomamos como sentido positivo el del movimiento (de traslacin de los
cuerpos y de giro de los puntos de la periferia de la polea) y analizamos lo que le ocurre a
cada uno de los cuerpos por separado tendremos:
Slido rgido 220

a) El cuerpo de masa m1: Realizar un movimiento hacia


abajo

siguiendo una trayectoria recta sometido a la accin de

P1 y T1 . Dado que la aceleracin normal es 0, podemos con-
siderar nicamente las componentes escalares tangenciales
de las fuerzas que actan (con el signo que les corresponda),
con lo que:

Fres t1 = m1 at1 P1 -T1 = m1 a (1)

b) El cuerpo de masa m2: Realiza un movimiento



rectilneo
hacia arriba sometido a la accin de T2 y P2 . La ecuacin
fundamental de la dinmica ser:

Fres t2 = m2 at2 T2 - P2 = m2 a (2)


En la ecuacin anterior hemos tenido en cuenta que T2 va en el sentido de la trayectoria

escogido como positivo mientras que P2 tiene sentido negativo, lo que hace que sus com-
ponentes escalares tangenciales sean, respectivamente, positiva y negativa.

c) La polea de masa m3 : Realiza



un movimiento

de rotacinalrededor

de su eje, sometida
a la accin de las fuerzas T'1 , T'2 , Fe y P3 . de las cuales Fe y P3 tienen momento nulo
respecto del eje de rotacin. La polea girar debido al momento
resultante M = I . De
acuerdo con el criterio de signos escogido, el momento de T '1 respecto del eje de rotacin

ser positivo y el de T ' 2 negativo, de manera que el valor del momento resultante se
podr expresar como:

T1 r - T2 r = I .

Si consideramos la masa de la cuerda a cada lado de la polea despreciable, sabemos que


T1 = T1 y que T2 = T2 . Por otra parte, la aceleracin de cualquier punto de la periferia de
la polea (que, segn hemos visto, es tambin a) se puede expresar como a = r con lo
que la ecuacin anterior puede escribirse como:

T1 r - T2 r = I a/r

Por qu no podemos considerar iguales T1 y T2 ?

Porque si lo fuesen, el momento resultante que actuara sobre la polea sera nulo y sta no
podra variar su rapidez angular por lo que, si inicialmente estaba en reposo, debera de
continuar estndolo.

Teniendo en cuenta ahora que el momento de inercia de un disco viene dado por I= mr 2/2,
nos queda que:

T1 r - T2 r = m3 r a/2 T1 -T2 = m3 a/2 (3)


Slido rgido 221

En este problema hemos de calcular la aceleracin a con que desciende la masa m 1, que
ser, segn hemos visto, la misma con que asciende m 2 o se mueven los puntos de la
cuerda (y periferia de la polea).

De qu depender el valor de la aceleracin a?

Podemos pensar que cuanto ms grande sea m 1 y ms pequea sea m2, mayor tendr que
ser la aceleracin a. Por otra parte, la masa de la polea tambin tendr que influir ya que
cuanto mayor sea sta mayor ser la inercia que presentara a la rotacin (y menor sera la
aceleracin a). Adems, es evidente que si las masas del cuerpo 2 y de la polea fuesen
despreciables frente a la del cuerpo 1, ste descendera con una aceleracin igual a la de la
gravedad.

Obtened la expresin de la aceleracin a y comprobad si en ella se contemplan las hip-


tesis anteriores.

Para determinar a podemos utilizar las ecuaciones que rigen el movimiento de cada uno
de los cuerpos:

Traslacin de m1: P1 -T1 = m1 a (1)


Traslacin de m2: T2 - P2 = m2 a (2)
Rotacin de la polea: T1 -T2 = m3 a/2 (3)

(m1 m 2 ) g
A partir de las mismas podemos obtener que: a = a = 26 m/s2
m3
m1 m 2
2

Analizando la expresin obtenida nos podemos dar cuenta en primer lugar de que es di-
-
mensionalmente homognea (dimensiones de la aceleracin LT 2 en ambos miembros de
la igualdad) y que en ella se recogen la hiptesis anteriores. As, podemos ver que la ace-
leracin a aumentar cuando aumente la diferencia entre las masas m 1 y m2 , y viceversa,
(como siempre a igualdad de los restantes factores) y que si m 2 y m3 se hacen desprecia-
bles frente a m1 ocurrir que a = g.

Tambin podemos darnos cuenta que si la masa m3 de la polea se considerase desprecia-


ble (lo que es equivalente a decir que no presente inercia alguna a cambiar de rapidez an-
gular), la expresin obtenida se transforma en otra, ya conocida, que manejamos en el
captulo de dinmica del punto, en un problema similar a este. Otra diferencia con aquel
problema es que aqu la tensin de la cuerda en ambos lados de la polea es diferente,
mientras que all suponamos que era la misma.

Cometimos una equivocacin? Qu cabe esperar que ocurra si en las expresiones de T1


y T2 introducimos la simplificacin de considerar la masa de la polea despreciable?

De acuerdo con las consideraciones anteriores, cabe esperar que las expresiones se trans-
formen de forma que ambas tensiones sean iguales. En efecto, del sistema de tres ecua-
ciones anterior obtenemos que:
Slido rgido 222

m1 m 2 m1 m 2
T1 m1 (g g) T2 m 2 (g g)
m3 m3
m1 m 2 m1 m 2
2 2

y haciendo en ellas m3 = 0 resulta T1 = T2 = 2m1 m2 g/(m1 +m2)

En adelante, pues, siempre que aparezca una polea de masa despreciable su I ser 0 y no
tendr otro efecto en el sistema en que participa, que cambiar la direccin de la cuerda
tensa que pase por ella.

25. Una polea de 1 kg de masa y 5 m de radio, cuyo radio de giro vale 25 m, lleva
arrollado un hilo inextensible y sin masa que pasa por otra polea, de masa desprecia-
ble y del que pende un cuerpo de 2 kg, como se indica en la figura. Determinad:

a) Aceleracin angular de la polea.


b) Tensin del hilo.
c) Rapidez angular de la polea a los dos segundos de iniciar el movimiento.

sol: = 178 rad/s2; T = 22 N; w = 356 rad/s

26. Un disco de 05 kg de masa y 25 cm de radio, puede girar acoplado a un eje de


masa despreciable y 2 cm de radio. En este eje se enrolla una cuerda de la que pende
un cuerpo de 2 kg y se deja en libertad. Determinad:

a) ngulo girado por el disco en 3 s.


b) Rapidez angular y lineal de un punto de la periferia del disco a los 10 s.

sol: a) = 28125 rad. b) w = 1875 rad/s; v = 312 m/s


Slido rgido 223

27. Un volante gira por la accin de un cuerpo de 4 kg de masa, que cuelga del extre-
mo de una cuerda arrollada a un eje de masa despreciable acoplado a aquel (vase la
fig. del ejercicio anterior). Sabiendo que al soltar el cuerpo ste desciende 3 m en 2 s,
determinad la tensin de la cuerda y el momento de inercia del volante (radio del eje
= 20 cm).

sol: T = 34 N; I = 09 kg m2

28. El sistema de la figura representa una polea


fija que puede considerarse como un disco de 4
F
kg de masa y radio R, junto con otras dos poleas
ms pequeas y de masas despreciables.

De un extremo de la cuerda se tira con una


fuerza de 60 N y del otro pende un cuerpo cuya
masa es de 5 kg. Calculad la aceleracin del
cuerpo y las tensiones de la cuerda a ambos
lados de la polea grande.

En este problema tenemos un sistema formado por cuatro cuerpos, pero de ellos hay dos,
que son las poleas de masa despreciable, que, como ya hemos visto, el nico efecto que
producen en el sistema es modificar la direccin de la cuerda tensa. Por tanto, para deter-
minar la aceleracin que se nos pide en el enunciado, nos ocuparemos del movimiento del
cuerpo (1) y de la polea (2). Para ello conviene que comencemos por considerar qu fuer-
zas actan sobre el cuerpo 1 y sobre la polea 2 y describir el movimiento de cada uno de
estos cuerpos con el mayor detalle posible.

Bajo la accin del peso P1 = 50 N, el cuerpo 1 descendera y la polea girara hacia la dere-
cha. Por el contrario, la fuerza F = 60 N, hara subir al cuerpo 1 y que la polea girase hacia
la izquierda. Como F es mayor que P1, el cuerpo 1 ascender y la polea girar hacia la
izquierda. Si tomamos como sentidos positivos el del movimiento del cuerpo 1 y el del
giro de la polea, las ecuaciones que rigen el movimiento de cada cuerpo sern:


a) El cuerpo 1: Sobre l actan las fuerzas T1
F
y P1 y se mueve sobre la trayectoria en el sen-
tido escogido como positivo. Utilizando un
tratamiento escalar a lo largo de la trayectoria,
la ecuacin fundamental de la dinmica apli-
cada al cuerpo 1 se expresa como:

T1 - P1 = m1 a (1)
Slido rgido 224


b) El cuerpo 2: Sobre la polea, de radio r y momento de inercia I, actan las fuerzas T'1 y

T2 (ejercidas ambas por la cuerda) as como Fe ejercida por el eje y el peso de la polea P2 .
Como estas dos ltimas estn aplicadas en el eje, su momento respecto al mismo ser nulo
y no tendrn efecto alguno en la rotacin de la polea. Por otra parte, si consideramos la
masa de la cuerda despreciable, sabemos que T 1 = T1 y T2 = F, con lo que:

Mres = I F r - T1 r = I

Conviene tener tambin en cuenta que como la cuerda no desliza sobre la polea sino
que en todo momento est en contacto punto a punto con la periferia de la misma, se
cumplir que la aceleracin sobre la trayectoria de cualquier punto de dicha periferia, ser
la misma que la aceleracin de cualquier punto de la cuerda inextensible y, por tanto, que
la aceleracin con que asciende el cuerpo 1. Ello nos permite expresar dicha aceleracin
como a = r. Si adems consideramos a la polea como un disco macizo, sabemos que,
en ese caso, I = m2 r2/2, con lo que podemos expresar la ecuacin anterior como:

F r - T1 r = (m2 r2/2 ) a/r F - T1 = m2 r a/2 (2)

En el problema se nos pide que calculemos la aceleracin con que asciende el cuerpo 1,
que, segn acabamos de ver, coincidir con la aceleracin de cualquier punto de la cuerda
o de la periferia de la polea.

Considerad, de acuerdo con las condiciones establecidas, qu factores cabe esperar que
influyan y cmo lo harn, en el valor de dicha aceleracin.

En primer lugar podemos razonar que cuanto mayor sea F y menor la masa del cuerpo 1 y
menor la masa de la polea 2 (menor inercia), tanto mayor tendra que ser la aceleracin
con que el cuerpo 1 ascendera. Por otra parte, parece evidente que si F fuese menor que
P1, la aceleracin tendra que salir negativa, es decir, el cuerpo 1 en esas condiciones des-
cendera (partiendo de una situacin inicial de reposo).

Sugerid y llevar a cabo un procedimiento para obtener la expresin de la aceleracin que


se nos pide.

Utilizando las dos ecuaciones anteriores:

Traslacin del cuerpo 1: T1 - P1 = m1 a (1)


Rotacin de la polea: F - T1 = m2 r a/2 (2)

F m1 g
Podemos obtener a partir de las mismas: a = a = 143 m/s2
m2
m1
2
Analizad el resultado literal obtenido

Podemos ver que el resultado, es dimensionalmente homogneo (LT-2 en ambos miem-


bros) y que en l se contemplan todas las hiptesis anteriores. Adems, nos permite darnos
cuenta de otros aspectos de inters, como por ejemplo, lo que ocurrira si la masa de la
Slido rgido 225

polea fuese despreciable. En ese caso, el momento de inercia de la polea tambin lo sera
y la aceleracin de ascenso del cuerpo sera la misma que si tirsemos directamente de l
hacia arriba mediante un fuerza F:

a = (F - P1)/m1 .

Tambin podemos plantearnos nuevos problemas, como, por ejemplo:

Cunto debera valer F para que el cuerpo 1 suba con movimiento uniforme?

Para resolver esta nueva cuestin no tenemos mas que hacer a = 0 en el resultado literal
obtenido y despejar F, con lo que resulta F = m 1g = 50 N.

Finalmente, el clculo de las tensiones T1 y T2, no ofrece ninguna dificultad ya que basta
con despejar de las ecuaciones (1) y (2), con lo que obtenemos:

T1 = m1 (a+g) = 5715 N; T2 = F = 60 N

29. Dado el dispositivo de la figura en el que


m1 = 4 kg, m2 = 1 kg y la polea puede ser con-
siderada como un disco de 2 kg de masa y 50
cm de radio, determinad la aceleracin del
sistema y la tensin de la cuerda (suponiendo
que el rozamiento sea despreciable y la cuerda
inextensible).

sol: a =5/3 m/s2 ; T2 = 25/3 N y T1 = 20/3 N

30. Un bloque de 5 kg de masa desliza a lo largo


de un plano inclinado de 37 con el que presenta
un coeficiente de friccin de 025. El bloque est
unido a un hilo, inextensible, perfectamente
flexible y de masa despreciable, enrollado en una
polea que puede girar libremente como indica la
figura.

Sabiendo que la polea tiene una masa de 20 kg,


un radio de 02 m y que su radio de giro es de
01 m, determinad la aceleracin con la que
desciende el bloque y la tensin del hilo.

sol: a = 2 m/s2; T = 10 N
Slido rgido 226

31. Dado el dispositivo de la figura adjunta,


se pide:
a) Aceleracin del sistema.
b) Tensin de la cuerda.
c) Numero de vueltas que dar la polea en 3 s.
(Datos: masa de la polea 4 kg, radio de la po-
lea 50 cm, m1 = 5 kg, m2 = 6 kg, = 01)
En este problema tenemos un sistema formado por dos bloques y una polea dispuestos
como se indica en la figura y se nos pide con qu aceleracin se movern. Para resolver el
problema conviene comenzar por analizar primero la situacin planteada estudiando las
fuerzas que actan sobre cada cuerpo:
Como la trayectoria es fija y conocida utilizaremos un tratamiento escalar considerando
nicamente las fuerzas que actan a lo largo de la trayectoria. Como sentido positivo es-
cogeremos el sentido del movimiento de los bloques y de rotacin de la polea, lo que nos
lleva a plantearnos en primer lugar en qu sentido se mover el sistema partiendo de una
situacin inicial de reposo.
Dado que la inercia de la polea y que la fuerza de rozamiento mxima de m 2 con el plano
no dependen del sentido con que se mueva el sistema, podemos razonar que puesto que P 1
es mayor que la componente tangencial Pt2 el sistema "intentar" desplazarse hacia la iz-
quierda, por lo que la fuerza de rozamiento actuar en sentido descendente. Se trata esta
de una reflexin previa esencial para resolver correctamente problemas en los que existan
fuerzas de rozamiento, ya que el sentido de movimiento determinar el sentido de la fuer-
za de rozamiento y como sta fuerza hay que tenerla en cuenta al calcular la fuerza resul-
tante que acta sobre el sistema, si nos equivocamos, el resultado de dicha fuerza resultan-
te (y por tanto de la aceleracin) ser distinto del real. En la figura siguiente hemos repre-
sentado esquemticamente las fuerzas que actan en la situacin inicial de reposo.

Para resolver el problema introduciremos


las simplificaciones habituales de supo-
ner que se trata de una cuerda inextensi-
ble (lo cual hace que ambos bloques se
muevan con la misma aceleracin) y que
su masa es despreciable (por lo que a
ambos lados de la polea se cumplir que
T1 = T1 y que T2 = T2).

Tomando como sentidos positivos el de despla-


zamiento de las masas y el de rotacin de la polea, analizaremos las fuerzas que rigen el mo-
vimiento de cada cuerpo cuando se deja el sistema en libertad y se inicia el movimiento:

a) Sobre el cuerpo de masa m1 actan la fuerza peso P1 y la tensin de la cuerda T'1 de
manera que al aplicar la ecuacin fundamental de la dinmica de la traslacin nos queda:
P1 -T1 = m1 a (1)
Slido rgido 227


b) Sobre el cuerpo de masa m2 actan la tensin de la cuerda T2 , la componente tangen-

cial de su peso Pt2 y la fuerza de rozamiento Fr opuesta al movimiento (no representada
en la figura anterior). Estas tres fuerzas son las que se ejercen a lo largo de la trayectoria,
pero adems actan tambin la componente normal del peso Pn y la fuerza normal que

ejerce el plano R , aunque stas se anulan entre si ya que al ser la trayectoria recta la com-
ponente normal del vector aceleracin es nula. De acuerdo con ello, la ecuacin funda-
mental de la dinmica de la traslacin aplicada a este cuerpo se expresar como:

T2 -Pt2 - Fr = m2 a (2)

c) Sobre la polea,

de

masa m3, actan

las tensiones ejercidas por la cuerda en ambos lados
de la misma T1 y T' 2 , su peso P3 y la fuerza ejercida por el eje Fe . Como las dos ltimas
estn aplicadas en el propio eje, su momento respecto del mismo ser nulo, con lo que la
ecuacin fundamental de la dinmica de la rotacin aplicada a la polea ser:

T1 r - T2 r = I T1 r - T2 r = I (3)

En el problema se nos pide que calculemos la aceleracin a . Conviene que antes de


operar se proceda a realizar una reflexin cualitativa sobre los factores que pueden influir
en dicha aceleracin, tratando de avanzar cmo cabe esperar que lo hagan.

Podemos razonar que cuanto mayor sea m 1 , menor m2 y menor la inercia de la polea (y
por tanto su masa si el radio no cambia) mayor tendr que ser la aceleracin. Por otra par-
te, es evidente, que si, por ejemplo, m 2 y m3 fuesen nulas, m1 debera de caer con la acele-
racin de la gravedad g. Adems, conforme vaya aumentando la inclinacin del plano (de
0 a 90, cabe esperar que a vaya disminuyendo), etc.

Cmo podemos determinar la aceleracin ?

Disponemos de tres ecuaciones que rigen el movimiento de cada cuerpo:

Traslacin del cuerpo 1: P1 -T1 = m1 a (1)


Traslacin del cuerpo 2: T2 -Pt2 - Fr = m2 a (2)
Rotacin de la polea: T1 r - T2 r = I (3)

Podemos tratar de obtener a. despejando de la ecuacin (1) o de la (2), pero descono-


cemos lo que vale la tensin de la cuerda en cada caso. El problema se podra solucionar
expresando la ecuacin (3) en funcin de la aceleracin a. Para ello hemos de tener en
cuenta que como la cuerda no resbala sino que se toca con la polea punto a punto, a =
r, con lo que la ecuacin (3) queda como T 1 r - T2 r = I a/r y tenemos as un sistema
de tres ecuaciones con tres incgnitas, que ya podemos resolver.

De la ecuacin (1) despejamos T1 = P1 - m1 a


De la ecuacin (2) despejamos T2 = Pt2 + Fr + m3 a

Sustituyendo ahora T1 y T2 en la tercera ecuacin nos queda:


Slido rgido 228

(P1 - m1 a)r - (Pt2 + Fr + m3 a) r = I a/r

Si identificamos la polea con un disco macizo su momento de inercia ser I = m 3 r2/2.


Por otra parte, sabemos que PN = mgcos y que Fr = mgcos. Sustituyendo estas expre-
siones en la ecuacin anterior y despejando a, obtenemos:

P1 Pt2 Fr m g m 2 gsen m 2 gcos m


a= 1 114
'
m m s2
m1 m 2 3 m1 m 2 3
2 2

Si analizamos el resultado literal obtenido, podemos comprobar que es dimensionalmente


homogneo (LT-2 en ambos miembros) y que, contempla las hiptesis que hicimos al co-
mienzo. As por ejemplo, vemos que, efectivamente, cuanto mayor sea m 1 y menores sean
m2 y m3, la aceleracin ser mayor y que si m 2 = m3 = 0, a = g. Adems, si comparamos el
resultado con el que se obtuvo en el captulo de dinmica para un ejercicio similar (ved
problema 24 de dinmica del punto) vemos que este se transforma en aquel sin ms que
considerar que la masa de la polea m 3 es despreciable (y por tanto no presenta inercia a
variar su rapidez angular), que es precisamente la simplificacin que hicimos all.

Cmo podemos determinar el nmero de vueltas N dadas por la polea a los 3 s de dejar
el sistema en libertad?

El nmero de vueltas se podr obtener dividiendo el ngulo girado por la polea expresado
en radianes entre 2 (que son los radianes que tiene una vuelta), es decir:

N = /2. Para calcular el ngulo recorrido bastar con hallar la aceleracin angular me-
diante a = r y calcular para un punto cualquiera de la periferia de la polea (que des-
cribir un movimiento circular uniformemente acelerado de aceleracin ). As pues:

= a/r = 114/05 = 228 m/s2 y si tomamos t0 = 0, w0 = 0 y 0 = 0, nos queda:

= t2/2 = 22832/2 = 1026 rad, con lo que N =1026/2 = 163 vueltas.

32. La doble polea esquematizada en la figura adjunta tiene


un radio de giro de 40 cm siendo su radio mayor de 50 cm y
el menor de 20 cm.

Sabiendo que la masa de la polea es de 10 kg, determinad el


valor F de la fuerza necesaria para que el cuerpo suspendido
(de 4 kg de masa) ascienda con una aceleracin de 15 m/s 2.
Cul ser
la energa cintica de la doble polea a los 5 s de
actuar F si inicialmente se encontraba en reposo?
Slido rgido 229

En este caso tenemos un sistema formado por la doble polea que gira (y que designaremos
como cuerpo 1) y un objeto suspendido que se traslada (cuerpo 2). Sobre cada uno de ellos
actan las fuerzas especificadas en la figura siguiente, y nuestro problema es determinar
cunto ha de valer F para que el cuerpo 1 ascienda con una cierta aceleracin. Para ello
comenzaremos por analizar el movimiento de cada uno de los cuerpos que forman el sis-
tema:

Analizad el movimiento de cada uno de los cuerpos considerados



a) Sobre la doble polea, actan las fuerzas T1 , T' 2 , P1 y Fe . Los momentos de la fuerza

peso P1 y de la fuerza que hace el eje Fe respecto al eje de rotacin son ambos nulos por-
que las dos fuerzas estn aplicadas en el propio
eje y, en consecuencia, solo tendrn efecto
en la rotacin los momentos de las fuerzas T1 y T' 2 ejercidas por las cuerdas, que, como
podemos apreciar en la figura, tienen sentido contrario. Como la trayectoria es fija y co-
nocida, podemos aplicar un tratamiento escalar. Para ello escogeremos como positivo el
del movimiento de rotacin de la doble polea (hacia nuestra izquierda), con lo que la
ecuacin fundamental de la dinmica de la rotacin ser: T 1 R - T2 r = I

Por otra parte, al ser la masa de las cuerdas despreciables T 1 = F y T2 = T2 con lo que la
ecuacin queda como:

F R - T2 r = I

En la expresin anterior I es el momento de inercia de la doble polea, R el radio mayor y r


el radio menor. (Dmonos cuenta que, a diferencia de lo que ocurra en la mquina de
Atwood, las distancias de las lneas de accin de las fuerzas al eje de giro no son iguales).

Dado que conocemos el radio de giro k, el momento de


inercia de la doble polea respecto de su eje de giro, se
puede evaluar como I = m k2

Finalmente, hemos de considerar que la aceleracin an-


gular de la doble polea est relacionada con la acelera-
cin con que asciende el cuerpo 2 mediante la expresin:
a2 = r = a2/r ya que la cuerda de la que pende
dicho cuerpo toca punto a punto a la periferia de la polea
de radio r.

Todo ello nos permite expresar la ecuacin fundamental


de la dinmica de la rotacin aplicada a la doble polea
como:

F R -T2 r = m k2 a2/r (1)


b) Sobre el cuerpo 2, actan la tensin T2 en sentido positivo y el peso P2 en sentido
negativo, realizando un movimiento de traslacin hacia arriba, con lo que, si aplicamos a
dicho cuerpo la ecuacin fundamental de la dinmica:
Slido rgido 230

T2 - P2 = m2 a2 (2)

En el problema se nos pide que determinemos la fuerza F que ser preciso realizar para
que el cuerpo 2 ascienda con una determinada aceleracin a 2.

Conviene que antes de operar reflexionemos sobre los factores que pueden influir en di-
cha fuerza.

Podemos pensar que cuanto mayor sea la aceleracin a 2 con que queremos que ascienda el
cuerpo 2 y mayor sea el momento de inercia de la doble polea (I = m k2 ), mayor ser el
valor de F necesario. Podemos plantearnos tambin alguna condicin lmite evidente, co-
mo por ejemplo, que si el momento de inercia de la doble polea y la masa m 2 fuesen nulas
F debera de resultar 0, etc.

Elaborad y aplicad una estrategia para obtener la fuerza pedida.

A partir de las ecuaciones:

F R -T2 r = m k2 a2/r (1)


T2 - P2 = m2 a2 (2)

podemos despejar F, con lo que nos queda:

a2 15
'
m1 k 2 m 2 (g a 2 ) r 10 0'42 4 (10 15
' ) 0'2
r 0'2
F= = 42'5 N
R 05'

Para calcular la energa cintica de la polea, dado que no se traslada, bastar aplicar la
expresin de la energa cintica de rotacin de un slido rgido alrededor de un eje:

Ec = Iw2/2. Para ello hemos de hallar w en el instante que se nos pide (t = 5s).

Como se trata de un movimiento de rotacin, podemos calcular el valor de la aceleracin


angular como:

= a2 /r = 15/02 = 75 rad/s2

Sustituyendo ahora en la ecuacin del movimiento circular uniformemente acelerado


(considerando w0 = 0 y t0 = 0), nos queda:

w = w0 + (t - t0) = t = 75 5 = 375 rad/s

y la energa cintica Ec = Iw2/2 = 10 042 3752 /2 = 1125 J


Slido rgido 231

33. En el dispositivo de la figura adjunta se muestran


dos cuerpos cuyas masas son m1 = 8 kg y m2 = 6 kg, que
cuelgan de una doble polea cuyos radios son 01 m y 02
m. Sabiendo que el momento de inercia de la doble polea
vale 5 kg m2. Determinad:

a) Aceleracin de cada uno de los cuerpos.


b) Tensiones de las cuerdas.
c) Momento cintico del sistema respecto del eje de giro.

En este caso tenemos un sistema formado por una doble polea y dos masas, que podemos
considerar como puntuales y que cuelgan de la misma como se muestra en la figura.
Cuando se deje el sistema en libertad (partiendo del reposo) la polea girar en torno a su
eje a la vez que uno de los cuerpos subir y el otro bajar. El problema nos pide calcular
la aceleracin con que se mover cada uno, para lo cual hemos de analizar el movimiento
de los cuerpos que forman el sistema. A este respecto, una primera cuestin que hemos de
plantearnos es hacia dnde se movern los cuerpos.

Un primer anlisis de la situacin, podra llevarnos a pensar que, como el peso P 1 del
cuerpo 1 es mayor que el peso P2 del cuerpo 2, el movimiento se producir de forma que
el primero baje y el segundo suba, en cuyo caso la doble polea girara hacia la izquierda.
Sin embargo, conviene recordar que lo que provoca cambios en el movimiento de rotacin
de un slido no son las fuerzas sino los momentos de dichas fuerzas respecto del eje de
rotacin, por tanto, para determinar el sentido de giro que se producir deberemos compa-
rar los momentos de las fuerzas y no los valores de dichas fuerzas.

En este caso al haber dos radios diferentes, no es inmediato determinar qu momento va a


ser mayor, por lo que tendremos que proceder a calcularlos:

Mdulo del momento de la fuerza P1 respecto del eje: M1 = P1 r = 80 01 = 8 Nm

Mdulo del momento de la fuerza P2 respecto del eje: M2 = P2 R = 60 02 = 12 Nm

Como vemos, aunque P1 es mayor que P2, el momento de esta ltima fuerza respecto del
eje de giro es mayor, por lo que la polea girar hacia la derecha y no hacia la izquierda.
Tomaremos como sentido positivo el de traslacin de los cuerpos y el de giro de la polea
(hacia la derecha).

A continuacin, analizaremos las fuerzas que actan sobre cada uno de los cuerpos que
forman el sistema, considerando, como siempre, que las cuerdas son inextensibles y sus
masas despreciables.
Slido rgido 232


a) Sobre m1 actan P1 y T1 de modo que la ecuacin fun-
damental de la dinmica de la traslacin aplicada a dicha
masa resulta ser:

T1 - P1 = m1 a1 (1)

b) Sobre m2 actan P2 y T2 y la ecuacin fundamental de la
dinmica de la traslacin aplicada a dicha masa vendr da-
da por:

P2 - T2 = m2 a2 (2)


c) Sobre la doble polea, de masa m 3, actan las fuerzas T'1 , T' 2 , su peso P3 y la fuerza

ejercida por el eje Fe pero estas dos ltimas estn aplicadas en el eje y su momento res-

pecto del mismo es nulo. No as el momento de T'1 (que ser negativo de acuerdo con el

criterio de signos adoptado) ni el de T' 2 (que ser positivo), de modo que la ecuacin
fundamental de la dinmica de la rotacin aplicada a la doble polea ser:

T2 R - T1 r = I y si tenemos en cuenta que, por ser la masa de las cuerdas desprecia-


ble T1 = T1 y T2 = T2 , nos queda:

T2 R - T1 r = I (3)

Cmo podemos calcular la aceleracin con que se mover cada uno de los cuerpos?

Tenemos tres ecuaciones en las cuales se hallan presentes las aceleraciones que nos inter-
esa determinar. No obstante, antes de proceder a realizar ninguna operacin, nos interesa
tener en cuenta que, en este caso, la aceleracin sobre la trayectoria con que ascender el
cuerpo 1 no ser la misma que la aceleracin con que descender el cuerpo 2, ya que a 1 =
r y a2 = R, es decir, dicha aceleracin resulta ser mayor para aquel cuya cuerda est
enrollada a una polea de mayor radio.

Las ecuaciones de que disponemos son pues:

Traslacin de m1: T1 - P1 = m1 r
Traslacin de m2: P2 - T2 = m2 R
Rotacin de m3: T2 R - T1 r = I

De ellas podemos obtener la aceleracin angular con que gira la doble polea sin ms que
despejar T1 y T2 de la primera y segunda ecuaciones y sustituir en la tercera, con lo que
nos queda:

T1 = m1 r + P1
Slido rgido 233

T2 = P2 - m2 R

(P2 - m2 R) R - (m1 r + P1 ) r = I y despejando la aceleracin angular:

P2 R P1 r 6 0'2 8 0'1
= 0075 rad/s2
m 2 R m1 r I
2 2
6 0'2 8 0'1 5
2 2

Si analizamos el resultado anterior nos podemos dar cuenta de que es dimensionalmente


homogneo (T-2 en ambos miembros), de que el signo de la aceleracin (positivo) es co-
herente con el criterio de signos adoptado (positivo en el sentido de giro de la polea), que
cuanto mayor sea la diferencia entre los momentos de las fuerza peso, mayor ser , que a
mayor inercia de la polea menor y que si P1 r fuese mayor que P2 R la aceleracin angu-
lar saldra negativa (es decir, la doble polea, partiendo del reposo, girara en sentido con-
trario).

El clculo de las restantes aceleraciones es inmediato ya que basta sustituir en las ex-
presiones de a1 y a2, con lo que se obtiene:

a1 = r = 0075 01 = 00075 m/s2


a2 = R = 0075 02 = 0015 m/s2

Anlogamente para calcular la tensin en cada una de las cuerdas, basta con sustituir los
resultados que tenemos en las ecuaciones correspondientes:

T1 = m1 r + P1 = 8006 N
T2 = P2 - m2 R = 5991 N

Podemos plantearnos ahora la determinacin del momento cintico del sistema respecto
del eje de giro.

Para ello tendremos que calcular el momento cintico de cada


uno de los cuerpos de que consta el sistema respecto de dicho eje
y sumarlos, es decir:

L L1 L 2 L 3 en donde cada uno de los momentos cinticos
vendr dado por:

L1 r1 p1 r1 m1 v1

L 2 r2 p 2 r2 m2 v 2

L3 I w '


El vector L 1 ser perpendicular al plano del papel y dirigido hacia adentro, e igual direc-

cin y sentido tendrn los vectores L 2 y L 3 , por lo que para calcular su suma bastar con
obtener los mdulos correspondientes y sumarlos:
Slido rgido 234

L1 = r1 m1 v1 sen = m1 v1 r = m1 w r2
L2 = r2 m2 v2 sen = m2v2 R = m2 w R2
L3 = I w

Sumando obtenemos L = m1 w r2 + m2 w R2 + I w = 532 w, es decir, el valor del


momento cintico total del sistema respecto del eje de rotacin, depender del valor que
tenga la rapidez angular en el instante en que queramos conocerlo.

34. La doble polea de la figura adjunta


tiene un radio de giro de 24 cm y una masa
m2 = 5 kg.

Sabiendo que el radio grande vale R = 40


cm y el pequeo vale r = 20 cm, determinad
el valor de F para que el cuerpo de masa
m1 = 10 kg, partiendo del reposo, alcance
una rapidez de 2 m/s tras ascender 4 m.

Considerando el sistema formado por la doble polea y el cuerpo de masa m 1, vemos que
para que dicho cuerpo ascienda se ejerce la fuerza exterior representada en la figura, la
cual har girar a la polea hacia la derecha con lo que la cuerda se arrollar sobre el disco
pequeo y el cuerpo ascender. Naturalmente, cuanto mayor sea la fuerza ejercida tanto
mayor ser la aceleracin con que ascender el cuerpo. En el problema se pide qu valor
hemos de dar a F para conseguir que la rapidez del cuerpo sea de 2 m/s despus de ascen-
der 4 m partiendo del reposo, o lo que es equivalente, para que suba con una cierta acele-
racin.

Cmo podemos determinar el valor de la fuerza que se nos pide?

Una posibilidad sera calcular la aceleracin que ha de llevar el cuerpo (a partir de los
datos de su movimiento) y despus proceder a plantear las ecuaciones de la dinmica que
rigen el movimiento del cuerpo y de la polea, tal y como hemos venido haciendo en pro-
blemas anteriores.

Otro mtodo de resolucin, que es el que vamos a desarrollar aqu, sera mediante consi-
deraciones de trabajo y energa, haciendo uso del principio de conservacin de la energa,
igualando el trabajo resultante que se realiza sobre el sistema al cambio de energa cinti-
ca que ste experimenta y despejando F de la expresin final que obtengamos. Para ello
consideraremos como estado inicial A cuando comienza a ascender el cuerpo (nivel 0 de
energa potencial gravitatoria) y como estado final B cuando haya recorrido una distancia
d = 4 m y lleve una rapidez de v1B = 2 m/s.
Slido rgido 235

a) Aplicando Wres = Ec al cuerpo 1, tendremos:

Wres A Ec1 A Ec1B Ec1A


B B

Como sobre dicho cuerpo solo actan la tensin y el


peso nos queda:

WP A WT A Ec1B Ec1A .
B B

Teniendo en cuenta que la fuerza peso es conservati-


va ( WP A Ep BA ), que el trabajo realizado por la
B

tensin ser T1 d cos0 = T1 d y que la rapidez ini-


cial v1A = 0, la expresin anterior queda como:
1
-Ep B T1 d Ec1B T1 d - m1 g d m1 v1 B 2 (1)
2

b) Para la polea (cuerpo 2), al tratarse de una rotacin pura Wrot A Ec rot2 B Ec rot2 A
B

Teniendo en cuenta que el trabajo de rotacin se puede calcular en este caso como M,
siendo M el momento resultante que acta sobre la polea (respecto del eje de giro), y que
inicialmente esta se encuentra tambin en reposo, podemos escribir:

M = Ecrot2 B y como M = T2 R - T1 r y Ecrot2 B = Iw2/2 = m2k2 w2 /2, nos queda:

(T2 R - T1 r) = m2k2 w2 /2 . Si consideramos la masa de la cuerda como despreciable


T2 = F, obtenemos finalmente:

(F R - T1 r) = m2k2 w2 /2 (2)

Cmo podemos calcular el valor de F que se nos pide?

Una posibilidad es despejar directamente de la ecuacin (2) con lo que:

m2 k 2 w 2
T1 r
F 2 .
R

Para poder hallar finalmente F necesitamos conocer T1 , y w o bien poder expresar


todo ello en funcin de otros datos conocidos.

Sabemos que = d/r (ya que d coincidir con la longitud de cuerda enrollada en la peri-
feria de la polea y sta con el arco descrito por un punto de la misma) y que w = v/r =
v1B/r. Por otra parte utilizando la ecuacin (1) podemos expresar T 1 como:
Slido rgido 236

2
m1 v 1 B
T1 m1 g
2d

2
m2 k 2 w 2 m 2 k 2 v 1B
2
m1 v 1 B
T1 r (m1 g )r
con lo que: F 2 = 2 d r 2d = 1725 N
R R

Podemos ahora analizar el resultado literal obtenido, comprobando que adems de ser
dimensionalmente homogneo (MLT-2 en ambos miembros), contempla, por ejemplo, que
cuanto mayor sea la rapidez que queramos conseguir (manteniendo constantes los dems
factores) ms fuerza se precisar. Tambin nos podemos dar cuenta de la influencia del
radio R y comprobar que cuanto mayor sea ste menor ser la fuerza necesaria (siempre a
igualdad de los restantes factores). Finalmente recoge casos lmite como, por ejemplo, que
si m1 y m2 fuesen despreciables tambin lo sera la fuerza F necesaria, etc.

Resolved el problema utilizando un tratamiento cinemtico-dinmico y comprobad que se


obtiene el mismo resultado.

35. Dada la doble polea de la figura deter-


minad la rapidez de cada uno de los cuerpos
cuando el 1 haya descendido 5 m. El radio
grande es R = 2 m y el radio pequeo r =
05 m. Considrese que la polea tiene un
2
radio de giro de m.
2

sol: v1 = 457 m/s; v2 = 115 m/s; w = 229 rad/s

36. Analizad el movimiento de un cilindro macizo de masa m y radio r que se encuen-


tra en reposo sobre un plano horizontal, con el que presenta un coeficiente de friccin
, al actuar una fuerza horizontal F aplicada en su eje de simetra de revolucin.

Para realizar el anlisis que se pide podemos comenzar por preguntarnos acerca de qu es
lo que ocurrira si llevsemos a cabo en la prctica la experiencia que se plantea en el
enunciado del problema.
Slido rgido 237

La realizacin de la experiencia nos mostrara que:



- Por pequea que sea la fuerza F , existe desplazamiento
(el cilindro rueda), lo que no sucedera si el cuerpo fuese
un prisma (con caras planas, sin capacidad de rodar) ya
que no existira aceleracin mientras que no se superase
el valor mximo de la fuerza de rozamiento.

- Hasta cierto valor crtico de F, el cilindro rueda sin deslizar. Eso significa que, a lo largo
de una vuelta completa del cilindro un punto cualquiera de su periferia tiene contacto una
sola vez con un nico punto del plano (decimos que la periferia toca punto a punto en el
plano). Sin embargo, a partir de ese valor crtico, se observa que el cilindro no solo rueda
sino que tambin desliza, es decir, mientras el cilindro da una vuelta, cada punto de su
periferia toca a ms de un punto del plano.

Cmo podemos interpretar estos hechos?



Al aplicar la fuerza F en el centro de masas, si el
cuerpo se encuentra inicialmente en reposo sobre
una superficie plana, se producira un movimiento

de traslacin en la misma direccin y sentido que F ,
pero entonces, el punto de contacto con el plano
deslizara y, como existe rozamiento, se producir
una fuerza de friccin que se opone a que el punto
Q del cilindro en contacto con el plano deslice, tal y
como se indica en la figura adjunta.

Cules son las ecuaciones que rigen el movimiento del cilindro en las circunstancias
descritas?

Como la trayectoria es fija y conocida, podemos aplicar un tratamiento escalar y si esco-


gemos como sentido positivo el del movimiento de traslacin y el del giro del cilindro,
podemos escribir:

Traslacin del centro de masas: F - Fr = m aC (1)


Rotacin alrededor del eje que pasa por del centro de masas: Fr r = I = mr2 /2 (2)

Para poder continuar el anlisis debemos distinguir previamente si estamos aplicando o no


una fuerza superior a la crtica.

a) Determinad la aceleracin del centro de masas, la aceleracin angular y la fuerza de


rozamiento (en funcin de la fuerza F aplicada, la masa m y el radio r del cilindro), para
el caso de que ste se mueva bajo la accin de una fuerza con un valor menor o igual al
crtico.
Slido rgido 238

En este caso el cilindro rueda sin deslizar y ello


implica que cuando un punto Q de la periferia
haya girado un ngulo , y, por tanto, haya des-
crito un arco eQ = r, el centro de masas se
habr desplazado sobre su trayectoria el mismo
espacio eC = eQ . As pues, en estas condiciones,
podremos expresar la posicin del centro de
masas eC en cualquier instante como eC = r y
derivando obtener su rapidez vC = w r y su ace-
leracin aC = r.

Utilizando esta ltima ecuacin junto con las (1) y (2) anteriores, podemos determinar lo
que se nos pide.

Traslacin del centro de masas: F - Fr = m aC (1)


Rotacin respecto eje de simetra: Fr r = mr2 /2 (2)
Rodadura sin deslizamiento: aC = r (3)

Tenemos un sistema de tres ecuaciones con tres incgnitas (a C, y Fr) que a continuacin
vamos a resolver:

De (3) obtenemos = aC /r que sustituida en (2) nos proporciona Fr = maC/2 Introduciendo


ahora esta expresin en la ecuacin (1) nos queda:

F = 3maC/2 de donde:

aC = 2F/3m y finalmente Fr = F/3 y = 2F/3mr

Conviene que nos detengamos en analizar brevemente estos resultados.

- Por pequea que sea F existir aC y el cilindro se mover.

- El valor de la fuerza de rozamiento que acta depende del valor de F de manera que si
fusemos aplicando fuerzas cada vez mayores, Fr ira tambin aumentando. Sin embargo,
aunque no hay ningn inconveniente a que podamos aumentar F indefinidamente, no ocu-
rre lo mismo con la fuerza de rozamiento ya que, como sabemos, sta no puede superar un
valor mximo. Ello hace que si vamos aumentando F lleguemos a un valor critico Fcrit
para el que la Fr tomar su valor mximo (Fr max = N = mg).

- Para valores de F superiores a Fcrit, la fuerza de rozamiento no podr aumentar y, en con-


secuencia como M = Fr r = I , tampoco podr aumentar la aceleracin angular . Sin
embargo, lo que s aumentar ser la aceleracin del centro de masas a C, de modo que en
dichas condiciones aC r por lo que en un tiempo dado el desplazamiento del centro
de masas sobre el plano ser mayor que el arco descrito por un punto de la periferia del
Slido rgido 239

cilindro, es decir, habr ms traslacin que rotacin y el cilindro adems de rodar desli-
zar sobre el plano.

Cul ser la mxima aceleracin aC posible sin deslizamiento?

Corresponder al caso de que Fr = Fr max y como aC = 2F/3m = 23Fr /3m, obtenemos que
aCmax = 23Fr max /3m = 2g, que corresponder a una fuerza actuante:

F = 3Frmax = 3 mg

b) Determinad la aceleracin del centro de masas, la aceleracin angular y la fuerza de


rozamiento (en funcin de la fuerza F aplicada, la masa m y el radio r del cilindro), para
el caso de que ste se mueva bajo la accin de una fuerza con un valor superior al crtico.

En este caso, como acabamos de ver, habr tambin deslizamiento por lo que la ecuacin
aC = r no se cumplir, pero en cambio la fuerza de rozamiento Fr ser conocida porque
presentar su valor mximo. Las ecuaciones sern pues:

F - Fr = m aC
Fr r = mr2 /2
Fr = mg
F
De las ecuaciones anteriores podemos obtener fcilmente: a C g y = 2g/r
m

Analizados estos resultado concluimos que cuando la fuerza F es tan grande que provoca
rodadura con deslizamiento, la aC coincide con la aceleracin con que se trasladara un
cuerpo si en lugar de ser cilndrico fuese prismtico (sin capacidad para rodar).

Quiere esto decir que para valores de F superiores al crtico dar lo mismo que el cuer-
po sea un cilindro que un prisma rectangular de la misma masa?

No, puesto que aunque los centros de masas de ambos se trasladen con la misma acelera-
cin, el cilindro posee tambin un movimiento de rotacin que el prisma no tiene. En
trminos de trabajo y energa, podramos decir que, mientras en el caso del prisma rectan-
gular el trabajo realizado por la fuerza de rozamiento supondra una disminucin de la
energa cintica del cuerpo, que se transformara ntegramente en energa interna, en el
caso del cilindro una parte del trabajo realizado por la fuerza de rozamiento se emplea en
aumentar la energa cintica de rotacin (si no deslizara y solo rodara todo el trabajo de
rozamiento se empleara en aumentar la energa cintica de rotacin).

Este ejercicio nos permite comprender el funcionamiento de las ruedas en los vehculos de
traccin y las enormes ventajas en cuanto a la utilizacin de la energa que supuso la in-
vencin de la rueda.
Slido rgido 240

37. Analizad el movimiento de un cilindro macizo de masa m y radio r que se deja en


reposo sobre un plano inclinado de pendiente con el que presenta un coeficiente de
friccin .

Si realizsemos las experiencias correspon-


dientes podramos darnos cuenta de que, por
pequeo que fuese el ngulo, el cilindro des-
cendera rodando por el plano. Por el contra-
rio, si se tratase de un cuerpo prismtico, co-
mo sabemos, no descendera hasta que el pla-
no presentase un cierto ngulo mnimo.

En el caso del cilindro si vamos aumentando el ngulo de inclinacin del plano llega-
remos a un valor (que llamaremos ngulo crtico C a partir del cual el cilindro comienza
a deslizar adems de rodar). Se trata pues, de un problema similar al anterior pero ahora:

F = Pt = mg sen y Fr max = N = Pn = mg cos.

Para resolver el problema comenzaremos por escribir las ecuaciones que rigen el movi-
miento del cilindro. Para ello escogeremos, como venimos haciendo, como sentido positi-
vo de la trayectoria el de traslacin del cilindro y en cuanto al giro, el sentido de rotacin
del mismo. En dichas condiciones las ecuaciones sern:

Pt - Fr = m aC (1)
Fr r = I = mr2 /2 (2)

A continuacin profundizaremos en lo que sucede con las ecuaciones anteriores segn el


ngulo sea inferior o superior al crtico:

a) Para C el cilindro rueda sin deslizamiento y se cumplir a C = r, que junto con


las ecuaciones anteriores nos dar el sistema:

(1) Pt - Fr = m aC
(2) Fr r = mr2 /2
(3) aC = r

A partir de dichas ecuaciones es fcil obtener:

aC = 2gsen/3; Fr = mgsen/3 y = 2gsen/3r

Nos detendremos, ahora, en analizar estos resultados:

- Por pequea que sea la inclinacin del plano el cilindro desciende ya que para cualquier
mayor que 0, habr aC.
Slido rgido 241

- Al aumentar aumenta aC pero tambin lo hace la fuerza de rozamiento Fr por lo que


existir un valor de para el cual Fr ser el mximo posible, esto es Fr max = mg cos.
Para ngulos mayores se producir rodadura acompaada de deslizamiento.

Cmo podemos determinar el ngulo crtico?

Para = crit se cumplir que la fuerza de rozamiento alcanzar su valor mximo, con lo
que de acuerdo con la expresin Fr = mgsen/3, podremos escribir:

Fr max = mg sencrit /3 mg cos = mg sencrit /3 tg crit = 3

b) Para C el cilindro rueda con deslizamiento de forma que a C r y Fr = Fr max .


Las ecuaciones sern ahora:

Pt - Fr = m aC
Fr r = mr2 /2
Fr = mg cos

Pt Fr mgsen mgcos
de donde concluimos que aC = de donde:
m m

aC = g (sen- cos) y como = 2 Fr /mr = 2g cos/r

De los resultados anteriores podemos concluir que cuando el ngulo es superior al crti-
co la aceleracin del centro de masas tiene el mismo valor que la aceleracin que le co-
rrespondera a un cuerpo de forma prismtica (sin capacidad de rodar) que se dejase sobre
el mismo plano (recordemos que, sin embargo, desde el punto de vista energtico no dar
lo mismo).

38. Estudiad el movimiento de una esfera maciza de masa m y radio r que se lanza
con una rapidez v0 (solo de traslacin), a lo largo de un plano horizontal con el que
presenta un coeficiente de rozamiento .

Al lanzar una esfera (sin que se encuentre inicialmente girando) sobre un plano horizontal
en direccin tangente a dicho plano, se observa que al principio desliza y rueda pero, a
medida que pasa el tiempo, la rapidez con que se traslada va disminuyendo mientras que
su rapidez de giro aumenta hasta que, en un cierto instante, comienza a rodar sin deslizar y
el movimiento se mantiene uniforme, es decir, tanto w como v C se mantienen constantes y
cumplindose que vC = w r. En realidad este ltimo efecto no se produce tal y como lo
hemos enunciado. La experiencia lo que nos muestra es que, en cualquier caso, el movi-
miento de la esfera se va debilitando hasta que finalmente sta se para. Sin embargo, ello
no es debido a un fallo en los razonamientos sino a que ningn cuerpo es totalmente rgido
(como estamos suponiendo en este captulo) y entre la superficie de la esfera y el plano se
producen deformaciones que complicaran el anlisis del problema pero cuyo resultado
ser frenar el cuerpo. De hecho ese efecto al que nos referimos es tanto ms acusado cuan-
to ms rgido es el material de la esfera y del plano.
Slido rgido 242

A continuacin vamos a proceder a realizar un estudio cinemtico-dinmico del movi-


miento de la esfera.

Al lanzar la esfera todos sus puntos tiene nicamente


movimiento de traslacin (no se encuentra inicial-
mente girando) con una rapidez v0. El punto que se
encuentra en contacto con el plano horizontal desli-
zar pues con dicha rapidez, pero debido a ese desli-
zamiento se produce una fuerza de rozamiento que
ser la mxima (Fr max = N = mg), en el sentido
especificado en la figura.

Si escogemos como sentido positivo el del movimiento de la esfera tanto en la traslacin


como en la rotacin:

Traslacin del centro de masas: - Fr max = m aC


Rotacin del cuerpo: M = I

siendo M el momento de la fuerza de rozamiento mxima (ya que tanto P como R tienen

momento nulo), respecto del eje de giro. El valor de M ser positivo ( M tiene el mismo

sentido positivo que w ). As pues tendremos el sistema:

- Fr max = m aC
M = Fr max r = I = 2mr2 /5

A partir de estas ecuaciones obtenemos:

aC = - Fr max/m ; = 5 Fr max/2mr y considerando que Fr max = mg nos queda:

aC = - g y = 5g/2r

Podemos ahora analizar los resultados obtenidos, prestando atencin particular a los
signos que les acompaan.

- Como vemos, la aceleracin del centro de masas es constante y negativa, lo que implica
que la rapidez inicial del mismo ir decreciendo de manera lineal conforme la esfera
avance: vC = vC0 + aC t vC = vC 0 - g t

- La aceleracin angular tambin es constante, pero positiva, lo que implica que la rapidez
angular con que gira la esfera alrededor de su centro de masas ir aumentando de manera
lineal: w = w0 + t w = (5g/2r) t
Slido rgido 243

Si la rapidez vC va disminuyendo desde vC 0 mientras que w va aumentando desde 0, lle-


gar un instante t en el que se cumplir que vC = w r. En ese instante, como sabemos, la
esfera solo rueda. Al no haber deslizamiento ni tampoco ninguna fuerza resultante exte-
rior, deja de haber fuerza de rozamiento con el plano (si no fuese as la rapidez angular de
la esfera aumentara sin lmite, lo cual es absurdo) y tanto vC como w permanecen cons-
tantes (suponemos que tanto la esfera como el plano son slidos totalmente rgidos).

En qu instante la esfera adquiere el movimiento de rodadura puro?

Igualando las expresiones de vC y wr tenemos vC 0 - g t = 5g t/2 y de aqu

t = 2vC0 /7g

El resultado obtenido nos muestra que cuanto mayor fuese la rapidez inicial v C0 y menor
el coeficiente de friccin, tanto ms tiempo tardara en aparecer la rodadura pura. Incluso,
si no hubiese rozamiento la rodadura pura no aparecera nunca.

Para qu valor de vC aparecer la rodadura pura?

Para resolver esta cuestin bastar sustituir t en la expresin de vC con lo que:

vC = vC 0 - g t = vC 0 - g 2vC0 /7g = 5vC0 /7

39. Estudiad el movimiento de una esfera maciza de masa m y radio r que se deja gi-
rando con rapidez angular w 0 (sin traslacin), sobre una superficie horizontal con la
que presenta un coeficiente de rozamiento .

Al dejar la esfera, que se encuentra girando, sobre el plano, se aprecia que avanza pero
inicialmente rueda ms que se traslada, porque resbala sobre el plano. Por tanto, rueda y
desliza. Sin embargo, a medida que transcurre el tiempo, se observa que la rapidez de tras-
lacin va aumentando mientras que la de rotacin va disminuyendo, hasta que, en cierto
instante, comienza a rodar sin deslizar y el movimiento se mantiene uniforme, es decir,
tanto vC como w se mantienen constantes y se cumple que vC = wr (al igual que suceda
en el problema anterior).

Para realizar un estudio del movimiento de la esfera, conviene que, partiendo de las con-
sideraciones anteriores, se proceda a obtener las ecuaciones que regirn el movimiento de
la misma.
Slido rgido 244

Tomaremos como sentidos positivos los de traslacin y


rotacin de la esfera. Por otra parte, hemos de tener en
cuenta que al dejar la esfera sobre el plano vC = 0 pero
existe una rapidez angular inicial w0, de manera que el
punto de la esfera en contacto con el plano intentar1
deslizar hacia la izquierda pero el rozamiento existente
tratar de impedirlo ejercindose una fuerza de rozamien-
to sobre la esfera como la que se muestra en la figura.

Esta fuerza se interpreta como la reaccin que el plano ejerce sobre la esfera como con-
secuencia de la accin que la esfera ejerce sobre el plano. De no existir rozamiento la
esfera deslizara (patinara) sin aparecer sobre ella fuerza alguna y permanecera en el
lugar que la dejsemos girando con rapidez angular constante.

Por otra parte, al haber deslizamiento, la fuerza de rozamiento presentar su valor mximo
(Fr max = N = mg). El momento de la misma respecto del eje de rotacin de la esfera

ser negativo (porque el vector M tiene, en este caso, sentido contrario a w , oponindose
a la rotacin). Las ecuaciones que rigen el movimiento de traslacin y de rotacin de la
esfera sern, pues, las siguientes:

Traslacin: Fr max = m aC
Rotacin: - (Fr max r) = I = 2mr2 /5

A partir de la dos ecuaciones anteriores se obtiene: a C = Fr max/m ; = - 5 Fr max/2mr


y considerando que Fr max = mg nos queda:

aC = g y = - 5g/2r

Podemos ahora analizar los resultados obtenidos, prestando atencin particular a los
signos que les acompaan.

-Si vale 0, no existirn ni aC ni y, como ya hemos sealado, la esfera permanecer


rodando donde se haya dejado.
- Existe una aceleracin aC constante y positiva que har que conforme transcurra el tiem-
po, la vC, que inicialmente era 0, vaya aumentando linealmente de acuerdo con la ecua-
cin: vC = vC 0 + aC t vC = g t
- La aceleracin angular tambin es constante, pero negativa, lo que implica que la rapidez
angular con que gira la esfera alrededor de su centro de masas ir disminuyendo desde su
valor inicial w0 de manera lineal segn: w = w0 + t w = w0 - 5g/2r t

Si la rapidez vC va aumentando desde 0 mientras que w va disminuyendo desde w0, llegar


un instante t en el que se cumplir que vC = w r. En ese instante, como sabemos, la esfe-
ra toca punto a punto en el plano y solo rueda. Al no haber intencin de deslizamiento,

1
Atencin: "intentar", es solo una forma de hablar (por eso lo entrecomillamos) . Por supuesto el cilindro
no tiene ninguna "intencin" de hacer nada.
Slido rgido 245

deja de haber fuerza de rozamiento con el plano (si no fuese as la rapidez v C de la esfera
aumentara sin lmite, lo cual es absurdo) y tanto vC como w permanecen constantes (su-
ponemos que tanto la esfera como el plano son slidos totalmente rgidos).

Obtened el valor de t y el valor de w para los cuales se producira la rodadura pura.

Para evaluar t basta con utilizar la expresin vC = w r sustituyendo t por t y despejando


con lo que:

g t = (w0 - 5g t/2) r y de aqu

t = 2w0 /7g

El resultado obtenido nos muestra que cuanto mayor fuese la rapidez inicial w 0 y menor el
coeficiente de friccin, tanto ms tiempo tardara en aparecer la rodadura pura. Incluso, si
no hubiese rozamiento la rodadura pura no aparecera nunca (ya hemos sealado que la
esfera permanecera girando en el lugar en que se hubiese dejado). Para hallar el valor de
w para el cual comienza la rodadura pura bastar sustituir t en la expresin de w con lo
que: w = w0 - 5g/2r t = w0 - (5g/2r) 2w0 /7g = 2w0/7

Qu diferencia hay en la fuerza de rozamiento con el suelo, entre un vehculo (con rue-
das) movido porque es empujado desde fuera y otro que lo hace gracias a un motor?

En el primer caso la fuerza de rozamiento se opone a la traslacin, mientras que en el se-


gundo tiene la misma direccin y sentido que sta. (Ved justificacin en este mismo pro-
blema y anteriores).

40. Estudiad el movimiento de un cilindro de masa m y radio r que se lanza, en trasla-


cin, desde la base de un plano inclinado un ngulo con rapidez inicial v0 (coeficien-
te de friccin ).

Al lanzar el cilindro se aprecia que asciende por el plano si bien se traslada ms que rueda
y, por tanto, se est produciendo una rodadura con deslizamiento. A medida que asciende,
la rapidez de traslacin disminuye mientras que la de rotacin aumenta . Llegar un ins-
tante en el que se cumplir que vC = w r. A partir de dicho instante, se observa que el
cilindro sigue ascendiendo pero sin deslizar y su vC y w van disminuyendo hasta que el
cilindro se para y ambas valen 0.

Al lanzar el cuerpo con una rapidez de


traslacin v0, sobre los puntos en contacto
con el plano inclinado actuar una fuerza
de rozamiento que se opondr al desliza-
miento de stos, sin conseguirlo, por lo
que tomar su valor mximo Fr max .
Slido rgido 246

El momento de la fuerza de rozamiento har que el cilindro comience a girar hacia la de-
recha y escogiendo como sentidos positivos los de traslacin y rotacin del cilindro, po-
demos establecer las ecuaciones que rigen el movimiento del mismo

Si tenemos en cuenta que ahora, a lo largo de la trayectoria, adems de la fuerza de roza-



miento acta la fuerza Pt componente tangencial del peso, la ecuacin que rige la trasla-
cin del centro de masas ser: - Pt - Fr max = m aC

La fuerza de rozamiento, por otra parte, es la nica cuyo momento respecto al eje de rota-
cin no es nulo, sino que ser positivo y valdr: M = Fr max r = I

De las dos ecuaciones anteriores obtenemos:

aC = -g (sen + cos) y = 2gcos/r

Analizad los resultados obtenidos y razonad en qu instante se producir la rodadura


pura y cuanto valdr vC en ese momento.

Como vemos la vC0 ir disminuyendo con una aC constante mientras que w ir aumentan-
do desde 0 con constante. Las ecuaciones cinemticas correspondientes sern:

vC = vC0 - g (sen + cos)t


w = 2gtcos/r

y en cierto instante t se cumplir que vC = w r (rodadura pura) de modo que:

vC0 - g (sen + cos)t = (2gtcos/r)r de donde podemos despejar t que resulta:

t = vC0/g (sen + 3cos)

En ese instante t el centro de masas se trasladar con v C = vC0 - g (sen + cos)t y


sustituyendo t':
g ( sen cos ) vC 2 vC
v ' C vC 0
v'C 0

0
g ( sen 3 cos ) 3 tg

Qu ocurrir a partir de ese momento?

A partir del instante t desaparecer la Fr max actuan-


te pero como existe Pt que disminuye vC pero no
altera la w, el cilindro intentar rodar ms que
trasladarse pero esto significa deslizar (patinar) y
como existe un coeficiente de friccin, se producir
una fuerza de rozamiento Fr que se opondr a ello,
actuando sobre el cilindro en el mismo sentido en
que ste se traslada. A partir de esta posicin se tra-
tar de una rodadura pura y las ecuaciones que re-
girn el movimiento sern:
Slido rgido 247

Para la traslacin del centro de masas: - Pt + Fr = m aC


Para la rotacin en torno al eje de simetra: - (Fr r) = I = (mr2 /2)
y se cumplir que aC = r

De las ecuaciones anteriores obtenemos que: aC = -2g sen/3 y = -2g sen/3r

Cmo podemos determinar el tiempo que tardar en detenerse el cilindro?

Las expresiones de la aceleracin obtenidas nos muestran que el centro de masas se mue-
ve con aceleracin constante y negativa lo que nos indica que su rapidez disminuir line-
almente con el tiempo. Si, por comodidad, tomamos nuevos orgenes de espacio y de
tiempo escogiendo e0 = 0 y t0 = 0 en la posicin e instante en que la fuerza de rozamiento
cambia de sentido, la ecuacin que nos da vC ser:

vC = vC - (2g sen/3)t y se detendr cuando vC = 0 es decir 0 = vC - (2g sen/3)t de


donde obtenemos t = 3vC /2g sen como el tiempo que tardar en detenerse (contado des-
de el instante en que empez la rodadura pura).

41. Un cilindro macizo, de 2 kg de masa y 50 cm de radio, asciende rodando y sin des-


lizar, por un plano inclinado de 300. Determinad la altura h hasta la que llegar (me-
dida sobre la base del plano), sabiendo que en dicha base la rapidez de su centro de
masas era de 4 m/s.

Para resolver el problema comenzaremos por realizar un estudio cualitativo de la situa-


cin, analizando las fuerzas que actan y las ecuaciones que van a regir el movimiento.

En primer lugar, hemos de tener en cuenta que para que el cilindro ascienda rodando sin
deslizar, es necesario suponer (aunque no se diga explcitamente en el enunciado), que el
centro de masas del cuerpo en el momento en que est en la base del plano ya posee una
rapidez inicial vC 0 tal que vC0 = w0r, es decir, que ya rueda sin deslizar.

En principio,
sobre el cilindro actan las fuer-
zas peso P y la componente normal R de la
fuerza ejercida por el plano. Como R se anula
con Pn , podemos suponer que la fuerza resul-

tante sobre el cilindro ser Pt . sta fuerza, que
acta sobre el centro de masas del cilindro,
solo producir una aceleracin del mismo a C,
que har que vC0 vaya disminuyendo. Pero en
cuanto esto empezara a suceder se cumplira
que vC sera menor que w0 r, es decir, que la
periferia del cilindro girara ms aprisa de lo
que se trasladara su centro de masas, produ-
cindose un deslizamiento.
Slido rgido 248

Sin embargo, como hay rozamiento, en el mismo momento en que la periferia del cilindro
intenta deslizar empujando al plano hacia atrs, se produce una fuerza de rozamiento que
empuja el cuerpo hacia adelante y que se opone al deslizamiento a costa de disminuir (debido
al momento de dicha fuerza respecto del eje de giro) la rapidez angular del cilindro.

De esta forma se cumple que vC = w r mientras el cilindro ascienda, lo cual explica que
este solo ruede hasta pararse, tal y como confirman las observaciones experimentales. Si
llevamos a cabo un tratamiento escalar tomando como positivos los sentidos de traslacin
y rotacin, tendremos que:

Traslacin: - Pt + Fr = m aC
Rotacin: - (Fr r) = I

Observemos que, al no haber deslizamiento, Fr no tomar su valor mximo y adems


aC = r, lo que nos permite obtener:
aC = -2gsen/3

y concluir que el centro de masas del cilindro lleva un movimiento uniformemente acele-
rado, de manera que su rapidez ir disminuyendo linealmente desde v C0 hasta 0 en el ins-
tante en que alcance la mxima altura h sobre la base del plano. La fuerza de rozamiento
que se dar es Fr = mgsen/3 y analizando este resultado vemos que si el ngulo fuera
tal que el valor de la Fr necesaria superase a la Frmax, no se podra dar este movimiento (la
fuerza de rozamiento nunca puede superar su valor mximo) y existira deslizamiento (ro-
dara ms que se trasladara).

Cmo podemos calcular la altura que alcanzar el cilindro sobre la base del plano?

Una posibilidad es escoger la base del plano como origen de espacios (ver figura) y t 0 = 0
el instante en que el cilindro se encuentra en ese punto. A continuacin aplicar las ecua-
ciones del movimiento uniformemente acelerado para obtener la posicin "e" en el instan-
te en que se pare y finalmente tener en cuenta que, segn se puede ver en la figura, h = e
sen. En efecto:

vC = vC0 + aC t
e = vC0t + aCt2/2

En el momento que alcance la altura mxima la rapidez ser nula, luego haciendo v C = 0
en la primera ecuacin podemos despejar t y sustituir en la segunda para obtener el espa-
cio:

v 2CO 3v 2CO 3 vC
2

e = y como h = e sen, obtenemos finalmente h = O = 12 m


2a C 4gsen 4 g

Si analizamos el resultado literal obtenido, nos daremos cuenta de que es dimensional-


mente homogneo (L en ambos miembros) y que contempla casos evidentes como, por
ejemplo, que cuanto mayor sea la rapidez inicial y menor la aceleracin de la gravedad,
mayor ser h; que si la gravedad g tendiera a 0, h tendera a , etc. Adems, podemos ver
Slido rgido 249

tambin, que en las condiciones en que se desarrolla el problema, la altura mxima alcan-
zada resulta ser independiente el ngulo de inclinacin del plano (el cual si tendr in-
fluencia, junto con el coeficiente de friccin, para que el cuerpo ascienda rodando sin des-
lizar).

Sugerid y llevad a cabo otro procedimiento alternativo para calcular la altura mxima.

Otro procedimiento para resolver este mismo problema consiste en utilizar las relaciones
entre trabajo y energa. Al igual que en el tratamiento por cinemtica y dinmica hemos
venido descomponiendo el movimiento en traslacin y rotacin, aqu tambin haremos lo
mismo. Para ello escogeremos dos estados que designaremos como A (al inicio del plano)
y B (cuando el cilindro se detiene) y aplicaremos la expresin que relaciona el trabajo
resultante con la variacin de energa cintica entre estos estados y a cada movimiento
(traslacin y rotacin) por separado:

Traslacin: WFr A WP A Ec tB Ec tA Fr d cos0 Ep A Ep B Ec tB Ec tA


B B

WFr A = Ec rB Ec rA Fr r = Ec rB Ec rA
B
Rotacin:

Si sumamos las dos ecuaciones anteriores obtendremos el trabajo resultante total:

Fr d Fr r Ec tA + Ec rA + Ep A = Ec tB + Ec rB Ep B

y como el cilindro rueda sin deslizar, se debe cumplir que la distancia recorrida por el
centro de masas sea la girada por un punto de la periferia del cilindro, es decir d = r, de
modo que sustituyendo en la expresin anterior queda como:

Ec tA + Ec rA + Ep A = Ec tB + Ec rB Ep B

La simplificacin producida nos muestra que la energa cintica y potencial se conserva


y puede interpretarse admitiendo que la energa que se pierde en la rotacin del cilindro
por efecto del trabajo que realiza Fr (negativo) pasa ntegramente a incrementar la trasla-
cin (trabajo positivo). As pues, en adelante siempre que se produzca una rodadura
pura, podremos ignorar el trabajo realizado por la fuerza de rozamiento, puesto que
resulta ser nulo.

Si escogemos como nivel 0 para la energa potencial gravitatoria la base del plano, po-
dremos escribir:

Ec tA + Ec rA = Ep B y sustituyendo cada energa por su expresin:

2
1 1 3 3 vC
mv 2A I w 2A mgh . Como I = mr2/2 queda: v 2A gh h = O = 12 m
2 2 4 4 g
Slido rgido 250

42. Un cilindro macizo, de 4 kg de masa y 50 cm de radio, se abandona a una altura


de 5 metros sobre un plano inclinado 30. Sabiendo que baja rodando sin deslizar,
calculad cul ser su rapidez angular en el instante en que llegue a la base.

Antes de resolver el problema, conviene proceder a un estudio cualitativo de la situacin.

Cuando se abandona un cilindro sobre un plano inclinado podemos comprobar que, para
valores pequeos del ngulo de inclinacin , este desciende sin deslizar, pero que al au-
mentar llegaramos a un valor critico (crit) a partir del cual el cilindro desciende ro-
dando y deslizando. Como en el enunciado del problema nos dicen que el cilindro baja
solo rodando, ello implica que el valor de dicho ngulo crtico ser, en este caso, superior
a 30.

Recordemos que mientras no haya deslizamiento entre dos superficies, la fuerza de fric-
cin que se estar dando podr tener cualquier valor comprendido entre 0 y un valor
mximo Frmax. Si hay deslizamiento la fuerza de rozamiento tomar siempre su valor
mximo. Aplicado todo esto a nuestro caso, nos lleva a concluir que la fuerza de roza-
miento ser tanto mayor cuanto mayor sea el ngulo de inclinacin del plano y que se al-
canzar un valor (crit) por encima del cual la fuerza de rozamiento siempre tomar su
valor mximo y el cilindro, adems de rodar, tambin deslizar.

Con el fin de poder estudiar el movimiento del cilindro, analizaremos a continuacin qu


fuerzas actan sobre el mismo mientras desciende.

En un principio, actuarn el peso P y la com-

ponente normal R de la fuerza que ejerce el
plano. La suma de ambas dar Pt (vector com-
ponente tangencial de la fuerza peso) ya que,

en este caso, R y Pn (vector componente
normal de la fuerza peso) se anulan entre si.


Debido a la accin de Pt el cilindro descendera deslizando (sin rotacin) puesto que di-
cha fuerza est aplicada en el centro de masas del cilindro y, por tanto, su momento res-
pecto del eje de rotacin es nulo. Sin embargo, como existe friccin, aparecer una fuerza
de rozamiento que se opondr al deslizamiento. Qu efectos tendr esta fuerza sobre el
cilindro?

La fuerza de rozamiento produce sobre el movimiento de descenso del cilindro los si-
guientes efectos:

a) Disminuir la aceleracin aC con que desciende el centro de masas, ya que su sentido es


opuesto al de Pt

b) Provocar una rotacin en el cilindro en el mismo sentido que este avanza, debido al
momento de Fr respecto al eje de rotacin.
Slido rgido 251

Si escogemos como sentido positivo el sentido del descenso y para la rotacin el sentido
en que se produce el giro, podemos utilizar un tratamiento escalar para escribir las ecua-
ciones fundamentales de la dinmica correspondientes al movimiento de traslacin y de
rotacin del cilindro, como:

Traslacin: Pt - Fr = m aC
Rotacin: Fr r = I

Por otra parte, para valores de crit se cumplir que el cilindro rodar sin deslizar y
ello, como ya hemos visto en problemas anteriores, implica que a C = r. Teniendo en
cuenta est relacin, junto con las dos ecuaciones anteriores, podemos obtener la acelera-
cin aC, la aceleracin angular y la fuerza de rozamiento Fr .

aC = 2gsen/3; = 2gsen/3r; Fr = mgsen/3

Vemos, pues, que para un ngulo determinado (menor que el crtico), el centro de masas
del cilindro descender por el plano inclinado con aceleracin constante.

Si aumentamos , la aceleracin aumentar y tambin la Fr requerida. Ello implica que


para un cierto ngulo (que ser crit ) la fuerza de rozamiento coincidir con la mxima
Frmax = N (N es, como sabemos, el mdulo de la fuerza normal que el cuerpo ejerce so-
bre la superficie, que, en este caso, tiene el mismo valor que P n). A partir de ah, si aumen-
tamos , se producir deslizamiento ya que a C aumentar pero no podr hacerlo , de
modo que: N = mgcoscrit mgcoscrit = mgsencrit/3 y simplificando:

tg crit = /3

Luego conocido el coeficiente de rozamiento , podemos hallar fcilmente el ngulo crit


a partir del cual el cilindro bajar rodando y deslizando.

En el problema se nos pide que calculemos con qu rapidez angular llegar el cilindro a la
base del plano. Cmo podramos hacerlo?

Como el cilindro solo rueda, se cumplir en todo momento que w = v C/r. Una posibilidad
es determinar vC cuando el cilindro llegue a la base del plano, utilizando las ecuaciones
cinemticas del movimiento uniformemente acelerado. Despus bastar dividir por r para
obtener w.

Otra posibilidad, que es la que vamos a desarrollar aqu, es determinar w mediante consi-
deraciones de trabajo y energa.

Si llamamos A al estado inicial en que dejamos libre el cilindro sobre el plano inclinado y
estado B cuando llega a la base, podemos aplicar la relacin, ya conocida, entre el trabajo
resultante y la variacin de energa cintica: Wres A = Ec BA
B

Traslacin: WFr A WP A Ec tB Ec tA Fr e cos180 Ep A Ep B Ec tB Ec tA


B B
Slido rgido 252

Rotacin: WFr A Ec rB Ec rA M Fr Ec rB Ec rA
B

Sumando ahora las ecuaciones anteriores obtenemos:

Fr e Fr r Ep A Ep B Ec tB Ec tA Ec rB Ec rA

Si escogemos como nivel 0 de Ep la base del plano y tenemos en cuenta que al rodar sin
deslizar e = r, la ecuacin anterior queda como:

Ep A Ec tB Ec rB

1 1
Desarrollando la ecuacin anterior nos queda: mghA = mv 2CB Iw 2CB
2 2

1 1
Como vCB = wCB r sustituimos y obtenemos: mghA = mw 2CB r 2 Iw 2CB
2 2

2mgh A
Despejando wCB de la ecuacin anterior: w CB .
mr 2 I

4gh A
Como en un cilindro macizo I = mr2/2, nos queda w CB = 1826 rad/s
3r 2

Obtened wCB por cinemtica y dinmica comprobando que se obtiene igual resultado

43. Una varilla homognea de 1 m de longitud puede girar en torno a un eje horizon-
tal que pasa por uno de sus extremos. La colocamos verticalmente y bajo una pequea
accin cae girando. Calculad la rapidez de su centro de masas cuando pase por el
punto inferior de la trayectoria.

La varilla en su cada realizar una rotacin pura alrededor


de un eje fijo. El centro de masas de la misma describir
una trayectoria circular de radio L/2 (L es la longitud de la
varilla), por lo que cabe esperar que su velocidad en el
punto ms bajo ser tanto ms grande cuanto mayor sea L.

Cmo podramos calcular la rapidez v C del centro de


masas cuando ste pase por el punto ms bajo de la tra-
yectoria?

Podemos tratar de resolver el problema por consideraciones de cinemtica y dinmica.


Para ello tendremos que calcular la aceleracin angular y por integracin obtener w en el
Slido rgido 253

punto considerado. Finalmente aplicaremos vC = w rC. Como el plano de rotacin es in-


variable, podemos utilizar un tratamiento escalar.

Proceded a resolver el problema mediante la estrategia sugerida en el prrafo anterior.

Escogeremos como sentido positivo el que corresponde a la rotacin y para calcular la


aceleracin angular aplicaremos a la varilla la ecuacin fundamental de la dinmica de la
rotacin M = I . Para calcular el momento total M hemos de analizar los momentos de

las fuerzas que actan. El momento de la fuerza ejercida por el eje sobre la varilla ( Fe )
respecto del propio eje es nulo (porque r = 0). El nico momento, que ser, por tanto, el

total, corresponder al de la fuerza peso P de la varilla, aplicada en el centro de masas,
cuyo valor ser positivo (de acuerdo con el criterio de signos adoptado) y viene dado por:

M = P L/2 sen = mgLsen/2

que, como vemos, es variable (depende del ngulo ), lo que significa que provocar una
aceleracin angular variable.

M = I mgLsen/2 = I = mgLsen/2I y como I = mL2/3 = 3gsen/2L

As pues, si queremos conocer la rapidez del centro de masas cuando pase por la vertical,
ser preciso que obtengamos primero la rapidez angular en ese punto por integracin:

dw/dt = 3gsen/2L . Para poder integrar debemos eliminar una de las tres variables (t, w,
) para lo cual multiplicaremos ambos miembros por d:

ddw/dt = 3gsend/2L wdw = 3gsend/2L

w
3gsen w2 3g 6g
cos 0 w

w
w dw
0 0 2L
d
2 0 2L L
y como vC = w L/2

3gL
obtenemos finalmente que: v C = 387 m/s
2

Si analizamos brevemente la expresin obtenida, podemos ver que es dimensionalmente


homognea (LT-1 en ambos lados de la igualdad) y que, como caba esperar, cuanto mayor
sea la longitud de la varilla mayor ser la rapidez del centro de masas.

Sugerid y llevad a cabo otra estrategia de resolucin del problema

El problema tambin se puede resolver mediante consideraciones de trabajo y energa, el


cual, como suele ocurrir, resulta un procedimiento ms sencillo y rpido.
Slido rgido 254

Si llamamos estado A a la posicin de partida del cdm y


estado B a cuando pasa por el punto ms bajo de su trayec-
toria, podemos aplicar el teorema que nos relaciona el traba-
jo resultante con la variacin de energa cintica, entre di-
chos estados: Wres A = Ec r A y como solo participa la fuer-
B B


za peso P (que, como sabemos, es conservativa), si tenemos
en cuenta que en A la varilla est en reposo y escogemos
como nivel 0 para la energa potencial gravitatoria el del
estado B, podemos hacer:

Wres A WP A E p A con lo que:


B B B

1 2 1 1 6g
Ep AB Ec r A Ep A Ec rB mgh A Iw B mL2 w 2B w B
B

2 2 3 L

Para obtener el mismo resultado que por el procedimiento anterior, basta multiplicar por
el radio correspondiente al centro de masas (L/2).

44. Se enrolla un hilo en la periferia de un cilindro macizo de


1 kg de masa y 10 cm de radio. A continuacin, y tras sujetar
en el techo el extremo libre del hilo, se deja caer el cilindro.
Determinad la rapidez del centro de masas cuando haya des-
cendido 12 m.


El cilindro descender cada vez ms aprisa debido a la fuerza peso P que acta sobre su
centro de masas,
pero a su vez girar tambin cada vez ms aprisa, debido al momento de
la fuerza T que la cuerda ejerce sobre su periferia, de modo que podemos interpretar el
movimiento del cilindro como traslacin de su centro de masas acompaada de una rota-
cin alrededor del mismo.

Por otra parte, si nos preguntamos por los factores que influirn en la rapidez v C que lle-
var el centro de masas cuando se encuentre en una cierta posicin, cabe pensar que de-
pender de la altura que haya descendido as como del momento de inercia del cuerpo, de
forma que cuanto mayor sea la altura que lleve bajada mayor sea vC .

Cmo podemos determinar la rapidez del centro de masas que se nos pide?

Una posibilidad es mediante consideraciones de trabajo y energa. Cuando se abandona la


polea, la energa potencial gravitatoria disminuye mientras que la energa cintica aumen-
ta. Si durante el descenso sucediera que el trabajo resultante no conservativo fuese nulo, el
principio de conservacin de la energa nos llevara a establecer una igualdad entre la
disminucin de la energa potencial gravitatoria y el aumento de energa cintica produci-
do (descompuesta en energa cintica de traslacin y energa cintica de rotacin) y de ah
podramos obtener la vC .
Slido rgido 255

Proceded a resolver el problema mediante la estrategia sugerida.

Si aplicamos la expresin Wres A = Ec BA a los estados A


B

(cuando soltamos el cuerpo) y B (cuando haya descendido


12 m), tendremos:

Traslacin: Wres A Ec tB Ec tA
B

Rotacin: Wres A Ec rB Ec rA
B

Para calcular los trabajos anteriores ser necesario que con-



sideremos las fuerzas actuantes (que son P y T ).

Tanto una como otra realizarn trabajo en la traslacin, pero solo T en la rotacin, ya que

el momento de P respecto del centro de masas es nulo).

Traslacin: WTt A WP A Ec tB Ec tA
B B

Rotacin: WTr A Ec rB Ec rA
B


Si llamamos d al desplazamiento efectuado por T y al ngulo descrito, por un punto de
la periferia de la polea, en dicho desplazamiento, podemos expresar los trabajos anteriores
como:

Traslacin: T d (-1) + EpA - EpB = EctB - EctA EctA + EpA - T d = EctB + EpB
Rotacin: M = EcrB - EcrA T r = EcrB - EcrA T r + EcrA = EcrB

y teniendo en cuenta que la cuerda toca punto a punto a la periferia de la polea d = r


podemos sustituir en la ecuacin correspondiente a la rotacin r por d, sumar ambas
ecuaciones y simplificar:

EctA + EcrA + EpA -T d +T d = EctB + EcrB + EpB

EctA + EcrA + EpA = EctB + EcrB + EpB .

Esta ltima ecuacin puede interpretarse diciendo que la energa mecnica que se perder-
a en la traslacin debido al trabajo realizado por T (negativo) en realidad no se pierde
sino que es invertida en energa cintica de rotacin, de modo que, al ser nulo el trabajo
total correspondiente a T , la suma de la energa potencial gravitatoria y la energa cintica
se conserva.

Si asignamos (como hacemos habitualmente) el nivel 0 de energa potencial gravitatoria a


la posicin ms baja (situacin B), y tenemos en cuenta que se parte del reposo y que w =
vC/r, desarrollando la ltima ecuacin, nos queda:
Slido rgido 256

mv 2C mv 2C Iv 2 2mgh
mgh = + Iw2 /2 mgh = + C2 v C
2 2 2r I
m 2
r

Si analizamos la expresin obtenida para vC, nos daremos cuenta que tal y como se pens,
cuanto mayor sea la altura h descendida mayor ser vC . Finalmente, si consideramos a la
polea como equivalente a un disco macizo de su misma masa y radio, podemos sustituir
en el resultado anterior I por mr2/2 y obtener:

4gh
vC = 4 m/s.
3

45. Se dejan en libertad sobre un plano inclinado y a la misma altura dos cuerpos
(una esfera y un cilindro), de igual masa y radio. Qu relacin existe entre las rapi-
deces de sus centros de masas al llegar a la base del plano sabiendo que ambos des-
cendieron rodando sin deslizar?

sol: ve =1 035 vc

46. A un disco macizo de 20 kg de masa y 20 cm de radio se le practica un orificio


concntrico de 10 cm de radio y se le hace rodar por una superficie horizontal de
forma que cuando llega a la base de un plano inclinado (que se encuentra sobre dicha
superficie) tiene un movimiento de rodadura pura y su centro de masas se est des-
plazando a 10 m/s. Se pide:

a) Valor mximo que puede tener el ngulo del plano inclinado para que el cuerpo
ascienda sin deslizar ( = 04).
b) Altura que alcanzar hasta detenerse si asciende sin deslizar.

Cuando el disco llega a la base del plano


inclinado lo hace con rodadura pura. Al
comenzar a ascender la fuerza Pt dismi-
nuir la rapidez vC del centro de masas. Si
ello ocurriese resultara que wr sera ma-
yor que vC, con lo cual el disco girara ms
de lo que se desplazara apareciendo un
deslizamiento.

Sin embargo, en cuanto el disco trata de deslizar, como existe rozamiento con el plano, se
produce una fuerza de rozamiento de valor Fr que se opone a ello y cuyo momento har
disminuir la rapidez angular w. Si el coeficiente de rozamiento es lo bastante grande
como para que la Fr requerida para disminuir la rotacin sea menor que el valor mximo
que puede tomar Fr, se producir rodadura sin deslizamiento. En estas condiciones, to-
Slido rgido 257

mando como sentidos positivos los de traslacin y rotacin del cuerpo, las ecuaciones que
regirn el movimiento de subida del disco, son:

Traslacin: -Pt + Fr = m aC (1)


Rotacin: -Fr R = I (2)

Adems de las ecuaciones anteriores, como no hay deslizamiento, a C = R (3)

Utilizando las tres ecuaciones anteriores, podemos obtener fcilmente:

Pt mgsen Pt mgsen
aC Fr 2

I I mR mR 2
m m 2 1 1
R2 R I I

Como vemos, la fuerza de rozamiento requerida para que el cuerpo ascienda por el plano
rodando sin deslizar, aumentar conforme aumente el ngulo de inclinacin del plano,
de manera que habr una valor crtico de dicho ngulo (crit) para el cual la Fr alcanzar
su valor mximo Fr max = mgcos, de forma que, podremos hacer:

mgsen crit mR 2
Frmax mg cos crit tg crit (1 )
mR 2 I
1
I

Para obtener el valor de crit ser necesario conocer el momento de inercia del disco agu-
jereado. Para ello, si llamamos m a la masa del disco completo (20 kg), m a la del disco
agujereado, m a la masa que extraemos al agujerearlo y a la densidad superficial del
cuerpo, se cumplir que:

= m/S = 20/022 = 1592 kg/m2 y m = S = 1592 012 = 5 kg, con lo que


el momento de inercia buscado se podr obtener como:

I = IDC - Iorif = mR2/2 - m r2/2

Sustituyendo los valores numricos nos queda I = 0375 kg m 2 , que aplicado a la expre-
sin de la tg crit nos conduce finalmente a:

crit = arctg 04 (1 + 15022/0375) = 461

Cmo podemos determinar la altura mxima que alcanzar el cilindro sobre la base del
plano?

Para determinar la altura que alcanzar podemos utilizar la expresin que relaciona el
trabajo resultante con la variacin de la energa cintica, aplicndola entre los estados A
(cuando se inicia el ascenso) y B (cuando alcanza la altura mxima).
Slido rgido 258

En este caso, haremos uso, adems, de algunas de las conclusiones que ya hemos justifi-
cado reiteradamente en problemas anteriores y plantearemos directamente la ecuacin
suma del trabajo resultante correspondiente a la traslacin y a la rotacin, considerando
que al no haber deslizamiento el trabajo neto de la fuerza de rozamiento es nulo. Por otra
parte, como la nica fuerza que realiza trabajo es la fuerza peso y sta es conservativa,
podemos escribir (tomando nivel 0 de Ep gravitatoria la base del plano) que:

EctA + EcrA = EpB ya que cuando alcanza la mxima altura el disco deja de moverse.

La ecuacin anterior se interpreta como que toda la energa cintica de que dispona en la
base se invierte ntegramente en aumentar la energa potencial y desarrollndola podemos
obtener la altura buscada:

1 1 1 1 v2
mv 2CA Iw 2A mgh B mv 2CA I CA2 mgh B de donde:
2 2 2 2 R

Iv 2CA I
mv 2CA 2
1
hB R mR 2 v 2
CA
2mg 2g

sustituyendo los valore numricos queda hB = 813 m

Podemos analizar el resultado literal obtenido comprobando cmo ste adems de ser
dimensionalmente homogneo (L en ambos miembros), contempla algunos casos eviden-
tes como, por ejemplo, que cuando mayor sea la velocidad inicial del centro de masas,
mayor ser la altura que se alcanzar y que si no hubiese gravedad la altura mxima sera
infinita (no se parara nunca).

47. Un cilindro macizo de 8 kg de masa y 20 cm de radio, capaz de girar libremente


alrededor de un eje que coincide con su eje de simetra, se encuentra sobre un plano
inclinado de 300. Se realiza el montaje de la figura (la polea es de masa despreciable)
y, a continuacin, dejamos en libertad los cuerpos aprecindose que el cilindro as-
ciende por el plano rodando sin deslizar. Determinad la rapidez angular del cilindro
cuando haya recorrido 10 metros. Cunto debe valer como mnimo el coeficiente de
friccin entre el cilindro y el plano? Dato: m1 = 6 kg
Slido rgido 259


El cilindro (o cuerpo 2) asciende por el plano bajo la accin de las fuerzas T2 ejercida por

la cuerda, su peso P2 , y la componente normal R de la fuerza ejercida por el plano. En

este caso al tratarse de una trayectoria rectilnea la fuerza R se anula con la componente

normal del peso Pn de modo que el cilindro, inicialmente en reposo, ascender debido a

que T2 es mayor que la componente tangencial del peso Pt . Como ambas fuerzas se apli-
can en el centro de masas, produciran traslacin del cilindro y, en consecuencia, inten-

tar deslizar sobre el plano inclinado a lo que se opondr la fuerza de rozamiento Fr que
aparecer provocando que el cilindro ruede.

En el enunciado se dice que el cilindro asciende sin deslizar. Ello implica que en todo
momento se cumplir que vC = wr, por lo que si calculamos vC podremos obtener fcil-
mente la rapidez angular w.

De qu depender vC?

Si atendemos a las condiciones en que se desarrolla el movimiento (solo rodadura, cuerda


inextensible y sin masa, situacin inicial de reposo, inercia de la polea nula y sentido del
movimiento segn el descenso del cuerpo 1), cabe esperar que cuanto mayor sea la distan-
cia recorrida y la diferencia de masas m 1 - m2, mayor tendr que resultar vC. Por otra par-
te, es evidente que si m2 fuese nula, m1 debera caer libremente (con la aceleracin de la
gravedad) y su rapidez en cualquier instante vendra dada, como sabemos, por la expre-
sin v = 2gh .

Cmo podremos hallar vC?

Como ya sabemos (ved problemas anteriores), al no haber deslizamiento el trabajo reali-


zado por la fuerza de rozamiento
es nulo.
Por otra parte, al ser la cuerda y la polea de ma-
sas despreciables, las fuerzas T1 y T2 tienen el mismo mdulo T y realizarn el mismo

trabajo, pero uno (el de T2 ) ser positivo y el otro negativo. Todo ello hace que al no
haber trabajo neto no conservativo, la suma de la energa potencial y cintica se conserve,
lo que nos lleva a establecer que la disminucin de la energa potencial gravitatoria aso-
ciada al descenso del cuerpo 1 se traducir en un aumento de la energa cintica de los dos
cuerpos y de la energa potencial gravitatoria asociada al cuerpo 2.
Slido rgido 260

Si aplicamos la expresin Wres A = Ec BA a los estados A (cuando soltamos el cuerpo 1) y


B

B (cuando haya descendido 10 m), tendremos:

Cuerpo 1: WT A WP1A Ec 1B Ec 1A
B B

Cuerpo 2: WT A WP2 A (Ec 2tB Ec 2rB ) (Ec 2tA Ec 2rA )


B B

Como puede verse en el cuerpo 2 no figura el trabajo de la fuerza de rozamiento al ser


ste nulo. Desarrollando las expresiones anteriores nos queda:

T d (1) Ep1A Ec 1A Ep1B + Ec 1B


T d 1 Ep 2A Ec 2tA Ec 2rA Ep 2B + Ec 2tB Ec 2rB

Tomando como nivel 0 para la energa potencial gravitatoria de cada uno de los cuerpos el
que corresponde a su posicin ms baja (la situacin inicial A para el 2 y la B para el 1) y
considerando que dichos cuerpos se hallan inicialmente en reposo, podemos sumar las
expresiones anteriores y obtener finalmente que:

Ep1A = Ec1B + Ep2B + Ec2 tB + Ec2 rB

La expresin obtenida nos muestra que la energa potencial gravitatoria inicial (respecto al
nivel tomado como 0) se ha transformado en energa cintica (de rotacin y traslacin del
cuerpo 2 y traslacin del cuerpo 1) y en energa potencial gravitatoria (asociada al cuerpo
2). Desarrollando dicha expresin podemos despejar vC.

m1 v B2 m v 2 Iv 2 2gh 1A (m1 m 2 sen )


m1gh1A = m2 gh2 B 2 B B2 de modo que v B
2 2 2r I
m1 m 2 2
r

Para obtener la expresin anterior, hemos tenido en cuenta que la altura a que asciende m2
respecto su nivel inicial, viene dada por la expresin sen = h2B /h1A ya que, al ser la
cuerda inextensible, la misma distancia que desciende el cuerpo 1 en vertical recorrer el
cuerpo 2 sobre el plano.

Finalmente, sustituyendo los valores numricos:

vB = 471 m/s y w2B = vB/r = 236 rad/s

Podemos analizar el resultado literal obtenido comprobando que es dimensionalmente


homogneo (LT-1 en ambos miembros) y que adems contempla las hiptesis realizadas
anteriormente, como, por ejemplo, que cuanto mayor sea la distancia que desciende el
cuerpo 1 (o que asciende el 2) mayor es la rapidez y que, anlogamente, cuanto mayor es
la diferencia de masas m1 - m2, mayor resulta vC, o que si m2 fuese nula, m1 debera caer
libremente (con la aceleracin de la gravedad) y su rapidez en cualquier instante vendra
dada por la expresin v = 2gh , etc.
7. CAMPO GRAVITATORIO

1. Diferenciad entre masa inercial y masa gravitacional de un cuerpo.

Si aplicsemos una fuerza a un bloque situado sobre una superficie horizontal y sin roza-
miento, comprobaramos que esta ha de tener un cierto valor para producirle una acelera-
cin determinada. En ello no interviene la gravedad para nada, es decir, se requerira una
fuerza idntica, para provocarle la misma aceleracin, si el bloque estuviese situado en un
lugar de gravedad nula. A la propiedad del bloque que hace necesaria la accin de una
fuerza resultante para cambiar su estado de movimiento, se le denomina masa inercial y
cuanto mayor es su valor, mayor es tambin la fuerza resultante que se necesita para pro-
ducir una determinada aceleracin. Dicha masa inercial (mi), es la que figura en la ecua-
cin fundamental de la dinmica:

F mi a

y, como sabemos, se mide en kilogramos. Su valor, para un objeto dado, se puede obtener
fcilmente sin ms que aplicar sobre dicho objeto una fuerza resultante determinada y
medir la aceleracin producida. A continuacin, basta dividir el mdulo de la fuerza, en
N, por el de la aceleracin en m/s2, para obtener el valor de la masa inercial, en kg.

Por otra parte, sabemos que cuando abandonamos un objeto en el seno de un campo gravi-
tatorio (como ocurre con cualquier cuerpo a una cierta altura sobre la superficie de la Tie-
rra), sobre l se ejerce una cierta fuerza. En este caso, la inercia no juega ningn papel. Se
trata de otra propiedad de la materia consistente en que los cuerpos se atraen entre s (por
ejemplo un objeto y la Tierra). Esta propiedad gravitatoria fue descubierta por Newton
cuyos trabajos en este campo le llevaron a establecer que la fuerza gravitatoria entre dos
cuerpos cualesquiera era directamente proporcional a su "masa gravitacional (mg) e inver-
samente proporcional al cuadrado de la distancia entre ellos. Esto qued reflejado en la
Ley de Newton de la Gravitacin Universal:
m g1 m g2
FK
d2

Son la misma cosa la masa inercial que la masa gravitacional?

Para responder a la pregunta supongamos que se dejan en libertad varios cuerpos de masas
inerciales mi1, mi2 ...en la proximidad a la superficie de la Tierra. En ese caso podemos
comprobar experimentalmente que todos ellos caen con el mismo valor de la aceleracin
a = 981 m/s2. Aplicando la ecuacin fundamental de la dinmica podemos obtener el va-
lor de la fuerza que estar actuando sobre cada cuerpo: F1 = mi1a; F2 = mi2 a ...

Por otra parte, esas fuerzas no son otras que la atraccin gravitatoria que ejerce la Tierra
(de masa gravitacional mgT ), sobre cada uno de los cuerpos (de masas gravitacionales
mg1, mg2 ) que, de acuerdo con la Ley de Newton de la Gravitacin vendrn dadas por:
Campo gravitatorio 262

m gT m g1 m gT m g2
F1 K ; F2 K ...
d2 d2

Igualando las fuerzas y despejando obtenemos la relacin existente entre m g y mi :

m g1 K m gT m g2 K m gT mg
; ... C (siendo C una constante).
m i1 a R 2
T
m i2 a R 2
T mi

Como vemos, se cumple que ambas masas son directamente proporcionales: m g = C mi

A continuacin analizaremos algunas consecuencias de la proporcionalidad anterior:

La unidad para medir la masa inercial (elegida arbitrariamente) es el kilogramo. Nos que-
da ahora decidir en qu unidad se va a medir la masa gravitacional. Segn sea nuestra
decisin, quedar fijado el valor de la constante de proporcionalidad C. Para comprender
el alcance de tal decisin, podemos hacer el siguiente supuesto:

Situamos dos masas inerciales iguales, de 1 kg cada una, separadas entre s por una dis-
tancia de 1 m y medimos la fuerza gravitatoria con que se atraen. En ese caso obtendra-
-
mos el valor de 66710 11 N y sustituyendo en la expresin de la Ley de la Gravitacin
Universal (LGU):
m g1 m g2
6'67 10 11 K
1

Podemos ahora diferenciar entre dos opciones en la seleccin de la unidad de m g, que


llamaremos "unidad de masa gravitatoria" y designaremos, de momento, como umg.

a) Que asignemos a cada masa inercial m i = 1 kg, una masa gravitacional mg = n (umg),
siendo n cualquier nmero positivo mayor que 1, como, por ejemplo, el 5. Ello significara
que a cada kg de masa inercial le corresponderan 5 (umg) y la ecuacin anterior quedara
como:
m g1 m g2 25(umg) 2 6' 67 10 11 N m 2
6'67 10 K
11
= K con lo que K =
1 1m 2 25 (umg) 2

umg
y la masa gravitacional de cualquier cuerpo vendra dada por: mg = 5 m i ( kg)
kg

b) Que asignemos a n el valor de 1, es decir que a cada masa inercial mi = 1 kg le haga-


mos corresponder una masa gravitacional mg = 1 umg. En este caso:

m g1 m g2 1(umg)2 Nm 2

6'67 10 11
K =K de modo K valdra: K = 6'67 10 11
1 1m 2
(umg)2
Campo gravitatorio 263

umg
y la masa gravitacional de cualquier cuerpo sera m g = 1 mi
kg

Si, adems, a la unidad de masa gravitatoria la llamsemos kilogramo gravitatorio en


lugar de unidad de masa gravitatoria y la representsemos por kg en lugar de umg,
11 N m
2
tendramos que K = G = 6' 67 10 y mi = m g .
kg 2

La igualdad anterior (mi = mg), no debe interpretarse como una identidad, sino simple-
mente como una coincidencia de valores numricos debida a la decisin tomada.

kg grav
m i 1 m i
kg iner

Dicha decisin presenta la ventaja de que solo hablamos de un nico valor para medir la
masa de cada cuerpo (representar tanto la masa inercial como la masa gravitacional),
pero tambin tiene el inconveniente de que, habitualmente, se confundan dos propiedades
distintas de la materia como son la inercia y la gravitacin.

La masa inercial es una propiedad de los cuerpos debida a la imposibilidad que tienen, por
ellos mismos, de cambiar su velocidad, mientras que la masa gravitacional es la propiedad
a la que se debe la atraccin gravitatoria entre los cuerpos. La proporcionalidad existente
entre estas dos propiedades distintas, se conoce con el nombre de Principio de equivalen-
cia. El que dos propiedades de naturaleza tan distinta, sean directamente proporcionales,
se consider durante mucho tiempo como una casualidad. Sin embargo, en la actualidad
este hecho se interpreta claramente en la Teora de la Relatividad Generalizada.

Qu consecuencia tiene en la cada de los cuerpos el hecho de que la masa inercial y la


gravitacional sean directamente proporcionales?

Una consecuencia es que todos los objetos (en ausencia de rozamiento) caern al suelo
(desde pequeas alturas) con igual aceleracin o, como suele decirse, ms impropiamente,
que la aceleracin de la gravedad no dependa de la masa. En efecto, cuanto mayor sea la
masa gravitatoria de un objeto, mayor ser la fuerza con que es atrado por la Tierra , lo
que podra llevarnos a pensar que debera caer con ms aceleracin. Sin embargo ello no
ocurre as porque al ser la masa inercial directamente proporcional a la gravitatoria, cuan-
do una aumenta la otra lo hace tambin en la misma proporcin, y, si bien es cierto que la
fuerza resultante hacia abajo aumenta cuando aumenta la masa gravitacional, lo mismo
ocurre con la inercia del cuerpo a cambiar de velocidad, por lo que ambos efectos se com-
pensan.

Para pequeas alturas sobre el suelo (despreciables frente al radio de la Tierra):

m gT m g
F G cte m g de modo que la aceleracin de cada ser:
R 2T
Campo gravitatorio 264

F cte m g
a = cte
mi mi

Qu ocurrira en un mundo hipottico en el que la masa inerte y la gravitacional fuesen


inversamente proporcionales?

De acuerdo con la expresin anterior (a = cte mg/mi), si la masa gravitacional y la masa


inercial fuesen inversamente proporcionales, los objetos de elevada masa gravitatoria ca-
eran con mayor aceleracin que los de pequea masa gravitatoria. Por otra parte, los obje-
tos de masa inerte elevada como, por ejemplo, un camin, caeran muy suavemente (con
muy poca aceleracin) ya que su masa gravitacional sera tanto ms pequea cuanto ma-
yor fuese su masa inercial.

2. Es lo mismo la intensidad del campo gravitatorio terrestre que la aceleracin de


cada de un cuerpo sometido a la accin de la fuerza peso (fuerza gravitatoria ejerci-
da por la Tierra)?

En ocasiones a la magnitud fsica g (intensidad del campo gravitatorio), cuando nos refe-
rimos al campo generado por la Tierra, se le llama aceleracin de la gravedad, cuando,
conceptualmente se trata de dos magnitudes distintas (si bien coinciden numricamente en

determinadas circunstancias). La g es una magnitud que se introduce para caracterizar el
campo gravitatorio existente en un punto cualquiera. Dicha magnitud, por tanto, tomar
un cierto valor, independientemente de que en ese punto haya o no masa alguna. Por el
contrario solo podremos comenzar a hablar de aceleracin cuando en el punto considerado
se coloque una masa y se la deje en libertad.

Para profundizar en la diferenciacin de ambos conceptos, analicemos lo que sucede con


un cuerpo de masa inercial mi que se abandona a cierta altura sobre la superficie de la
Tierra. Si la altura es pequea, comparada con el radio terrestre, el cuerpo se mover
hacia la superficie de la Tierra con una aceleracin constante, que, medida experimental-
mente, resulta ser 981 m/s2. Apoyndonos en la ecuacin fundamental de la dinmica,
podemos decir que sobre el cuerpo acta una fuerza de valor:

F = mi a (siendo a = 981 m/s2).

Esta fuerza es, como sabemos, la fuerza gravitacional ejercida por la Tierra sobre el cuer-
po, que se puede expresar como:

F = = mg g

(siendo g la intensidad del campo gravitatorio terrestre en el punto en donde se deja el


cuerpo y mg la masa gravitacional de este).

mi
Igualando ambas fuerzas: mi a mg g g a
mg
Campo gravitatorio 265

Si tenemos en cuanta ahora que los valores de ambas masas coinciden, obtenemos que:

kg iner m N
g 1 9'81 2 9'81
kg grav s kg grav

y, dado que la masa inercial mi y la gravitacional mg son conceptualmente diferentes,


aunque, por conveniencia, se hayan hecho coincidir numricamente (ved ejercicio ante-
rior), lo mismo ocurrir con la aceleracin "a" y la intensidad del campo gravitatorio "g".
Ambas tendrn el mismo valor numrico. No obstante, la unidad que conviene que acom-
pae a la intensidad del campo gravitatorio ser N/kg y no m/s2 ya que, evidentemente: 1
m/s2 = 1N/kg (masa inercial) pero 1 m/s2 1 N/kg (masa gravitacional). Si la unidad de
masa gravitacional hubiese recibido, en lugar de kilogramo, otro nombre distinto, esta
confusin no se dara.

3. Las masas m1, m2, y m3, de la figura Y(m)


adjunta valen 8103 kg, 9103 kg y 103 kg,
m1
respectivamente. Se pide:

a) La intensidad del campo gravitatorio


en el punto (4, 3) m.
b) Fuerza que actuara sobre una masa de
m3 m2 X (m)
10 kg situada en dicho punto.

Como el campo gravitatorio se debe, en este caso, a tres masas, la intensidad en un punto
cualquiera del mismo, se obtendr como la suma de la intensidad correspondiente a cada

una de las masas por separado, esto es: g g 1 g 2 g 3

Y(m)
Para determinar g podemos expresar cada ur1 g1
uno de los vectores en funcin de un vector m1 P

unitario u r que vaya desde la masa que crea g3 g2
el campo hacia el punto en el que se desea
conocer la intensidad, tal y como se propone ur3
ur2
en la figura adjunta:
O m3 m2 X (m)

Expresad los vectores intensidad de campo en P, en funcin de los correspondientes vec-


tores unitarios:

Gm 8 103 -
g 1 2 1 u r1 = 6'67 1011 2 (1 , 0) = (-33410 8 , 0) N/kg
d1 4
Campo gravitatorio 266

Gm 9 103 -
g 2 2 2 u r2 = 6'67 1011 2 (0 , 1) = (0 , -66710 8 ) N/kg
d2 3

Gm
g 3 2 3 u r3 Cmo podemos determinar d3 y u r3 en este caso?
d3

Un procedimiento que nos permite evaluar d 3 y u r3 (incluso cuando trabajemos con tres
componentes) es el siguiente:

Consideramos el vector OP = (4 , 3) m. Su mdulo ser OP = 4 2 32 = 5 m y coinci-


dir con la distancia d3. Si dividimos ahora dicho vector por su propio mdulo, obtendre-

mos el vector unitario u r3 :

u r3 = OP/OP = (4/5 , 3/5), de modo que sustituyendo en la expresin de g 3 nos queda:
3
Gm 11 10 4 3 - -
g 3 2 3 u r3 = 6'67 10 2 ( , ) = (-21310 9 , -16 10 9) N/kg
d3 5 5 5


Podemos ahora calcular g g 1 g 2 g 3 simplemente sumando analticamente los vec-
tores obtenidos, con lo que resulta:

g = (-33410-8, 0)+ (0, -66710-8 )+ (-21310-9 , -16 10-9 ) = (-35510-8, -68310-8 ) N/kg

Cmo podemos hallar la fuerza a que estar sometido un cuerpo cuando se coloque en
el punto P considerado?

La intensidad del campo gravitatorio en un punto, es una caracterstica intrnseca del


campo y su valor coincide con el de la fuerza que actuara sobre la unidad de masa si la
colocsemos en dicho punto (de este modo, podemos comparar las intensidades corres-
pondientes a distintos puntos del campo). Esto hace, que si conocemos la intensidad en un
punto P, resulte sencillo determinar la fuerza gravitatoriaque actuar sobre cualquier ma-

sa que coloquemos en ese punto, mediante la expresin: F m g , lo que aplicado a nues-
tro caso ser:
- - - -
F m g = 10 (-35510 8, -68310 8 ) = (-35510 8, -683 10 8 ) N

4. Determinad el valor de la Energa potencial gravitatoria de una masa m1 que se


encuentra en el seno del campo gravitatorio generado por otra masa m2.

Supongamos un cuerpo de masa m1 que se mueve desde un punto A hasta otro punto B
sometido a la accin del campo gravitatorio generado por otro cuerpo de masa m 2. El tra-
bajo realizado por la fuerza gravitatoria sobre la masa m1 ser:

rB


WAB F dr
rA
Campo gravitatorio 267

G m 2 m1
donde la fuerza ser F u r y por tanto el trabajo se podr expresar como:
r2

rB G m 2 m1 rB u dr
W
B
A u r dr = G m 2 m1 r
.
rA r2 rA r2

Para resolver esta integral necesitamos, en


principio, conocer la trayectoria seguida por el
cuerpo. Tomemos una trayectoria particular-
mente sencilla (y til, como veremos a conti-
nuacin) representada en la figura adjunta.

Podemos descomponer el trabajo en dos sumandos: WAB WAC WCB y, como en el tra-
yecto AC la fuerza es siempre perpendicular a la trayectoria (se trata de un trayecto circu-

lar con centro en m2), el trabajo realizado por F a lo largo del mismo ser nulo, luego:

rB u r dr
W W
B
A
B
C = G m 2 m1
rC r2

siendo u r un vector unitario que, en todo momento tendr la misma direccin y sentido

que el vector r , es decir: r = r u r , y, como en esta segunda parte del trayecto, dr tiene la

misma direccin que el vector unitario u r , podemos expresarlo como dr = dr u r (donde el
escalar dr ser positivo si nos alejamos de m 2 y negativo si nos aproximamos a ella).

En estas condiciones u r dr (u r u r ) dr dr y sustituyendo en la integral:

rB
u r dr
rB rB dr 1
W W = Gm 2 m1
B
A
B
C = G m 2 m1 2 = G m 2 m1
rC r2 rA r
r rC
y como rC vale lo mismo que rA, tendremos:

1 1 Gm 2 m1 Gm 2 m1
r
1
B

WAB Gm 2 m1 Gm 2 m1
r rA rB rA rB rA

Qu sucedera para cualquier otra trayectoria entre los puntos A y B?

No es difcil darse cuenta que siempre podramos


descomponerla en una serie de infinitos y sucesi-
vos desplazamientos infinitesimales segn un
arco (centrado en m2) y radiales, tal y como se
aprecia en la figura adjunta. En sta la lnea que-
brada que se forma no coincide exactamente con
la trayectoria, pero es fcil imaginar que para
desplazamientos infinitesimales s que coincidir-
a prcticamente.
Campo gravitatorio 268

Como solo existe trabajo en los desplazamientos radiales, el trabajo total a lo largo de la
trayectoria indicada se podr evaluar como la suma de los correspondientes a los despla-
zamientos radiales infinitesimales y su valor coincidir con el de antes ya que llegaramos
a la misma expresin en funcin de rA y rB .

As pues, sea cual sea el trayecto seguido entre A y B, el trabajo realizado por F ser el
mismo y, por tanto, la fuerza gravitatoria es una fuerza conservativa. Si recordamos el
concepto de funcin energa potencial, podemos escribir:

WAB Ep = (Ep B Ep A ) e igualando con la expresin del trabajo obtenida antes:

Gm 2 m1 Gm 2 m1 Gm 2 m1
= (Ep B Ep A ) , lo que implica que: Ep = C
rB rA r

En la expresin obtenida C es una constante, y su valor depender de la situacin para la


que nos convenga tomar Ep = 0. Lo habitual, por ser lo ms cmodo, es definir dicha si-
tuacin de modo que C resulte 0. Ello supone que para r = hagamos Ep = 0, ya que as
se cumple:

Gm 2 m1
0= C C=0

Recordemos que la energa potencial (gravitatoria o de otro tipo) es una magnitud fsica
que permite determinar de forma sencilla el trabajo realizado por una fuerza conservativa
cuando el cuerpo sobre el que acta se desplaza desde un punto A a otro B. Dicho trabajo,
como acabamos de ver, se puede evaluar como una diferencia de Ep entre esos dos pun-
tos, que es independiente de la forma de la trayectoria, de modo que lo que realmente tie-
ne significado es hablar de variaciones de Ep, pero no de valores absolutos de Ep. La Ep
puede tomar infinitos valores distintos (tanto como valores distintos podamos asignar a la
constante C), mientras que Ep solo tendr un valor.

Una vez que definimos Ep = 0 para r = qu interpretacin fsica puede tener el afir-
mar que la Ep de un cuerpo de masa m1 situado en un punto Q de un campo gravitatorio
creado por otro cuerpo de masa m2 es Ep = -Gm2 m1 /r?

Si analizamos el trabajo realizado por la fuerza gravitatoria cuando el cuerpo de masa m 1


se traslada desde Q hasta el , podemos escribir: WQ Ep Q Ep Ep Q y, de aqu
concluimos el significado fsico que podemos dar a la Ep de m 1 situada en un punto dado
de un campo gravitatorio: Es una magnitud cuyo valor nos indica el trabajo que realizara
la fuerza gravitatoria cuando dicha masa se aleje a una distancia infinita de dicho punto.

5. Diferenciad entre Energa potencial de una masa en una distribucin de masas y


Energa potencial de la distribucin.

Para comenzar, supongamos un sistema formado por un conjunto de cuerpos cuyas masas
se puedan considerar como puntuales. Cul sera la Ep gravitatoria de una de esas masas?
Campo gravitatorio 269

Si seguimos la definicin dada en el ejercicio anterior, el valor de esa energa potencial


debera de coincidir con el del trabajo realizado por la fuerza gravitatoria cuando dicha
masa se trasladase desde la posicin que ocupa hasta el infinito. Para simplificar, vamos a
considerar un sistema formado solo por tres masas m1, m2 y m3 puntuales, como el repre-
sentado en la figura adjunta.

Cuando m1 de traslada desde su posicin inicial Q hasta el infinito, el trabajo realizado


por la fuerza gravitatoria que acta sobre ella se podr evaluar como la suma de los traba-
jos realizados por cada una de las fuerzas gravitatorias actuantes sobre m 1:


WQ WF2 Q WF3 Q y, por tanto, la Ep de la masa m1 en la distribucin ser:

Gm 2 m1 Gm 3 m1
Ep1 = Ep12 + Ep13 =
r12 r13

Es decir: La Ep de m1 ser la suma de las energas potenciales que a dicha masa le corres-
pondern por el hecho de encontrarse en el seno del campo gravitatorio de cada una de las
restantes masas que conforman el sistema.

De igual forma podramos evaluar la energa potencial gravitatoria de la masa m2 y la de


la masa m3 como Ep2 = Ep21 + Ep23 y Ep3 = Ep31 + Ep32 respectivamente.

Cmo podramos definir ahora la Ep gravitatoria de una distribucin de masas?

Por paralelismo con la definicin establecida para una masa en un sistema, podramos
decir que: La energa potencial gravitatoria de un sistema es una magnitud cuyo valor co-
incide con el trabajo total realizado por la fuerza gravitatoria cuando se trasladen todas
estas masas (sucesivamente) desde la posicin que ocupan en la distribucin hasta el infi-
nito.

Para evaluarla bastar, pues, con imaginar que vamos llevando sucesivamente cada una de
las n masas puntuales de que consta la distribucin, desde su posicin inicial hasta el infi-
nito y calcular, en cada caso, el trabajo realizado por la fuerza gravitatoria del campo
creado por las masas que queden. La suma de todos esos trabajos coincidir con la Ep
gravitatoria del sistema.

Para simplificar los clculos, consideraremos, de nue-


vo un sistema formado solo por tres masas puntuales
m1, m2 y m3, y comenzaremos por calcular el trabajo
realizado por la fuerza gravitatoria cuando m 1 se tras-
lada hasta el infinito:
Campo gravitatorio 270

Gm 2 m1 Gm 3 m1
WA Ep 12 Ep 13 = . Si ahora se traslada m2 hasta el infinito:
r12 r13
Gm 3 m 2
WB Ep 23 = (ya que solo queda m3 en la distribucin). Finalmente, para m3:
r23
WC 0 (ya que al estar cada una de las otras masas a una distancia la fuerza ser nula).

Gm 2 m1 Gm 3 m1 Gm 3 m 2
Luego la energa potencial del sistema ser: Epsis =
r12 r13 r23

Comparad la expresin que acabamos de obtener con la que resultara de sumar las Ep
correspondientes a cada una de las tres masas consideradas.

Si sumamos las Ep de cada una de las tres masas (Ep1 + Ep2 + Ep3) obtenemos:

Gm 2 m1 Gm 3 m1 Gm1 m 2 Gm 3 m 2 Gm1m 3 Gm 2 m 3
+
r12 r13 r21 r23 r31 r32

Como podemos ver: La suma de las energas potenciales de cada una de las partculas de
un sistema no coincide con la energa potencial correspondiente a dicho sistema, aunque,
ambas magnitudes estn relacionadas, ya que:

1 1 in
Epsis =
2
(Ep 1 Ep 2 Ep 3 ) y, en general: Epsis Ep i
2 i 1

Otra forma, equivalente, de expresar el resultado anterior es decir que la energa potencial
gravitatoria (o de otro tipo) de un sistema formado por ms de dos masas (m 1, m2, m3)
interaccionando entre ellas, se puede obtener calculando la suma de las energas potencia-
les de cada pareja de masas, es decir:

Epsis = (Ep12 + Ep13 + Ep23 ) y, en general: Epsis = ij


Ep i j (i j)

Conviene, pues, diferenciar entre los conceptos de energa potencial gravitatoria (y de


cualquier otro tipo) de una partcula situada en un punto dado de un campo (obviamente,
si no hay campo y partcula no se puede hablar de Ep) y energa potencial correspondiente
a un sistema de partculas.
Campo gravitatorio 271

6. En la figura adjunta m1 = 106 kg y m2 = 4106 kg. Ambas se hallan sobre el eje X a 4


m y 3 m del origen respectivamente. Determinad:

Y(m)

m1 m2 X (m)

a) Punto en el que la intensidad del campo gravitatorio es nula.


b) Potencial gravitatorio en el origen de coordenadas.
c) Energa potencial gravitatoria de una masa m3 = 105 kg situada en el origen.
d) Energa potencial gravitatoria del sistema formado por m1, m2 y m3.

La intensidad del campo gravitatorio resultante creado por las dos masas viene dada por la

expresin g g 1 g 2 . Buscamos un punto en el que g 0 . Antes de proceder a realizar
ningn clculo convendr que nos planteemos en qu zona podra estar dicho punto.

El hecho de que g 0 significa que g 1 g 2 0 y, por tanto, que los vectores g 1 y g 2
han de tener el mismo mdulo y sentidos contrarios. Como el vector intensidad del campo
gravitatorio, creado por una masa que pueda considerarse puntual, siempre se dirige hacia
la masa creadora del campo, las posibilidades quedan reducidas al eje X (para que tengan
la misma direccin) y entre las dos masas (para que tengan sentido contrario). Adems, el
punto buscado deber de estar ms prximo a m 1 por ser sta la masa ms pequea de las
dos (si fuesen iguales debera de hallarse justo en el punto medio, pero al no serlo se ha de
compensar la diferencia de masa con la distancia).

m1 g1 g2 m2
*
d1 d2


Proponed un procedimiento para determinar el punto en que g 0 y llevadlo a cabo.

La intensidad del campo gravitatorio creado por una masa puntual m a una distancia d de
la misma, se obtiene mediante:
g = Gm/d2

Podemos expresar el mdulo de la intensidad del campo debido a cada una de las masas e
igualarlos con el fin de obtener la distancia a que se halla el punto que buscamos de algu-
na de las dos masas. De esta forma:
m m m m
G 21 G 22 21 22
d1 d2 d1 d2

En la ecuacin anterior hay dos incgnitas, pero si tenemos en cuenta que d1 + d2 = d = 7 m, el


problema queda reducido a resolver un sistema de dos ecuaciones para hallar d1 o d2:
Campo gravitatorio 272

m1 d 22 m 2 d 12 d 2 m1 d 1 m 2

d 2 m1 d d 2 m 2 y despejando d2:
d1 d 2 d d 1 d d 2

d m2 d 7 7 7
d2 d2 = 4' 7m.
m 2 m1 m1 1 1 0' 5 1' 5
1 1
m2 4


El resultado es, pues, que el punto en el que g 0 est 47 m a la izquierda del cuerpo de
masa m2 y que, por tanto, sus coordenadas segn el sistema de referencia explcito en la
figura del enunciado, sern P (-07, 0) m. Si nos fijamos en el resultado literal, podemos
ver que, tal y como habamos supuesto, cuando las masas sean iguales, el punto en el que

g 0 se situar en la mitad de la recta que las une (d2 = 35 m). El resultado tambin re-
fleja lo que ocurrir cuando cambie el valor de las masas. Podemos ver, por ejemplo, que
si (manteniendo constantes los dems factores) se aumenta el valor de m 2 la distancia d2 a
la que g = 0, aumentar, es decir el punto P en el que g = 0 se alejar de m2, etc.

Cmo podemos calcular el valor del potencial del campo gravitatorio en el punto origen
de coordenadas?

El potencial que buscamos ser la suma de los potenciales en ese punto correspondientes a
cada una de las masas por separado, y, como se trata de una magnitud escalar:

Gm1 Gm2 6'67 10 11 10 6 6'67 10 11 4 10 6


V0 =V10 +V20 = = = 88910-5 J/kg
r1 r2 3 4

El potencial del campo gravitatorio en un punto es una magnitud cuyo valor numrico
coincide con el que tendra la energa potencial de una masa unidad colocada en dicho
punto. Por tanto, si conocemos el potencial del campo gravitatorio en un punto, resulta
sencillo evaluar la energa potencial que correspondera a una masa cualquiera colocada
en dicho punto, sin ms que aplicar la expresin: Ep = mV

En nuestro caso, la ecuacin anterior queda como: Ep0 = m3V0 = 105 (-88910-5 ) = -889 J

Cmo podemos hallar la energa potencial gravitatoria del sistema formado por las tres
masas?

Podemos utilizar la expresin: Epsis = (Ep12 + Ep13 + Ep23 )

que aplicada a nuestro caso concreto conduce a:

m1 m 2 m1 m 3 m 2 m 3 -11 4 10
12
1011 4 1011
Epsis = G = - 66710
r21 r31 r32 7 3 4

y operando obtenemos finalmente: Epsis = -4996 J


Campo gravitatorio 273

7. Una masa de 1000 kg se desplaza desde un punto de potencial V1 = -5 J/kg a otro


cuyo potencial es V2 = -7 J/kg. Calculad el trabajo realizado por la fuerza gravitato-
ria e indicad si se trata de una transformacin espontnea o forzada. dem si el cuer-
po se aleja desde el punto de potencial V1 hasta el infinito.

sol: W12 = 2000 J (transformacin espontnea); W1 = -5000 J (transformacin forzada).

8. Determinad la intensidad del campo gravitatorio generado por una varilla


homognea de masa m y longitud L, en un punto situado a distancia d de uno de sus
extremos y en la direccin de la varilla.

Sabemos que la intensidad del campo gravitatorio debido a una masa puntual m en un
punto P situado a una distancia r de la misma, se puede calcular mediante la expresin:

Gm
g 2 ur
r

en la que u r es un vector unitario con origen en m y dirigido hacia el punto P. Sin embar-
go, la varilla no es una masa puntual y es evidente que no toda su masa se encuentra a la
misma distancia del punto P, de manera que la parte izquierda de la varilla, por estar ms
alejada, contribuir menos al campo en P que la parte derecha.

Cmo podramos pues hallar la intensidad del campo gravitatorio en P?

Podemos descomponer la varilla en infinitos elementos de masa dm fraccionndola en


infinitos elementos de longitud dL. Cada una de estas masas infinitesimales producir en
el punto P una intensidad:
G dm
dg ur
r2


y, a continuacin, proceder a sumar (integrar) todas las intensidades: g dg

Para ello, conviene elegir un sistema de referencia apropiado. En el de la figura adjunta,


cada elemento de la varilla tiene una longitud dL = dx y una masa dm, estando situado a
una distancia r = (L + d) - x del punto P, de modo que:
Campo gravitatorio 274

G dm
g dg i
m
i
0
(L d x) 2

donde "dg" es la componente escalar cartesiana del vector dg y el vector i es un vector
unitario siempre en el sentido positivo del eje X.

En la integral anterior existen dos variables cmo podemos reducirlas a una sola?

Como ya hemos hecho en temas anteriores, podemos utilizar la densidad lineal de la


varilla (suponiendo que su seccin es despreciable frente a su longitud), de modo que:

= dm/dL dm = dL = dx , y sustituyendo dm en la integral ya podemos resolverla:

Gdx L 1 L
L dx 1
g=
0 (L d x) 2
i = G i
0 (L + d - x) 2
G i
d L d 0
y como = m/L, queda:

Gm
g i
(L d) d

La expresin obtenida nos muestra que cuanto mayor sea la distancia d a la que el punto P
se encuentra de la varilla, menor ser la intensidad del campo gravitatorio en dicho punto.

Qu debera ocurrir con la intensidad g en el punto P si ste se alejase tanto de la vari-


lla que la longitud L de la misma fuese mucho menor que la distancia d?

En ese caso, la intensidad del campo gravitatorio creado por la varilla debera de tender a
la que corresponde a una masa puntual. Este hecho queda perfectamente reflejado en el
resultado obtenido ya que, como podemos ver, para valores de d mucho mayores que L (es
decir, para L despreciable frente a d) la expresin se transforma en:

Gm
g i
d2

que es, justamente, la que corresponde a una masa puntual.

Mediante un proceso similar (aunque con clculos un poco ms complejos) se puede de-
mostrar que la intensidad del campo gravitatorio debido a una esfera hueca de masa m y
radio R, en un punto exterior a la misma (situado a una distancia de su centro r R), es la
misma que correspondera a una masa puntual m situada en el centro de la esfera, y, que
para aquellos puntos situados a una distancia r R, la intensidad es nula (como si no exis-
tiera masa). De ello haremos uso en problemas posteriores.
Campo gravitatorio 275

9. Determinad la intensidad del campo gravitatorio generado por una esfera maciza y
homognea de masa m y radio R, en un punto P situado a distancia r de su centro.

De acuerdo con lo que hemos sealado en el problema anterior, podemos descomponer la


esfera maciza en una serie de infinitas capas esfricas concntricas de espesor "dr" y masa
"dm" (algo parecido a la estructura de una cebolla).

Sugerid y llevad a cabo un procedimiento para calcular la intensidad del campo gravita-
torio en un punto P, cuando ste se encuentre a una distancia r R del centro de la esfera
maciza considerada.

En este caso cada una de las capas


esfricas infinitesimales se comporta
como una masa puntual dm situada en

su centro y su dg correspondiente
vendr dado por la expresin:

G dm Gdm
u r de modo que g dg =
m
dg
r 2 0 r 2
u r y como r y u r son constantes:


Gu r m Gm
g 2
r dm =
0 r2
ur .

Vemos, pues, que en el caso considerado, la esfera se comporta como una masa puntual
colocada en su centro, con lo que g va disminuyendo con el cuadrado de la distancia r al
centro de la esfera.

Sugerid y llevad a cabo un procedimiento para calcular la intensidad del campo gravita-
torio en un punto P, cuando ste se encuentre a una distancia r R del centro de la esfe-
ra.

En este caso hemos de distinguir entre dos gru-


pos de capas esfricas: Aquellas cuyo radio sea
mayor que r (cuyo efecto en la intensidad del
campo gravitatorio en P ser nulo) y las que su
radio sea inferior o igual a r (que se comportarn
como masas puntuales). De esta forma, si lla-
mamos m a la masa total de todas las capas cuyo
radio sea r, tendremos que:


m' Gdm Gu r m' Gm'
g 2 ur 2 dm ur
0 r r 0 r2

Utilizando la densidad de la esfera, podemos poner este resultado en funcin de m y R:


Campo gravitatorio 276

m 4
m= dV= r 3 = mr3/R3.
4 3 3
R
3

Gmr
Sustituyendo m' en la ecuacin anterior, nos queda: g ur
R3

Vemos que el mdulo de g, en este caso, es una funcin lineal de la distancia r al centro
de la esfera, de modo que va aumentando conforme aumenta r.

A la vista de los resultados obtenidos razonad que forma tendr la grfica que representa
la variacin de g con la distancia desde que r = 0 hasta un punto en el que r R.

Es evidente que en dicha grfica se obser-


varn dos tramos:
El primero para valores de r comprendi-
dos entre O y R, donde la funcin g au-
menta linealmente con r.
El segundo, para valores de r mayores que
R, donde la funcin g disminuye con el
cuadrado de r.

10. A qu altura de la superficie terrestre la intensidad del campo vale la mitad que
en la superficie de la Tierra? (Dad el resultado en funcin del radio de la Tierra R T).

El problema es equivalente a que nos pregunten a qu distancia r (mayor que el radio te-
rrestre RT) del centro de la Tierra el mdulo de la intensidad del campo gravitatorio tiene
un valor determinado. Sabemos que, en ese caso, la Tierra se comporta como si toda su
masa estuviera concentrada en su centro y que, por tanto, la intensidad vendr dada por:

G mT G mT G mT
g 2
= 2
y en la superficie de la Tierra ser g0 = 2
r (R T + h) RT

Cmo podemos calcular el valor de h en el que g vale justamente g0 /2?

Si imponemos la condicin de que g sea igual a g0 /2 y despus sustituimos el valor de g0,


podemos despejar la h buscada. En efecto:

g0 G mT G mT G mT
2
y sustituyendo el valor de g0, nos queda: 2

2 (R T + h) 2R T (R T + h) 2

Simplificando obtenemos 2R 2T (R T h) 2 2 R T R T h y despejando h:


Campo gravitatorio 277

h = ( 2 1) R T = 041 RT

Si tomamos como valor medio para el radio de la Tierra 6350 km, la altura a la que la
intensidad del campo gravitatorio se hara la mitad, corresponde a los 26035 km. Una
persona situada a esa altura pesara la mitad de lo que pesa en la superficie de la Tierra.

Cunto valdra g a una altura igual al radio de la Tierra?

Como la intensidad del campo es menor cuanto mayor es la distancia al centro, a primera
vista podra pensarse que si la distancia se duplica, la intensidad del campo gravitatorio se
reducir a la mitad. Sin embargo, basta reflexionar un poco para darse cuenta que ello no
es as, puesto que g es inversamente proporcional a la distancia al cuadrado (no a la dis-
tancia) y, por tanto, lo que ocurrir es que g se har cuatro veces menor que en la superfi-
cie. En efecto:
G mT G mT
Sustituyendo h por RT en la expresin g = 2
obtenemos g = 2
= g0 /4
(R T + h) 4R T

De acuerdo con el resultado anterior, un astronauta situado en una rbita de radio r =2R T,
pesara all cuatro veces menos de lo que pesa en la superficie de la Tierra.

11. Si el peso de una persona fuera de la atmsfera terrestre no es 0, por qu se dice que
los astronautas que se encuentran en una estacin espacial se hallan en estado de ingra-
videz?

La sensacin fsica que tenemos acerca de nuestro propio peso se debe a la existencia de otras
fuerzas que habitualmente lo equilibran. As, por ejemplo, cuando nos colocamos encima de
una balanza de bao en nuestra casa, la fuerza peso con que la Tierra nos atrae es equilibrada
por la fuerza ejercida sobre nosotros por el resorte de la balanza. Nosotros notamos esa fuerza,
lo mismo que la que nos hace el suelo cuando permanecemos de pie en l y esto nos da la
sensacin de que pesamos.

A veces, la superficie sobre la que estamos nos hace una fuerza mayor que nuestro peso (y
nosotros a ella), por eso notamos como si pesramos ms (aunque la Tierra nos sigue atrayen-
do con la misma fuerza y realmente seguimos pesando igual). Esto ocurre, por ejemplo, en el
momento en que un ascensor arranca y acelera hacia arriba. En otros casos ocurre lo contrario
y la fuerza que nos hace la superficie (y nosotros a ella) es menor que nuestro peso y, conse-
cuentemente, nos da la sensacin de que pesamos menos. Qu ocurrir en aquellos casos en
los que la superficie no ejerce ninguna fuerza sobre nosotros o, simplemente, no hay ninguna
superficie y estamos en cada libre? En esos casos nos parecera que no pesamos nada. Sen-
timos un estado de "ingravidez" pero eso, naturalmente, no debe interpretarse como que no
hay gravedad o que la Tierra ha dejado de atraernos y realmente no pesamos. Esta sensacin
la experimentan por un tiempo los saltadores de trampoln, los paracaidistas y tambin (de
forma continua) los astronautas que se hallan en rbita en satlites alrededor de la Tierra. As
pues cuando se dice que un astronauta est en estado de ingravidez debe entenderse que se
halla en cada libre, sometido a la accin de la fuerza gravitatoria terrestre sin ninguna otra
fuerza que la equilibre, pero no que se encuentre en un lugar donde no exista gravedad (como
a veces se piensa identificando, errneamente, espacio vaco con gravedad nula).
Campo gravitatorio 278

12. Determinad la masa de la Tierra sabiendo que la intensidad del campo en su su-
perficie vale 981 N/kg y que el radio medio de la Tierra es 6350 km.

sol: mT = 593 1024 kg

13. Determinad el valor de la intensidad del campo gravitatorio en la superficie de la


Luna sabiendo que su masa es 81 veces menor que la de la Tierra y su radio 366 ve-
ces menor. Qu pesar en su superficie un astronauta que en la superficie de la Tie-
rra tiene un peso de 1000 N? (g0T = 981 N/kg).

El mdulo de la intensidad del campo gravitatorio de la Luna en un punto de su superficie


vendr dado por la expresin:
G mL
g0L = 2
RL

Si supiramos lo que vale la masa m L de la Luna y su radio RL, el clculo de g0L sera in-
mediato. Sin embargo no conocemos ninguna de las dos cosas, por lo que se hace necesa-
rio buscar otro procedimiento.

Podemos expresar mL y RL en funcin de mT y de RT, ya que en el enunciado se nos indica


que RT = 366RL y que mT = 81 mL , de modo que:

mT
3'66 2 G m T G mT G mT
g0L = G 81 2 = 2
'
= 017 2
y como g0T = 2
tendremos:
RT 81 RT RT RT

3' 66
g0L = 017 g0T

El resultado obtenido nos muestra que la intensidad del campo gravitatorio lunar (en la
superficie de la Luna) es solo el 17 % de la intensidad del campo gravitatorio en la super-
ficie de la Tierra. Si consideramos que g0T = 981 N/kg, obtenemos que:

g0L = 017 981 = 167 N/kg

Cmo podramos calcular el peso de un astronauta en la Luna?


Campo gravitatorio 279

Dado que conocemos la intensidad del campo gravitatorio en su superficie no tendramos


sino que aplicar la expresin P = mg sustituyendo g por g0L. La masa m del astronauta,
evidentemente, sera la misma que la que tiene en la Tierra. Para calcularla, podemos uti-
lizar la expresin que nos proporciona su peso en la superficie de la Tierra y despejar m.

P0T = mg0T de donde m = P0T /g0T = 1000/981 = 10194 kg , con lo que su peso en la su-
perficie de la Luna ser: P0L = mg0L = 10194 167 = 17024 N, es decir, se reducira
casi a la sexta parte de lo que pesa en la Tierra, o lo que es lo mismo, el astronauta en la
Luna pesara aproximadamente, lo mismo que un nio de 17 kg de masa pesa en la Tierra.

14. En qu punto de la recta que pasa por dos astros la intensidad del campo gravita-
torio resultante es 0?

Supongamos un sistema formado por dos astros como, por ejemplo, la Tierra y la Luna,
separados entre s por una cierta distancia. Si una nave se dirige hacia la Luna siguiendo la
recta que pasa por ambos astros, resultar del mayor inters conocer en qu punto del tra-
yecto, la fuerza gravitatoria resultante que el sistema ejerce sobre la nave, deja de oponer-
se a su movimiento y comienza a favorecerlo (anlogamente cuando se dirige de la Luna
hacia la Tierra). Es evidente que ello se producir a partir del punto en el que la intensidad
del campo gravitatorio sea 0 y que dicho punto deber estar situado entre ambos astros
para que los vectores intensidad tengan sentidos contrarios y que su suma pueda valer 0.

F1 F2

El problema planteado tiene que ver, pues, con algo ms general, cmo es el aprovecha-
miento de los campos gravitatorios en el movimiento de naves y sondas espaciales.

Nos vamos a centrar en el caso de dos astros de masas m 1 y m2 separados por una gran
distancia d tal que ambos se puedan considerar como masas puntuales y vamos a calcu-
lar a qu distancia r1 de m1 el campo gravitatorio de dicho sistema es nulo.

r2
m1 m2
g1 g2

r1
d
Campo gravitatorio 280

Cabe pensar que r1 dependa de la distancia d, as como de los valores de m 1 y de m2, de


tal forma que: cuanto mayor sea m1 y menor sea m2, tanto mayor ser r1; cuanto mayor sea
d, mayor ser tambin r1. Tambin podemos pensar en algn caso lmite o evidente como,
por ejemplo: que si m2 tiende a 0, r1 tender a d; o que si las masas de ambos astros fue-
sen iguales, r1 = d/2, etc.

Sabemos que en el caso del campo gravitatorio creado por una masa m puntual (o que
pueda considerarse como tal), la intensidad del campo en un punto del mismo es una
magnitud vectorial cuyo mdulo viene dado por g = Gm/r2 .

En nuestro caso, la intensidad del campo gravitatorio ser g g1 g 2 . Para que la suma
de dos vectores que tienen la misma direccin y sentidos contrarios valga 0, es necesario
que sus mdulos sean iguales. Por tanto una forma de resolver el problema podra ser
igualar g1 con g2 y a partir de la ecuacin obtenida hallar r1.

Gm1 Gm2 m m
g1 = g2 2
2 21 22
r1 r2 r1 r2

m1 m2
y teniendo en cuenta que r2 = d - r1 , nos queda que:
r12
(d r1 ) 2

d
Despejando r1 en la ltima ecuacin, se llega a: r1
1 m 2 / m1

A partir de ese punto (suponiendo el caso de la figura anterior), la fuerza con que m 2 atra-
era a cualquier objeto de masa m sera mayor que la fuerza con que ese mismo objeto
sera atrado por m1 (recordemos que F = mg).

El resultado se puede cuantificar sin ms que sustituir por valores reales. Por ejemplo, m 1
podra ser la Tierra y m2 la Luna. En el caso de la Tierra y la Luna, sabiendo que la masa
de la primera es unas 81 veces la de la segunda y que la distancia media entre ambos as-
tros es de 384000 km, nos quedara que: r1 = 345600 km del centro de la Tierra.

Si nos fijamos en el resultado final obtenido podemos ver en primer lugar que es dimen-
sionalmente homogneo (L en ambos miembros). Tambin que se cumplen nuestras hip-
tesis de partida ya que, por ejemplo: si m1 aumenta, r1 tambin aumenta; si m2 tiende a 0,
r1 tiende a d; si m1 = m2, r1 = d/2, etc.

Una vez resuelto este problema podramos plantearnos otros ms complejos como, por
ejemplo, qu hacer en el caso de sistemas con ms de dos astros o cmo aprovechar el
campo gravitatorio de distintos planetas cuando queremos enviar una sonda espacial a la
periferia de nuestro sistema solar, etc.
Campo gravitatorio 281

15. Calculad la energa potencial gravitatoria de un cuerpo de 2000 kg de masa


a) En la superficie de la Tierra.
b) A una altura sobre su superficie igual al radio de la Tierra (6350 km).

sol: a) Ep = -126 1011 J; b) Ep = -63 1010 J

16. Determinad la rapidez con que llegara a la superficie de la Tierra un cuerpo de


masa m que se abandonase a una altura igual al radio terrestre. (Para simplificar,
ignorar el rozamiento con el aire, intensidad del campo gravitatorio en la superficie
terrestre g0T = 981 N/kg, radio de la Tierra RT = 6370 km).

Vamos a manejar el sistema formado por un cuerpo de masa m (considerado puntual) y la


Tierra (a la que consideraremos inmvil). Supondremos que el cuerpo se halla a una altura
inicial h lo bastante grande como para que no se pueda considerar constante a la acelera-
cin de la gravedad.
En cuanto lo soltemos, el cuerpo caer sometido a la accin de la fuerza gravitatoria ejer-
cida por la Tierra. Como dicha fuerza siempre va dirigida hacia el centro del planeta, el
cuerpo tendr un movimiento rectilneo hacia el centro de la Tierra, aumentando su velo-
cidad respecto de la Tierra (aunque no de manera uniforme ya que F no es constante sino
que va aumentando conforme el cuerpo se acerca a la Tierra). Se trata pues de un movi-
miento variado y, como consecuencia, la determinacin cinemtico-dinmica de la rapi-
dez al llegar al suelo, no es una tarea sencilla).

A m B
vA=0

rA h m
F
m vB?

mT RT mT RT RT
mT

No obstante, cabe esperar que la rapidez v con la que choca, para una masa y un radio de
la Tierra que tienen unos valores dados, depender de la altura inicial h desde la que lo
soltamos de modo que v aumentar cuanto mayor sea el valor de h. Es evidente que si h
fuera 0 la v valdra 0. Adems, en el caso de que la altura fuese lo bastante pequea como
para que pudisemos considerar constante la aceleracin de la gravedad, el objeto llevara
un movimiento rectilneo uniformemente acelerado y la rapidez valdra: v = 2 g 0 h en
donde g0 tendra el valor de la aceleracin de la gravedad al nivel del mar (981 m/s 2).

Se trata de un problema que tiene un indudable inters prctico en el tema de lanzamiento


de satlites, proyectiles, e incluso el posible impacto de meteoritos.
Campo gravitatorio 282

En el sistema considerado no hay fuerzas exteriores y, por tanto, el trabajo exterior es 0.


Adems, por tratarse de una masa puntual no se produce calor.

Dado que el trabajo exterior es 0, y no hay calentamiento, podemos concluir que, aunque
cambie la energa cintica y la energa potencial del sistema, la suma de ambas (energa
mecnica) permanecer constante. Por tanto, una forma sencilla de obtener la rapidez pe-
dida sera aplicar la expresin Wext = E (donde E = Ec + Ep), tomando como estado ini-
cial (A) del sistema cuando se suelta el cuerpo y como estado final (B) la situacin del
sistema en el momento en que el cuerpo impacta contra el suelo.

Otra posibilidad sera aplicar al cuerpo el teorema de las fuerzas vivas W res = Ec en don-
de la fuerza resultante sobre el cuerpo sera la fuerza gravitatoria con que la Tierra lo atrae
(cuyo valor ira cambiando con la distancia r al centro de la Tierra).

Siguiendo la primera estrategia y llamando mT y RT a la masa y radio de la Tierra respec-


tivamente:

Wext = E; como Wext = 0 E = 0 Ec + Ep = 0 y sustituyendo:

1 GmT m GmT m 2GmT h


mv B 0 = 0 v B
2
.
2 RT rA rA RT
2 g 0 RT h
Teniendo en cuenta que GmT = g 0 RT2 y que rA = RT + h: v B .
RT h

2 g 0 RT
Dividiendo arriba y abajo por h obtenemos: vB
( RT / h) 1

Tras esta resolucin literal, podemos sustituir los datos numricos que nos den y obtener
el valor de la rapidez que se demanda, que en este caso resulta ser: vB = 7905 m/s. Se trata
de una rapidez enorme (ms de 28000 km/h) que conlleva tambin una enorme energa
cintica. Incluso teniendo en cuenta el efecto de frenado de la atmsfera, el posible cho-
que con la Tierra de cualquier satlite espacial averiado, podra tener muy graves conse-
cuencias.

Si nos fijamos en la ltima expresin obtenida, podemos ver en primer lugar que es di-
mensionalmente homognea (L/T en ambos miembros); si no lo fuese es seguro que el
resultado sera incorrecto. Por otra parte, tal y como habamos supuesto, cuanto mayor sea
el valor de h, mayor es el valor de la rapidez con que el cuerpo choca contra el suelo.

En cuanto a los casos lmite considerados, es evidente que si h = 0 la vB = 0. Adems si h


es muy pequeo frente a RT, podemos despreciar el 1 del denominador frente a RT/h con
lo que nos quedara:
2 g 0 RT
vB 2g 0 h
( RT / h)
Campo gravitatorio 283

que es, precisamente, el resultado obtenido cuando la altura es pequea y se puede hacer
la simplificacin de suponer que el movimiento de cada es un movimiento rectilneo y
uniformemente acelerado.

El resultado literal obtenido tambin nos permite percatarnos de algo que no sabamos. En
principio, cabe pensar que cuanto ms lejos de la Tierra se deje caer al cuerpo, mayor
ser la rapidez con que impactar contra el suelo. Ahora bien: se trata de un proceso que
no tiene ningn lmite? En otras palabras: la rapidez del impacto crece indefinidamente
con la distancia h?

Analizando el resultado anterior podremos contestar a esta importante pregunta:

En efecto, si lo hacemos, podemos ver que cuando h resulta que vB 2 g0 R T

de modo que sustituyendo, obtenemos que la mxima rapidez de impacto (partiendo del
reposo), resulta ser de unos 112 km/s (equivalente a 40320 km/h).

Naturalmente los resultados y conclusiones a que hemos llegado solo son vlidos para las
condiciones que hemos considerado imperantes en el problema (el objeto parte del reposo
hacia la Tierra inmvil y no incluimos el rozamiento), con lo que el problema podra pro-
seguir cambiando alguna de estas condiciones y viendo cmo eso afecta al resultado.
Tambin es posible proponer el problema inverso: Con qu velocidad debera lanzarse
un objeto desde la superficie terrestre para que no regresara a ella?

17. Si la Luna se encontrara a solo 100000 km de la Tierra, cul sera su periodo de


revolucin alrededor de la misma?

El movimiento de la Luna alrededor de la Tierra puede considerarse aproximadamente


como un movimiento circular y uniforme (a lo largo del tema haremos uso siempre de esta
simplificacin para estudiar el movimiento de planetas y satlites). De acuerdo con ello la
Luna describir una trayectoria circular sometida exclusivamente a la fuerza de atraccin
gravitatoria que le ejerce la Tierra. Dicha fuerza estar dirigida hacia el centro de la Tierra
y su mdulo ser siempre constante e igual a Gm TmL/r2 siendo r la distancia entre el cen-
tro de la Tierra y el de la Luna.

Para resolver el problema conviene comenzar


obteniendo las ecuaciones que rigen el movi- F
miento de la Luna alrededor de la Tierra.
v

Las ecuaciones que en las condiciones ante- r


riores regirn el movimiento de la Luna (de
trayectoria conocida) son:

Ft = mL at
Fn = mL an

y sustituyendo las fuerzas:


Campo gravitatorio 284

0 = mL at at = 0 (movimiento uniforme).
F = mL an GmT mL/r2 = mLv2/2
Gm T
De la segunda ecuacin, podemos obtener la rapidez con que gira la Luna v =
r

En el problema se nos pide que calculemos el periodo de rotacin T de la Luna (para un


cierto valor de r). La ltima ecuacin anterior nos muestra que cuanto menor sea la dis-
tancia r, mayor tendr que ser la rapidez de giro v (para que el movimiento sea circular y
uniforme), por tanto, cabe esperar que el periodo T (tiempo que tarda en dar una vuelta
completa) disminuya conforme lo haga r (ya que la v debe ser mayor). El problema puede
resolverse, pues, obteniendo la ecuacin que relaciona v con T.

Al tratarse de un movimiento circular la rapidez lineal v estar relacionada con la angular


mediante v = wr y como adems es uniforme, w ser constante y valdr w = /t . Para
una vuelta = 2 radianes e t = T, luego w = 2/T T = 2/w = 2r/v

2 r
Sustituyendo ahora v por su expresin nos queda: T =
GmT
r

Por otra parte, hemos visto que G m T g 0T R 2T y sustituyendo en la expresin anterior:

2
T= r 3 = 36 das.
R T g 0T

Si analizamos el resultado literal anterior, vemos que es dimensionalmente homogneo (T


a ambos lados de la igualdad) y que, efectivamente, el periodo de rotacin depende de la
distancia r, de forma que al disminuir r disminuye T (ms precisamente, el cuadrado del
periodo es directamente proporcional al cubo del radio). En este caso concreto, el periodo
de rotacin de la Luna alrededor de la Tierra pasara de ser 273 das a solo 36 das. El
resultado tambin es aplicable a otros astros en su movimiento de rotacin alrededor del
Sol (en la ecuacin habra que cambiar R T y g0T por los valores correspondientes al Sol),
lo que hace que el periodo de rotacin vaya disminuyendo desde los aproximadamente
250 aos que tarda Plutn hasta los 88 das del planeta Mercurio.

18. Sabiendo que la Luna tiene un periodo de revolucin T = 273 das. Determinad la
distancia Tierra-Luna. (Masa de la Tierra mT = 6 1024 kg).
sol: r = 384108 m = 384.000 km

19. Determinad el radio de la rbita geoestacionaria de la Tierra ( R T = 6350 km).


sol: r = 41.910 km (35.560 km sobre la superficie terrestre).
Campo gravitatorio 285

20. Determinad la mnima rapidez con que debera lanzarse, desde la superficie de la
Tierra, un objeto de masa m para que, en ausencia de rozamiento, consiga escapar del
campo gravitatorio terrestre. (Considerad RT = 6350 Km y g0T = 981 N/kg).

Para que un objeto de masa m no vuelva a caer sobre la superficie de la Tierra, tras ser
lanzado desde su superficie con una cierta rapidez, ser necesario que consiga escapar del
campo gravitatorio terrestre. Para ello debe alejarse a una distancia infinita de la Tierra,
donde la fuerza de atraccin gravitatoria ejercida por sta valdr 0. Por tanto, el valor
mnimo de la velocidad de lanzamiento (o velocidad de escape) ser el menor valor nece-
sario para que el objeto pueda llegar al infinito con velocidad final nula. Durante el tra-
yecto la fuerza de atraccin gravitatoria ejercida por la Tierra, har que disminuya la velo-
cidad con que el objeto se aleja de ella. Utilizando consideraciones de trabajo y energa,
podemos decir que el trabajo resultante producir una disminucin de la energa cintica.

En el problema se nos pide el valor de la velocidad de escape. Cmo podra obtenerse?

Una forma de abordar el problema es aplicando la expresin que relaciona el trabajo re-
sultante sobre el objeto con el cambio experimentado por la energa cintica entre los es-
tados A (cuando es lanzado) y B (cuando llega al infinito con rapidez final nula) y, a partir
de la ecuacin obtenida, despejar vA.

Procederemos, pues a resolver el problema de acuerdo con la estrategia indicada:

Wres A Ec WP A Ec
B B

(Estamos suponiendo que la nica fuerza que acta sobre el objeto es la gravitatoria ejer-
cida por la Tierra).

Como la fuerza gravitatoria es conservativa: WP A Ep , de modo que: Ep Ec .


B

Esta expresin puede ponerse como EpA + EcA = EpB + EcB , en la que:

mv 2A
EpA = -GMm/RT ; EpB = 0 ; EcB = 0; EcA = y sustituyendo:
2

mv 2A 2GM
-GMm/RT = - . Despejando obtenemos vA =
2 RT

Finalmente, como G m T g 0T R 2T , obtenemos que vA = 2g 0T R T = 112104 m/s

El resultado obtenido es aplicable tambin a cualquier otro planeta (sustituyendo g 0 y R


por los valores correspondientes a dicho planeta). En el caso de la Tierra, el valor mnimo
de la velocidad con que habra que lanzar un cuerpo (de cualquier masa) para que no re-
grese de nuevo atrado por la gravedad terrestre (velocidad de escape) sera de 112 km/s
(considerando nula la friccin con la atmsfera).
Campo gravitatorio 286

21. Un satlite artificial, de 103 kg de masa, se eleva hasta


cierta altura desde la superficie de la Tierra. Una vez all, es
impulsado mediante cohetes propulsores para que pueda
describir una rbita circular alrededor de la Tierra, con un
movimiento circular uniforme, de periodo T = 15 horas.
Determinad:
a) Radio de la rbita que describe.
b) Rapidez con que se mueve en dicha rbita.
c) Energa total suministrada para situarlo en la rbita.

Datos: RT = 6370 km; g0 = 9'8 N/kg

Para resolver el problema, conviene que obtengamos primero las ecuaciones que rigen el
movimiento del satlite en rbita alrededor de la Tierra.

Ft = mS at 0 = mS at at = 0 (movimiento uniforme).
Fn = mS an F = mS an GmT mS/r2 = mSv2/r
Gm T
De la segunda ecuacin, podemos obtener la rapidez de giro v = , que, como po-
r
demos ver, tendr que ser tanto mayor cuanto mayor sea el radio que se quiera dar a la
rbita del satlite.

Cmo podramos calcular el radio de la rbita?

Si conocisemos la rapidez v con que se mueve el satlite la obtencin de r mediante la


ecuacin anterior sera inmediata, pero lo que conocemos es el periodo T de revolucin.
No obstante, dicho periodo est relacionado con v, ya que es evidente que cuanto menor
sea T, con mayor rapidez ha de girar el satlite. Obteniendo, pues, la relacin entre v y T
podremos resolver el problema.

Como la rapidez lineal v est relacionada con la angular w mediante v = wr y al ser un


movimiento circular uniforme w = 2/T, podemos expresar v como v = 2r/T, de forma
que sustituyendo en la ecuacin anterior:

Gm T Gm T GmTT2
v= , obtenemos 2r/T = y despejando, r = 3
r r 4 2

El resultado obtenido puede ponerse en funcin de g0T y de RT si tenemos en cuenta que,


como ya sabemos, G m T g 0T R 2T y obtener as finalmente, que:

g 0T R 2T T2
r 3 = 665106 m
4 2

Podemos ahora proceder a calcular la rapidez lineal del satlite.


Campo gravitatorio 287

Basta con sustituir el resultado obtenido para r en la ecuacin v = 2r/T, con lo que se
obtiene v = 7737'6 m/s

Para terminar, hemos de calcular la energa suministrada para situarlo en rbita

Dicha energa proviene del propio satlite. El proceso consiste pues en una disminucin
de energa interna (de origen qumico) que se traduce en un aumento de la energa mec-
nica. Como es lgico, considerando el sistema satlite-Tierra, esa energa ser la diferen-
cia entre la energa mecnica del sistema en la situacin B (satlite en rbita) y la energa
mecnica del sistema en la situacin A (satlite en reposo sobre la superficie terrestre).

E EB E A EcB EpB EpA

Teniendo en cuenta que: EpA = -GmT mS/RT, EpB = -GmT mS/r, EcB = mS v2/2

ms v 2 GmT ms GmT ms
E
2 r RT

GmT
Teniendo en cuenta que v
r

GmT ms GmT ms GmT ms = GmT ms GmT ms


E
2r r RT 2r RT

El primer parntesis de la expresin anterior, corresponde a la energa mecnica del sis-


tema formado por la Tierra y un satlite en rbita (MCU) alrededor de ella. Anlogamen-
te, el segundo parntesis corresponde a la energa mecnica del sistema en la situacin
inicial (satlite en reposo sobre la superficie terrestre).

Teniendo ahora en cuenta que GmT g 0 RT2 y simplificando, obtenemos finalmente:

R2
E g 0 m S R T T y sustituyendo valores numricos: E = 3'210 J
10

2r

Conviene tener en cuenta que lo que hemos obtenido es el valor terico de la energa que
se necesitara para colocar en rbita un satlite de una masa dada en unas condiciones
determinadas. Como dicha energa ha de obtenerse de un combustible, en realidad se ne-
cesitar una energa mayor, ya que junto con el satlite es necesario elevar tambin los
tanques llenos de los materiales utilizados como combustible (que se irn consumiendo,
con lo que la masa cambiar). Adems, hemos ignorado el efecto del rozamiento con el
aire, el cual no es nada despreciable, dada la gran velocidad a la que el satlite atraviesa la
atmsfera.
Campo gravitatorio 288

22. Un satlite de 1000 kg de masa se encuentra en


rbita terrestre de radio 3RT. Determinad: a) Rapi-
dez y periodo de revolucin; b) Trabajo realizado
por el motor del satlite al pasar a otra rbita de ra-
dio 2RT . Datos: g0 = 981 N/kg ; RT = 6350 km.

sol: a) v = 45566 m/s; T = 03 das. b) -52 109J

23. Determinad la energa necesaria para situar en rbita terrestre de un radio igual
a dos veces el de la Tierra un cuerpo de 104 kg de masa.
sol: E = 4671011 J

24. Un satlite artificial de 100 kg est girando alrededor de la Tierra y a una altura
de 400 km sobre su superficie. Calculad:
a) La rapidez del satlite.
b) Supuesto que no existe rozamiento, la energa necesaria para situarlo en rbita.
sol: v = 776 103 m/s; E = 339109 J

25. Un satlite de 104 kg se sita en rbita terrestre de periodo 3 horas. Sabiendo que
g0 = 981 N/kg y que RT = 6350 km. Determinad:
a) Radio de la rbita que ocupa, rapidez con la que se desplaza y aceleracin a la que
se encuentra sometido.
b) Si desde la rbita anterior cambia a otra de radio 15000 km, calculad el trabajo
realizado por la fuerza gravitatoria y la energa necesaria para dicho cambio.
2
sol: a) r = 10533'4 km, v = 6128 m/s, a = 356 m/s ; b) WFg = -112 1011 J, E = 56 1010 J

26. Un astronauta sale al exterior de la estacin espacial a arreglar una avera, al


mismo tiempo que un paracaidista se deja caer desde un avin. Comparad el movi-
miento de ambas personas sealando las semejanzas y diferencias existentes.
Campo gravitatorio 289

Para resolver este problema comenzaremos por estudiar cada uno de los movimientos por
separado, analizando las fuerzas que actan en cada caso.

a) La estacin espacial se desplaza en su rbita sin necesidad de ninguna propulsin, ya


sobre ella se supone que solo acta la fuerza gravitatoria terrestre, cuyo nico efecto es
impedir que la nave contine con movimiento rectilneo y uniforme en la direccin de la
tangente (como hara si en un momento dado desapareciera el campo gravitatorio en el
que se encuentra). Por tanto, la fuerza gravitatoria, perpendicular en todo momento a la
velocidad de la estacin y dirigida hacia el centro de la Tierra, produce un cambio cons-
tante solo en la direccin del vector velocidad, haciendo que el movimiento sea circular y
uniforme, lo que cinemticamente significa que:

at = 0; an = v2/r = constante.

Y dinmicamente:

Ft = meat Ft = 0
Fn = mean Fn = mev2/r

La estacin espacial, pues, deber de estar sometida a una fuerza constante, perpendicular
a la trayectoria y de valor mev2/r (siendo me su masa, v la rapidez lineal con que gira y r el
radio de la rbita descrita). Quin ejerce dicha fuerza?

La nica fuerza que acta sobre la estacin (en el sistema considerado) es la de atraccin
gravitatoria Fg, que ejerce la Tierra. Dicha fuerza al estar dirigida en todo momento hacia
el centro de la Tierra ser siempre normal a la trayectoria. Por tanto, deber cumplirse
que:

Gm T
Fg = Fn Fg = mev2/r GmT me/r2 = mev2/r v .
r

La expresin anterior nos muestra la rapidez que debe llevar la estacin espacial para
mantenerse en una rbita dada y que su valor depende del radio de la rbita (cuanto menor
sea r mayor ser la rapidez requerida) y de una constante, pero no de la masa de la esta-
cin. Cualquier cuerpo que se encuentre a la misma distancia de la Tierra y con la misma
Gm T
velocidad (perpendicular a Fg y de mdulo ) describir el mismo movimiento
r
orbital. Esto es lo que le ocurrir al astronauta cuando salga, ya que su velocidad es la
misma que la de la estacin, por lo que permanecer constantemente junto a la nave ani-
mado del mismo movimiento circular y uniforme que ella.

b) El avin, si suponemos que se desplaza paralelamente a la superficie terrestre, describe


tambin un movimiento circular (aunque su rbita es de mucho menor radio que la de la
estacin espacial) y si su rapidez es constante ser tambin uniforme, de manera que las
condiciones cinemticas y dinmicas sern las mismas que para la estacin espacial:
Campo gravitatorio 290

at = 0; an = v2/r = cte y Ft = maat Ft = 0; Fn = maan Fn = mav2/r

Gm T
Si sustituysemos Fn por Fg = GmTma /r2 obtendramos v'
r'

Esta es la rapidez a la que debera de volar el avin, pero si la calculamos, veremos que
nos sale un valor tremendamente elevado (ms de 28000 km/h), muy superior a la rapidez
mxima que puede alcanzar cualquier avin.

Cmo puede entonces el avin mantenerse en su trayectoria, con un movimiento circular


y uniforme si la rapidez con que vuela (que llamaremos v) es mucho menor que la que,
en principio se requiere, v?

El movimiento del avin, debido a que se realiza dentro de la atmsfera, no es tan sencillo
como el de la estacin espacial. Para empezar, gracias a que el rozamiento con el aire no
es despreciable, los aviones pueden volar y los paracaidistas sobrevivir. Sobre el avin

actan dos tipos de fuerzas: Una la de rozamiento por deslizamiento Fr , que se ejerce

siempre en sentido contrario al vector velocidad del avin v ' ' y otra la debida a la dife-
rencia de presin que se origina en la parte superior e inferior de sus alas (que llamaremos

Fs o de sustentacin), que acta en sentido contrario a la fuerza gravitatoria F g .

Por tanto, sobre el avin se ejercen en realidad cuatro



fuerzas: Fm debida a los motores1, F g , Fs y Fr .

Las ecuaciones del movimiento sern ahora:

Ft = maat Fm - Fr = 0 Fm = Fr
Fn = maan Fg - Fs = mav2/r

Analizando la primera ecuacin, constatamos que la fuerza que ejercen los motores com-
pensa a la de rozamiento con el aire y de esa forma la rapidez se mantiene constante. De
la segunda se aprecia que parte de la fuerza de atraccin gravitatoria se compensa con la
fuerza de sustentacin, lo que explica que con una rapidez v mucho menor que la, en
principio, requerida v el avin pueda describir un movimiento circular.

Fg Fs Gm T F
v" = s
ma r' ma

c) Respecto al paracaidista, en cuanto abandone el avin, sobre el no actuarn ni Fm ni

Fs . De inmediato, pues, al estar dotado de la misma velocidad que el avin pero sin Fs,
existir una fuerza en la direccin normal y hacia el centro de la Tierra (cuyo valor vendr

1
En realidad la fuerza que empuja a un avin a reaccin hacia delante es ejercida por el aire y otros gases que
son expulsados hacia atrs, como consecuencia de la compresin y combustin que tiene lugar en los turbo-
rreactores.
Campo gravitatorio 291

dado por Fg = GmT mp/r2, siendo mp la masa del paracaidista). Si el rozamiento del para-
caidista con el aire se pudiese considerar despreciable, su movimiento sera como el del
astronauta, sometido nicamente a la accin de la fuerza gravitatoria, pero con la diferen-
cia de que, en este caso, dicha fuerza resulta demasiado grande para la rapidez v con que
se mueve el paracaidista y el radio de la trayectoria r que estaba describiendo hasta en-
tonces, es decir: Fg mpv2/r y, en consecuencia, el radio r disminuira.

Si queremos estudiar el movimiento del paracaidis-


ta con ms detenimiento, podramos considerar un
fragmento de la superficie de la Tierra comparable
con la altura a que se encuentra el avin. En esas
circunstancias y debido al valor tan grande del radio
terrestre (comparado con dicha altura), podramos
suponer que la superficie de la Tierra es plana y el
movimiento del paracaidista sera el representado
en la figura adjunta. En ella se puede apreciar que
al salir del avin tiene la misma velocidad horizon-
tal que ste.


A partir de ese momento, queda sometido a la accin de Fr y de Fg producindose una
disminucin, desde v, de la componente X del vector velocidad y un aumento, desde 0,
de su componente Y (aunque en el sentido escogido como negativo). En poco trayecto, la
componte X de la velocidad se habr anulado prcticamente y a partir de entonces el pa-
racaidista experimentar una cada casi vertical, con una rapidez creciente (en valor abso-
luto) hasta que se alcance un valor lmite debido a que la fuerza de rozamiento con el aire
aumenta conforme aumenta la rapidez con que cae. Si tal y como le conviene, utiliza el
paracadas, el valor de dicha rapidez lmite ser mucho menor impidiendo que colisione
con el suelo a gran velocidad.

27. En una nave espacial que gira en rbita alrededor de la Tierra con movimiento
circular y uniforme, hay un pndulo simple que cuelga del techo de la misma. Ra-
zonad cmo ser el periodo de oscilacin del pndulo en dicha situacin respecto a su
valor en la superficie de la Tierra:
L
(Dato: El periodo de un pndulo simple viene dado por T 2 )
g

sol: En las condiciones dadas en el enunciado el pndulo no oscila porque la tensin del
hilo es 0. El pndulo se desplaza a la misma velocidad que la nave, que es la velocidad
idnea para la rbita en la que se encuentra y la fuerza gravitatoria existente sobre l tiene
como nico efecto modificar la direccin de su velocidad (el hilo no hace nada sobre el
cuerpo).
Campo gravitatorio 292
8. CAMPO ELCTRICO


1. Obtened la expresin que nos permite calcular la intensidad E del campo elctrico
creado por una carga elctrica q, que puede considerarse como puntual, a una distan-
cia r de la misma.

Sabemos que siempre que se site una carga q en cualquier punto de un campo elctrico,
se ver sometida a una fuerza. En caso de que el campo se deba a una carga q que poda-
mos considerar como puntual, dicha fuerza vendr dada mediante la expresin de la ley de
Coulomb:

q q' q F
F K 2 ur r
r q ur

en la que al valor numrico de cada carga hay que acompaarlo del signo correspondiente

para indicar as el sentido de la fuerza F (en el ejemplo de la figura ambas cargas son del
mismo signo). Dicha fuerza, no constituye una magnitud apropiada para caracterizar el
campo elctrico creado por la carga q, ya que en cada punto puede tomar infinitos valores
diferentes (tantos como pueda tomar la carga q all situada). Para poder solucionar este
problema, es necesario introducir una nueva magnitud que cumpla las siguientes condi-
ciones:

a) Que su valor en cada punto del campo dependa nicamente de la carga creadora del
campo y de la distancia a que dicho punto se encuentre de ella.

b) Que, conocida esa magnitud en un punto dado, se pueda calcular fcilmente la fuerza
que se ejercera sobre cualquier carga q que se colocase en ese punto.

Proponed una magnitud que cumpla las condiciones anteriores.

Si nos fijamos en la expresin anterior, que nos sirve para calcular la fuerza ejercida sobre
una carga q situada a una distancia r de q, nos podemos dar cuenta que una funcin que
cumplira con los requisitos impuestos sera:
q
E K 2 ur
r

ya que en cada punto tomar un valor distinto y, conocido este, podemos calcular fcil-
mente la fuerza que
se ejercera
sobre cualquier carga q que se colocase en ese punto sin
ms que aplicar: F q' E .
Campo elctrico 294


A la funcin E anterior, se le denomina intensidad del campo creado por la carga puntual
q. Tal y como ha sido definida, la intensidad del campo elctrico en un punto ser total-
mente independiente de que en dicho punto se coloque o no carga alguna, de manera que

siempre podemos caracterizar un campo elctrico por el vector E que corresponde a cada
punto del mismo. El mdulo, de dicho vector depender del valor de la carga q creadora
del campo y de la distancia r a que el punto considerado se encuentre de ella. Su direccin
ser la de la recta que une la carga q con el punto, y el signo de q determinar el sentido

de E (hacia q si sta es negativa y al contrario si es positiva).

Por otra parte, si analizamos la expresin F q' E , podemos ver que el valor de la inten-

sidad del campo E en un punto cualquiera coincidir con el valor de la fuerza que actuar-
a sobre la unidad de carga positiva si se colocase en dicho punto 2. Cuanto mayor sea la
intensidad del campo elctrico en un punto, mayor fuerza se ejercer sobre cualquier car-
ga dada que se coloque en dicho punto.

2. Representad el campo generado por una carga puntual y positiva mediante las
lneas de campo y las superficies equipotenciales.

Las lneas de campo se definen como unas lneas que en todos sus puntos son tangentes al
vector intensidad de campo y tienen su mismo sentido. Por otra parte, la intensidad del
campo generado por una carga puntual viene dada por:
q
E K 2 ur ,
r

donde r es el mdulo del vector r con ori-
gen en q y sentido hacia el punto donde se va

a evaluar E , y u r un vector unitario de la

misma
direccin y sentido que r . Por tanto,
E es un vector que en todo punto tiene una
direccin radial con centro en la carga q y,
como consecuencia, la lnea del campo
tendr que ser tambin radial para permane-

cer en todos sus puntos tangente a E . En la
figura adjunta se ha representado esta situa-
cin para el caso del campo creado por una
carga q puntual y positiva.

Fijmonos que al alejarnos de la carga q las lneas del campo se separan a la vez que el
mdulo de E disminuye. Esta propiedad se cumple en cualquier campo elctrico, lo que
permite con una simple observacin determinar en qu lugares ste es ms intenso (all
donde las lneas del campo estn ms prximas).

2
Atencin: Esa coincidencia es nicamente numrica y nunca debe interpretarse como que la intensidad del
campo elctrico en un punto ES la fuerza ejercida sobre la unidad de carga positiva Una cosa es Fuerza y
otra Intensidad de campo. Son dos magnitudes diferentes.
Campo elctrico 295

Se entiende por superficie equipotencial aquella superficie constituida por puntos en los
que el potencial vale lo mismo. Si la carga creadora del campo se puede considerar como
puntual, el potencial a una cierta distancia r viene dado por:
q
V K
r
Por tanto, en todos los puntos situados a la misma distancia de la carga q el potencial del
campo ser el mismo y como consecuencia la superficie equipotencial correspondiente
tendr forma esfrica.

Si representamos las lneas del campo y las superficies equipotenciales, vemos que resul-
tan perpendiculares, tal y como se muestra en las figuras siguientes, correspondientes a un
campo creado por una carga puntual positiva (izquierda) y a otro campo creado por una
carga puntual negativa (derecha).

E1 E2 V1 V2 E1 E2
V1 V2
E2 E2
Lnea de fuerza
E1 E1

V1 V2 V1 V2

Superficies
equipotenciales

Si en el seno de un campo elctrico se realiza un desplazamiento infinitesimal de una car-


ga q desde un punto A a otro B a lo largo de una superficie equipotencial cualquiera, el
trabajo realizado por la fuerza electrosttica se podr evaluar como dWFe A q' dV = 0
B

ya que el potencial no cambia. Dicho trabajo tambin se puede calcular mediante la ex-

presin general dWFe A Fe dr y para que sea nulo no sindolo la fuerza ni el desplaza-
B

miento, es necesario que ambos vectores sean perpendiculares y como el vector campo
elctrico siempre tiene la misma direccin que el vector fuerza ejercida por el campo, se
concluye que el vector campo elctrico tambin ha de ser perpendicular a la superficie
equipotencial.

3. Dadas las cargas: q1 = 10-8 C y q2 = -210-8 C, situadas en los puntos (0,0) y (2,0) m.
Determinad: a) Intensidad del campo elctrico en los puntos: A(1,0) m, B(0,1) m y
C(2,1) m. b) En qu punto se anula la intensidad del campo?

sol: a) E A = (270, 0)N/C; E B = (322, 739)N/C; E C = (161, -172) N/C. b) P(-48 , 0) m
Campo elctrico 296

4. Dadas las cargas: q1 = -72 C, q2 = - 40 C y q3 = 64 C representadas en la figu-


ra adjunta, determinad:
Y(m)
a) Intensidad del campo elctrico en el q1 q2
origen de coordenadas y fuerza que ac-
tuara sobre una carga de -10-6 C situada
en dicho punto.
b) Trabajo realizado por la fuerza elec-
trosttica al desplazar q2 hasta el origen.
c) Energa potencial de la distribucin q3
inicial Cul es su significado fsico?
X(m)

En este caso tenemos un sistema formado por tres cargas puntuales. Como sabemos, aso-
ciado a dicho sistema existir un campo elctrico. Tanto aqu, como en todos los ejerci-
cios siguientes, supondremos que el medio en el que se hallan las cargas es el aire (K
9109 U.I.). En el problema se nos pide que calculemos la intensidad de dicho campo en
un punto determinado (el origen de coordenadas).Cmo podramos hacerlo?

Para obtener la intensidad E del campo elctrico en cualquier punto bastar con que cal-

culemos los vectores E 1 , E 2 y E 3 en dicho punto y los sumemos.

Calculad la intensidad del campo elctrico generada


por cada una de las cargas en el
origen de coordenadas de la figura y, obtened E = E 1 + E 2 + E 3

Sabemos que la intensidad del campo elctrico en un punto, cuando la carga generadora
q
del campo sea una carga puntual, se puede determinar como: E K 2 u r
r

siendo u r un vector unitario que siempre tiene la misma direccin y sentido que el vector

r que va desde la carga generadora del campo hasta el punto donde se desea calcular su
intensidad (no confundirlo con el vector de posicin de las cargas). Aplicando esta expre-
sin al problema que nos ocupa tendremos:

Kq 9 10 9 (7'2 10 6 )
E 1 2 1 u r1 = u r1 = -1800 u r1 Y(m
r1 6 2
) q1 q2
ur1
Kq 9 10 9 ( 40 10 6 ) ur2
E 2 2 2 u r2 = 2
u r2 = -3600 u r2
r2 10
E1
Kq 9 10 9 (6'4 10 6 ) E3 E2 ur3 q3
E 3 2 3 u r3 = 2
u r3 = 900 u r3
r3 8
X(m
)
La determinacin de los vectores u r1 y u r3 es inmediata ya que de la figura anterior se

desprende que u r1 = (0, -1) y u r3 = (-1, 0). Por el contrario la determinacin del vector

u r2 es algo ms laboriosa:
Campo elctrico 297


r2 = (-8, -6) m; r2 = 8 2 6 2 = 10 m; u r2 = r2 /r2 = (-8, -6)/10 = (-08, -06)

Sustituyendo los vectores unitarios en las expresiones que nos dan las intensidades de
campo y sumando obtenemos:

E = E 1 + E 2 + E 3 = (0, 1800) + (2880, 2160) + (-900, 0) = (1980, 3960) N/C

El resultado coincide numricamente con la fuerza que actuara sobre la unidad de carga
positiva si se colocase en el origen de coordenadas. Si en lugar de la unidad de carga posi-

tiva se sita una carga q, la fuerza que actuar sobre la misma viene dada por: F q E , de
modo que sustituyendo los valores correspondientes a este caso:

F q E = -10-6 (1980, 3960) = (-19810-3, -39610-3) N

Cmo podramos determinar el trabajo realizado por la fuerza electrosttica cuando la


carga q2 se desplace desde su posicin inicial (punto A) hasta el origen de coordenadas?

La carga q2 se mover en el seno del campo elctrico generado por q 1 y q3. La fuerza resul-
tante que acte sobre ella ser la fuerza electrosttica ejercida por el campo. Al ser dicha
fuerza conservativa, podemos escribir: WFe A Ep AO = - (EpO - EpA).
O

En el caso del campo elctrico, cuando se trata de resolver problemas mediante conside-
raciones de trabajo y energa, lo habitual es manejar la funcin potencial elctrico. Expre-
sando Ep en funcin del potencial (Ep = qV), la ecuacin anterior queda como:

WFe A Ep AO = q VAO = -q (VO - VA) = q (VA - VO)


O

Para calcular, pues, el trabajo que se nos pide tendremos que calcular el potencial del
campo creado por las cargas q1 y q3 en los puntos A y O:

q1 q (7'2 10 6 ) 6'4 10 6
VA = VA1 + VA3 = k k 3 = 9 10 9 9 10 9 = 1500 V
rA1 rA3 8 6
q1 q (7'2 10 6 ) 6'4 10 6
VO = VO1 + VO3 = k k 3 = 9 10 9 9 10 9 = -3600 V
rO1 rO3 6 8

WFe A = -q (VO - VA) = 4010-6 (-3600 -1500) = -0204 J


O

El hecho de que resulte un trabajo negativo se interpreta como que la transformacin con-
siderada (desplazamiento de q2 desde su posicin inicial A hasta el origen de coordenadas
O), no es espontnea, lo que significa que la fuerza electrosttica que acta sobre q 2 se
opone al desplazamiento. Por tanto, para que ste se produzca ser necesario que dismi-
nuya la energa cintica de la partcula de carga q 2 en 0204 J, o bien acte una fuerza
exterior que realice un trabajo (suministre una energa) de al menos 0204 J.
Campo elctrico 298

La determinacin de la energa potencial electrosttica del sistema se puede hacer sin


ms que aplicar la expresin correspondiente:

Kq 2 q 1 Kq 3q 1 Kq 2 q 3
Epsis = Ep12 + Ep13 + Ep23 Epsis = y sustituyendo:
r12 r13 r23

288 10 12 9 (46 10
12
) 9 (256 10
12
)
Epsis = 9 10
9
9 10 9 10 = -0101 J
8 10 6

Este problema nos permite tambin plantearnos nuevas preguntas, como por ejemplo: En
qu direccin se mover espontneamente una pequea carga q abandonada en el seno
de un campo elctrico, hacia potenciales crecientes o decrecientes?

B
La ecuacin general WFe A = -q (VB - VA) nos permite contestar a la cuestin ya que si q
es negativa, para que el trabajo realizado por el campo sea positivo (transformacin es-
pontnea) VB tendr que ser mayor que VA y al contrario si q es positiva, lo que nos lleva
a concluir que las cargas negativas siempre se movern espontneamente hacia potencia-
les crecientes y las positivas hacia decrecientes.

5. Una carga puntual q1 = 36 C se halla en el punto (0,0) de un sistema de coordena-


das cartesianas y otra q2 = -36 C, tambin puntual, en el (8,0) cm. Calculad:

a) Intensidad del campo electrosttico en el punto A (0,6) cm.


b) Qu trabajo realizara la fuerza electrosttica del campo si se trasladara una car-
ga de 2 C, situada en A, hasta el punto B (4,0) cm?

sol: a) E = ( 26107, 7107) N/C. b) WAB = 432 J


6. Qu relacin existe entre las magnitudes intensidad E y potencial V, de un campo
elctrico?

Dada una carga q que se sita en un punto de un campo elctrico, se ver sometida a una
fuerza: F q'E y dispondr de una energa potencial Ep = qV

Si se produce un desplazamiento dr de la carga q, el campo realizar un trabajo (elemen-

tal) de valor: dWFe F dr , que se podr evaluar tambin (al ser F conservativa) como:
dWFe dEp q'dV .

Si igualamos tendremos: F dr = q'dV y como F q'E , nos queda finalmente:

q'E dr q'dV E dr dV
Campo elctrico 299

7. Sabiendo que la longitud del lado del tringulo equiltero de la figura es 2 m, y que
q1 = -210-5 C, q2 = 10-5 C y q3 = -10-5 C, se pide:

a) Intensidad del campo electrosttico en su centro.


b) Valor de la energa potencial de la carga q3
c) Energa potencial de la distribucin.

Para resolver el problema nos conviene, en primer lugar, localizar el centro de la figura
que corresponder al punto en donde se cortan las bisectrices. Si llamamos r a la distancia
del centro de la figura a cualquiera de los vrtices, d a la distancia entre dicho centro y el
punto medio de cualquiera de los lados y L a la longitud del lado, tendremos:

L
3 L L 2
cos 30 = 2 r = m
r 2 2r 3 3

L 1 L 1
Por otra parte, d = rsen30 =
m
3 2 2 3 3

De qu factores depender la intensidad del campo elctrico resultante en el punto con-


siderado? Cmo podramos obtenerla?

La intensidad del campo elctrico creado por una carga puntual q a una distancia r de la
misma viene dada, por la expresin:
q
E K 2 ur
r

siendo u r un vector unitario que siempre tiene la misma direccin y sentido que el vector

r que va desde la carga generadora del campo hasta el punto donde se desea calcular su
intensidad. La intensidad del campo
elctrico
en el centro del tringulo depender de los
vectores intensidad de campo E 1 , E 2 y E 3 correspondientes a cada una de las tres cargas
y podremos obtenerla sumando dichos vectores.

Para ello es necesario que escojamos un sis-


tema de referencia cmodo, tal y como el
que se propone en la figura adjunta, en la
que se han representado los vectores anterio-
res. En dicho sistema, las coordenadas del
centro del tringulo son (0, L / 3 ). y la ex-
presin analtica de cada uno de los vectores
vendr dada por:
Campo elctrico 300

Kq 9 10 9 ( 2 10 5 )
E 1 2 1 u r1 = 2
u r1 = -135105 u r1 = -135105 (0 , -1)
r1 2

3

E 1 (0, 1' 35 105 ) N/C .

Kq
E 2 2 1 u r2 = 0675105 u r2 .
r2

La expresin de u r2 en funcin de sus componentes escalares no es inmediata (como

ocurra con u r1 en el caso anterior). Para obtenerla dividiremos el vector r2 por su mdulo

r2. El vector r2 podemos hallarlo sin ms que restar las coordenadas de su origen a las co-
ordenadas de su extremo (centro del tringulo), de modo que:

1 1
0, 1, 0 1, 3
3 3 3 1
u r2 = r2 /r2 = , y sustituyendo en E 2 queda:
2 2 2 2
3
3 1
E 2 = 0675105 u r2 = 0675105 , = (0584105 , 0338105) N/C
2 2

Kq
E 3 2 3 u r3 = -0675105 u r3 . Conocido u r2 resulta muy sencillo determinar u r3 .
r3

En efecto, si analizamos la figura vemos que solo difieren en el signo de la componente X,


con lo que:
3 1
E 3 = -0675105 , = (0584105 , -0338105) N/C
2 2

Finalmente, sumamos: E = E 1 + E 2 + E 3 = (117105 , 135105) N/C

El mdulo podemos hallarlo como: E = E 2x E 2y = 179105 N/C.

En cuanto a la direccin y sentido del vector, estos, vienen dados por los ngulos directo-
res o ngulos que forma el vector con cada uno de los semiejes positivos:

cos = 117/179 = 065 = arccos 065 = 49


cos = 135/179 = 075 = arccos 075 = 41

El resultado obtenido se puede interpretar diciendo que si en el centro del tringulo se


colocase una hipottica carga puntual positiva (+ 1 C), sobre ella el cempo ejercera una
fuerza de 179105 N en la direccin y sentido obtenidos.
Campo elctrico 301

En cuanto a la determinacin de las energas potenciales que se demanda, el procedimien-


to a seguir es similar al que se utiliz en el caso del campo gravitatorio:

La energa potencial de la carga q3 en el sistema ser:

Ep3 = Ep13 + Ep23 = (Kq1q3/L ) + (Kq2q3/L) = (q1 +q2) Kq3/L y sustituyendo valores:

Ep3 = (-210-5 + 10-5 ) 9109 (-10-5 )/2 = 045 J.

De qu otra forma podramos obtener el valor de la Ep pedida?

Otra posibilidad es utilizar el concepto de potencial del campo elctrico. El valor de dicha
magnitud en un punto dado coincide con el de la energa potencial que tendra la unidad
de carga positiva si se colocase en dicho punto, de modo que si conocemos el potencial en
un punto, para hallar la Ep de una carga cualquiera que se coloque en ese punto bastar
aplicar la expresin: Ep = qV

En nuestro caso, el potencial del campo en el punto ocupado por q 3 valdr:

Kq 1 Kq 2 K 9 10 9
V3 = V13 + V23 = (q 1 q 2 ) = (2 10 5 10 5 ) = 45104 V
L L L 2

de modo que: Ep3 = q3V3 = 10-5 45104 = 045 J

La energa potencial del sistema, se puede evaluar mediante: Epsis = ij


Ep i j (i j) que
en nuestro caso queda como: Epsis = Ep12 + Ep13 + Ep23 , y sustituyendo:

9 10 9 (2 10 5 ) 10 5
Epsis = Ep12 + 045 = + 045 = -09 + 045 = -045 J
2

8. En cada uno de los vrtices de un tringulo equiltero de 10 cm de lado hay una


carga de 1 nC. Determinad el potencial y la intensidad del campo elctrico en el cen-
tro del tringulo.

sol: El potencial 465 V y la intensidad es nula.

9. Tres cargas puntuales se encuentran en los vrtices de un tringulo rectngulo cu-


yos catetos miden 3 m y 4 m. La que se encuentra en el vrtice del ngulo recto es q1 =
4 nC y las otras dos q2 = q3 = -2 nC. Hallad la fuerza que acta sobre cada carga y el
potencial en el punto en que se encuentra la carga positiva.

sol: F1 = (4510-9, 810-9 ) N; F2 = (-33510-9, -8610-10 ) N;

F3 = (-11510-9, -71410-9) N; V1 = -105 V
Campo elctrico 302

10. Dado el esquema de cargas adjunto,


determinad:

a) Intensidad del campo y el potencial


que generan en el otro vrtice.
b) Fuerza que actuara sobre una car-
ga de 100 C situada en dicho punto.


sol: a) E (8000, -6000) N/C; V= -180 V. b) F = (08, -06) N

11. Dada la figura adjunta y sabiendo que en el punto M se encuentra una carga q1 de
5 nC y en el punto N otra carga q2 de valor -10 nC, determinad:

a) Intensidad del campo y potencial en Z(m)


A. Qu fuerza actuara sobre una A
carga q = - 2 C que se situase A?
b) Energa potencial del sistema de
cargas formado por q1 y q2 . M
c) Trabajo realizado por la fuerza elec- O
trosttica si desplazamos q1 hasta O. q1 Y(m)
q2 N
X(m
)
Hasta ahora hemos trabajado con cargas distribuidas en un plano. En este tipo de proble-

mas podramos haber calculado la intensidad E del campo elctrico resultante sin apoyar-

nos en el vector unitario u r , obteniendo las componentes escalares de E por considera-
ciones trigonomtricas. Sin embargo, no lo hemos hecho porque este procedimiento no
sera til en problemas como el que aqu se plantea, en donde se tiene una distribucin
espacial y hemos de trabajar en tres dimensiones.

Procederemos a calcular E como la suma de los vectores E 1 y E 2 que crean en dicho
punto las cargas q1 y q2 aplicando el procedimiento general que venimos utilizando.

Kq 9 10 9 5 10 9 E1
E 1 2 1 u r1 = 2
u r1 = (9/5) u r1 Z(m)
r1 5
A
E2
donde r1 = (0, 4, 3) m; r1 = 5 m;
ur1
u r1 = r1 /r1 = (0, 4, 3)/5 = (0, 4/5, 3/5) ur2
q1 Y(m)
de modo que: E 1 = (0, 144, 108) N/C
q2
X(m
)
Campo elctrico 303

Kq
E 2 2 2 u r2 , donde: r2 (0, 0, 3) (2, 3, 0) (2, 3, 3) m; r2 = 22 m.
r2
9 10 9 (10 10 9 )
con lo que: E 2 = u r2 , siendo u r2 = r2 /r2 = (-043, -064, 064)

2
22

Sustituyendo u r2 y operando: E 2 = (176, 262, -262) N/C

Sumando: E = E 1 + E 2 = (0, 144, 108) + (176, 262, -262) = (176, 406, -154) N/C

Cmo podemos calcular la fuerza que actuar sobre una carga q situada en A?

Para obtener la fuerza actuante sobre una carga de -2C situada en el punto A, bastar

sustituir en la expresin: F q E

F q E = -210-6 (176, 406, -154) = (-35210-6, -81210-6, 30810-6) N

Como podemos ver, el vector fuerza obtenido tiene sentido contrario al vector intensidad
del campo. Ello se debe, evidentemente, al signo negativo de la carga q colocada y a la
propia definicin de la intensidad del campo elctrico.

Vamos a calcular, ahora, el potencial en el mismo punto A. Como se trata de una magni-
tud escalar:

Kq1 Kq 2 5 109 (10 109


V = V1 + V2 = 9 109 = 9 + (-199) = -1019 V
r1 r2 5 4' 69

En cuanto a la energa potencial del sistema, como se trata de dos cargas, su valor coinci-
dir con el de la energa potencial de cualquiera de dichas cargas en el sistema:

Kq 1 q 2 9 10 9 5 10 9 (10 10 9 )
Ep = -61810-8 J
r12 7' 28


(El valor de la distancia entre las cargas r 12 se puede obtener calculando el vector r12 y
determinando su mdulo o tambin aplicando el teorema de Pitgoras en la figura).

Cmo podemos calcular el trabajo realizado por la fuerza electrosttica cuando q 1 se


desplaza desde el punto que ocupa ( M) hasta el origen (O)?

O
Podemos aplicar la expresin: WFeM q VMO q(VO VM ) q(VM VO )

Para ello hemos de determinar previamente VO y VM.

Kq 2 9 10 9 (10 10 9 )
VO = = = -2493 V
r2O 13
Campo elctrico 304

Kq 2 9 10 9 (10 10 9 )
VM = = = -1236 V
r2M 7'28

Sustituyendo: WFe M = -q1 (VO -VM) = -510-9 (-2493 +1236) = 62910-8 J


O

Cmo se interpreta que el trabajo realizado sea positivo?

El carcter positivo del trabajo realizado por el campo, nos muestra que se trata de una
transformacin en la que la fuerza ejercida por el campo no se opone al desplazamiento
experimentado por la carga. Si la nica fuerza actuante en el desplazamiento considerado
fuese la electrosttica todo el trabajo realizado por esta producira un aumento de la
energa cintica.

12. En el sistema de tres cargas puntuales representado en la figura, q1 = -10-5 C,


q2 = 210-5 C y q3 = 410-5 C. Se pide:
Z(m) q3
a) Intensidad del campo elctrico que crean q2
en el origen de coordenadas.
b) Fuerza que sufrira una carga de
- 10-4 C si se colocase en el origen.
c) Trabajo realizado por la fuerza elec-
trosttica si desplazamos q3 hasta el origen. Y(m)
X(m)
q1

sol: a) E = (104, -576103, -443104) N/C; b) F = (-1, 058, 443) N; c) W = 071 J

13. Dado un anillo de radio R, cargado con una carga q uniformemente distribuida,
determinad la intensidad del campo elctrico y el potencial que crea en un punto de la
recta perpendicular al plano del anillo y que pasa por su centro.

Hasta aqu, hemos calculado la intensidad E del campo elctrico en un punto, en situacio-
nes tales que las cargas creadoras del campo podan ser consideradas como puntuales.
Cmo podemos abordar ahora un problema como este, en el que la carga no puede ser
considerada como puntual?

Como ya hicimos en el campo gravitatorio, podemos descomponer la carga total q del


anillo en infinitas cargas dq
que s podrn considerarse como puntuales, hallar la expre-
sin del campo elctrico dE correspondiente a cada una (en el punto considerado) y final-
mente obtener E como la suma de todos los dE .

Si suponemos el grosor del anillo despreciable frente a su longitud L, podremos simplifi-


car el problema y considerar el anillo como una distribucin lineal de carga. En ese caso
para descomponer la carga q bastar que dividamos el anillo en elementos de longitud dL
a cada uno de los cuales le corresponder una carga dq,
Campo elctrico 305

En la figura adjunta podemos ver la intensidad del


campo elctrico generado en el punto P por uno de tales
elementos de carga dq.

Podemos pensar cmo sera la contribucin del resto de


los elementos que conforman el anillo y preguntarnos
hacia dnde ira dirigida la intensidad del campo elc-
trico resultante en P.

Por la simetra que presenta la figura, es fcil darse



cuenta de que el vector E se encontrar sobre la recta
perpendicular al plano del anillo que pasa por su centro
y dirigido hacia afuera.


Z En efecto, cada vector dE puede ser descompuesto en
dos vectores, uno de los cuales ser horizontal ( dE y ) y
dE z
el otro vertical ( dE z ). Podemos ver que las componen-
tes horizontales de tales vectores estarn situadas en un
dE d dE ' plano perpendicular al eje Z y siempre se anularn por
pares. En consecuencia, slo existir intensidad del
dE y dE y'
campo en la direccin del eje Z, por lo que para calcu-
Y
lar E bastar con sumar todos los dE z .


Teniendo en cuenta lo anterior proceded a calcular el vector E que se pide.

dq q dq
dE z K 2 cos u z y la suma ser: E =
e dE z =
0
K
e 2
cos uz

K cos q q
Sacando fuera de la integral las constantes: E = 2
u z 0 dq E K 2 cos u z
e e

El resultado tambin se puede expresar en funcin de la distancia z existente desde el cen-


z
tro del anillo hasta el punto P si tenemos en cuenta que cos y sustituimos:
e
q
EK z uz
z R
3
2 2 2

Para determinar el potencial en el mismo punto bastar considerar que dV = Kdq/e y


sumar todos los dq:

dq K q q q
= dq V K
q
V=
0
K
e e 0 e
En funcin de z y R queda: V K
z2 R2
Campo elctrico 306

14. Determinad la intensidad del campo elctrico y el potencial generados por una
carga q, uniformemente distribuida en una capa esfrica de radio R, en un punto P
situado a una distancia r de su centro.

Revisad el ejercicio anterior y sugerid una posible estrategia para contestar ste.

Para resolver este ejercicio podemos utilizar


el resultado del anterior y considerar a la capa
esfrica como una superficie hueca compues-
ta de infinitos anillos a cada uno de los cuales
Rsen
le corresponder una carga dq. Para realizar
esta descomposicin bastar intersectar la
capa esfrica con planos infinitamente
prximos y perpendiculares a la recta que une
el centro de la esfera con el punto en el cual
queremos hallar la intensidad del campo elc-

trico E . Cada uno de los anillos, generar
una intensidad dE que, de acuerdo con la fi-
gura adjunta (en la que hemos supuesto que q
es positiva) se podr expresar como:
dq
dE K 2 cos u r donde dq ser la carga de la superficie dS del anillo, la cual podre-
e
mos expresar en funcin de la densidad de carga como dq = dS siendo = q/S =
q/4R2 y dS = 2 (Rsen)Rd.

Para calcular la intensidad del campo creado por la distribucin esfrica decarga en el
punto P situado a una distancia r del centro O, podemos sumar todos los dE correspon-
dientes a los infinitos anillos que integran la superficie esfrica de radio R.

Calculad E en un punto P situado a una distancia r R del centro de la esfera.

dq 2 R 2
E = dE K 2 cos u r K 2
sen cos u r d
0 e 0 e

En la integral anterior tenemos tres variables: e, , , que estn relacionadas y para poder
resolver hemos de expresarlas en funcin de una sola. Conviene hacerlo en funcin de e
cuyos lmites de integracin corresponden a r-R (para = 0) y r+R (para = rad).

Sugerid un procedimiento que nos permite relacionar y con e.

Aplicando el teorema del coseno en la figura tenemos: e 2 = r2 + R2 + 2 r R cos (1).


Tambin por el teorema del coseno: R2 = e2 + r2 - 2 e r cos (2).

Derivando la ecuacin (1): 2 e de = 2 r R sen d y despejando: sen d = ede/rR

Despejando cos de la ecuacin (2): cos = (e2 + r2 - R2 )2e r, y sustituyendo:


Campo elctrico 307

2 R 2 K R rR
r 2 R2


E= K sen cos u r d u r 1 de
0 e2 r2 rR e2

rR
K R r 2 R2 K 4 R 2
E = 2
u r e 2
u r . Sustituyendo = q/4R2 queda:
r e rR r

q
E K 2 ur
r

Como vemos, la superficie esfrica cargada se comporta como si toda la carga q estuviese
situada en el centro de la misma.

Cmo podramos ahora calcular la intensidad del campo en un punto P situado a una
distancia r R del centro de la esfera? Sugerid un procedimiento y llevadlo a cabo.

Bastara con cambiar los lmites de integracin ya que en el caso propuesto es fcil ver en
la figura que la distancia e variara desde R-r hasta r+R. Podemos comprobar que al

sustituir se obtiene E = 0. Es decir, que la intensidad del campo elctrico en cualquier
punto situado en el interior de una esfera hueca cargada, es nula.

En un ejercicio posterior veremos cmo es posible llegar a estos mismos resultados de una
forma mucho ms sencilla mediante la aplicacin del teorema de Gauss.

Sugerid y llevad a cabo un procedimiento para determinar el potencial del campo elctri-
co generado por la capa esfrica en cada uno de los dos puntos anteriores.

Para obtener el potencial podemos utilizar un procedimiento anlogo al que hemos segui-
do para el campo elctrico, es decir, consideraremos en primer lugar el potencial en un
punto P debido a un anillo de carga infinitesimal y despus sumaremos todas las contribu-
ciones debidas a los infinitos anillos de este tipo que forman la capa esfrica.

El potencial correspondiente a uno de los anillos de carga dq en un punto P ser:

dq
dV K
e
dq
de forma que el que se debe a toda la esfera se podr obtener como V K
e
Se trata ahora de resolver esta integral.

Para ello, hemos de reducir las dos variables a una sola. Como ya hemos visto, dq = dS
siendo = q/S = q/4R2 y dS = 2 (Rsen)Rd. Con lo que para un punto P exterior a la
capa esfrica nos quedar:
Campo elctrico 308

dq 2R 2 sen d sen d
V= K = K = K 2R 2
e 0 e 0 e

Teniendo en cuenta que sen d = ede/rR, y que e vara entre r-R y r+R:

sen d r R de Kq q
e r R de donde V K
rR
V = K 2 R 2 = K 2 R 2
r R r R
0 e 2rR r

El resultado muestra que el potencial es el mismo que correspondera a una carga q situa-
da en el centro de la esfera. En el caso de que el punto P estuviese situado sobre la esfera,
bastara sustituir, en la expresin anterior, r por el radio de la misma R.

Calculad V para el caso de un punto situado en el interior de la esfera (r R).

El procedimiento ser el mismo, pero los lmites de la integral cambiarn siendo en este
caso desde R-r hasta R+r, con lo que sustituyendo e integrando en la misma expresin
anterior, obtenemos que el potencial en cualquier punto interior de la esfera es:

q
VK
R

Los resultados anteriores nos indican que:

a) En todo el volumen de la esfera conductora el potencial es el mismo y coincide con el


que existe en su superficie.

b) La superficie de la esfera es una superficie equipotencial.

Estas conclusiones tienen una gran importancia ya que se dan en todos los conducto-
res cargados y en equilibrio (aunque no sean esfricos ni huecos). Se trata, por otra
parte, de conclusiones que se pueden explicar de forma cualitativa ya que si en el propio
conductor hubiesen dos puntos a diferente potencial, las cargas se desplazaran por accin
del campo que existira entre esos puntos.

Para terminar, podemos visualizar estas conclusiones representando de forma cualitati-


va el mdulo de la intensidad del campo elctrico y el potencial, en funcin de la distancia
r desde el centro de la esfera hasta el punto donde queramos evaluarlos:
Campo elctrico 309

15. Determinad la intensidad del campo elctrico en el interior y en la superficie de


un conductor cargado y en equilibrio.

Al introducir cargas en un conductor stas se repelern entre s y, como pueden moverse


libremente, se alejarn unas de otras hasta que se alcance una situacin estable en la que
ya no se movern (equilibrio). Ello se traduce en que las distancias relativas entre las car-
gas debern ser las mayores posibles y para que esto suceda tendrn que situarse necesa-
riamente en la superficie del conductor.

Para evaluar la intensidad del campo elctrico en un pun-


to del interior del conductor utilizaremos el teorema de
Gauss aplicndolo a una superficie cerrada cualquiera en
el interior del conductor. Segn dicho teorema el flujo
del campo elctrico a travs de una superficie cerrada
cualquiera S es directamente proporcional a la carga total
existente en el interior de dicha superficie.

Se cumplir que: E dS 4 Kq y como no puede haber carga en su interior (ya

que toda se distribuye en la superficie), tendremos que: E dS 0 , y la nica posi-
bilidad de que esta integral valga 0, cualquiera que sea la superficie interior elegida, es

que E = 0.

Esta propiedad explica que cuando se desea que un campo elctrico no acte sobre un
determinado objeto, se recurra a encerrar a ste dentro de una caja metlica.

Podemos proceder ahora a determinar E en la superficie del conductor cargado y en equi-

librio, plantendonos en primer lugar qu direccin podra tener E en cualquier punto de
la superficie del conductor.

Como las cargas se encuentran en la superficie y en reposo, necesariamente E , si es que
existe, tendr que ser perpendicular a la superficie, ya que, de lo contrario, su componente
tangencial (segn la superficie) hara que sobre las cargas situadas en ese punto el campo
ejerciese una fuerza que provocara su desplazamiento, pero, ya hemos dicho que se haba
alcanzado el equilibrio (las cargas permanecen en reposo).

Visto ya que E debe ser perpendicular a la superficie del conductor, nos falta determinar
su valor, aplicando el teorema de Gauss.

Para ello escogeremos una superficie imaginaria cilndrica


de tamao infinitesimal cuya generatriz sea perpendicular a
la superficie del conductor y cuyas bases queden una en el
interior y otra en el exterior del mismo. Este cilindro englo-
bar una carga dq que corresponde a la carga que el conduc-
tor tiene en la superficie dS, interseccin del cilindro con la
superficie del conductor.
Campo elctrico 310

El flujo a travs de la superficie del cilindro infinitesimal considerado ser, segn el teo-
rema de Gauss: d = 4 K dq. ste flujo puede ser evaluado descomponindolo en la suma
de los flujos a travs de las superficies de las bases interior (1) y exterior (2) del cilindro y
de su superficie lateral:

d = d 1 + d 2 + d L = 4 K dq. Si evaluamos d 1 y d 2 tendremos:



d 1 = E int dS = 0 porque tal y como hemos visto E int = 0.

d L = Eint dSLint Eext dSLext 0 Eext dSLext cos 90 0 , y sustituyendo:

d2 = E ext dS 2 4 K dq .

Efectuando el producto escalar y poniendo dq en funcin de la densidad de carga superfi-


cial, obtenemos finalmente:

Eext dS2 cos0 = 4 K dq Eext dS2 = 4 K dS y como dS2 = dS: Eext = 4 K

Como podemos ver, la intensidad del campo elctrico en la superficie de un conductor


cargado y en equilibrio es directamente proporcional a la densidad de carga. Si el conduc-
tor fuera esfrico, la densidad de carga en todos sus puntos sera constante y, por tanto, la
intensidad de campo elctrico en cualquiera de ellos la misma.

Qu ocurrira si el conductor tuviese algn extremo acabado en punta?

Como sabemos, las cargas tienden a separarse lo ms posible unas de otras, por ello se
acumularan especialmente en las puntas, producindose all una gran densidad de carga
y, en consecuencia, un campo elctrico muy intenso, que llega a ionizar el aire que rodea a
la punta, lo cual provoca que el conductor se descargue rpidamente. En este efecto se
basan precisamente los pararrayos que no son sino conductores acabados en una o varias
puntas y conectados a tierra por el otro extremo. Anlogamente ocurre con las pequeas
varillas metlicas acabadas en punta que los aviones llevan en las alas, por las que se es-
capa la carga electrosttica acumulada en el fuselaje a causa del rozamiento con el aire,
que, de otra forma, podran producir descargas elctricas muy peligrosas. Tambin explica
que sea peligroso colocarse en das de tormenta debajo de postes o rboles, por los que es
ms probable que vayan las descargas.

16. Determinad por aplicacin del teorema de Gauss, la intensidad del campo elctri-
co creado por una esfera conductora de radio R, cargada con una carga q.

Al cargar un conductor sabemos que las cargas se distribuyen por su superficie de forma
que la distancia relativa entre ellas sea lo mayor posible. En nuestro caso, por la simetra
de la superficie esfrica, se distribuirn uniformemente por toda ella (densidad de carga
constante en todos los puntos). El problema consiste, pues, en determinar la intensidad del
campo elctrico generado por una carga uniformemente distribuida en una superficie esf-
rica, problema que ya hemos resuelto, pero que ahora vamos a volver a resolver, mucho
ms rpidamente, utilizando el teorema de Gauss.
Campo elctrico 311

El teorema de Gauss es til para calcular la intensidad del campo elctrico en aquellos
casos en los que la distribucin de cargas que genera el campo presenta una cierta si-

metra, de forma que conozcamos la direccin de E en todos los puntos y su mdulo sea
constante a lo largo de algunas superficies sencillas. ste es precisamente el caso que se

nos plantea en el problema ya que por la simetra existente la direccin de E ser radial y
su mdulo tendr que ser el mismo en todos los puntos que se encuentren a la misma dis-
tancia del centro de la esfera.

En primer lugar procederemos a calcular E en un
punto situado a una distancia r R. Para ello
hemos de tomar una superficie esfrica que pase
por este punto y que sea concntrica con la esfera
conductora. Aplicando el teorema de Gauss a
esta superficie:

E dS 4 Kq E dS cos0 4Kq

en donde el crculo de la integral significa que S es una superficie cerrada y que la integral
se extiende a toda ella. Como E es constante, nos queda:

q q
ES = 4 K q E 4R2 = 4 K q E K y vectorialmente: E K 2 ur
r2 r

En el caso de que el punto se encuentre sobre la propia superficie, bastar hacer r = R para
calcular la intensidad del campo correspondiente.

Utilizad ahora el teorema de Gauss para calcular E en el interior de la esfera.

Para un punto situado a una distancia r R del centro de la esfera,


tomamos de nuevo una superficie esfrica que pase por dicho
punto y que sea concntrica con la esfera conductora. Aplicando
el teorema de Gauss a esta superficie:

E dS 0

ya que no encierra carga alguna, y para que sta integral sea nula, es necesario que la in-
tensidad E = 0.

Vemos, pues, que los resultados son los mismos que los obtenidos anteriormente mediante
la aplicacin directa de la ley de Coulomb y que el proceso resulta ser mucho ms breve y
sencillo.
Campo elctrico 312

17. Sabiendo que el aire se hace conductor (se ioniza), cuando se encuentra en un
campo elctrico cuya intensidad sea igual o mayor de 310 6 V/m, determinad:

a) Mxima carga que podr almacenar un conductor esfrico de radio R en el aire.


b) Potencial mximo que podr alcanzar.

Como ya se ha visto, al cargar un conductor esfrico , la carga


se distribuye uniformemente por toda su superficie y su com-
portamiento para un punto P exterior al mismo es equivalente
a que si toda la carga estuviese situada en su centro, de modo
que la intensidad del campo elctrico en P se puede calcular
mediante la expresin:
q
E K 2 ur
r

De acuerdo con la expresin anterior, los puntos en los que el campo elctrico generado
por la esfera alcanzar su mxima intensidad sern precisamente los de la superficie, en
q
los que el mdulo de la intensidad ser: E K 2
R

Si imponemos la condicin de que E valga 310 6 V/m (en realidad debera ser un infinit-
simo menor), obtendremos el valor de la mxima carga que podr adquirir la esfera des-
pejando q de la expresin anterior:

qmax = ER2/K =3106 R2/9109 = 10-3 R2/3

As, una esfera conductora de 1 m de radio, podra (en las condiciones expresadas en el
enunciado) acumular como mximo una carga de 333 10-4 C . A partir de ese valor, si le
suministrramos ms carga el aire que rodea a la esfera se ionizara y sta se descargara.

Este efecto tiene gran importancia en los generadores de Van der Graaf ya que en ellos se
va acumulando carga en una esfera conductora y aunque, en principio, se podra almace-
nar en ellos cualquier carga, la realidad es que la cantidad de carga se halla limitada por el
efecto de la ionizacin del aire que rodea a la esfera.

En cuanto al potencial electrosttico V en un punto dado situado a una distancia mayor o


igual que el radio de la esfera, sabemos que ste viene dado por la expresin:

q
V= K
r

de modo que para obtener el valor mximo del potencial en la superficie de la esfera,
haremos r = R (radio de la esfera) y sustituiremos q por su valor mximo:

3
q max 9 10 R2
Vmax = K = 9 10 3 10 6 R . Para R = 1 m V = 3106 V
R 3R
Campo elctrico 313

18. Dos conductores esfricos A y B de 10 cm de radio cada uno, estn separados una
distancia de 2 m y con las cargas q1 = 310-8 C y q2 = - 610-8 C respectivamente. De-
terminad el potencial en cada uno de los conductores (suponed que la distancia entre
las esferas es lo suficientemente grande, comparada con el radio, para que no afecte a
la distribucin de las cargas en su superficie).

Si, como se dice en el enunciado, la distancia entre las esferas es lo suficientemente gran-
de como para despreciar los efectos de induccin, podremos considerar que cada una de
ellas se comporta como una carga puntual.

El potencial de cualquiera de las esferas se podr determinar sumando el debido a su pro-


pia carga ms el que se debe a la carga de la otra esfera. As:

q1 q 3 10 8 (6 10 8 )
VA = VA1 + VA2 = K K 2 9 10 9 9 10 9 = 2430 V
RA rAB 01' 2

q2 q 3 10 8 (6 10 8 )
VB = VB2 + VB1 = K K 1 9 10 9 9 10 9 = -5625 V
RB rAB 2 01'

Qu ocurrira si conectsemos ambas esferas mediante un hilo conductor?

Al ser el potencial de la esfera A mayor que el de la esfera B, se producira un movimiento


de cargas (corriente elctrica) de forma que la carga positiva de A disminuira mientras
que la de B aumentara (pasando electrones desde la B hacia la A) hasta que los potencia-
les de ambas se igualasen. Cmo conseguir mantener constante una diferencia de poten-
cial entre dos puntos y aprovechar la energa cintica del movimiento de cargas que se
puede dar entre ellos, son problemas que se abordan en el estudio de la corriente elctrica.

Es posible que un objeto est cargado elctricamente y el potencial a que se encuentre


sea 0?

De acuerdo con lo que hemos visto en este ejercicio s que es posible ya que para ello bas-
tara con situar un segundo objeto tambin cargado a una cierta distancia del primero, de
manera que la suma del potencial debido a la propia carga del primero ms el que se de-
bera a la carga del segundo, diese 0. As, por ejemplo, se puede comprobar que si en este
mismo ejercicio la esfera B tuviese una carga q 2 = -6010-8 C, el potencial de la esfera A
(cargada con q1 = 310-8 C) sera 0.
Campo elctrico 314

19. Determinad lo que suceder cuando una esfera conductora de 10 cm de radio se


conecte al polo positivo de un generador que suministra una diferencia de potencial
de 900 V, estando el polo negativo conectado a tierra.

El punto A, al ser de la superficie de la Tierra, se encontrar en todo instante a potencial


0. El punto B, en principio, estar a un potencial desconocido pero inferior en 900 V al
punto C ya que VC - VB = 900 V.

El punto D, inicialmente, se encuentra a potencial 0 ya que la esfera est descargada y no


hay en la proximidad ninguna otra carga.

Conectamos, en primer lugar, A con B mediante un conductor. Como entre sus extremos
hay una diferencia de potencial, se producir una corriente que durar hasta que el poten-
cial en B y en A sea el mismo. Como el del punto A, por ser la superficie de la Tierra, vale
en toda situacin 0, el punto B alcanzar el potencial 0. En estas condiciones el potencial
del punto C ser de 900 V y si conectamos mediante un conductor con la esfera, tendre-
mos entre sus extremos una diferencia de potencial de 900 V ya que en el punto D el po-
tencial es 0. Se producir entonces una corriente de forma que, a medida que la esfera
vaya adquiriendo carga positiva, el potencial en D ir creciendo hasta que valga 900 V. En
ese momento cesar la corriente y dispondremos de una esfera conductora cargada a po-
tencial de 900 V. La carga adquirida por la esfera ser:

q Vr 900 0'1
V= K q= = = 10-8 C = 10 nC
r K 9 10 9

20. Determinad la capacidad de un conductor esfrico de radio R.

Dado un conductor cualquiera, se puede comprobar que al conectarlo a distintos poten-


ciales V1, V2, V3 ... adquiere distintas cargas q1, q2, q3 ... tal y como se desprende del ejer-
cicio anterior. Experimentalmente se observa que en ese caso, el cociente entre la carga
adquirida y el potencial es siempre un valor constante caracterstico del conductor en
cuestin. A la magnitud q/V se le denomina capacidad C del conductor, puesto que su
valor nos da idea de la carga que puede almacenar (para cada potencial determinado).
Campo elctrico 315

En el caso de que el conductor sea esfrico podemos calcular su capacidad sin necesidad
de recurrir a medidas experimentales ya que, como sabemos, el potencial de una esfera
conductora de radio R cargada con una carga q es:

q
V = K y su capacidad ser, por tanto, C = q/V = R/K
R

Vemos que cuanto mayor sea el radio de la esfera mayor ser su capacidad. De hecho la
capacidad de un conductor esta relacionada con la forma del mismo. As, por ejemplo, si
aplastsemos poco a poco a una esfera conductora, podramos comprobar cmo su capa-
cidad ira cambiando.

21. Cmo podramos aumentar la capacidad de un conductor?

En ocasiones resulta tcnicamente conveniente almacenar mucha carga elctrica y para


ello se utilizan los conductores. Podremos conseguir almacenar cargas elevadas, en prin-
cipio, de dos formas:

a) Con conductores muy grandes (por ejemplo, esferas de un gran radio).


b) Con conductores pequeos pero a elevadsimos potenciales.

Ninguna de las dos opciones resulta muy adecuada ya que la primera exigira en la ma-
yora de los casos demasiado espacio y la segunda porque, como ya hemos visto, si se ele-
va demasiado el potencial, el aire o cualquier otro material dielctrico que rodee al objeto,
se hara conductor y ste se descargara.

Cmo podra aumentarse la capacidad de un conductor sin estos inconvenientes?

Esto puede conseguirse aproximando a un conductor cargado (cuya capacidad queremos


aumentar) otro conductor con carga de signo opuesto. Para comprenderlo podemos imagi-
nar el siguiente proceso:

1) Cargamos el conductor A conectndolo a un genera-


dor entre cuyas placas existe una diferencia de potencial
de valor V, tal y como se muestra en la figura adjunta.
V Generador
Tierra

2) Si ahora lo desconectamos del generador, el conductor habr adquirido una carga q a


un potencial V, de modo que su capacidad ser C = q/V.

3) Situamos en las proximidades del conductor A, otro conductor B con carga negativa.
Campo elctrico 316

El potencial elctrico en el conductor A ser la suma de su propio potencial ms el que la


carga que el otro conductor cree en l (que ser negativo), es decir: V = V + V AB y al ser
VAB negativo, tendremos que V V, lo que significa que el conductor A tiene la misma
carga q pero con menos potencial, es decir, que su nueva capacidad, mientras no retiremos
el conductor B, ser C = q/V (mayor que la anterior).

4) Si ahora conectamos el conductor A de nuevo al generador, al ser V V, pasarn ms


cargas hasta que el conductor se encuentre de nuevo a potencial V. En esta situacin el
conductor tiene el mismo potencial que al principio pero con ms carga (q) y su capaci-
dad ser C = q/V.

Claro est, que cuanto ms influya el conductor B, mayor ser el aumento que se produ-
cir en la capacidad de A. La influencia mutua de los dos conductores depender de su
forma y de su situacin. Los casos ms favorables se producen cuando un conductor en-
cierra al otro y cuando los conductores son superficies planas paralelas y separadas por
una distancia muy pequea. En estos casos se dice que ambos conductores constituyen un
condensador.

22. Determinad la intensidad del campo elctrico creado por dos placas conductoras
paralelas y de superficies planas, con cargas iguales pero de distinto signo, en un pun-
to situado entre las mismas.

Entre las placas existir un campo elctrico debido a una y otra distribucin de cargas, por lo
que en primer lugar conviene que nos planteemos qu caractersticas tendr ese campo.

En principio,
cabe pensar que la direccin del
vector E en la superficie de cualquiera de las
placas tendr que ser perpendicular (de otra
forma las cargas no estaran en equilibrio).
Para comprobar si esta hiptesis es cierta, po-
demos utilizar el teorema de Gauss aplicndolo
al cilindro de la figura, cuyas bases quedan en
el interior de cada una de las placas3:

T = 4 K q siendo q la carga total encerrada en el cilindro.

y como la carga existente en la interseccin del cilindro con cada placa es la misma pero
de signo contrario, q = 0. Por tanto, T = 0 y si descomponemos este flujo, considerando
las bases del cilindro y la superficie lateral:

T = 1 + 2 + L = 0

3
Atencin, las superficies de las placas son planas, pero las propias placas tienen un cierto grosor, de forma
que tanto S1 como S2 figura que se hallan en el interior de cada placa. Las lneas roja y azul del dibujo, repre-
sentan solo las caras enfrentadas de cada placa (vistas de canto).
Campo elctrico 317

Como 1 y 2 son nulos porque se hallan en el interior del conductor y all la intensidad
del campo es 0, hemos de concluir que:

L = E dS = 0, y como ni E ni dS son nulos, necesariamente han de ser perpendicula-
res y, por tanto, el vector intensidad del campo elctrico tendr que ser perpendicular a las
placas del condensador y dirigido de la positiva hacia la negativa.

En estas condiciones las lneas del campo entre las placas


sern paralelas, lo que, como ya
vimos, significa que en todos los puntos el mdulo de E ser el mismo, es decir, se trata
de un campo elctrico uniforme.

De qu factores depender E? Cmo podramos hallarlo?

Si llamamos VAB a la diferencia de potencial


existente entre las placas positiva (A) y negativa
(B), de forma que VAB = VA -VB 0 y d a la
distancia a que se encuentran separadas, pode-
mos pensar que cuanto mayor sea VAB y menor
la distancia que las separa, ms intenso deber
ser el campo existente entre dichas placas.


rB
Para determinar E podemos utilizar la expresin
rA
E dr (VB VA ) aplicndola al
trayecto de la figura.

rB d V

rA
E dr E dr E d (VB VA ) = VAB E AB
0 d

23. Dos bolitas conductoras idnticas y de 2 gramos de masa cada una, se suspenden
de sendos hilos de 1 metro sujetos de un mismo punto del techo. Si al suministrarles la
misma carga se repelen hasta que los hilos formen un ngulo de 60 o, qu valor tiene
su carga?

sol: q = 11310-6 C

24. Dada una carga de 10-5 C uniformemente distribuida sobre una superficie esfrica
de radio 50 cm. Determinad:

a) Intensidad del campo y potencial en un punto de su superficie.


b) Idem en un punto P situado a 1 m del centro de la esfera.

sol: a) ES = 36105 N/C, VS =18105 V ; b) EP = 9104 N/C, VP = 9104 V


Campo elctrico 318

25. Sea un condensador plano vertical cargado a una dife-


rencia de potencial de 5000 V. Entre sus placas, separadas
20 cm, se sita un pndulo elctrico cuya esfera metlica
tiene 1 mm de radio y una densidad de 5 g/cm3. Determi-
nad el ngulo que formar el pndulo si la esfera del mis-
mo se carg a 2000 V.

Como hemos visto, un condensador es un dispositivo formado por dos partes metlicas
(llamadas armaduras) que estn separadas por un aislante. En nuestro caso las partes
metlicas son dos placas de superficies planas y paralelas como se indica en la figura.

Qu es lo que sucede cuando dos lminas conductoras, iguales, descargadas y paralelas,


se conectan a los extremos de un generador?

Al estar las lminas inicialmente descargadas, el potencial en ellas ser nulo, de modo que
por los hilos conductores circularn cargas hasta que el potencial en A sea igual que el del
polo positivo B del generador (y el potencial en D igual que el de C), es decir, hasta que la
diferencia de potencial entre las lminas sea la misma que entre los extremos del genera-
dor, siendo la carga adquirida por las lminas igual y de signo contrario (A positiva y D
negativa). Las cargas suministradas por el generador a cada lmina se distribuirn por su
superficie alejndose lo ms posible unas de otras (ya que se repelen y las lminas son
conductoras) hasta alcanzar una situacin de equilibrio (reposo).

Entre las lminas del condensador cargado existir, un campo elctrico creado por las
distribuciones de carga existentes en ellas. La intensidad de dicho campo ha de ser per-
pendicular a las lminas (sentido desde la positiva a la negativa) y su mdulo vale E =
VAB/d siendo VAB la diferencia de potencial existente entre las lminas (VA -VB) y d la
distancia a que se hallan separadas.

En el problema se nos dice que entre las lminas de un condensador plano se introduce un
pndulo cuya esfera se halla cargada elctricamente.

Qu le ocurrir a la esfera?

Para comprender lo que le sucede, conviene comenzar por


plantearse qu fuerzas actuarn sobre ella. Al encontrarse en
el seno de un campo elctrico y estar cargada,
sobre la esfera,
adems del peso P y de la tensin T del hilo, actuar una
tercera fuerza que ser la fuerza electrosttica Fe ejercida por

el campo. Al actuar Fe la esfera perder la situacin de equi-
librio en la que se encontraba (hilo vertical y tensin igual al
peso) y pasar a otra en la que se cumplir que:


Fe + P + T = 0
Campo elctrico 319

Como podemos ver en la figura, el hilo del pndulo se desviar un cierto ngulo res-
pecto de la vertical. Podemos plantearnos ahora de qu depender dicho ngulo y cmo
podramos hallarlo.

Cabe esperar que cuanto mayor sea la carga q de la esfera, mayor la intensidad del campo
elctrico entre las lminas y menor el peso, ms grande sea el valor de . Para determi-
narlo podemos aplicar la ecuacin anterior expresando cada uno de los vectores en fun-
cin de sus componentes escalares y despejando . De esta forma tenemos:

Fe = q E = q (E, 0) = (qE, 0)

P = (0, -P) = (0, -mg)

T = (Tx , Ty ) = T cos(90 ), Tcos = (-Tsen , Tcos )

Sustituyendo en: Fe + P + T = 0 (qE, 0) + (0, -mg) + (-Tsen , Tcos ) = 0

La ecuacin vectorial obtenida puede descomponerse en dos escalares:

qE -Tsen = 0 Tsen = qE
Tcos - mg = 0 Tcos = mg
qE
y dividiendo entre si ambas ecuaciones obtenemos: tg
mg

Podemos ahora, analizar este primer resultado, comprobando que contempla las hiptesis
iniciales siendo fcil ver, por ejemplo, que cuanto mayor sea la carga q mayor ser el
ngulo (siempre a igualdad de los restantes factores), o que si el campo gravitatorio fue-
se nulo tg sera , lo cual corresponde a un ngulo de 90 y que si no hubiese campo
elctrico tg sera 0 (lo que corresponde a un ngulo de 0 o hilo en la vertical).

Para calcular el valor de hemos de poner E en funcin de los datos que se nos dan y
averiguar el valor de q y de la masa de la esfera conductora.

El valor E viene dado, en este caso, por: E = VAB/d = 5000/02 = 25 104 V/m

q
Como sabemos que la esfera se carg a 2000 V, a partir de V = K , podemos despejar
R
VR 2000 10 3
q, de modo que: q = = = 22210-10 C
K 9 10 9

En cuanto a la masa, como conocemos la densidad basta hacer m = V siendo V el vo-


lumen de la esferita y su densidad, de manera que:

4
m = 5000 3 (10 3 ) 3 = 2110-5 kg
3
qE
Sustituyendo los valores numricos hallados, en la expresin de tg =
mg
obtenemos finalmente que tg = 264210-2 = 15
Campo elctrico 320

26. Determinad el valor de la diferencia de potencial que debemos aplicar a dos lmi-
nas planas, paralelas separadas 20 cm y situadas horizontalmente para que una
partcula de 1 mg de masa y con una carga de 10 nC quede en suspensin entre ellas.

sol: 200 V

27. La esfera de la figura tiene una masa


de 2 mg. Al dejarla en libertad en la
lmina inferior asciende y llega a la
superior al cabo de 04 s. Si la diferencia
de potencial aplicada entre las lminas
es de 103 V y la distancia entre ellas es
de 20 cm, qu carga tiene la esfera?

Al dejar la esfera encima de la lmina cargada positiva-



mente, sobre ella actan dos fuerzas: una, Fe , debida al
campo elctrico existente entre las lminas y otra el peso,

P , debida al campo gravitatorio terrestre. Si la esfera, al
dejarla en libertad asciende, significa que la fuerza elec-
trosttica tiene un valor mayor que el de la fuerza peso.
En esas condiciones la esfera describir una trayectoria
rectilnea conocida.

Cmo podramos calcular el valor de la carga q de la esfera?

Es evidente que del valor que tenga q va a depender


la fuerza electrosttica que acte so-
bre la esfera ya que, como sabemos, Fe q E , de forma que si conocisemos Fe y E
tendramos determinada la carga q haciendo q = Fe /E.

La intensidad del campo E entre las armaduras del condensador se puede calcular median-
te la expresin: E = VAB /d, siendo VAB la diferencia de potencial entre sus placas y d la
distancia entre las mismas. En cuanto a Fe, dado que la trayectoria que sigue la esferita es
conocida, podemos aplicar un tratamiento escalar para su obtencin.

Sugerid y llevad a cabo un procedimiento para calcular el valor Fe de la fuerza elec-


trosttica que se ejerce sobre la esfera durante su movimiento.

Como la trayectoria es rectilnea no habr componente normal de la aceleracin y por


tanto, si tomamos como sentido positivo el del movimiento, la ecuacin fundamental de la
dinmica aplicada a la esfera se podr enunciar como:

Fres t = m at Fe - P = m at Fe = mg + m at
Campo elctrico 321

Como vemos hemos de calcular la aceleracin tangencial con que se mueve la esfera si
queremos obtener el valor de Fe. Como el movimiento de la esfera es rectilneo y unifor-
memente acelerado, podemos utilizar las ecuaciones de dicho movimiento, tomando como
origen de espacios y de tiempos el punto de la lmina positiva donde se deja la esferita y
el instante en que ello ocurre (y como sentido positivo el del movimiento):

t0 = 0; v0 = 0; e0 = 0.

v = at t
e = at t2/2

De la ecuacin de e podemos obtener la aceleracin si consideramos el instante en que la


esfera llega a la otra placa, con lo que a t = 2d/t2 = 202/042 = 25 m/s2

Sustituyendo ahora en la expresin de q, obtenemos:

Fe (mg ma t ) d 2 10 6 (10 2' 5) 0' 2


q= = 3
= 526 10-9 C = 526 nC
E VAB 10

28. Se conectan dos lminas planas y paralelas a una diferencia de potencial de 106 V.
A continuacin se deja en libertad, junto a la lmina positiva, una partcula de 01 mg
de masa y carga 10 nC. Se pide:

a) La rapidez con que llegar a la lmina negativa sabiendo que el efecto de la fuerza
peso resulta despreciable.
b) Energa cintica con que llega a la lmina expresada en eV.

Comenzaremos por realizar un anlisis cualitativo de la situacin planteada:

Al dejar una partcula cargada positivamente junto


a la lmina positiva, estar sometida a la accin de
dos fuerzas: la fuerza electrosttica , Fe y el peso

P , que le provocarn un movimiento de trayectoria
curva, en principio, desconocida y que podremos
estudiar si tomamos un sistema referencial carte-
siano y utilizamos un tratamiento vectorial, para
deducir las caractersticas del movimiento y poder
dar as respuesta a lo que se nos pregunta.
Campo elctrico 322

La aceleracin con que se mueve la partcula cargada ser:


Fres Fe P (qE, 0) (0, mg) qE
a , g
m m m m

Suele ser habitual, como ocurre en nuestro caso, que el valor de la fuerza electrosttica
sea mucho mayor que el peso, de forma que podamos despreciar el efecto de esta ltima.
Con esta aproximacin el problema se simplifica puesto que si se tiene en cuenta slo la
fuerza electrosttica, la partcula describir un movimiento rectilneo y uniformemente
acelerado, movindose desde la lmina positiva hacia la negativa, que podremos estudiar
aplicando un tratamiento escalar. En ese caso la aceleracin ser:

Fe qE
a= y considerando que t0 = 0; v0 = 0 y e0 = 0 v = at y e = at2/2
m m

En el problema hemos de calcular la rapidez con que llegar la partcula a la lmina nega-
tiva. Reflexionad sobre los factores que pueden influir en dicha rapidez y sugerid una
posible estrategia para determinarla.

Cabe esperar que dicha rapidez sea tanto mayor cuanto mayor sea la carga de la partcula
y la intensidad del campo elctrico donde se encuentra, as como cuanto menor sea el va-
lor de su masa, ya que entonces se mover con una aceleracin ms grande y, por tanto,
adquirir, en un tiempo dado, una rapidez mayor.

Un posible mtodo para obtener la rapidez que se nos pide consiste en utilizar la ecuacin:
v = at , determinando previamente el instante t en que la partcula alcanza la otra lmina.
Para ello hemos de darnos cuenta que en ese instante e = d.

1 2 2d 2qEd
d= at' t = v = at = 2ad v =
2 a m

El resultado obtenido confirma las hiptesis que realizamos anteriormente respecto a los
factores que influiran en la rapidez y cmo lo haran. Pero en l figura adems la distan-
cia d entre las lminas del condensador mostrando, al parecer, que cuanto mayor sea dicha
distancia con mayor rapidez llegar la partcula cargada a la otra placa. Sin embargo, la
dependencia de v con esta variable es preciso matizarla ya que lo que hemos dicho slo es
cierto a igualdad de los restantes factores y, como ya sabemos, en el caso de un condensa-
dor de lminas planas y paralelas, el campo elctrico entre ellas es prcticamente unifor-
Campo elctrico 323

me y vale: E = VAB/d, de modo que si d aumenta (manteniendo constante la diferencia de


potencial), la E disminuir en la misma proporcin.

Sustituyendo E = VAB/d en la expresin anterior, nos queda:

2qVAB
v= = 1414 m/s
m

que como, vemos, nos muestra que la rapidez buscada no depende de la distancia exis-
tente entre las dos lminas.

Sugerid y llevad a cabo otra estrategia para obtener la rapidez que se demanda.

El problema podra resolverse de una forma ms breve mediante consideraciones de traba-


jo y energa.

En efecto, si designamos como estado A el correspondiente a cuando se abandona la


partcula cargada sobre la lmina positiva y como estado B cuando sta llega a la lmina
negativa, podemos aplicar entre dichos estados la ecuacin que relaciona el trabajo reali-
zado por la fuerza resultante con la variacin producida en la energa cintica y de ah
tratar de obtener la rapidez vB.

Como la nica fuerza a considerar es la electrosttica (que adems es conservativa), po-


demos escribir que:

Wres A E c A WFe A Ec B Ec A Ep e Ec B Ec A
B B B

Si ahora expresamos la variacin de energa potencial electrosttica en funcin de la dife-


rencia de potencial, nos queda que:

-Ep = -qV = EcB - EcA y como EcA = 0 y -V = VAB , qVAB = EcB

mv 2B 2q VAB
qVAB = y despejando vB obtenemos finalmente: v B .
2 m

La energa cintica con que la partcula positiva llega a la lmina negativa se puede de-
terminar pues, como EcB = q VAB .

Vemos que el valor de dicha energa depende exclusivamente de la carga de la partcula y


de la diferencia de potencial existente entre las placas, de modo que cuanto ms grandes
sean ambos factores con mayor energa cintica llegar la partcula. El dispositivo estu-
diado (lminas conductoras conectadas a una diferencia de potencial elevada), constituye
la base de los aceleradores lineales de partculas, que se utilizan para dotar a diversas
partculas cargadas, de una gran energa cintica, con lo que se pueden utilizar como pro-
yectiles con los que romper los ncleos atmicos e investigar as la estructura ntima de la
materia.
Campo elctrico 324

Sustituyendo valores: EcB = qVAB = 10 10-9 106 = 10-2 J

En el problema se nos pide tambin que expresemos la energa cintica en electrn-


voltios (eV). Se trata de una unidad muy utilizada en fsica de partculas, que significa la
energa cintica que adquiere un electrn por la accin exclusiva de un campo elctrico,
cuando se desplaza de un punto a otro entre los que existe una diferencia de potencial de 1
V. (Ntese que en este caso VAB = -1V ya que el electrn, por ser una carga negativa, es
desplazado por el campo hacia potenciales crecientes). Para determinar su equivalencia en
julios basta con aplicar la expresin, ya manejada anteriormente:

Wres A Ec A WFe A EcB q VAB


B B B

Sustituyendo q por la carga del electrn qe = -1610-19 C y VAB por -1 voltio, nos queda:

1 eV = -1610-19 (-1) = 1610-19 J

En nuestro caso como EcB = 10-2 J, tendremos: EcB = 10-2 /1610-19 = 6251016 eV

29. Una esfera conductora de 10 g de masa y


1 mm de radio se carga conectndola a un 5 cm 5 cm
potencial de -9000 V. Si se deja en el punto
medio entre dos lminas verticales separadas
10 cm y conectadas a una diferencia de poten-
cial V, determinad:

a) Valor de V si sabemos que llega hasta la lmina correspondiente con rapidez de 10


m/s (se desprecia el efecto de la fuerza peso).
b) Energa cintica con la que llega a la lmina en eV.

sol: a) V =1000 V; Ec = 31.1012 eV

30. Un haz de electrones penetra en el espacio comprendido entre las placas de un


condensador plano, paralelamente a stas, con una rapidez v. Si entre las placas se
aplica una diferencia de potencial V se observa que, a la salida, la trayectoria forma
un ngulo de 200 con la direccin inicial .

a) Cul ser la desviacin si se duplica la rapidez inicial?


b) Y si dejando la misma rapidez inicial se duplica la diferencia de potencial apli-
cada? (Se desprecia el efecto del peso).

sol: a) = 520 , b) = 360.


9. MOVIMIENTO ONDULATORIO

1. Razonad cmo se propaga una onda longitudinal a lo largo del conjunto de pndu-
los de acero de la figura.

Supongamos, en primer lugar, que tenemos un nico pndulo. Si lo separamos ligeramen-


te de su estado natural estamos introduciendo una perturbacin. sta accin lleva
consigo el suministro de una energa que quedar, en principio, acumulada en la propia
perturbacin. Si soltamos el pndulo, desde un ngulo pequeo, oscilar con un movi-
miento armnico simple y por tanto su rapidez en cualquier instante se podr expresar
mediante la ecuacin v = Awcos(wt) (siendo A la amplitud de oscilacin, w =2/T y T el
periodo).

El valor de la energa E asociada a una partcula que realiza este tipo de movimiento,
permanece constante y puede ser evaluado en cualquier posicin como la suma de la
energa cintica y la potencial. En particular, cuando pase por la posicin de equilibrio su
energa potencial ser nula y su energa cintica mxima, de modo que podemos escribir:

1 1 1
E = Ecmax = mv 2max m A 2 w 2 m A 2 4 2 2 E = 2 2 m 2 A 2
2 2 2

en donde es la frecuencia de la oscilacin ( =1/T). En ausencia de fuerzas de friccin,


esta energa permanecera indefinidamente en el pndulo.

Si realizamos el mismo proceso con el pri-


mero de los pndulos del conjunto represen-
tado en la figura adjunta, al soltarlo colisio-
nar con el siguiente. Qu ocurrir enton-
ces con la energa?

Como se trata de un choque elstico y directo entre dos cuerpos de la misma masa, inter-
cambiarn sus velocidades, de modo que el pndulo 1 quedar en reposo y el 2 colisionar
con el 3 con la misma velocidad con que el 1 lo hizo con l. Este proceso se repetir con
todos los pndulos. Vemos pues, que cada pndulo transmitir ntegramente la energa al
siguiente, permaneciendo en reposo en la posicin que ocupaba originalmente.

A los fenmenos fsicos de este tipo, en los que se produce un transporte de energa sin
que exista un transporte de materia, se les da el nombre de movimientos ondulatorios.

Qu ocurrira si en lugar de desplazar el pndulo 1 en la direccin de la fila de pndulos


lo hiciramos perpendicularmente al plano del papel?
Movimiento ondulatorio 326

Veramos que permanecera oscilando sin interaccionar con el resto, es decir, que la per-
turbacin (energa) queda localizada (no se propaga). En los casos en que esto ocurre se
dice que el medio no es elstico frente a la perturbacin introducida.

Qu condiciones son necesarias para que se produzca un movimiento ondulatorio en un


medio material?

Podemos resumirlas diciendo que es preciso introducir una perturbacin (energa) y que el
medio sea elstico para dicha perturbacin.

Conviene tener en cuenta que, en general, los medios no son perfectamente elsticos y
por esta causa, la perturbacin se va amortiguando a medida que avanza (va disminuyendo
su amplitud). En nuestro ejemplo, bastara con separar ligeramente los pndulos para
apreciar que tras la colisin no quedaran en reposo y, por tanto, que no se ha transmitido
ntegramente la energa de uno a otro. Cuanto ms separados estn, menos elstico ser el
medio y ms se amortiguar la perturbacin.

Si realizsemos la experiencia propuesta con los pndulos ligeramente separados unos de


otros, no veramos lo que sucede con cada uno en particular debido a la rapidez del fen-
meno, pero s que tendramos una sensacin visual que nos permitira apreciar una
compresin que parecera desplazarse de izquierda a derecha (suponiendo que slo intro-
ducimos una perturbacin en el primer pndulo).

Qu deberamos hacer para conseguir que el conjunto de los pndulos estuviera recorri-
do por una serie sucesiva de perturbaciones?

Tendramos que ir introduciendo en el primer pndulo la misma energa por unidad de


tiempo, que la que se est propagando. En ese caso visualizaramos compresiones y enra-
recimientos que se desplazaran hacia la derecha (equivalentes, respectivamente, a las
crestas y valles de las ondas transversales).

El fenmeno descrito suceder siempre que la perturbacin tenga la misma direccin que
la de propagacin de la onda y se le da el nombre de onda longitudinal

2. Obtened la ecuacin que representa el avance de un movimiento armnico simple


(onda armnica) por un resorte que se encuentra sobre un plano horizontal.

Tomemos un muelle elstico (tambin podra valer una cuerda) y situmoslo sobre un
plano horizontal con el que presente un rozamiento despreciable.

Si aplicamos en uno de sus extremos un movimiento armnico simple cuya direccin se


encuentre sobre el plano y sea perpendicular al muelle (ver figura adjunta), veremos que
la perturbacin no queda localizada en el punto en el que se ha introducido (foco) sino que
viaja a lo largo del resorte.
Movimiento ondulatorio 327

Conviene diferenciar bien entre lo que es el efecto visual de la propagacin y el movi-


miento que realmente se propaga. Para ello nada mejor que tomar una serie de instantne-
as en las que se pueda apreciar la posicin de todos los puntos que constituyen el resorte
vistos desde arriba (para simplificar, consideraremos que el medio es perfectamente els-
tico y que, por tanto, no hay amortiguamiento).

En la figura, las flechas indican el sentido en que va a desplazarse el punto considerado


(lo que nos ayudar a hacernos una idea de cmo va a evolucionar el conjunto). De la ob-
servacin de cmo se mueven los puntos del medio y cmo se propaga la perturbacin,
podemos concluir que:

a) Se aprecian una serie sucesiva de crestas y de valles que avanzan por el resorte.

b) En el tiempo de un periodo T, la perturbacin avanza una longitud que abarca una cres-
ta y un valle. Dicha longitud (medida sobre la recta, no sobre el muelle), recibe el nombre
de longitud de onda y se representa por .

c) Si coloresemos varios puntos del resorte para concentrar en ellos nuestra atencin,
dejaramos de ver las crestas y los valles, para apreciar que dichos puntos describen exac-
tamente el mismo movimiento armnico simple que el foco, pero con un cierto retraso.
Movimiento ondulatorio 328

Para poder estudiar el fenmeno con mayor profundidad nos interesar cuantificarlo. Con
este fin, trataremos de expresar matemticamente el movimiento armnico simple que se
propaga por el resorte (medio elstico). Se trata de obtener una ecuacin que nos permita
conocer el valor de la perturbacin en un punto P cualquiera del resorte, en cualquier ins-
tante t (ecuacin de propagacin de una onda armnica).

Si tomamos unos ejes como los de la


figura adjunta y consideramos que se
introduce en el foco (situado en el
punto O) un movimiento armnico
simple de amplitud A y frecuencia ,
la ecuacin que determinar el movi-
miento de este punto ser:

YF = Asen2t.

En la ecuacin anterior t es el tiempo que indica un cronmetro que se puso en marcha en


el momento en que el punto O (en este caso el foco) inici el ascenso, de modo que susti-
tuyendo t por el valor correspondiente a un instante dado, obtendremos el valor de la
elongacin del foco YF en ese preciso instante.

Cul ser la ecuacin correspondiente a un punto P situado a una distancia x del foco?

Si tenemos en cuenta que la perturbacin que se inici en el foco en el instante 0, tarda un


cierto tiempo t en llegar al punto P, es lgico pensar que en cualquier instante t posterior
a t el punto P llevar vibrando (t -t) s y en consecuencia la ecuacin que determinar su
movimiento ser:

YP = Asen2 (t-t)

Como la perturbacin se propaga con movimiento uniforme, podemos escribir que la ra-
pidez con que lo hace viene dada por v = x/t y sustituyendo en la ecuacin anterior:

YP = Asen2 (t-x/v)

Si ahora tenemos en cuenta que el periodo T y la frecuencia son inversamente propor-


cionales ( = 1/T), obtenemos que: YP = Asen2 (t/T-x/vT)

La expresin obtenida corresponde a la ecuacin del movimiento ondulatorio analizado.


En ella se denomina fase a 2 (t/T-x/vT), es decir, al ngulo del cual se calcula el seno y
que ir expresado en radianes.

La funcin obtenida es peridica, ya que se trata del producto de una constante (A) por
una funcin peridica (sen ). Si la analizamos con ms detenimiento podemos compro-
Movimiento ondulatorio 329

bar que es doblemente peridica y que los valores de Y se repiten en el tiempo (para un
valor de x determinado) y en el espacio (para un instante dado).

a) En el tiempo. Si nos situamos en un punto P cualquiera del medio por donde se propa-
ga la onda (fijamos el valor de x) y dejamos transcurrir el tiempo, veremos que para que
en dos instantes dados t1 y t2 la elongacin YP tome idntico valor, se debe cumplir que:

sen2 (t1 /T-x/vT) = sen2 (t2 /T-x/ vT)

Esta igualdad se cumplir siempre que los ngulos se diferencien en un nmero entero de
veces 2 radianes. Por tanto:

2 (t2 /T-x/vT) - 2 (t1 /T-x/vT) = n2 (donde n es un nmero entero cualquiera).

Simplificando: t2 - t1 = nT. Consecuentemente, el menor intervalo de tiempo para el cual


se repetir el estado de vibracin del punto P ser precisamente cada periodo T (n = 1).

b) En el espacio. Si en un instante determinado t (fijamos el valor de t), evaluamos la per-


turbacin en distintos puntos del medio por el que se realiza la propagacin (como si
tomsemos una fotografa), veremos que hay una serie de puntos que se encuentran en
idntico estado de vibracin. Si nos fijamos en dos de ellos (dados por x1 y x2) podremos
escribir que para que la elongacin YP de cada uno tome idntico valor, se debe cumplir
que:

sen2 (t/T-x1 /vT) = sen2 (t /T-x2 /vT)

Esta igualdad se cumplir siempre que:

2 (t/T-x1 /vT) - 2 (t /T-x2 /vT) = n2 (siendo n un entero cualquiera).

Simplificando: (x2 /vT) - (x2 vT) = n , y despejando obtenemos que x2 - x1 = nvT, de mo-
do que la distancia existente entre dos puntos consecutivos del medio que se encuentren
en el mismo estado de vibracin se obtendr haciendo n = 1 en la expresin anterior. A
dicha distancia se le denomina longitud de onda , y vale = vT.

Cmo variara la expresin que nos da el valor de la perturbacin en un punto cualquie-


ra P del medio si al poner el cronmetro en marcha, la perturbacin en el foco (x = 0) no
hubiera sido nula?

Basta considerar que, en este caso, al existir una fase inicial (0), la ecuacin se transfor-
mar en:
t x
YP A sen 2 0
T

en la que, como podemos ver, cuando t = 0 y x = 0, obtenemos que Y = A sen 0 (estado


de vibracin del foco en el instante inicial). En el caso particular de que 0 sea 0 la ex-
presin se convierte de nuevo en la anterior.
Movimiento ondulatorio 330

3. Un sonido de una frecuencia de 200 Hz se desplaza por un cierto medio slido a


3000 m/s y en el aire lo hace a 340 m/s. Determinad la longitud de onda que tendr la
onda sonora en cada uno de estos medios.

sol: En el slido: s = 15 m y en el aire a = 17 m.

4. Determinad la longitud de onda que presenta en el aire una onda electromagntica


de 100 Hz sabiendo que se propaga a 300.000 km/s. Y en el agua, donde se propaga a
225.000 km/s?

sol: En el aire, = 3106 m . En el agua, = 225106 m.

5. Sabiendo que la funcin de una onda viene dada en metros para t en segundos y
x y d en metros. Determinad las caractersticas de los movimientos ondulatorios
cuyas ecuaciones de propagacin son:

a) = 2 sen 2 (t/001- x/30)


b) = 003 sen 2 (60t -2d)
c) = sen 2 (t/2 + x/20)
d) Y = 001 sen (x - t/2)

La ecuacin general que representa la propagacin de una onda plana armnica longitudi-
nal cuya direccin de propagacin coincide con el eje OX de coordenadas es de la forma:

t x
Asen 2 0
T

en la que representa el valor de la perturbacin en un punto cualquiera del medio (si la


onda fuese plana y transversal se representara por Y). Tambin puede suceder que, si la
direccin de propagacin no coincide con OX, figure d en lugar de x.

Cada onda plana armnica est caracterizada por unos valores de A (amplitud), T (perio-
do), (longitud de onda) y 0 (fase inicial), no siendo necesario explicitar los valores de
la frecuencia ni de la velocidad de propagacin de la onda v, puesto que estn relacio-
nados con los anteriores mediante las expresiones = 1/T y v = /T. Si nos dan los valores
de las magnitudes citadas, no tenemos ms que sustituirlos en la expresin de para ob-
tener la ecuacin de propagacin correspondiente a dicha onda plana armnica.

Cmo deberamos proceder si lo que conocemos es la ecuacin de propagacin y que-


remos determinar los valores de las magnitudes caractersticas?

Es razonable pensar, que tendremos que comparar la ecuacin dato con la expresin gene-
ral e identificar los coeficientes. Apliquemos este procedimiento en los casos en que se
nos plantea en el enunciado de este ejercicio.
Movimiento ondulatorio 331

a) = 2 sen 2 (t/001- x/30). Identificando coeficientes tenemos:

A = 2 m (ya que va en m y la funcin sen no tiene unidades).


T = 001 s (porque t va en segundos y el ngulo debe ir en radianes).
= 30 m (porque x va en m y el ngulo debe ir en radianes).
0 = 0 (en la ecuacin no figura ninguna fase inicial).

Se trata, pues, de una onda plana armnica que se desplaza segn OX (+) de forma que en
el instante t = 0 el punto que se encuentra en x = 0 tiene fase 0. Sus magnitudes caracters-
ticas son:

A = 2m, T = 001 s, = 1/T = 100 Hz, = 30 m y se propaga con v = /T = 3000 m/s.

b) = 003 sen 2 (60t -2d).

Se trata de una onda plana armnica que se desplaza en una cierta direccin de forma que
a una distancia d = 0 del punto tomado como origen de distancia (que suele ser el foco) en
t = 0 tiene fase 0. Identificando coeficientes vemos que:

A = 003 m; 1/T= 60 T= 1/60 s; = 1/T = 60 Hz; 1/ = 2 = 05 m; v =/T = 30 m/s

Otra posibilidad es poner la ecuacin dada en el enunciado en la forma:

t d
= 0'03 sen 2
1 /60 1 /2

e identificar los coeficientes directamente.

c) = sen 2 (t/2 + x/20).

Esta expresin presenta una novedad respecto de las dos anteriores y es que la x lleva sig-
no positivo. Cmo se interpreta esto? Si nos fijamos veremos que en estas condiciones
aquellos puntos que tengan mayor valor de x tendrn tambin una fase mayor, es decir,
comenzaron a vibrar antes, lo que significa que la perturbacin est viajando hacia valores
de x decrecientes, o lo que es lo mismo, segn OX (-).

Identificando coeficientes: A = 1 m, T = 2 s, = 05 Hz, = 20 m , v = 10 m/s, 0 = 0.

d) Y = 001sen (x - t/2)

En esta expresin apreciamos dos variaciones respecto a la general:

En primer lugar, ha sido sustituida por Y, lo que significar que se trata de una onda
armnica plana transversal de forma que la perturbacin que se propaga tiene la direccin
de OY.
Movimiento ondulatorio 332

En segundo lugar, en la fase figuran cambiadas de signo la x y la t. Si consideramos la


relacin trigonomtrica sen = sen (- +), veremos que la ecuacin anterior puede ex-
presarse como:

t
Y 0' 01sen x
2

Por tanto, se trata de una onda armnica plana transversal que viaja segn OX (+) de for-
ma que en el instante t = 0, el punto x = 0 tiene una fase de rad. La expresin anterior se
puede escribir de forma coincidente con la general como:

t x
Y 0' 01sen 2 con la que resulta muy sencillo identificar coeficientes:
4 2

A = 001 m, T = 4 s, = 1/4 Hz, = 2 m , v = 05 m/s, 0 = rad.

6. La ecuacin de propagacin de una onda es: =35 sen (03x - t + 02), ( en mm


si t en s y x en m), representa la propagacin de una onda plana armnica. Expresad
de tantas formas como sepis la ecuacin de este movimiento y sealad cules son sus
caractersticas.

Anteriormente hemos visto que la ecuacin

t x
Asen2 0
T

nos sirve para representar una onda armnica plana que se propaga en un medio sin absor-
cin. La ecuacin que se nos da en el enunciado no presenta la misma forma. No obstante
no se trata de un error ya que una ecuacin de este tipo puede ser expresada mediante
otras formas distintas pero todas ellas equivalentes. Para comprobarlo, vamos a escribir la
ecuacin del enunciado de tres formas diferentes haciendo uso de las conocidas relacio-
nes trigonomtricas:

a) sen = sen (- + )
b) sen = cos ( - /2)
c) cos = cos (-)

Aplicando las relaciones anteriores la ecuacin que se nos da en el enunciado de este ejer-
cicio, =35sen (03x-t +02), se puede expresar de tres formas distintas:

a) = 35sen (t- 03x-02+) = 3' 5sen t 0' 3x ( 0' 2)


b) = 35cos (03x-t +02-/2) = 3' 5cos 0' 3x t (0'2 /2)
c) = 3' 5cos 0' 3x t (0'2 /2) = 3' 5cos t 0' 3x ( /2 0'2)
Movimiento ondulatorio 333

Si consideramos la primera de ellas, veremos que se presenta en la forma en que habi-


tualmente trabajamos:

t x
Asen 2 0
T

En efecto, = 3' 5sen t 0' 3x ( 0' 2) y comparando con la ecuacin general:

A = 35 mm; 2/T= T= 2 s = 05 Hz; 2/ =03 =209 m; v = /T = 105 m/s

En cuanto a la fase inicial, 0 = - 02 = 29 rad. Se trata pues de una onda armnica pla-
na que se desplaza segn OX (+) con una amplitud de 35 m, una frecuencia de 05 Hz y
una longitud de onda de 209 m, a una velocidad de 105 m/s.

7. Qu caractersticas tiene una onda cuya ecuacin de propagacin viene dada por:
Y= 2 cos2 (500t - 05x) (donde x e Y van en m y t en s)?

sol: Onda plana transversal que se propaga hacia OX (+), A = 2 m, = 500 Hz


= 2 m, 0 = /2 rad (si tomamos como ecuacin de la onda la funcin seno)

8. Una emisora de radio emite una onda electromagntica cuyo campo elctrico viene
dado por la ecuacin: E(x,t) = 10-3 cos (x - .... t). Completa la ecuacin sabiendo que
la velocidad con que se propaga la onda es c = 3108 m/s.

sol: E(x,t) =10-3 cos (x - 3108 t)

9. Obtened la frecuencia y la longitud de onda del campo elctrico definido por la


ecuacin: E(x,t) = 10-3 cos (200x - 51010 t), donde x se mide en m y t en s. Cul es el
ndice de refraccin del medio? (c = 3 108 m/s).

sol: = 795109 Hz; = 0031 m; n =12

10. Dada una onda armnica plana de amplitud A = 02 m y frecuencia = 05 Hz


que se propaga con rapidez v = 025 m/s en un medio dado, se pide:

a) Ecuacin de la perturbacin en un punto P situado a 3 metros del foco sabiendo


que el cronmetro se puso en marcha al iniciarse la perturbacin en el foco.
b) Valor de la perturbacin en P en los instantes 5, 13 y 155 (todos ellos en segundos).

Si hacemos coincidir la direccin de propagacin con OX(+) y el origen con el foco, la


ecuacin de la onda ser de la forma:
Movimiento ondulatorio 334

t x
Asen 2 0
T

Esta ecuacin quedar perfectamente definida cuando conozcamos los valores de A, T,


y 0, los cuales podemos averiguar con los datos que se nos dan en el enunciado.

A = 02 m; = 05 Hz T= 1/ = 2s; v = 025 m/s = v T= 05 m

Para t = 0, en x = 0 la fase es 0 = 0, luego la ecuacin para cualquier punto ser:

t x
0'2 sen2
2 0'5

y si ahora particularizamos para el punto P que se encuentra en x = 3 m:

t
P 0' 2sen2 6
2

Podemos ahora proceder a calcular el valor de la perturbacin en el punto P en los ins-


tantes demandados 5 s, 13 s y 155 s.

Para ello basta sustituir los valores de t correspondientes en la expresin anterior:

Para t = 5 s: P 0' 2sen2 2'5 6 .

Como vemos la diferencia dentro del parntesis es negativa Cmo se interpreta esto? El
tiempo que tarda la perturbacin en llegar desde el foco a un punto P situado a 3 m de
distancia del mismo es t = x/v = 12 s, es decir, mayor de 5 s. Por tanto no debemos de
seguir operando porque al no haber llegado todava la perturbacin al punto P, ste se en-
contrar en reposo en la posicin de equilibrio, es decir: P = 0.

Para t = 13 s: P 0' 2sen2 6'5 6 = 0.

Como el periodo vale T = 2 s y la perturbacin emplea en llegar 12 s al punto P, conclui-


mos que dicho punto, en el instante t =13 s, se encuentra en la posicin de equilibrio, des-
pus de haber realizado medio ciclo.

Para t = 155 s:

P 0' 2sen2 7'75 6 = 02sen35 = 02sen (2 + 15) = 02sen15

Por tanto, en el instante t = 155 s tendremos P = - 02 m y el punto P habr realizado


3/4 de ciclo.

Cul sera la elongacin del punto P en el instante t = 175 s?


Movimiento ondulatorio 335

Sin realizar ningn clculo podemos decir que ser la misma que acabamos de calcular
para t = 155 s, ya que al ser el periodo T = 2s, cada 2 s se repetir el estado de vibracin.

11. Determinad la diferencia de fase que habr en un mismo instante entre dos puntos
dados de un medio por el que se propaga una onda, sabiendo que se encuentran a dis-
tancias de 10 m y 16 m del foco, que la velocidad de propagacin de la onda es de 300
m/s y su periodo T = 004 s.

Como en el enunciado no se especifica nada al respecto, consideraremos el caso sencillo


de una onda armnica plana que se propaga segn el eje OX(+) con el foco en el origen,
de modo que la ecuacin que nos da el valor de la perturbacin en cualquier punto e ins-
tante ser:

t x
Asen 2 0
T

Como ya hemos indicado en ejercicios anteriores, la fase es el ngulo expresado en radia-


nes, que siendo funcin de t y de x determina el valor de de un punto en un instante
dado.

Si consideramos los puntos P (xP = 10 m) y Q (xQ = 16 m), la perturbacin en cada uno de


ellos viene dada por:

t x t xQ
P = Asen 2 P 0 y Q = Asen 2 0 respectivamente.
T T

En un instante cualquiera, la fase correspondiente a cada uno de esos puntos ser:

t xP t x
P 2 0 para el punto P, y Q 2 Q 0 para el Q.
T T

Para evaluar la diferencia de fase existente entre ambos, tendremos que expresarla como
= P - Q ya que al estar Q ms alejado, comenzar a moverse ms tarde y su fase en
un mismo instante ser menor que la de P.

Sustituyendo obtenemos que:

t x t x 2
P - Q = 2 P 0 - 2 Q 0 P Q (x Q x P )
T T

Si analizamos el resultado anterior, nos daremos cuenta de que la diferencia de fase de-
pende de la diferencia de distancias entre los puntos y el foco as como del valor de la
longitud de onda. Cuando dos puntos se encuentran en idntico estado de vibracin se
dice que estn en concordancia de fase. Ello exige que en cada instante el valor de la per-
turbacin en cada uno de esos dos puntos sea el mismo, para lo cual se debe de cumplir que:
Movimiento ondulatorio 336

P - Q = n2 (siendo n un nmero entero).

Si introducimos esta condicin en la ecuacin obtenida, nos queda:

2
(x Q x P ) = n2 , y despejando: xQ - xP = n

As pues:

Para que dos puntos situados sobre el mismo eje OX en el que se halla el foco se encuen-
tren en fase, la distancia existente entre ellos ha de ser igual a un nmero entero de veces
la longitud de onda.

La conclusin es lgica pues, como ya vimos en un ejercicio anterior, la longitud de onda


se puede interpretar como la distancia existente entre dos puntos consecutivos que se
hallan en idntico estado de vibracin (y, por tanto en fase).

As, por ejemplo, todos los puntos sealados con un * en la figura siguiente, se encuentran
en fase ya que para cualquier pareja de ellos la distancia que los separa es n

Podemos, proseguir con este anlisis y plantearnos ahora qu deber ocurrir para que dos
puntos se encuentren en oposicin de fase (es decir que en cada instante la elongacin o
valor de la perturbacin sea el mismo pero con distinto signo).

Para que dos puntos se encuentren en oposicin de fase, la diferencia de fase existente
entre ellos deber de ser igual a un nmero impar de veces rad. De ese modo sen para
cada uno ser igual y opuesto y lo mismo ocurrir con el valor de la perturbacin en
todo momento. As pues, si introducimos esta condicin en la ecuacin anteriormente
obtenida para la diferencia de fase entre dos puntos, nos queda que:

2
(x Q x P ) = (2n+1) , y despejando: xQ - xP = (2n 1)
2

Por tanto: Para que dos puntos se encuentren en oposicin de fase, la diferencia de dis-
tancias al foco deber ser igual a un nmero impar de semilongitudes de onda.
Movimiento ondulatorio 337

En la figura siguiente los puntos sealados con * se encuentran en oposicin de fase, ya


que la distancia entre ellos es de 5/2.

En el ejercicio que nos ocupa, para calcular el valor de la diferencia de fase, necesitamos
saber primero el de la longitud de onda:

2 2
= vT = 300 004 = 12 m, de modo que: P - Q = (x Q x P ) = (16 10) = rad
12

Al ser la diferencia de fase un nmero impar de veces , cuando la perturbacin alcance el


mximo valor en uno de los puntos, tomar el valor mnimo en el otro. Conviene darse
cuenta tambin de que la distancia entre los puntos P y Q es una semilongitud de onda,
coherentemente con los razonamientos anteriores.

12. Una onda cuya frecuencia es de 500 Hz tiene una velocidad de propagacin, en un
cierto medio, de 350 m/s. Qu distancia hay entre dos puntos de ese medio que en un
instante dado tienen una diferencia de fase de 600 ?

sol: d = 0117 m

13. Una onda est representada por la ecuacin: Y = 2 cos 2 (t/4 + x/160) (donde x e
Y van en cm y t en s). Determinad:

a) Caractersticas de la onda.
b) Diferencia de fase para dos posiciones de la misma partcula cuando el intervalo de
tiempo transcurrido sea de 2 s.
c) Diferencia de fase en el mismo instante entre dos partculas separadas entre si 120
cm en la direccin de avance de la onda.

sol: a) Onda plana transversal que se mueve hacia x decreciente A= 2 cm; T= 4s; =160 cm;
v = 04 m/s; b) = rad; c) = 3/2 rad.

14. Una onda sonora de una frecuencia de 486 Hz se propaga en el aire con amplitud
constante de 025 mm. Sabiendo que su longitud de onda es de 70 cm, calculad la ve-
locidad de propagacin de la onda y la velocidad de las partculas del medio.
Movimiento ondulatorio 338

Como se nos indica en el enunciado que la onda se propaga con una amplitud constante,
debemos considerar que se trata de una onda armnica plana que avanza en un medio en
el que la absorcin es despreciable.
Tomando la propagacin segn OX(+) y el origen en un punto en el que la fase sea nula
en el instante inicial, la ecuacin ser:
t x
Asen 2
T

Su velocidad de propagacin ser en la misma direccin y sentido que OX(+) y su mdulo


valdr: v =/T = = 07 486 = 3402 m/s (velocidad de propagacin del sonido en el aire).

La velocidad con que se propaga una onda mecnica por un medio dado, no es lo mismo
que la velocidad con que vibran las partculas que transmiten la perturbacin. Cmo
podramos conocer esta ltima?

Hemos de pensar que lo que se propaga es una perturbacin armnica de modo que las
partculas del medio describen un movimiento armnico simple y, en consecuencia, vi-
brarn sobre su trayectoria con la velocidad que corresponda a dicho movimiento. sta
tendr la misma direccin y sentido que la de propagacin de la perturbacin en el caso de
las ondas longitudinales, como el sonido en el aire, o perpendicular a ella (ondas transver-
sales). Sabemos que para una partcula que describe un movimiento armnico simple, la
rapidez con que vibra en cualquier instante viene dada por:

t x
v = d/dt = Aw cos2 .
T

Este resultado muestra que las partculas del medio vibran con una rapidez que vara sinu-
soidalmente entre vmax = +Aw y vmin = -Aw.

En nuestro caso:
t x 2 t x
v = Aw cos2 = 0' 25 10-3 cos2
T 1 /486 1 /486 0'7

de donde resulta: v = 0763 cos [2 (486t -143 x)]

+
15. Una onda longitudinal plana y armnica, se propaga segn OX con una rapidez
de 2400 m/s, siendo su longitud de onda 18 cm. En el punto x = 0 la elongacin es
mxima en el instante 24 s y vale 01 mm. Hallad:

a) La ecuacin que describe la propagacin de la onda.


b) La elongacin y la velocidad del punto de coordenada 1 m en el instante 10-3 s.

sol: a) = 10-4 sen [2 (t/7510-5 - x/018) - 044; b) = - 9510-5 m; v = -219 m/s


Movimiento ondulatorio 339

16. La ecuacin de una onda es Y(x,t) = 2 sen (t + x/80) donde x e Y se expresan en


cm y t en s. Determinad:

a) Caractersticas de la onda.
b) Elongacin, velocidad y aceleracin del punto situado en x =160 cm para t = 5 s.
c) Diferencia de fase entre dos puntos separados 120 cm en un mismo instante.

sol: a) Onda plana transversal movindose hacia x decreciente, A=2 cm, = 05 Hz, T=2 s,
= 160 cm, v = 80 cm/s; b) Y= 0, v = -2 cm/s, a = 0; c) = 15 rad.

17. Una onda armnica longitudinal se propaga a lo largo de un resorte horizontal en


el sentido negativo de OX, siendo 20 cm la distancia entre dos puntos sucesivos que
estn en fase. El foco emisor vibra con una frecuencia de 25 Hz y una amplitud de 3
cm. Determinad:

a) Ecuacin de propagacin sabiendo que el foco emisor se encuentra en el origen de


coordenadas y que en el instante 0'1 s su elongacin es nula y su rapidez positiva.
b) Velocidad y aceleracin mximas de cualquier partcula del resorte.

Al tratarse de una onda armnica plana que viaja segn OX(-), la ecuacin de propagacin
se puede expresar de forma general como:

t x
A sen 2 0
T

para determinar la ecuacin correspondiente a nuestro caso concreto necesitamos conocer


los valores de A, T, , y 0 .

En el enunciado nos dan directamente la amplitud A = 003 m y la frecuencia = 25 Hz.


El periodo podemos obtenerlo fcilmente como T = 1/ = 004 s. Nos falta conocer la
longitud de onda y la fase inicial 0. Cmo podramos calcular el valor de cada una de
estas dos magnitudes?

La longitud de onda se puede conocer de forma inmediata si recordamos que la distancia


entre dos puntos consecutivos que se encuentren en concordancia de fase ha de ser preci-
samente igual a la longitud de onda, por tanto, = 02 m.

propagacin Foco

P 20 cm Q O X

En efecto, si calculamos la diferencia de fase entre dos puntos P y Q separados 20 cm en-


tre s, tal y como se expresa en la figura de arriba, obtenemos que:
Movimiento ondulatorio 340

t x t x 2
Q - P = 2 Q 0 - 2 P 0 Q - P = (x x P )
T T Q

Fijmonos que en este caso, el punto Q tendr una fase mayor que el P porque es alcanza-
do antes por la perturbacin. Si despejamos de la ecuacin anterior:

(x Q x P )
= 2 .
Q P

Si, como se nos dice, se trata de dos puntos consecutivos y en fase, la diferencia de fase
entre ambos deber de valer 2 rad. Adems, xQ - xP = 02 m (advirtase que tanto xP co-
mo xQ son negativos). Por tanto, sustituyendo en la expresin anterior nos queda que, tal y
como habamos previsto:
0'2
= 2 = 02 m
2

Para la determinacin de la fase inicial 0, debemos considerar que en el instante t = 01 s


el foco, que se encuentra en x = 0, presenta elongacin nula = 0 y rapidez positiva. Sus-
tituyendo estas caractersticas en la ecuacin de la onda tenemos:

0' 1
0 0'03 sen 2 0 0 sen (5 + 0) = 0
0' 04

Para calcular 0 quitaremos de la fase el nmero entero de veces 2 rad que exista, ya que
se tratar de ciclos completos y no influirn en el valor del sen, por tanto:

sen (5 + 0) = sen ( + 0) = 0 ( + 0) = arcsen 0

El arcsen 0 puede ser 0 rad (en cuyo caso 0 valdra - rad) o bien rad (en cuyo caso 0
valdra 0 rad). En cul de las dos situaciones nos encontramos en este ejercicio?

Como se nos dice que la rapidez del foco en ese instante es positiva y sabemos que el va-
lor de sta se puede evaluar en general como:

vF = dF/dt = Aw cos (wt + 0) = Aw cos ( + 0)

podemos ver que para que dicho valor sea positivo 0 tiene que ser - rad ya que si fuese 0
(la otra posibilidad), la rapidez del foco en ese instante (01 s) sera negativa (cos = -1).
As pues = 02 m y 0 = - rad. Aadiendo estos datos a los ya conocidos, podemos
expresar la ecuacin de este movimiento ondulatorio como:

t x t x
A sen 2 0 = 0'03 sen 2
T 0' 04 0'2
Movimiento ondulatorio 341

Cmo podemos calcular la rapidez y la aceleracin de cualquier partcula del resorte?

Si conocemos , resulta sencillo obtener v y a de cualquier partcula ya que no tenemos


sino que derivar respecto de t. En efecto:

2 t x
v = d/dt = 0'03 cos 2 vmax = 003 (2/004) = 471 m/s
0'04 0' 04 0'2

2 t
2
x
a = dv/dt = - 0'03 sen 2 amax = -003 (2/004) = -740 m/s
2 2
0'04 0' 04 0'2

18. Hallad la ecuacin de propagacin de un movimiento ondulatorio armnico de 2


mm de amplitud y 400 Hz de frecuencia, que se propaga a lo largo del OX (+) con una
velocidad de 840 m/s. Calculad tambin la velocidad con que vibra un punto que dista
5 m del foco, 001 s despus de comenzar la vibracin del foco.


sol: = 2sen 2 400t x / 2'1 ; v = -367 m/s

19. Determinad la ecuacin de propagacin de una onda esfrica en un medio en el


que la absorcin sea despreciable.

Hasta ahora hemos estado trabajando con la ecuacin de propagacin de una onda arm-
nica plana que se propaga en un medio sin absorcin. Esto nos ha permitido considerar en
todos los casos que la amplitud, que determina la energa con se propaga una onda de una
frecuencia dada, permaneca invariable.

Qu cabe esperar que ocurra en el caso de que la perturbacin tenga un frente de onda
esfrico?

Este caso se dar cuando al introducir la perturbacin en un punto dado de un medio, di-
cho medio sea homogneo e istropo, lo que har que la velocidad con que propaga la
onda sea la misma en todas direcciones. Conforme se propaga la perturbacin por el me-
dio, la misma cantidad de energa ha de repartirse cada vez entre ms puntos y, en conse-
cuencia, la energa que corresponder a cada punto del medio ser tambin cada vez me-
nor y, por tanto, su amplitud de vibracin tambin lo ser.

Cmo variar la amplitud con la distancia al foco?


Movimiento ondulatorio 342

Para responder a esta cuestin podemos razonar con-


siderando el frente de onda en un cierto instante t 1, R1
con un radio R1. Si designamos por m1 la suma de
todas las masas de las partculas que lo integran, la
energa de dicho frente ser: R2

1 1
E1 m1 w 2 A 12 E 1 m1 4 2 2 A 12
2 2

en donde A1 ser la amplitud de vibracin de cada uno de los puntos de ese frente.

Al cabo de cierto tiempo, en el instante t 2 , el frente se encontrar a una distancia R 2 del


foco abarcando a ms puntos del medio, de forma que su energa vendr dada ahora por:

1 1
E2 m 2 w 2 A 22 E 2 m 2 4 2 2 A 22
2 2

en donde A2 ser la amplitud de vibracin de cada uno de los puntos de ese frente y m2 la
suma de todas sus masas, pero como la energa del frente 2 ha de ser la misma que la que
haba en el frente 1 (admitiendo que no haya absorcin), tendremos que:

m1 A 22
E1 = E 2
m 2 A 12

Como m2 m1 la ecuacin anterior ya nos muestra que A2 A1, pero si queremos averi-
guar qu relacin concreta existe entre ambas amplitudes tendremos que hallar la rela-
cin existente entre dichas masas.

La densidad superficial, en cada una de las esferas vendr dada por:

m1 m2
1 y 2 , pero como el medio es homogneo ambas sern iguales y:
4R12 4R22

m1 m2 m1 R 12 R 12 A 22 R A R
2
= 2
luego 2
y sustituyendo: 2
2 1 2 A 2 A1 1
R1 R2 m2 R 2 R2 A1 R2 A1 R2

El resultado obtenido muestra cmo va disminuyendo la amplitud conforme el frente de


onda esfrico va avanzando (aumentando su radio). Esta disminucin de la amplitud, de-
bida a que la energa que transporta el frente de onda se reparte cada vez entre ms pun-
tos, recibe el nombre de atenuacin y no hay que confundirla con el fenmeno de absor-
cin que provoca tambin una disminucin de la amplitud pero que se debe a la existencia
de rozamiento y otros factores que podemos globalizar diciendo que el medio no es en
realidad perfectamente elstico y que aqu, para simplificar, hemos ignorado.
Movimiento ondulatorio 343

Si la amplitud no es constante sino que va disminuyendo, cmo habr que expresar la


ecuacin del movimiento ondulatorio para el caso que estamos analizando?

Una posibilidad es comparar la amplitud A a una cierta distancia R del centro del foco,
con la amplitud A0 a una distancia determinada del foco (que por comodidad tomaremos
igual a 1 m). En estas condiciones y teniendo en cuenta la expresin anterior, obtenemos:

1 A0
A A 0
R2 R

y sustituyendo en la ecuacin de propagacin:

A0 t x
sen 2 0 donde R ser la distancia entre el frente de onda y el foco.
R T

20. Un foco emite sonido en un medio homogneo e istropo (frente de onda esfrico)
con una potencia de 100 W. Cul es la intensidad de la onda a una distancia de 10 m
del foco? (Supngase que no hay absorcin).

El foco emite con una cierta potencia P que mide la energa que lanza al espacio por uni-
dad de tiempo. Si suponemos que no se producen prdidas en el medio, dicha energa
atravesar cualquier superficie cerrada imaginaria que contenga al foco que la emite. No
obstante, aunque dicha energa por unidad de tiempo sea siempre la misma, conforme nos
alejemos del foco se hallar cada vez ms repartida (al ir aumentando la superficie cerrada
a atravesar). Para cuantificar este fenmeno se introduce una nueva magnitud, que se de-
nomina intensidad del movimiento ondulatorio I en un punto, y que se define como una
magnitud cuyo valor coincide con la cantidad de energa que cada unidad de tiempo atra-
viesa una superficie unidad colocada en dicho punto perpendicularmente a la direccin de
propagacin de la onda.

En nuestro caso, al ser el medio homogneo e istropo, los frentes de onda sern esfricos.
Toda la energa que emita el foco por unidad de tiempo (o potencia) ser la misma que
atravesar cualquier superficie esfrica con centro en el foco (sea cual sea su radio) en la
unidad de tiempo. De acuerdo con lo anterior, la intensidad del sonido emitido por el foco
en un punto cualquiera situado en una superficie esfrica de radio R con centro en dicho
foco, se podr expresar como:

I = Pfoco /Sesfera = Pfoco /4R2

La expresin obtenida muestra que la intensidad disminuir con el cuadrado de la distan-


cia al foco (siempre en ausencia de absorcin). Para calcular el valor de la intensidad bas-
tar sustituir los datos del enunciado en la expresin anterior:

P 100 W
I 0' 08 2
4 R 2
4 102
m
Movimiento ondulatorio 344

21. Un foco sonoro emite con una potencia de 20 W. Calculad la intensidad en dos
puntos situados a 10 m y 20 m del foco. Cul ser la relacin entre las amplitudes?
(Supngase que no existe absorcin del medio).

sol: I1 = 0016 W/m2 ; I2 = 0004 W/m2; A1 =2 A2

22. Una onda sonora armnica que tiene una amplitud de 2 mm y una frecuencia de
400 Hz, se propaga a lo largo del eje X con una velocidad de 340 m/s, y se sabe que en
el punto x = 0 alcanza su mxima velocidad positiva en el instante t = 0. Su intensidad
en un punto P es de 27 W/m2. Determinad:

a) La elongacin y la velocidad de la vibracin del punto que dista 5 m del origen en


el instante t = 01 s.
b) Si se duplica la frecuencia dejando la amplitud constante, cules sern las nuevas
longitud de onda e intensidad?

Como se trata de una onda armnica plana que se propaga segn OX(+), las expresiones
generales de la elongacin y de la rapidez v de vibracin sern:

t x t x
A sen 2 0 y v Aw cos 2 0 respectivamente.
T T

Si queremos conocer estas ecuaciones para el caso que nos ocupa, tendremos que averi-
guar los valores que presentan la amplitud A, el periodo T, la longitud de onda , la fre-
cuencia angular w y la fase inicial 0. Del enunciado sabemos que:

A = 2 mm, = 400 Hz T = 1/ = 1/400 s, w = 2 = 800 rad/s,


v = 340 m/s = vT = 3401/400 = 085 m

Nos falta conocer el valor de la fase inicial. Cmo podramos determinar ste? Como el
valor de la fase inicial es una constante, bastar conocer la elongacin y la rapidez en un
punto cualquiera del medio, en un instante dado, para poder determinar 0 .

Sabemos que en el instante t = 0, el punto x = 0 tiene su rapidez mxima y positiva. En


dicho instante los valores de y de v en el punto considerado sern:

= Asen 0 y v = Aw cos 0 y como v = vmax = Aw cos 0 = 1, es decir 0 = 0

Ahora que ya conocemos las caractersticas necesarias del movimiento, podemos determi-
nar las ecuaciones de la elongacin y de la rapidez para cualquier punto del medio:

t x x
A sen 2 0 = 2 10 3 sen 2 400t
T 0' 85
t x x
' cos 2 400t
v Aw cos 2 0 = 503
T 0' 85
Movimiento ondulatorio 345

Para calcular la elongacin y la rapidez en el punto x = 5 m en el instante t = 01s, bastar


con sustituir en las ecuaciones anteriores estos valores de x y de t, de modo que:

5
= 2 10 3 sen 2 400 0'1 = 210 sen 2 (3412) = 13510 m
-3 -3
0' 85

5
v = 5'03 cos 2 400 0' 1 = 372 m/s
0' 85

Qu es lo que ocurre con la longitud de onda y con la intensidad de la onda si se duplica


la frecuencia?

La velocidad con que se propaga una onda depende de la naturaleza de dicha onda y del
medio en el que lo hace, por tanto el hecho de duplicar la frecuencia no va a alterar la
velocidad de propagacin, de modo que la longitud de onda ser:

= v/ = 340/800 = 0425 m, es decir, se reduce a la mitad.

En cuanto a la intensidad, si tenemos en cuenta que la energa de una partcula de masa m


que describe un movimiento vibratorio armnico simple es

1
E= m (2 ) 2 2 A 2
2

como en este caso aumenta la frecuencia al doble, cada una de las partculas del medio
dispondr del cudruple de energa que antes y la intensidad en el mismo punto tambin
ser el cudruple:

I = 4I = 427 = 108 W/m2.

23. El Sol posee una potencia emisiva de 271020 MW. Qu intensidad luminosa re-
cibimos en la Tierra que dista del Sol 15 10 11 m. (Considrese que no existe absor-
cin en ningn punto del medio).

sol: I = 955 W/m2

9-24. Un altavoz emite un sonido con una potencia


de 40 W. Determinad:

a) La intensidad a 10 metros del altavoz.


b) Nivel de intensidad que producir en el punto
anterior sabiendo que la intensidad umbral es de
10-12 W/m2.
Movimiento ondulatorio 346

Para resolver este ejercicio podemos considerar que el medio en el que se encuentra el
altavoz es homogneo e istropo, por lo que la propagacin se realizar por ondas esfri-
cas desde el altavoz o foco.

Para calcular el valor de la intensidad sonora en un punto situado a 10 m del altavoz bas-
tar con sustituir en la expresin correspondiente y operar:

I = Pfoco /4R2 = 40/4102 = 00318 W/m2

En general la potencia sonora de la mayor parte de los focos corrientes son extraordina-
riamente bajas. A ttulo de ejemplo, podemos decir que con la energa sonora emitida en
un concierto o por unas motos a escape libre durante una hora, apenas tendramos sufi-
ciente para preparar un par de tacitas de caf. Sin embargo ello no impide que, debido a la
extraordinaria sensibilidad del odo para captar vibraciones, la emisin de esa energa nos
afecte especialmente debido a la sensacin sonora que nos produce (incluso an estando
bastante alejados de la fuente).

A primera vista podra pensarse que la intensidad fsica con que nos llega un sonido es
directamente proporcional a la sensacin sonora que nos produce en el odo. Pero si esto
fuese as, dos focos sonoros idnticos (doble intensidad en cada punto que uno solo) ac-
tuando simultneamente deberan producirnos una sensacin fisiolgica doble que uno
solo y, sin embargo, no es eso lo que ocurre. En realidad, se comprueba experimentalmen-
te que la sensacin que nos produce un sonido vara con la intensidad pero no de forma
lineal sino logartmica. Por esta razn se define una nueva magnitud denominada nivel de
intensidad (S) que vara con el logaritmo decimal de la intensidad, tal y como se aprecia
en la expresin siguiente:
S-S0 = log I/I0

donde S representa el nivel de intensidad que nos produce un sonido de una intensidad I,
y S0 el que nos produce otro de intensidad I0 .

Si designamos por Iu la intensidad umbral o mnima intensidad sonora que es capaz de


detectar el odo humano (cuyo valor es de 10 -12 W/m2 para una frecuencia de 1000 Hz), la
expresin anterior se podr escribir como:

S = log I/Iu

ya que si Iu es la intensidad umbral, le corresponder un valor de S u = 0, lo cual no quiere


decir que no haya sonido sino que simplemente, no somos capaces de detectarlo con nues-
tro odo. En general a la intensidad umbral se le designa como I 0 y al nivel de intensidad
umbral como S0, pero con objeto de evitar confusiones (ved ejercicio 26) hemos preferido
designarlos respectivamente como Iu y Su.

La unidad de medida de S recibe el nombre de Bel y de acuerdo con la expresin anterior,


ser el nivel de intensidad provocado por una intensidad 10 veces superior a la umbral.
Movimiento ondulatorio 347

Habitualmente se utiliza un submltiplo del Bel (el decibel o decibelio, dB) de modo que
si queremos trabajar con sta ltima unidad hemos de modificar la expresin y escribir:

S = 10 log I/Iu

que aplicada a nuestro caso nos conduce a S = 10 log I/Iu = 10 log 00318/10-12 = 105 dB

Conviene tener en cuenta que la sensacin fisiolgica que produce un sonido depende,
adems de la intensidad, de su frecuencia. Tanto es as que para frecuencias inferiores a
20 Hz (infrasonidos) y superiores a 16000 Hz (ultrasonidos) sea cual sea su intensidad, no
son detectados por las personas. A ttulo de ejemplo, sealaremos que mientras que un
sonido de 1000 Hz comienza a percibirse a partir de un nivel de intensidad de 0 dB, otro
sonido de 100 Hz no podr ser apreciado hasta que su nivel de intensidad no llegue a los
30 dB.

Una cuestin interesante podra ser averiguar a qu distancia nos tendramos que alejar
de una fuente sonora para dejar de orla.

Teniendo en cuenta que I = Pfoco /4R2 y sustituyendo en la expresin de S, obtenemos:

S = 10 log I/Iu = 20 log Ru /R

En la ecuacin obtenida Ru es la distancia al foco para que la intensidad sea la umbral (es
decir S = 0) y por tanto, la distancia que hemos de hallar.
S
log Ru/R = S/20 R u R 10 20
105
Esta expresin resulta ser en nuestro caso Ru = 10 10 20 = 1.778.279 m = 1778279 km

Naturalmente, la distancia a la que dejamos de or el altavoz es muchsimo ms pequea


que la evaluada, ya que en su clculo no hemos tenido en cuenta el fenmeno de absor-
cin que inevitablemente se produce en el aire.

25. A qu distancia de un foco sonoro que emite con una potencia de 50 W debemos
situarnos para que nos produzca un nivel de intensidad de 80 dB? (Ignorar la absor-
cin que se produce en el medio de propagacin).

sol: A una distancia de 1995 m.

26. Una onda sonora plana se propaga en el aire, aprecindose que tras recorrer 1 km su
nivel de intensidad disminuye en 7 dB. Cul es el coeficiente de absorcin del aire?

En primer lugar conviene tener en cuenta que al tratarse de una onda plana, la disminu-
cin de intensidad (de amplitud) ser debida a la absorcin del medio (que se produce a
causa del rozamiento, viscosidad y otros factores que hacen que ste no sea perfectamente
elstico).
Movimiento ondulatorio 348

De qu magnitudes cabe esperar que dependa la amortiguacin que sufre una onda por
absorcin cuando atraviesa un medio dado?

Un factor que debe influir es el espesor x del material de que est hecho el medio que ten-
ga que atravesar, de forma que cuanto mayor sea x ms tendr que reducirse su intensidad.
Otro ser la naturaleza del material, a travs de un parmetro (distinto para cada sustan-
cia) que se denomina coeficiente de absorcin. Finalmente, tambin puede influir la inten-
sidad I0 con que llegue la onda, de modo que, en principio, podemos pensar que cuanto
mayor sea I0, a igualdad de los restantes factores, mayor tendr que ser la intensidad que
tendr la onda despus de recorrer una cierta distancia x en un medio dado.

Experimentalmente se comprueba que


cuando una onda plana atraviesa un Aire
medio de espesor x, su intensidad dis- I0 I
minuye exponencialmente en la forma:
x

I I0 e x
Expresin que recoge adecuadamente las hiptesis anteriores.

Consideraremos pues que la perturbacin tiene en un punto dado del medio (aire) una
intensidad I0 y que, tras recorrer 1 km en ese mismo medio, su intensidad pasar a ser I (7
dB menor que I0). Tomando logaritmos neperianos en la igualdad anterior obtenemos que:

1 I
ln I/I0 = -x y despejando = ln
x I0

Cmo conocer la relacin entre las intensidades?

Si recordamos que S = 10log I/Iu (Iu corresponde a la intensidad umbral) y como nos
indican en el enunciado que el nivel de intensidad disminuye en 7 dB, si llamamos I 1 a la
intensidad antes de recorrer 1 km en el aire e I2 a la intensidad despus de recorrer el cita-
do km, podremos hacer:

S1 = 10log I1/Iu y S2 = 10log I2/Iu y restando: S2 - S1 = 10 log I2/I1 de donde:

-7 = 10log I2/I1 y de aqui log I2/I1 = -07 de donde I2/I1 = 10-07 = 0200

y sustituyendo en la expresin del coeficiente de absorcin , obtenemos finalmente:

1
= 3
ln 0' 2 = 1610-3 m-1
10

27. El espesor de semiabsorcin de un cierto material es 30 cm. Qu espesor debe-


remos situar para que al ser atravesado por una onda plana armnica su intensidad
se reduzca de 08 W/m2 a 03 W/m2 ?
Movimiento ondulatorio 349

El espesor de semiabsorcin de una sustancia nos indica el espesor d de la misma que


es necesario para que al ser atravesado por una onda, la intensidad de esta se reduzca a la
mitad.

En este ejercicio se nos pide que calculemos el espesor de un cierto material que habr
que interponer frente a una onda para que su intensidad al atravesarlo se reduzca en una
cantidad dada.

Cul puede ser el inters de este problema?

Una aplicacin importante es, por ejemplo, la utilizacin de materiales adecuados para el
aislamiento acstico de viviendas, habitculos de vehculos, etc.

Sabemos que para una onda plana al atravesar un medio de espesor x, se produce una va-
riacin en su intensidad que viene dada por la expresin:

I = I0e-x

Sin embargo en ella no figura para nada el espesor de semiabsorcin d o espesor de ma-
terial que la onda ha de atravesar para que su intensidad se reduzca a la mitad

Cmo podramos relacionar con d?

Si en la expresin anterior hacemos I = I0/2 el valor de x coincidir con d, por tanto:

I0/2 = I0e-x y tomando logartmos neperianos: -ln 2 = -d d = 0693/

Como en este caso conocemos d, tendremos que: = 0693/d = 0693/03 = 231 m-1
y sustituyendo en la expresin que da la intensidad emergente:

I = I0e-x 03 = 08 e 3' 31x ln 0375 = -231x x = 042 m

As pues deberemos de utilizar un espesor de 42 cm del material dado en el enunciado


para conseguir reducir la intensidad de la onda de 08 W/m 2 a 03 W/m2.

28. Cul es el coeficiente de absorcin del agua del mar si un buceador comprueba
que a 10 m de profundidad la luminosidad ha decrecido al 13% de la existente en la
superficie?

sol: = 02 m-1

29. Si una onda atraviesa un espesor de pared de 10 cm, su intensidad se reduce de 6


pW/cm2 a 05 pW/cm2. Hallad el coeficiente de absorcin del material de que est
hecha la pared.

sol: = 0248 cm-1


Movimiento ondulatorio 350

30. Una onda penetra en un medio cuyo coeficiente de absorcin es 04 cm-1. Qu


espesor recorrer para que su intensidad se reduzca a la cuarta parte?

sol: x = 347 cm

31. Dos ondas planas que se propagan a 400 m/s pasan por los puntos P y Q de la figu-
ra en direccin a M. Si la perturbacin en cada uno de los puntos se puede represen-
tar por las funciones:
108 m M
P = 4 sen 200 t
Q 100 m
Q = 3 sen 200 t
P

( se expresa en cm cuando t se mide en s) Cul sera el valor de la perturbacin en


el punto M? Y si Q estuviese a 102 m de l?

Para resolver el problema, antes de comenzar a operar conviene detenerse en analizar el


tipo de perturbaciones que se estn propagando y ver cules son sus caractersticas.

La perturbacin dada por: P = 4 sen (200 t) tiene una amplitud de 4 cm, su frecuencia es de
100 Hz, el periodo 001 s, la longitud de onda es de 4m y se propaga con una rapidez de 400 m/s.

En cuanto a: Q = 3 sen 200 t, corresponde a una onda de 3 cm de amplitud y el resto de las


caractersticas iguales a la anterior. Las perturbaciones en P y en Q se encuentran, pues, en fase.

Cul sera el valor de la perturbacin que existira en el punto M si cada una de las on-
das avanzara sola?

Para averiguarlo tendremos que utilizar la ecuacin general que nos da el valor de la per-
turbacin en cualquier punto del medio por el que se propaga, de este modo obtenemos:

2 d P 2 d Q
MP 4 sen 200 t y MQ 3 sen 200 t

Como son las dos ondas las que se propagan, la perturbacin en el punto M, de acuerdo
con el Principio de Superposicin, ser la suma algebraica de ambas perturbaciones:

2 d P 2 d Q
M = MP + MQ = 4 sen 200 t + 3 sen 200 t

En el punto M se producir, pues una interferencia de las dos ondas. Podemos preguntar-
nos de qu depender que dicha interferencia sea constructiva (se refuercen los efectos de
ambas perturbaciones) o destructiva (se opongan). A ttulo de hiptesis cabe pensar que,
como la longitud de onda es la misma, si la diferencia entre las distancias d P y dQ desde
cada foco al punto M es un mltiplo entero de dicha longitud de onda, se producir una
concordancia de fase y los efectos se reforzaran al mximo.
Movimiento ondulatorio 351

Para obtener M necesitamos sumar las expresiones anteriores. Para ello vamos a recurrir
al mtodo vectorial, pero antes, para mayor comodidad, expresemos las funciones como:

M = MP + MQ = A P sen wt P + A Q sen wt Q

En donde = 2d/. Toda funcin del tipo A sen (wt - ) se Y


puede representar en unos ejes de coordenadas XY mediante A
un vector de mdulo A, que gira con movimiento circular Asen(wt-)
wt-
uniforme en sentido contrario a las agujas del reloj, de ma-
nera que en cada instante forma con el eje X un ngulo que X
vale (wt - ) y su proyeccin sobre el eje Y nos proporciona
el valor de la funcin sinusoidal que representa.

En nuestro caso, si representamos las dos funciones en un mismo sistema de coordenadas


podemos comprobar que si sumamos los dos vectores, al proyectar el vector suma sobre el
eje Y, nos da en todo instante la suma de ambas funciones.

Asen(wt-M)

Asen(wt-P)

Asen(wt-Q)

wt-Q

wt-M

wt-P

La perturbacin en M ser pues: M = AM sen (wt - M).

Cmo podemos conocer AM y M ?


Aplicando el teorema del coseno en la figura anterior sabemos que:

A 2M A 2P A Q2 2A P A Qcos ( Q P ) .

Por otra parte, de la misma figura se obtiene tambin que:

A P sen(wt - P ) A Q sen(wt - Q )
tg( wt M ) y particularizando para t = 0, teniendo
A P cos(wt - P ) A Q cos(wt - Q )
Movimiento ondulatorio 352

en cuenta que tg (-) = -tg , que sen (-) = -sen y que cos (-) = cos , nos queda que:

A P sen P A Q sen Q
tg M de donde podemos calcular fcilmente M
A P cos P A Q cos Q

Analizando los resultados obtenidos vemos que el punto M describir un movimiento


armnico simple de igual frecuencia que las perturbaciones que llegan al mismo, pero con
una amplitud de vibracin que depender de la diferencia de distancias a que dicho punto
se encuentra de los focos ( = 2d/). As, podemos ver que hay dos casos extremos:

a) Cuando cos (Q - P) = 1 AM = AP + AQ (interferencia constructiva)


b) Cuando cos (Q - P) = -1 AM = AP - AQ (interferencia destructiva).

Qu ocurrir con la diferencia de distancias en cada uno de los casos anteriores?

2
a) cos (Q - P) = 1 cos (d Q d P ) = 1, lo que significa que: dQ - dP = n

2
b) cos (Q - P) = -1 cos (d Q d P ) = -1, lo que significa que: dQ - dP = (2n+1)/2

Es decir: La mxima amplitud AM = AP + AQ se dar cuando la diferencia de distancias


sea un nmero entero de veces la longitud de onda, y la mnima amplitud AM = AP - AQ se
dar cuando esta diferencia sea un nmero impar de semilongitudes de onda.

Concretando el resultado anterior para el caso que nos ocupa (d P =100 m y dQ =108 m)
tendremos que:

2
A 2M A 2P A Q2 2A PA Qcos ( Q P ) = A 2M A 2P A 2Q 2A P A Q cos 8
4
de donde: AM = AP + AQ = 4 + 3 = 7 cm.

A P sen P A Q sen Q 4sen50 3sen54


Por otra parte tg = tg = 0 = 0.
A P cos P A Q cos Q 4cos50 3cos54

As pues, la perturbacin en el punto M ser: M = 7 sen 200 t y la interferencia produ-


cida ser constructiva.

Qu hubiera ocurrido si Q hubiese estado a 102 m del foco?

Utilizando las mismas ecuaciones que anteriormente, se llega a la conclusin que:

AM = AP - AQ = 4 - 3 = 1 cm, que = 0 y que M = 1 sen 200 t, siendo la interferencia


destructiva. En estas condiciones si las dos ondas que concurriesen en P tuviesen la misma
amplitud ocurrira que en M se producira una zona de sombra, no aprecindose all nin-
guna perturbacin.
Movimiento ondulatorio 353

32. Sean dos ondas planas que viajan a lo largo del eje X, una segn OX(+) y la otra
segn OX(-). Suponiendo que tienen ambas la misma amplitud y frecuencia y que
cuando alcanzan el punto x = 0 se encuentran en fase, determinad el valor de la per-
turbacin en cualquier punto del eje.

En este ejercicio se plantea un fenmeno de interferencia particularmente interesante.

En el esquema adjunto estn representadas las condiciones expresadas en el enunciado.

10 = Asenwt
P

O X
20 = Asenwt

Como en el punto x = 0 ambas ondas estn en fase, la perturbacin correspondiente a cada


una en ese punto ser la misma, es decir: 10 = 20 = Asenwt. En un punto P cualquiera
del eje, las perturbaciones valdrn:

Para la onda que viaja segn OX (+): 1P = Asen w (t - t) ya que cuando la onda 1 lle-
gue al punto P habrn transcurrido t s desde que pas por el punto x = 0, de modo que la
fase del punto P (si slo actuase esta onda) sera siempre menor, en un instante dado, que
la del punto x = 0.

Para la onda que viaja segn OX (-): 2P = Asen w (t + t) ya que cuando la onda 2 llegue
al punto x = 0, como viaja a la misma velocidad que la 1, habrn transcurrido t s desde
que pas por el punto P, por tanto, el punto P en este caso, estar ms tiempo oscilando de
lo que est el punto x = 0 (concretamente t s ms) y su fase (si slo actuase esta onda)
sera mayor que la del punto x = 0.

Teniendo en cuenta que t = x/v, la perturbacin resultante en el punto P ser pues:

x x
P = 1P + 2P = A sen w t sen w t de donde:
v v

wt wx/v wt wx/v wt wx/v wt wx/v


P = A 2sen cos y simplificando:
2 2

P = (2Acos wx/v) sen (wt) P = A sen (wt), siendo A = 2Acos (wx/v).

Analizando el resultado obtenido vemos que debido a la interferencia de las dos ondas:

a) Todos los puntos se encuentran en fase (wt es igual para todos en cada instante).

b) Cada punto realiza el mismo movimiento armnico simple pero con distinta amplitud de-
pendiendo de su posicin, la frecuencia y de la velocidad con que se propaga la perturbacin.
Movimiento ondulatorio 354

c) El efecto visual es una serie de


nodos (A = 0) y vientres (A = 2A)
que, a diferencia de una onda viaje-
ra, da una sensacin esttica y por
eso a este fenmeno de interferen-
cia se le denomina onda estaciona-
ria. Los nodos corresponden a aque-
llos puntos en los que las ondas
llegan en todo instante en oposicin
de fase y, por tanto, no vibran,
mientras que los vientres corres-
ponden a los puntos en los que lle-
gan en concordancia de fase.

Si el periodo de vibracin es pequeo, el


ojo superpone las imgenes anteriores y se
aprecia una figura como la siguiente:

Cul ser la posicin de aquellos puntos en los que A valga 0?

La amplitud A valdr 0 cuando cos (wx/v) = 0 es decir, cuando wx/v = (2n+1)/2 rad (n =
0, 1, 2, ...). Como v = /T y w = 2/T ello implica que x = (2n+1) /4. A cada uno de
estos puntos en los que en cualquier instante la amplitud es 0 se le denomina nodo. El
primer nodo (n=0) se encuentra en x=/4, el segundo (n=1) en x = 3/4, el tercero (n=2)
en x = 5/4 y as sucesivamente, de modo que la distancia entre dos nodos consecutivos
cualesquiera ser /2.

Cul ser la posicin de aquellos puntos en los que A vare entre -2A y +2A?

La amplitud A valdr 2A cuando cos (wx/v) = 1 es decir cuando wx/v = n rad


donde n ser un nmero entero cualquiera. Siguiendo un razonamiento anlogo al del
prrafo anterior, se concluye que la posicin de los puntos buscados ser x = n/2 y a que
la distancia mnima entre ellos ser de /2.
En el resto de los puntos, A variar entre dos extremos los cuales sern (en valor absolu-
to) mayores que 0 y menores que 2A.
Este tipo de ondas se dan en medios limitados, como por ejemplo en una cuerda de guita-
rra en la que las vibraciones se reflejan en los dos extremos de modo que se originan dos
ondas movindose en sentido contrario. En general, las ondas estacionarias son de gran
importancia en msica y tambin en otros campos como, por ejemplo, la arquitectura ya
que los edificios altos, puentes colgantes, etc., cuando vibran (viento, temblores de tierra,
etc.) pueden originarlas, por lo que es necesario tener esto en cuenta en su diseo, para
que puedan soportarlas y no derrumbarse.
Movimiento ondulatorio 355

33. Cmo se pueden obtener ondas estacionarias en una cuerda por reflexin?

Ya hemos visto que mediante dos perturbaciones de igual frecuencia y amplitud que via-
jan en sentido contrario a lo largo de una misma direccin (por ejemplo una cuerda), se
puede obtener una onda estacionaria.

Analicemos cmo podemos conseguir una onda estacionaria con una sola onda que inter-
fiera con ella misma tras sufrir una reflexin. Para ello seguiremos con el ejemplo de la
cuerda. Como la reflexin se puede producir en un extremo fijo o libre de la cuerda, con-
sideremos las condiciones que se debern dar en cada caso para que se produzca la onda
estacionaria:

a) Si el extremo de la cuerda se fija (por ejemplo sujetndolo a un tabique), la onda al


llegar a dicho extremo se refleja cambiando su fase en rad tal y como se puede compro-
bar experimentalmente si hacemos avanzar un pulso de onda por una cuerda en esas con-
diciones.

En este caso ser necesario que la longitud de la cuerda sea un nmero impar de veces
/4, para que la onda reflejada cumpla la condicin de estar en fase con la onda que esta-
mos introduciendo al alcanzar los puntos S, Q y O (y en oposicin en los T, R y P).

b) Si el extremo de la cuerda est libre (por ejemplo mediante una anilla que puede desli-
zar a lo largo de un eje vertical como el que se muestra en la figura), ser necesario que la
longitud de la cuerda sea un nmero par de veces /4, para que la onda reflejada (que no
cambia de fase al reflejarse) cumpla la condicin de estar en fase con la onda que estamos
introduciendo, en los puntos U, S, Q y O.
Movimiento ondulatorio 356

Tambin se pueden obtener ondas estacionarias fijando la cuerda por los dos extremos e
introduciendo una perturbacin (pulsando la cuerda) tal y como sucede con las cuerdas de
una guitarra. En este caso la perturbacin se reflejar en ambos extremos y se producirn
ondas estacionarias siempre que se cumpla que L = n/2 ( n = 1, 2, 3 ...):

/2

3/2

Conviene darse cuenta que en un medio limitado como el descrito, no se pueden producir
ondas estacionarias de cualquier longitud de onda sino slo aquellas que cumplan la con-
dicin de que:
= 2L/n

Podemos decir que en este caso las ondas estn cuantizadas. Esto contrasta con lo que
ocurre en medios no limitados, en los cuales pueden darse ondas estacionarias de cual-
quier longitud de onda. La importancia de este hecho es fundamental en la interpretacin
del comportamiento de partculas subatmicas (como el caso de los electrones en un
tomo).

34. Al pulsar una cuerda de 2 m de longitud sujeta por ambos extremos, se observa
que vibra apareciendo 9 nodos. Si la amplitud mxima es de 4 cm y la velocidad de
propagacin de la onda es de 6 m/s, obtened la ecuacin que representa la onda esta-
cionaria producida.

La ecuacin que buscamos tendr la forma: Y = (2Acos wx/v) sen wt o lo que es equiva-
lente Y = 2A cos (2x/)sen (2t/T), en la que 2A = 4 cm ("A" corresponde a la amplitud
de una de las ondas que por interferencia originan la onda estacionaria). Para obtener la
ecuacin correspondiente a nuestro caso concreto, nos falta pues conocer la longitud de
onda y el periodo T.
Movimiento ondulatorio 357

Si nos fijamos en la figura anterior podemos apreciar que existe una relacin entre la longitud
de la cuerda L, el nmero de nodos n y la longitud de la onda que puede darse. Dicha rela-
cin ha de ser tal que L sea un mltiplo entero de /2 es decir, un mltiplo entero del nmero
de vientres y como el nmero de vientres es el de nodos menos 1, podemos escribir que: L =
(n de nodos - 1) /2, de modo que sustituyendo los valores correspondientes:

2 = 8/2 y despejando, = 05 m.

En cuanto al periodo T, si recordamos la relacin v = /T a partir de la misma obtenemos:

T = /v = 05/6 = 1/12 s.

Sustituyendo en la expresin general:

Y = 2A cos (2x/)sen (2t/T) Y = 4 cos 4x sen 24 t

(en la que Y va en cm para x en m y t en s).

35. Una cuerda sujeta por ambos extremos, vibra de acuerdo con la ecuacin
Y = 2 cos (x/3) sen 50 t (x e Y se expresan en cm cuando t se mide en s). Se pide:
a) Amplitud y velocidad de las ondas que por interferencia dan lugar a la onda estacionaria.
b) Rapidez del punto de la cuerda situado en x = 10 cm en los instantes 135 s y 15 s.

sol: a) A = 1 cm; v = 150 cm/s. b) v = 0 y v = -1539 cm/s respectivamente.

36. Un submarinista ve pasar un pez que se aleja de l. Al cabo de unos segundos y


estando el submarinista a 10 m de profundidad, mira hacia la superficie y, de pronto,
ve aparecer la imagen del mismo pez que parece estar volando fuera del agua.

Determinad la distancia horizontal que separa a ambos sabiendo que en ese momento
el pez se encuentra a 7 m de profundidad.
Movimiento ondulatorio 358

Los rayos que partan del pez hacia la superficie, por pasar de un medio de mayor ndice de
refraccin a otro en el que dicho ndice es menor, se separan de la normal. Si va aumen-
tando el ngulo de incidencia el rayo refractado, de acuerdo con las leyes de la refraccin,
cada vez se separa ms de la normal. En consecuencia existe un ngulo, llamado ngulo
lmite, al que le corresponde un ngulo de refraccin de 90. A partir de ese valor se pro-
duce una reflexin.

Aire
90

L Agua

buceador pez

Este ngulo lmite para el caso del agua y el aire ser:

' sen L 1 sen 90 L 48'75


n1 sen i n2 sen r 133

La imagen del pez comenzar pues a ser apreciada por reflexin por el buceador cuando la
situacin sea la de la figura siguiente:

Aire
Q P R

7m
10 m 4875
4875
4875
Agua

La distancia horizontal entre submarinista y pez coincide en el instante especificado ante-


riormente con la longitud de QR Cmo podramos calcularla?

Analizando la figura anterior podemos darnos cuenta de que:

QR = QP + PR. Por otra parte tg 4875 = QP/10 QP = 114 m.

Adems tambin se cumple que tg 4875 = PR/7 PR = 8 m, de modo que sumando las
dos distancias obtenemos: QR = 114 + 8 = 194 m.
Movimiento ondulatorio 359

37. Determinad cul ser la mxima profundidad que puede tener una piscina com-
pletamente llena de agua, para que una persona sentada a 33 m del borde y cuya
vista queda a una altura de 12 m sobre el suelo, pueda ver un objeto que se encuen-
tra en el fondo y al centro de la piscina. (Anchura de la piscina 15 m. ndice de re-
fraccin del agua 133)

El objeto situado en el fondo refleja la luz en todas direcciones y esto podemos represen-
tarlo mediante rayos o lneas rectas que parten del mismo y solo si alguno de dichos rayos
alcanza al ojo de la persona, sta podr ver el objeto. Por otra parte, como los rayos parten
de un objeto sumergido en el agua, experimentarn una refraccin desvindose de la nor-
mal, por lo que este hecho alterar las condiciones necesarias para la observacin del ob-
jeto desde la superficie. (De hecho, el propio suelo de la piscina acta como un objeto y
como el ndice de refraccin del aire es menor que el del agua, el fenmeno de la refrac-
cin hace que nos parezca que ste se encuentra a menor profundidad de lo que realmente
est, con el consiguiente peligro de confusin en personas que no saben nadar).

En este problema se nos pide la mxima profundidad que podra tener la piscina para que
pudiera verse un objeto situado en su fondo, en determinadas condiciones. Conviene que
analicemos el problema y reflexionemos en primer lugar sobre lo que ocurrira en una
situacin ms sencilla que la planteada como sera el caso de que la piscina se encontrase
totalmente vaca.

h
En el caso propuesto los rayos que salen del
d
objeto no se desviaran. Podemos representar P
la situacin mediante la figura adjunta en la
Q H
que se observa que desde la posicin P hasta
la R se vera el objeto, pero en la posicin S R
ya no se vera. D
S

Cabe pensar que cuanto mayor sea la altura h desde la que se observa, mayor ser la pro-
fundidad mxima H a la que sera posible ver el objeto (y que si h fuese 0 la profundidad
H tambin lo sera). Por otra parte, cuanto mayor fuese la distancia d menor sera H, ya
que al alejarnos del borde el rayo debera inclinarse ms para llegar a nuestros ojos, de
Movimiento ondulatorio 360

modo que para cuando d tienda a infinito H tender a 0 y viceversa. Finalmente, cuanto
mayor sea la distancia D del objeto a la pared de la piscina, mayor podr ser H.

As pues: H = H (h, D, d), que son datos presentes en el enunciado del problema.

Cmo podramos hallar H?

Si nos fijamos en la figura anterior podemos darnos cuenta que los dos tringulos que se
forman son semejantes y aplicar las relaciones de proporcionalidad entre sus lados para
obtener la H buscada.

En efecto, la semejanza de tringulos nos permite escribir que:

H h h D 1'2 7 '5
de modo que despejando H obtenemos que: H = = 273 m
D d d 3' 3

El resultado obtenido no solo es dimensionalmente homogneo (L en ambos lados de la


igualdad) sino que adems contempla todas las hiptesis enunciadas anteriormente.

Podemos ahora ir ms all y plantearnos qu es lo que ocurrir cuando la piscina se llene


completamente de agua.

Ya hemos visto al comienzo que los rayos al pasar del agua al aire se refractan alejndose
de la normal, por lo que podrn llegar al ojo del observador rayos desde puntos situados a
mayor profundidad (como, por ejemplo el punto S de la figura anterior). No obstante,
tambin aqu habr una profundidad mxima HH a partir de la cual ya no se ver el ob-
jeto (que es precisamente la que nos piden en el problema).

Dicha profundidad mxima depender de los N


r
mismos factores h, d y D que antes, pero h
adems influir la diferencia en los ndices de d
refraccin entre los medios 1 (agua) y 2 (aire),
de modo que, de acuerdo con la ley de la re- i
fraccin ( n1 sen i n2 sen r ), cuanto mayor H
Q
sea n1 y menor sea n2, mayor ser el ngulo
r de refraccin y, por tanto la profundidad
mxima a la que se podr ver el objeto au-
mentar. Naturalmente, si n1 y n2 fuesen igua- P D
les, H se debera de hacer igual a H.

Cmo podramos obtener la profundidad mxima H?

Para determinar H bastara con conocer tg i ya que, como se puede ver en la figura:

D
tg i
H'
Movimiento ondulatorio 361

De acuerdo con la ley de la refraccin:

n2
sen i sen r
n1
n2 sen r
de modo que dividiendo por cos i tenemos: tg i .
n1 cos i
sen r
Por tanto, si pudisemos calcular tendramos resuelto el problema.
cos i
d H'
De la figura anterior sen r cos ; cos
i
d 2 h2 H'2 D2

n2 d H'2 D2
De modo que: tg i
n1 d 2 h2 H'

D
de donde podemos obtener finalmente: H ' h 2 n12 d 2 (n12 n22 )
n2 d
Y sustituyendo los datos: H = 75 m

Si analizamos el resultado literal que acabamos de obtener vemos en primer lugar que,
efectivamente, tal y como habamos supuesto, la profundidad es ahora mayor que la obte-
nida para la piscina sin agua. Por otra parte contempla todas las hiptesis, incluyendo la
de que cuanto mayor fuese n1 y menor n2, ms grande sera la profundidad permitida. In-
cluso, podemos ver que para el caso particular de que n 1 = n2 (por ejemplo cuando se vac-
a la piscina), el resultado se convierte en el anterior, es decir, H = H.
Otro aspecto interesante es cul sera la profundidad aparente que una persona dira que
tiene una piscina al observar su fondo desde una cierta distancia de la orilla.

Analizando la ltima figura, podemos ver que dicha profundidad coincidira con la pro-
fundidad H calculada cuando hemos supuesto que la piscina estaba vaca, que es de donde
parece provenir el rayo refractado, de modo que:

D h h n1 d (n1 n2 ) h 2 n12 d 2 (n12 n22 )


2 2 2 2 2

H' H
d n2 h n2 h

y despejando obtenemos finalmente:

n2 h
H H '
h 2 n12 d 2 (n12 n22 )

As el fondo de una piscina de 2 m de profundidad, visto desde la posicin indicada, pare-


cera estar a 073 m y resultar un serio peligro para quienes adems de no conocer el
fenmeno de la refraccin tampoco supieran nadar.
Movimiento ondulatorio 362
10. NATURALEZA DE LA LUZ: DUALIDAD ONDA CORPSCULO

1. Al experimentar con una clula fotoelctrica como la de la figura, se obtienen las


grficas de la derecha: interpretadlas.

Consideremos la grfica 1 en la que est representada la intensidad de la corriente elctrica,


medida por el microampermetro, que se produce cuando sobre la placa A de la clula (co-
nectada al polo negativo de un generador) se hace incidir radiacin de una cierta frecuencia.
La energa radiante que incide sobre la placa negativa libera electrones de la misma. Dichos
electrones son atrados por la placa positiva (B) situada enfrente, lo que permite que se pue-
da cerrar el circuito y producirse una corriente elctrica.

Si hacemos que aumente la diferencia de potencial entre las dos placas VBA (siendo VBA =
VB -VA), se produce tambin un aumento del nmero de electrones que habiendo sido libe-
rados de la placa A consiguen llegar a la placa B (hemos de tener en cuenta que no todos los
electrones que saltan de A lo hacen en direccin a B). Sin embargo, a partir de un cierto
valor, por mucho que aumentemos VBA , la intensidad de corriente ya no sigue aumentando.
En esta situacin la intensidad del campo elctrico existente entre las placas es tan grande
que todos los electrones arrancados de A llegan a B, haciendo que la intensidad de la co-
rriente alcance un valor mximo (corriente de saturacin) ya que no pueden llegar a B ms
electrones de los que se liberan en A debido a la radiacin incidente (la cual no se ha modi-
ficado).

Si invertimos la polaridad del generador la placa A ser positiva y la B negativa de forma


que la diferencia de potencial VBA = VB-VA ser ahora un nmero negativo. No obstante, en
la placa A sigue habiendo electrones y si la iluminamos podemos conseguir arrancar algu-
nos de ellos. En estas condiciones el campo elctrico existente entre las placas hace que
sobre los electrones (cargas negativas) acte una fuerza que se opone a su desplazamiento
de A hacia B, ya que dichos electrones son atrados por A y repelidos por B, de modo que
nicamente aquellos que sean emitidos con la suficiente energa cintica (y en una direccin
adecuada), pueden conseguir llegar a B venciendo el efecto del campo. Si seguimos dismi-
nuyendo VBA, es decir, aumentando la fuerza de frenado sobre los electrones, llegaremos a
Naturaleza de la luz 364

un valor negativo de la diferencia de potencial (-V0) a partir del cual ningn electrn alcan-
zar la placa B de modo que la intensidad de la corriente ser nula.

Al valor absoluto V0 de la citada diferencia de potencial, se le denomina potencial de fre-


nado o de detencin. (Tambin suele representarse como Vf).

Podemos considerar que para diferencias de potencial


mayores que -V0 los electrones que llegan a la placa
B lo hacen con una cierta energa cintica, pero que
conforme nos vamos acercando a -V0 dicha energa
cintica disminuye hasta anularse justo cuando se
alcanza VBA = -V0.

El valor del potencial de frenado se puede medir fcilmente con un voltmetro y nos permite
calcular la energa cintica mxima con que estn siendo arrancados los electrones por la
radiacin incidente. En efecto, basta aplicar la conocida relacin entre el trabajo resultante y
la variacin de energa cintica a uno de los electrones, con lo que:

Wres = Ec WFe Ec y como la fuerza electrosttica es conservativa, WFe Ep , de


modo que si consideramos el desplazamiento de A a B e igualamos las expresiones anterio-
res nos queda que:

Ep BA = Ec BA es decir: (Ep B Ep A ) Ec A ya que, en nuestro caso, EcB = 0.

Si ahora expresamos la variacin de energa potencial electrosttica en funcin de la dife-


rencia de potencial entre las placas (Ep = qV) nos queda:

q e (VB VA ) Ec A siendo qe la carga del electrn (negativa) y VB - VA la diferencia de


potencial entre las placas (tambin negativa e igual a -V0). La expresin anterior se suele
escribir como:
Ec qe V0

En la que ambas magnitudes sern positivas.

Si manteniendo el mismo tipo de radiacin (misma frecuencia) aumentamos la intensidad de


la misma en un cierto valor, obtenemos la grfica 2 y si repetimos el proceso la 3 (y as su-
cesivamente), comprobando que el nico efecto que conseguimos es aumentar el nmero de
fotoelectrones o electrones liberados (lo que se traduce en un aumento de la intensidad de
corriente), pero que, sorprendentemente, la energa cintica con que surgen de la placa sigue
siendo la misma.

El resultado anterior fue realmente sorprendente para los cientficos de principios del siglo
XX, porque contradice la teora clsica (ondulatoria) de la radiacin ya que, de acuerdo con
ella, al aumentar la intensidad de la luz incidente se aumenta la energa que transporta el
frente de onda uniformemente distribuida por el mismo, de modo que al incidir sobre la pla-
ca metlica los electrones deberan de adquirir ms energa cintica. En 1905 Einstein pu-
blic una serie de trabajos por los que consigui el premio Nobel y en uno de los artculos
Naturaleza de la luz 365

daba la explicacin de este problema, admitiendo que la radiacin luminosa era una distri-
bucin de cuantos de luz (fotones) cada uno de los cuales posea una energa proporcional
a la frecuencia de dicha radiacin E = h.

2. En las experiencias sobre el efecto fotoelctrico se obtiene que:

a) Para un metal dado, el efecto slo se presenta a partir de una cierta frecuencia um-
bral (0) de manera que si la luz utilizada es de una frecuencia menor, por muy alta
que sea su intensidad no se detecta el paso de ninguna corriente.

b) Si la frecuencia de la radiacin es mayor que la umbral, la intensidad de la corriente


elctrica que se produce aumenta conforme aumentamos la intensidad luminosa de la
radiacin utilizada (manteniendo la frecuencia constante), pero no ocurre as con la
energa cintica de los electrones liberados que se mantiene constante.

c) A partir del valor umbral de la frecuencia, la energa cintica de los electrones libe-
rados crece linealmente con la frecuencia.

Explicad los resultados anteriores, mediante la naturaleza discontinua de la radiacin


electromagntica propuesta por Einstein.

La luz y, en general cualquier radiacin electromagntica, est formada por fotones. Cada
uno de estos fotones posee una energa dada por E = h (h es una constante llamada cons-
tante de Planck y cuyo valor es 6610-34 Js), de modo que el efecto fotoelctrico se produce
al chocar un fotn con un electrn, siempre que la energa del fotn sea superior a la energa
mnima que se necesita comunicar al electrn para arrancarlo del metal (venciendo la atrac-
cin de los iones positivos). Dicha energa mnima recibe el nombre de trabajo de extrac-
cin (Wextr). En consecuencia, habr una frecuencia mnima 0 (frecuencia umbral) a partir
de la cual se conseguir extraer electrones del metal y tal que E 0 = h0 = Wextr.

Las consideraciones anteriores permiten comprender el hecho de que por mucho que au-
mentemos la intensidad luminosa de una radiacin de cierta frecuencia (lo que significa au-
mentar el nmero de fotones incidentes), si su frecuencia es inferior a la umbral, (0) el
efecto fotoelctrico no se presente en ningn caso, puesto que la energa que los fotones
pueden comunicar a los electrones ser inferior siempre a la energa mnima necesaria para
arrancarlos venciendo las fuerzas elctricas de atraccin de los restos atmicos positivos.

Tambin explica que la energa cintica con que surgen los electrones (para radiaciones que
tengan frecuencias superiores a la umbral) vare linealmente con la frecuencia puesto que la
energa de un fotn que incide sobre un electrn se destinar en parte a extraerlo del metal
(h0) y el resto se comunicar al mismo como energa cintica. Este tipo de razonamientos
condujeron a Einstein a establecer que la energa cintica con que surgen los electrones en el
efecto fotoelctrico viene dada por:

Ec = E - E0 Ec = h( - 0)

En la expresin anterior h representa la energa del fotn incidente y h 0 la energa


mnima necesaria para arrancar el electrn (o trabajo de extraccin). Conviene darse cuenta
Naturaleza de la luz 366

de que en ella se halla implcito el resultado que se enuncia en el apartado c) segn el cual la
energa cintica crece linealmente con la frecuencia (a partir de 0 ) y que el valor de la
pendiente de la recta ser precisamente el de la constante h.

1
La ecuacin se suele escribir como: h = h0 + me v 2
2

donde mev2/2 es la energa cintica con que el electrn sale de la placa, que podemos ex-
presar tambin como:
h = h0 + qe V0

ya que, como ya hemos visto en el primer ejercicio, la energa cintica con que salen los
electrones se puede medir conociendo el potencial de frenado V 0 correspondiente.

3. El potencial de frenado que elimina el efecto fotoelctrico en funcin de la frecuencia


() de la luz incidente est dado (en valor absoluto) por una de las grficas siguientes:

Determinad cul de ellas es la correcta, justificando la respuesta.

Si analizamos las grficas anteriores podemos darnos cuenta en primer lugar de que tanto la
primera como la ltima de ellas no son posibles, porque en ambas la frecuencia de la radia-
cin incidente aparece limitada, cuando est claro que se trata de la variable independiente y
que puede tomar, en principio, cualquier valor entre 0 e infinito (ya que podemos utilizar
radiaciones de la frecuencia que queramos).

En cuanto a la tercera grfica (c), vemos que para una frecuencia nula, es decir, sin iluminar,
se producira el efecto fotoelctrico ya que existe un potencial de frenado distinto de 0 (y
por tanto una energa cintica inicial con que salen electrones), lo cual es absurdo puesto
que si no hacemos incidir ninguna radiacin los electrones no pueden adquirir la energa
necesaria para vencer las fuerzas de atraccin de los iones positivos del metal.

Finalmente, la segunda grfica (b) nos indica que existe una frecuencia 0 por debajo de la
cual el potencial de frenado vale 0, es decir, no se arrancan electrones y a partir de dicho
valor 0 el potencial de frenado crece linealmente con la frecuencia. Este resultado es co-
herente con la ecuacin de Einstein h = h0 + qe V0 (ved ejercicio anterior), ya que si
despejamos V0 obtenemos:
h
V0 = ( 0 )
qe
Naturaleza de la luz 367

expresin que representada grficamente nos dara, precisamente, una recta como la de la
grfica b, cuya pendiente debera ser siempre la misma independientemente del metal utili-
zado en la experiencia.

De hecho, la determinacin experimental de la pendiente de esta grfica (conocida la carga


del electrn) permite obtener el valor de la constante h. Millikan durante la segunda dcada
del siglo XX obtuvo de esta forma el valor dicha constante y comprob que era concordante
con el obtenido por el fsico alemn Max Planck 15 aos antes por mtodos muy diferentes,
lo que constituy un importante apoyo a la hiptesis corpuscular de Einstein sobre la natura-
leza de la luz.

4. Calculad la energa cintica de los electrones liberados por un metal que forma par-
te de una clula fotoelctrica si el potencial de frenado es de 5 V. ( Carga del electrn
1610-19 C).

sol: Ec = 5 eV = 810-19 J

5. Una superficie metlica emite electrones cuando sobre ella incide luz verde pero no
lo hace cuando es amarilla Emitir cuando la luz incidente sea azul? Y si es roja?
(Justificad la respuesta).

sol: Con azul s y con roja no.

6. Que un determinado metal presente o no efecto fotoelctrico, al ser iluminado con


una radiacin de una cierta frecuencia, depende de:

a) La amplitud de la onda incidente.


b) El valor de la frecuencia.
c) Otros factores (especificar en su caso).

Sealad qu propuesta es la que se acepta como correcta razonando el porqu.

sol: Depende del valor de la frecuencia, de modo que el efecto fotoelctrico slo se presen-
tar si la frecuencia de la radiacin utilizada es superior a la umbral, sea cual sea el valor de
la amplitud.

7. Una fuente de luz monocromtica emite una radiacin de longitud de onda 4810 -7 m,
con una potencia de 20 W Cuntos fotones emite por segundo?

Sabemos que la luz est formada por fotones y que la energa de cada uno de ellos viene
dada por la expresin Ef = h siendo h la constante de Planck y la frecuencia que tenga la
luz en cuestin. La energa que emite una fuente luminosa por segundo coincide precisa-
mente con el valor de la potencia de dicha fuente, por tanto cuanto mayor sea dicha potencia
Naturaleza de la luz 368

P mayor ser tambin el nmero de fotones que se emitan por segundo. Por otra parte, como
la energa de cada fotn es h, cabe pensar que, para una potencia dada, cuanto menor sea
la frecuencia de la radiacin, tantos ms fotones se estarn emitiendo cada segundo.

Cmo podramos determinar el nmero de fotones N emitidos por la fuente al cabo de un


tiempo t?
De acuerdo con los razonamientos anteriores, podemos hallar la energa E emitida durante
ese tiempo y dividirla por la energa correspondiente a uno de los fotones que forman la
radiacin, de este modo:
E P t
N
E f h
Hemos de ver ahora, cmo podemos calcular la frecuencia (no se nos da directamente en el
enunciado) y finalmente determinar el nmero de fotones emitidos por segundo (que es lo
que se nos pide).

Aunque no conocemos la frecuencia, s que sabemos la longitud de onda de la radiacin


utilizada y como ambas magnitudes se hallan relacionadas, podemos obtener fcilmente la
frecuencia. En efecto:


c de modo que c siendo c la velocidad de propagacin de la luz.
T

Pt Pt
Consecuentemente N =
h hc

de donde podemos despejar el nmero de fotones emitidos cada segundo sin ms que dividir
N por t:

N P N 20 4'87 10 7
y sustituyendo numricamente = = 4891019 fotones/s
t h c 34
t 6'63 10 3 10 8

8. La frecuencia umbral de un metal dado es de 851014 Hz. Determinad la energa cinti-


ca mxima de los electrones emitidos cuando ste se ilumina con luz de 131015 Hz.

sol: Ec = 186 eV

9. El trabajo de extraccin para el aluminio es 42 eV. Si se ilumina una superficie de


o
aluminio con radiacin de 200 A . Determinad:

a) Longitud de onda umbral para el aluminio.


b) Potencial de frenado necesario para detener los fotoelectrones.

Recordemos que el trabajo de extraccin nos indica el valor de la mnima energa (E0) que
hay que suministrar a un electrn del metal para arrancarlo del mismo. En nuestro caso esta
Naturaleza de la luz 369

energa deber ser comunicada por un fotn de la radiacin incidente, de modo que
E0 = h0, por tanto:
Wextr = h0

Si conocemos el trabajo de extraccin ser inmediato conocer h0 y, por tanto 0.

c ch
En efecto: 0
0 Wextr

No obstante, antes de sustituir, hemos de expresar el Wextr en J:

Wextr = 42 eV = 421610-19 J = 67210-19 J.

de modo que sustituyendo ahora numricamente, obtenemos:

3 108 6' 63 1034 o


0 2 '96 10 7
m 2960 A
6' 72 1019
o
Si iluminamos con luz de = 200 A ser ms energtica (de acuerdo con la expresin c =
, a menor longitud de onda mayor frecuencia y por tanto mayor energa). En consecuen-
cia, no slo extraer los electrones sino que adems los dotar de cierta energa cintica Ec
que podremos conocer gracias al potencial de frenado V0 necesario.

h hc 1 1
Ec = h ( - 0) qeV0 = h ( - 0) de donde V0 = ( 0 ) = ( )
qe qe 0
y sustituyendo: V0 = 578 V.

10. Un metal emite fotoelectrones de energa cintica 2 eV al iluminar con luz de fre-
cuencia 111015 Hz. Calculad la frecuencia de la luz con que hay que iluminar para
que la energa mxima de los fotoelectrones sea superior, en un 25%, a la del caso an-
terior.

En este problema se nos indica que al irradiar un metal con fotones de una cierta frecuencia
, se arrancan electrones con una energa cintica de 2 eV. Esto significa que el fotn inci-
dente posee una energa E = h superior a la de extraccin (E0 = h0) en 2 eV. Si introdu-
cimos estas condiciones en la ecuacin fotoelctrica de Einstein nos queda:
h = h0 + Ec Ec = h - h0 = h ( - 0 ) = 2 eV

Cmo podemos aumentar la energa cintica con que surgen los electrones?

Si queremos que cada electrn extrado incremente la energa cintica con que sale en un
25% habr que iluminar el metal con una radiacin de mayor frecuencia ( ), para que
los fotones sean ms energticos (E = h), cumplindose entonces que:

Ec = E- E0 = h- h0 = h (- 0 ) = 2 + 0252 = 250 eV = 250 1610-19 J


Naturaleza de la luz 370

Tenemos pues las dos expresiones siguientes:

Ec = h- h0
Ec = h- h0

y necesitamos obtener la frecuencia . Como no sabemos el valor de la frecuencia umbral


0 podemos eliminarla multiplicando la segunda ecuacin por -1 y sumando ambas:

Ec- Ec = h- h y despejando obtenemos finalmente:

Ec' Ec h
' y sustituyendo numricamente: = 1221015 Hz.
h

Si analizamos el resultado anterior, comprobaremos que es dimensionalmente homogneo


(T-1 en ambos lados). Adems la frecuencia pedida es mayor que la inicial (como deba
de ser para aumentar la energa cintica de los fotoelectrones) y coincide con ella si impo-
nemos la condicin de que la diferencia entre las energas cinticas sea nula.

11. Si la frecuencia umbral para la plata es 113 10 15 Hz, Cul deber ser la frecuen-
cia de la radiacin incidente para que la energa cintica de los fotones emitidos sea de
26 eV?

sol: = 1751015 Hz

12. Si la frecuencia umbral de una superficie metlica es 46310 14 Hz, determinad la


rapidez inicial con que son emitidos los fotoelectrones al iluminar la superficie con una
frecuencia de 81014 Hz ( h= 66310-34 J s; me = 9110- 31 kg).

sol: v = 7105 m/s

13. Si los fotoelectrones del cinc exigen un trabajo de extraccin de 43 eV, Qu longi-
tud de onda mxima ser capaz de extraer electrones?
o
sol: = 2890 A

14. El trabajo de extraccin para el caso del metal sodio es de 23 eV.

a) Cul ser la mxima longitud de onda que producir emisin de fotoelectrones en


dicho metal?
o
b) Si la luz incidente fuera de 2000 A , Cul sera la energa cintica mxima de los
electrones extrados?

Si conocemos la energa que, como mnimo, debemos suministrar para conseguir arrancar
un electrn del sodio, podremos saber tambin la frecuencia umbral o mnima necesaria
para ello, la cual viene dada por la expresin:

E0 = h0 0 = E0/h
Naturaleza de la luz 371

y como la frecuencia de una radiacin est relacionada con la longitud de onda mediante c =
, podemos obtener fcilmente que:

0 = c/0 = ch/E0 = 5410-7 m


o
Si se hace incidir sobre el sodio una radiacin de 2000 A (210-7 m), al ser de menor longi-
tud de onda (y por tanto ms energtica), s que ser capaz de arrancar electrones, pero
adems, el exceso de energa del fotn se invertir en energa cintica del electrn. Ello vie-
ne expresado en la ecuacin fotoelctrica de Einstein en la forma:

1 1
Ec = h - h0 Ec = h c ( ) = 62610-19 J = 391 eV
0

15. Determinad la longitud de onda de la radiacin emergente que se produce en el


efecto Compton para un ngulo de 300, sabiendo que la radiacin incidente es de 310 20
Hz (h = 66310-34 Js, masa del electrn = 93110-31 kg).

El denominado efecto Compton se produce cuando un haz de fotones llega a una fina lmina
de un cristal de calcita y, tras incidir en ella, se observa que una parte del haz atraviesa la
lmina sin sufrir modificacin alguna y la otra se dispersa apareciendo radiacin en todas
direcciones (valores de entre 0 y 180). Adems se comprueba que la radiacin que ha
sido dispersada tiene distinta longitud de onda y, por tanto, distinta frecuencia, segn sea el
ngulo de desviacin.

Mediante medidas experimentales Compton lleg a establecer la relacin existente entre las
longitudes de onda de la radiacin incidente y emergente, que viene dada por la expresin:

h
' (1 cos )
mc
Como veremos en el ejercicio 24, esta expresin queda justificada tericamente al interpre-
tar el fenmeno como un choque elstico entre los fotones incidentes y los electrones de los
tomos del cristal.

En la ecuacin anterior se aprecia que la variacin de la longitud de onda que se produce


depende del valor que tome la funcin 1-cos. Dicha funcin alcanza su valor mximo
cuando = rad, en cuyo caso 1-cos = 1+1 = 2 e = 2h/mc. Analizando la ecuacin es
fcil darse cuenta tambin que aquella radiacin que atraviese la lmina sin desviarse,
tendr un = 0.
Naturaleza de la luz 372

Para el caso que nos plantean en el que el ngulo de dispersin es = 30, lo conocemos
todo menos (incgnita) y . Esta ltima podemos determinarla si la relacionamos con la
frecuencia: c = = c/, as pues:

c h
' (1 cos ) y sustituyendo valores:
m0 c

3 108 6' 63 1034


' 31
' 1012 m
(1 cos 30 ) 13
3 10 20
' 10 3 10
91 8

Como podemos ver la longitud de onda de la radiacin emergente es mayor que la de la in-
cidente ( = c/ = 10-12 m), lo cual es coherente con que parte de la energa de la radiacin
incidente se transfiera a los electrones con los que se interacciona, de forma que el fotn
emergente tendr menos energa que el incidente ( y por tanto que ).

16. Para qu ngulo de dispersin es mxima la transferencia de energa en el efecto


Compton? Cul es la variacin relativa de la longitud de onda que experimenta un
haz de rayos X de = 71 nm, cuando es dispersado en dichas condiciones?

sol: 1800 ; - / = 6810-4 m

17. Averiguad la variacin de la cantidad de movimiento que sufren ciertos fotones,


por efecto Compton, para que la variacin mxima de su longitud de onda sea del 1%.

sol: p = 1%

18. Qu significado hay que atribuir a la famosa expresin de Einstein: E = m c2 ?

En ocasiones se afirma que la expresin E = mc 2 significa que la masa se puede transfor-


mar en energa pura, en el sentido que es posible desmaterializar una cierta masa m y obte-
Naturaleza de la luz 373

ner a cambio una energa mc2 habindose convertido masa (algo material) en energa (algo
inmaterial).

En realidad, dicha expresin debera escribirse ms propiamente como:

E0 = mc2

donde E0 es la llamada energa propia de un cuerpo o sistema dado y representa la energa


de dicho sistema en reposo, independientemente de lo que tenga alrededor.

La ecuacin anterior debe interpretarse diciendo que no es posible que cambie la energa
propia de un sistema sin que a la vez lo haga el valor de su masa. Ambas cosas van ligadas,
de modo que todo aumento (disminucin) de energa propia va acompaado siempre de un
aumento (disminucin) en el valor de la masa.

Con esta afirmacin no se est indicando que la masa se convierta en energa sino que cuan-
do un cuerpo pierde una cantidad de energa, tambin est perdiendo masa, existiendo una
proporcionalidad entre ambas cantidades, que viene dada por la expresin:

E0
m
c2
Este razonamiento no solo es vlido para aquellos fenmenos (como los radiactivos) en los
que un sistema emite energa mediante determinadas radiaciones sino que se puede genera-
lizar a cualquier proceso en el que la energa propia cambie, sea cual sea el mecanismo por
el cual lo hace como, por ejemplo, al comprimir un muelle, formarse un dipolo, etc.

19. Calculad el valor de la energa cintica relativista y comprobad que para v<<c co-
incide con la expresin utilizada en la mecnica clsica.

Sea un objeto de masa m y energa propia E 0. Ambas cantidades m y Eo se refieren a una


propiedad (masa-energa) intrnseca del objeto, que se puede determinar en un sistema de
referencia inercial (SRI) en el que el objeto est en reposo. Ahora bien, en cualquier otro
SRI en el que el objeto se est moviendo con una determinada velocidad, su energa total E
ser la suma de Eo y la correspondiente energa cintica Ec :

E = Eo + Ec

De acuerdo con la mecnica relativista, sabemos que la energa total de un sistema viene
dada por la expresin:
E m c2

Sustituyendo obtenemos: mc2 = mc2 + Ec y despejando: Ec = mc2- mc2

La expresin anterior es la expresin relativista de la energa cintica de un cuerpo en mo-


vimiento en un SRI dado, que podemos escribir tambin como:
Naturaleza de la luz 374

1 v
Ec = mc2 ( - 1) = mc 2 1 donde
1 2 c

Si sustituimos en esta expresin por v/c y suponemos que v es mucho menor que c, como
se nos dice ene el enunciado, podramos pensar que (v/c) 2 es prcticamente 0 y en conse-
cuencia obtendramos:
Ec = mc2 (1-1) = 0

La conclusin anterior es absurda puesto que si se trata de una partcula en movimiento debe
tener energa cintica. Dnde puede estar el error?

En el razonamiento anterior se ha cometido el fallo de suponer que (v/c) 2 es despreciable,


cuando, si bien es cierto que su valor es muy pequeo, en este clculo, como veremos a con-
tinuacin, no se puede ignorar

1
1
Si tomamos (1 2 ) 2
y desarrollamos este binomio, obtendremos:
1 2
1
1 2 3 4
(1 2 ) 2
1 ...
2 8

y ahora, si consideramos que por ser v c podemos despreciar a partir de 4 (inclusive) y


no a partir de 2 (como hicimos antes), tendremos:

1 2 1
Ec = mc2 (1 1) = mv que es la expresin utilizada en mecnica clsica.
2

2 2

20. Justificad que un fotn sea una partcula de masa nula.

Como sabemos, la radiacin electromagntica est constituida por unas partculas especiales
o cuantos llamadas fotones que se desplazan en el vaco con una rapidez c de prctica-
mente 300.000 km/s (en el aire es aproximadamente la misma). Cada uno de esos fotones
dispone de una energa E = h en donde h es una constante llamada constante de Planck
(66310-34 Js) y la frecuencia de la radiacin de la que forma parte el fotn en cuestin.

Por otra parte, de acuerdo con la Teora Especial de la Relatividad, la energa de cualquier
partcula libre viene dada por la expresin: E = mc2. Si queremos aplicarla al caso del
fotn, bastar sustituir en ella v por c, con lo que:

1 1 1 2
E m c2 m c2 m c m c2
1 v/ c 1 c/ c 0
2 2

Esta ltima expresin tiene dos soluciones:


Naturaleza de la luz 375

a) m 0 y E = , cosa que no puede suceder ya que por otro procedimiento hemos visto que
la energa de un fotn es finita y de valor h.

b) m = 0 y E indeterminado, lo que s puede ocurrir. (E = mc2 = 0 c2 = indeterminado)

As pues, la masa del fotn es nula y su energa no puede obtenerse con la expresin
E = mc2, sino como E = h.

21. Determinad el valor de la cantidad de movimiento de un fotn.

Segn la Teora Especial de la Relatividad, la expresin de la cantidad de movimiento de


traslacin de una partcula es p = mv, que aplicada al caso del fotn resulta p = 0 c =
indeterminado.

As pues no podemos conocer la cantidad de movimiento de un fotn haciendo uso de la


expresin anterior. Sin embargo, s podemos hacerlo utilizando la expresin:

E2 = p2c2 + m2c4

La ecuacin anterior expresa una ley fundamental de la dinmica relativista y nos informa
de que la energa, la masa y la cantidad de movimiento de cualquier partcula o sistema en
general, estn interrelacionadas. Aplicndola al caso de un fotn (m = 0), nos queda que:

E
E = pc con lo que: p
c

Para terminar, indicar tan solo que si, equivocadamente, hubisemos utilizado p = mv, vli-
da nicamente para casos en los que v c, habramos obtenido p = 0c = 0, en franca con-
tradiccin con las experiencias que sealan inequvocamente que el fotn, aunque de masa
nula, s tiene cantidad de movimiento.

22. Un electrn de un tomo de hidrgeno pasa desde un estado cuya energa es - 085 eV
a otro que es -102 eV Cunto valdr la cantidad de movimiento del fotn emitido?

sol: p = 510-27 kgm/s

23. En el efecto Compton se considera que el electrn sobre el que impacta el fotn
incidente tiene, tras el impacto, una energa E, dada por la expresin:

E m2 c 4 p 2 c 2

Comprobad que dicha expresin es correcta.

Para justificar la expresin que se nos da en el enunciado, consideremos que la energa del
electrn tras el impacto (lo mismo que la de cualquier otra partcula en movimiento), valdr:
Naturaleza de la luz 376

m
E = mc2 =
2
c 2 y su cantidad de movimiento ser:
v
1
c

m
p = mv =
2
v
v
1
c

Si elevamos al cuadrado estas dos expresiones, obtenemos:

2 m2 c6 2 m2 c2 v2
E = 2 para la energa y p = para la cantidad de movimiento.
c v2 c2 v2
y si multiplicamos por c2 esta ltima expresin y la restamos de la primera nos queda:

m 2 c 6 m 2 c 4 v 2 m 2 c 4 c 2 v 2
E2 - p2 c2 = - = m2 c4
c v
2 2
c v
2 2
(c v )
2 2

de modo que E m 2 c 4 p 2 c 2 tal y como queramos demostrar.

24. Deducid de forma terica la expresin:

h
' (1 cos )
mc

obtenida al interpretar los resultados experimentales de la experiencia de Compton.

Compton comprob en 1923 que al hacer incidir rayos X (radiacin electromagntica de


elevada frecuencia) sobre un cristal de calcita, la radiacin emergente sufra una cierta des-
viacin respecto a la direccin original pero lo que resultaba ms sorprendente, era que sala
con una frecuencia menor que la que tena al incidir sobre el cristal. Este ltimo hecho era
absolutamente inexplicable si se conceba la radiacin como una onda (la frecuencia de una
onda no cambia cuando esta pasa de un medio a otro). En cambio, se justifica fcilmente si
se interpreta como un choque elstico entre dos partculas: una el fotn incidente y la otra
un electrn de la corteza de los tomos presentes en la calcita. En efecto, el electrn gana
energa mientras que el fotn la pierde, como la energa del fotn viene dada por h, si
pierde parte de la misma su frecuencia debe disminuir.
Naturaleza de la luz 377

Compton constat experimentalmente que las frecuencias de la radiacin incidente y


emergente estaban relacionadas a travs de la expresin:

1 1 h h
(1 cos ) o lo que es equivalente: ' (1 cos )
' mc 2
mc
siendo m la masa correspondiente al electrn.

A estas mismas expresiones se puede llegar considerando el fenmeno como un choque


elstico y oblicuo entre un fotn incidente y un electrn. Por tratarse de un choque, se cum-
plir el principio de conservacin de la cantidad de movimiento y por ser elstico se conser-
var tambin la energa cintica.

E E
'
E'
Conservacin de p : f , 0 f cos , f sen p'cos , p'sen
c c c
Conservacin de Ec: Ef + Ec = Ef + Ec
y suponiendo despreciable la energa cintica del electrn antes del choque:
Ef = Ef + Ec

Descomponiendo la ecuacin vectorial en dos escalares, nos queda:


Ef Ef '
cos p' cos h = hcos + p'c cos (1)
c c
E '
0 f sen p sen 0 = h sen - p'c sen (2)
c
y en cuanto a la energa cintica: h = h+ Ec (3)
Naturaleza de la luz 378

Despejando en (1) y en (2):

p'c cos = h - hcos


p'c sen = hsen

Elevando al cuadrado y sumando: p'2c2 = (h)2 -2 h h cos + (h)2 (4)

Por otra parte, la energa del electrn (E) y su cantidad de movimiento p, estn relaciona-
dos en la forma: E2 = p2c2 + m2c4 y despejando: p2c2 = E2 - m2c4.

Si adems consideramos que E = E0 + Ec = mc2 + Ec obtenemos:

p2c2 = (mc2 + Ec) 2 - m2 c4 p2 c2 = E' c2 + 2mc2 Ec

Sustituyendo en la expresin anterior la Ec dada por la ecuacin (3):

p2 c2 = (hf - hf )2 + 2m c2 (hf - hf ) y operando nos queda:

p2 c2 = (hf)2 -2 hf hf + (hf )2 + 2mc2 (hf hf ) (5)

Podemos ahora igualar las expresiones (4) y (5) y simplificar:

(hf)2 -2 hf hf cos + (hf )2 = (hf)2 -2 hf hf + (hf )2 + 2m c2 (hf hf )

hf hf (1- cos) = m c2 (hf hf ) y dividiendo por h2c2 obtenemos:

1 mc 1 1
f f '
1 m
(1 cos ) (1 cos ) y multiplicando en am-
' h ' h '
bos miembros por y despejando obtenemos:

h
' (1 cos )
mc

25. Qu idea bsica introduce De Broglie, acerca del comportamiento de la materia?

Tanto el efecto fotoelctrico como el efecto Compton se pueden explicar, como hemos vis-
to, asignando a la luz y a las radiaciones electromagnticas en general un carcter disconti-
nuo o cuantizado. Esto podra interpretarse como un apoyo fundamental a la teora corpus-
cular de la luz cerrndose as una histrica polmica sobre si se trataba de una onda o estaba
formada por partculas. Sin embargo, los fenmenos de interferencia y de difraccin que
experimenta la luz, seguan sin poder explicarse mediante la teora corpuscular y haba que
hacerlo otorgndole naturaleza ondulatoria.

De acuerdo con todo ello hubo que admitir para los fotones un doble comportamiento onda-
corpsculo, que interpretaremos asignndoles una naturaleza dual de tal forma que se
pondra de manifiesto uno u otro segn el tipo de fenmeno en el que interviniese la radia-
Naturaleza de la luz 379

cin. El fotn aparece pues como un concepto unificador de las propiedades corpusculares y
ondulatorias de la radiacin electromagntica, tal y como queda reflejado en la expresin p
= h/ que relaciona su cantidad de movimiento (carcter corpuscular) con la longitud de
onda (carcter ondulatorio).

Esta situacin, junto con algunos problemas fsicos que en la dcada de 1920-30 permane-
can todava sin resolver, llev al francs Louis De Broglie a exponer una nueva y atrevida
hiptesis consistente esencialmente en extender esa doble naturaleza de onda-corpsculo de
los fotones a todos los dems objetos en movimiento ya sean tan pequeos como un electrn
o tan grandes como un planeta.

De Broglie generaliz la ecuacin = h/p, aplicable a los fotones que se mueven siempre a
la velocidad de la luz, a cualquier otro objeto en movimiento. Ms tarde, los cientficos
americanos Davisson y Germer, verificaron la hiptesis de De Broglie al obtener fotografas
que mostraban que los electrones tambin experimentaban el fenmeno de la difraccin
(tpicamente ondulatorio) y obtener a partir de las mismas la longitud de onda asociada,
comprobando que coincida con el valor predicho por la relacin anterior ( = h/p).

As pues, la idea bsica que se introdujo en la fsica a partir de la hiptesis de De Broglie


fue que todos los objetos materiales y no solamente el fotn, posean una doble naturaleza
ondulatoria-corpuscular.

La idea de la dualidad onda-corpsculo planteaba no obstante nuevas interrogantes, como,


por ejemplo, en qu podan consistir las ondas asociadas a los objetos. Conviene sealar que
se han dado varias interpretaciones errneas. Una de las ms frecuentes consiste en imaginar
fotones y electrones como partculas clsicas que se mueven acompaadas de una onda,
cuando en realidad partcula y onda son la misma cosa, es decir, el concepto clsico de
partcula material y de onda no existe. Todos los sistemas fsicos tienen potencialmente los
dos comportamientos de modo que, segn el tipo de fenmeno que estemos analizando po-
dremos apreciar el ondulatorio o el corpuscular. A diferencia de lo establecido en la Mec-
nica Clsica, estos comportamientos no son excluyentes sino complementarios. Es algo as
como si slo nos mostrasen un cilindro en dos posiciones tales que en unos casos nos pare-
ciese un rectngulo y en otros un crculo, aunque sabemos que en realidad no es lo uno ni lo
otro ni tampoco la simple suma de ambas cosas.

26. Los electrones de un microscopio electrnico son acelerados con una diferencia de
potencial de 12 kV Cul ser su longitud de onda asociada?

Si tenemos en cuenta la naturaleza ondulatorio-corpuscular de la materia y, en consecuencia


de los electrones, la longitud de onda correspondiente a los mismos ser de acuerdo con la
ecuacin introducida por De Broglie:
= h/p
Naturaleza de la luz 380

de modo que para conocer bastar con determinar la cantidad de movimiento p.

Cmo podemos obtener la cantidad de movimiento correspondiente a uno de los electro-


nes que se indican en el enunciado del problema?

Si consideramos la definicin de cantidad de movimiento de una partcula p = mv, necesita-


remos conocer qu rapidez adquiere un electrn al ser acelerado bajo la accin del campo
elctrico. Como se desplaza entre dos puntos cuya diferencia de potencial es 12 kV, bastar
aplicar el principio de conservacin de la energa al movimiento del electrn entre dichos
puntos para conocer la rapidez que alcanza.

La energa cintica que adquiere el electrn ser Ec = qeV (donde, como ya justificamos
en el captulo de campo elctrico, tanto qe como V son valores absolutos) y considerando
que la velocidad que adquiere el electrn es mucho menor que c:

1 2q e V
me v2 q e V v
2 me

Si al sustituir los datos se obtiene un valor de "v" comparable a "c", deberemos utilizar la
expresin relativista para Ec y para p. (Si hallamos v, veremos que no es nuestro caso).

h
Como p = mv tendremos que p 2q e me V , con lo que: = h/p =
2q e me V

Dmonos cuenta que la longitud de onda depender de la diferencia de potencial a que sea
sometido el electrn. Si las V son elevadas conseguiremos longitudes de onda muy peque-
as. En nuestro caso:

6'63 10 34 o
= = 1' 12 10 12 m 0' 112 A
2 1'6 10 19 9'1 10 31 12 10 3

Qu ventajas puede suponer el hecho de poder utilizar longitudes de onda muy cortas en
un microscopio de este tipo?

En primer lugar conviene sealar que la mxima resolucin de un microscopio es del orden
de la longitud de onda que se utilice para poder ver los objetos. Esto hace que un microsco-
pio convencional tenga limitada su resolucin por el mnimo valor de la longitud de onda a
o
que es sensible el ojo humano (unos 4000 A ). Para objetos menores de este tamao, al ser
ste comparable a la longitud de onda utilizada, se producen efectos de difraccin y dan una
imagen borrosa.

En la dcada de 1930 se construyeron unos nuevos microscopios en los que la ilumina-


cin del objeto se lleva a cabo mediante un haz de electrones. Estos haces pueden tener una
o
longitud de onda efectiva de unos 004 A , con lo que se pueden examinar objetos tan pe-
queos como los virus, cuya estructura en un microscopio normal permanecera oculta debi-
Naturaleza de la luz 381

do a la difraccin. La imagen producida por el haz de electrones se observa finalmente en


una pantalla fluorescente o se fotografa.

27. Calculad la longitud de onda asociada a un electrn que se desplaza con una rapi-
dez de c/25.

sol: = 610- 11 m

28. Cul ser la longitud de onda asociada a un automvil de 2000 kg, que se desplaza
a 144 km/h?

sol: = 82910-39 m

29. Determinad el cociente entre las longitudes de onda asociadas a un neutrn y a un


electrn de igual energa cintica. (mn = 167510 - 27 kg; me = 91110- 31 kg).

sol: n /e = 23310- 2

30. Comparad la longitud de onda asociada a un electrn, un fotn y un neutrn, si


todos ellos tienen una energa cintica de 1 MeV.

Se trata de un ejercicio de aplicacin en el que se ha de obtener el valor de la longitud de


onda correspondiente a distintas partculas (o ms precisamente cuantones). En todos los
casos deberemos utilizar la expresin de De Broglie:

= h/p

Nos bastar conocer la cantidad de movimiento p de cada una de las partculas para poder
determinar el valor de la longitud de onda :

Para el fotn: p = E/c

y sustituyendo: = h h h c 6'63 10 3 10 1'24 10 12 m


34 8

p E E 6
10 1'6 10
19

Para el electrn y el neutrn la cantidad de movimiento se puede obtener mediante el pro-


ducto mv, en el que v habr de calcularse a partir de la energa cintica de cada uno de
ellos teniendo en cuenta la expresin relativista de la misma:


Ec E E 0 m c - m c m c
2 2
2 1
1
1 v/c 2

1
y despejando: v c 1 2
Ec
1
mc
2
Naturaleza de la luz 382

Sustituyendo para el electrn los valores correspondientes de Ec = 1 MeV y de su masa m =


9110-31 kg, nos queda: v = 094c (cercana a la de la luz) de modo que:

m 9' 1 10 -31
p = mv = 094c = 0' 94 c = 75510-22 kgm/s
1 v / c 1 0' 94
2 2

Sustituyendo en = h/p llegamos a = 66319-34 /75510-22 = 87810-13 m

Mediante las mismas expresiones pero utilizando ahora los valores correspondientes al
neutrn, cuya masa es de 167510-27 kg, obtenemos:

v = 138107 m/s, p = 2310-20 kgm/s y = 28810-14 m.

Qu hubiera sucedido si para calcular v en el caso del electrn y del neutrn hubisemos
utilizado la expresin aproximada: Ec 1 m v 2 ?
2

2 Ec
Al aplicarla al electrn hubisemos obtenido ve = = 59108 m/s.
me

Como vemos, se trata de un valor casi el doble que el de la velocidad de la luz en el vaco,
lo que segn la fsica relativista es imposible (de acuerdo con la Teora de la Relatividad
ningn objeto se puede desplazar a mayor velocidad que la de la luz). En este caso, pues, no
puede ser utilizada la expresin aproximada de la energa cintica (se debe a que la rapidez
con que se propaga es 094 c y, por tanto, nada despreciable frente a c).

En cambio, al aplicarla al neutrn (cuya masa es ms de 1800 veces la del electrn), obte-
nemos:

2Ec
vn = = 138107 m/s
m0 n

que coincide con el valor anterior (al desplazarse con una rapidez mucho menor que c).

31. Determinad el valor de la longitud de onda correspondiente a:

a) La Tierra en su movimiento de traslacin alrededor del Sol. (Masa aproximada de


la Tierra 61024 kg, rapidez lineal respecto del Sol 29'8 km/s.
b) Una pelota de 150 g lanzada a 200 km/h
c) Una mota de polvo de 10 -10 g de masa movindose a 1 cm/s
d) Un electrn de energa cintica igual a 49 eV (masa del electrn 910 -31 kg)

sol: a) 3710-63 m; b) 79610-35 m; c) 66310-19 m; d) 17510-10 m


Naturaleza de la luz 383

32. De Broglie pensaba que la onda asociada a un electrn que describe una rbita cir-
cular deba ser una onda estacionaria (una circunferencia es un medio limitado) que se
cerrara sobre s misma (de otra manera se producira interferencia destructiva y la
onda cesara rpidamente). De acuerdo con lo anterior Qu relacin matemtica de-
ber existir entre la longitud de la rbita circular y la longitud de la onda asociada al
electrn que describe dicha rbita?

Parece evidente que la circunferencia de la rbita que sigue el electrn deber tener una lon-
gitud igual a un nmero entero de longitudes de onda.

As pues las nicas ondas posibles sern aquellas que cumplan la relacin: 2r = n siendo
r el radio de la rbita y n = 1, 2, 3 ...

Si en la expresin 2r = n sustituimos = h/mv no es difcil obtener que mvr = nh/2 que


es, precisamente, uno de los postulados de Bohr. Adems, conviene tener en cuenta que es a
partir de dicho postulado de donde se obtuvieron los radios de las rbitas permitidos, as
como los valores de energa permitidos.

33. El principio de incertidumbre de Heisenberg es uno de los principios fundamenta-


les de la mecnica cuntica. En l se dice que resulta imposible conocer en determina-
do instante y con toda precisin, la posicin que ocupa una partcula en movimiento y
su velocidad. Esta indeterminacin hay que atribuirla a (sealad la propuesta correcta
y explicadla):

a) El hecho de que toda medida implica, mientras se lleva a cabo, una perturbacin en
aquella magnitud que se mide.
b) La propia imprecisin de los instrumentos de medida utilizados.
c) Otra respuesta.

El principio de incertidumbre, de Heinserberg afirma la imposibilidad de determinar si-


multneamente y con una precisin absoluta la posicin y la cantidad de movimiento de una
partcula determinada, como por ejemplo un electrn.

Se trata de algo contrario a los postulados de la Mecnica Clsica en donde, como sabemos,
si se conocen la posicin y velocidad de un objeto en un instante dado y la fuerza resultante
que acta sobre el mismo, es posible conocer con total exactitud la posicin y la velocidad
Naturaleza de la luz 384

que dicho objeto tendr en cualquier instante posterior y, por tanto, la trayectoria que se-
guir.

El principio de incertidumbre es una consecuencia de la dualidad onda-corpsculo de la


materia. En efecto, dicha dualidad conlleva una cierta deslocalizacin de todos los objetos
en movimiento (aunque sea apreciable slo en partculas submicroscpicas como, por ejem-
plo, los electrones), que nos impide hablar de trayectorias perfectamente definidas.

Heinserberg demostr que dada una partcula que se desplace a lo largo del eje OX, el pro-
ducto de la imprecisin x con que se conozca su posicin en un instante dado y la impreci-
sin p de su cantidad de movimiento (p = mv) en ese mismo instante, debe ser siempre
mayor (o, al menos igual) que h/2, es decir:

px h/2

De la ecuacin anterior se desprende, que si en un instante dado conseguimos determinar la


posicin de una partcula en movimiento con mucha exactitud (x pequesimo), ello ser a
cuenta de una mayor imprecisin en la cantidad de movimiento y viceversa (para que el
producto de ambos factores permanezca siempre por encima de h/2.

Hemos de sealar que nada se opone a que podamos determinar con la precisin que quera-
mos, la posicin o la velocidad de, por ejemplo, un electrn (este principio se refiere a la
determinacin simultnea de ambas con precisin absoluta).

El principio de incertidumbre tal y como acaba de ser formulado, establece unos lmites a la
precisin con que podemos determinar los valores de las magnitudes cantidad de movi-
miento y posicin de un mvil, con lo que, cabe esperar que esos lmites vuelvan a apare-
cer cuando se midan otras magnitudes relacionadas con las anteriores. De hecho, el princi-
pio de indeterminacin no solo se cumple para la cantidad de movimiento y posicin sino
para cualquier par de magnitudes cuyo producto tenga las mismas dimensiones que ste.
Observemos que el producto anterior tiene dimensiones de energa tiempo:

kg m/s m = kg m/s2 m s = N m s = J s

Por tanto, otro par de magnitudes que cumplen dicho principio son la energa y el tiempo.
As, si la energa de un sistema se mide con una imprecisin E y dicha medida se realiza
durante un intervalo de tiempo t, se cumplir que:

Et h/2

Segn la expresin anterior, la determinacin simultnea de la energa y el tiempo viene


afectada de un cierto lmite, de tal forma que la imprecisin en el valor de la energa de un
objeto es tanto mayor cuanto menor es el intervalo de tiempo considerado y viceversa. Un
caso lmite especialmente interesante es que, si queremos medir con absoluta precisin la
energa de un sistema, ser necesario utilizar un tiempo infinito.

Conviene tener presentes algunas interpretaciones errneas que se relacionan con el Princi-
pio de Incertidumbre:
Naturaleza de la luz 385

Una de ellas (probablemente la ms extendida), consiste en atribuir la indeterminacin en l


implcita al hecho (por otra parte cierto), de que toda medida perturba aquello que se mide.
Podemos pensar, por ejemplo, en una partcula de la que deseamos saber su posicin y can-
tidad de movimiento en un cierto instante. Un modo ordinario de medir la posicin de un
objeto es examinarlo con un haz luminoso (envindole fotones). Cabe pensar que si el obje-
to es muy grande, ni su posicin ni su velocidad se vern afectadas por este hecho, pero
podemos afirmar lo mismo si se trata, por ejemplo, de un electrn? Sabemos que, debido al
efecto de la difraccin, para poder medir una distancia que nos de la posicin con precisin
hemos de utilizar una luz cuya longitud de onda sea inferior al tamao de aquello que se
quiere medir. Aunque en principio no hay ningn inconveniente en disponer de luz con una
longitud de onda muy corta, resulta que cuanto ms pequea sea , mayor ser la frecuencia
y, por tanto, la energa de los fotones incidentes sobre el electrn, que afectarn a su canti-
dad de movimiento. Podramos tratar de disminuir la perturbacin utilizando una luz de lon-
gitud de onda mayor, pero en ese caso, debido a la difraccin, la imprecisin en la determi-
nacin de la posicin aumentara.

El fenmeno anterior, no obstante, no debe ser identificado con el principio de incertidum-


bre, que se seguira cumpliendo an en el hipottico caso de que pudisemos observar el
movimiento de partculas sin perturbarlo o aunque tuvisemos instrumentos de medida
totalmente precisos. No son estas las causas del lmite a la precisin con que podemos cono-
cer la trayectoria de una partcula sino la propia naturaleza dual de la materia, su carcter
ondulatorio-corpuscular.

34. Determinad el lmite que impone el Principio de Incertidumbre de Heisenberg a la


localizacin de una partcula macroscpica de 1 mg de masa y 10-6 m de dimetro, que
se desplaza a lo largo del OX con rapidez de 10 m/s, medida con una imprecisin de 10 -
3
m/s. (Considrese que la masa de la partcula se ha medido con la suficiente exactitud
como para no influir en la imprecisin de la cantidad de movimiento).

Segn el principio de incertidumbre el producto de las imprecisiones con que se midan para
un instante dado la posicin y la cantidad de movimiento de una partcula debe ser igual o
mayor que h/2:

xp h/2 de donde x h/2p

Como vemos, para conocer el lmite que impone este principio a la localizacin de la part-
cula, necesitamos conocer la imprecisin con que se mide la cantidad de movimiento (p).
Ahora bien, la cantidad de movimiento no es una magnitud que se haya medido directamen-
te, sino que se obtiene al sustituir los valores de m y de v en la expresin p = mv, de forma
que la imprecisin en p depender de la imprecisin con que se midan m y v. Si, como se
dice en el enunciado, la imprecisin cometida al medir la masa se puede considerar despre-
ciable frente a la de v, tendremos que:

p = m v = 10-610-3 = 10-9 kgm/s y sustituyendo en la expresin inicial:


-
x h/2p 66310 34/210-9 = 10610-25 m.
Naturaleza de la luz 386

Como vemos, el lmite que impone el principio de incertidumbre es totalmente irrelevante


en el mundo macroscpico ya que, ni somos capaces de alcanzar tal nivel de precisin ni,
por otra parte, nos interesara. Una cuestin que podemos plantearnos es:

Cmo cambiara el resultado si en lugar de la partcula considerada se tratase de un


electrn que se desplaza con igual rapidez y medida con la misma imprecisin?

Un anlisis superficial de la situacin (una partcula que se desplaza con la misma rapidez,
etc.), podra llevarnos a contestar que el resultado debera ser del mismo orden. Sin embar-
go, es preciso tener en cuenta que la masa de un electrn es de 9110 -31 kg lo que hace que
al sustituir en la expresin que nos da la imprecisin con que vendr afectada su posicin,
obtengamos:

6'63 10 34
x h/2p 0'12 m
2 9'110 31 10 3

Es fcil comprender que el lmite impuesto por el principio de incertidumbre es muy impor-
tante en este caso ya que tendramos el electrn (que se considera como si fuera prctica-
mente puntual) deslocalizado en una longitud de 12 cm.

35. Un lser cuya longitud de onda es 630 nm emite un pulso de duracin 10 ns. Hallad la
imprecisin mnima de la frecuencia del pulso.

h h E
E t E . Por otra parte, E = h f E = h f f
2 2 t h
E h 1 1
Sustituyendo: f = 1'59 10 7 Hz
h 2 t h 2 t 2 10 8

36. En la mecnica clsica el estado de un sistema en un instante dado queda caracteri-


zado por los valores de su posicin y su velocidad. Sin embargo, debido a la doble na-
turaleza ondulatorio-corpuscular de la materia, esto no es posible cuando se trabaja
con partculas submicroscpicas. Cmo se procede en este caso?

Una vez aceptado el carcter dual de la materia se desarroll una nueva mecnica llamada
Mecnica Cuntica, para la que todo sistema fsico est descrito por una funcin de onda
(x, t) en la que x representa la posicin de un objeto que se mueve sobre una recta y t el
tiempo. Esta nueva mecnica, si bien es absolutamente general, al aplicarla al mundo ma-
croscpico se obtienen los resultados de la Mecnica Clsica, por lo que nicamente tiene
inters cuando se aplica a sistemas fsicos extraordinariamente pequeos o submicroscpi-
cos (electrones, protones, fotones, etc.).

Qu representa (x, t) para una partcula?

El valor 2 dx representa la probabilidad de que la partcula se encuentre entre las posi-


ciones x y (x+dx) en un cierto instante t. En efecto, si en un instante dado representsemos
Naturaleza de la luz 387

en funcin de la posicin x, obtendramos una curva como la de la figura adjunta, que


2

nos indica la probabilidad de encontrar la partcula en distintas posiciones.

Por tanto, la posicin de una partcula en movimiento no est perfectamente definida y


nicamente se puede hablar de la probabilidad de encontrarla en cierta posicin. Esto es
algo que se comprueba experimentalmente ya que cuando se realizan medidas repetidas de
la posicin de una partcula en las mismas condiciones, se obtiene la curva de distribucin
anterior.

Si como hemos dicho (x, t) describe el estado de un sistema fsico en el instante t, qu


expresin determina en la mecnica cuntica cmo evoluciona este sistema y, por tanto su
, con el tiempo?

En la dinmica clsica la ecuacin de Newton F m a o ecuacin fundamental de la din-
mica, determina cmo evoluciona un sistema fsico macroscpico con el tiempo. Para la
mecnica cuntica, fue Schrdinger quien formul la ecuacin fundamental que describe
cmo evoluciona un sistema fsico. Dicha ecuacin puede escribirse para una partcula co-
mo:

d 2 4 m
dx 2

h
E U ( x) 0

en la que x representa el valor de la coordenada X, E la energa total, m la masa de la part-


cula y U la energa potencial a que est sometida. Se trata de una ecuacin que, como se
puede apreciar presenta una cierta complejidad. Sin embargo ha tenido una importancia ex-
traordinaria en el estudio de la estructura subatmica de la materia y se le considera, con
toda razn, como la ecuacin fundamental de la mecnica cuntica.
Naturaleza de la luz 388

PROBLEMAS COMPLEMENTARIOS

C-1. Los trabajos de extraccin de tres metales diferentes son: 1'9 eV, 4'2 eV y 5'0 eV. Se
tiene una fuente de luz monocromtica de = 514 nm. Se pide:
a) De cul de los tres metales anteriores se podrn extraer electrones?
b) Para obtener una fotocorriente de 2 mA qu potencia deber tener la luz incidente?

Para poder extraer un electrn es necesario que la energa del fotn incidente E = hf supere
a la energa mnima necesaria para liberar al electrn, dada por h f0 (donde f0 es la fre-
cuencia umbral, caracterstica de cada metal). El valor de esa energa coincide con el trabajo
de extraccin. Por tanto, lo primero que tendremos que hacer ser conocer la frecuencia de
la radiacin utilizada y, a continuacin, calcular la energa en eV y compararla con los tra-
bajos de extraccin que se nos dan en el enunciado.

f = c/ = 3108/(51410-9) = 5841014 Hz

La energa del fotn sera E = hf = 66310-34 5841014 = 38710-19 J = 24 eV

El resultado anterior muestra que, con ese tipo de luz, slo podrn extraerse electrones del
primer metal que es el que tiene un trabajo de extraccin inferior a la energa del fotn inci-
dente.

Para hallar la potencia de la luz incidente que se necesita (potencia ptica), hemos de averiguar
qu relacin liga dicha potencia con la intensidad de fotocorriente que se desea obtener.

Supongamos el caso ms favorable en el que cada fotn incidente extrajera un electrn y


que ste llegase a la otra placa. Si consiguiramos relacionar por una parte el nmero de
fotones incidentes en un tiempo t con la potencia ptica y por otra el nmero de electrones
liberados con la intensidad de saturacin, tendramos resuelto el problema. En efecto:

Sabemos que P = E/t = NEfotn/t = Nhf/t donde N es el nmero de fotones que inci-
den en t. Por tanto:
N P
(1)
t h f

Por otra parte, la intensidad de corriente I = Q/t = Nqe/t donde N es el nmero de elec-
trones que atraviesan una seccin dada del conductor en un tiempo t. Por tanto:

N I
(2)
t q e

P I I h f
Igualando las expresiones (1) y (2), obtenemos que: P
h f qe qe

Sustituyendo valores nos queda que: P = 4810-3 W


11. FSICA NUCLEAR

1. Una muestra radiactiva emite partculas , y . Determinad la energa cintica que


corresponder a una partcula que sale con una rapidez de 107 m/s, a una cuya
rapidez es de 098c y a una de frecuencia 1020 Hz.

Datos: masa del electrn 9110-31 kg; constante de Planck h = 66310-34 Js

Sabemos que las partculas estn constituidas por dos protones y dos neutrones (ncleos
de 24 He ), las son electrones y las son fotones. Estas ltimas son las que tienen un mayor
poder de penetracin siendo capaces de atravesar incluso paredes gruesas de hormign. El
clculo de la energa cintica de cada una de ellas deber hacerse atendiendo al valor de la
rapidez con que se estn moviendo. Dicha rapidez, en el caso de los fotones (en el vaco y
en el aire) es siempre de 3108 m/s (aproximadamente) y en el caso de las otras dos puede
variar dependiendo de si han sido o no aceleradas de algn modo. En este problema nos dan
la rapidez con que se mueve cada partcula (y la frecuencia de la radiacin ). Con estos
datos, podemos calcular fcilmente la energa cintica correspondiente, teniendo en cuenta
que si el valor de la rapidez es cercano al de la luz en el vaco, ser necesario utilizar la ex-
presin relativista de dicha energa.

a) Para la partcula , como su rapidez (107 m/s) es bastante menor que c (3108 m/s),
podremos hacer uso de la expresin clsica o aproximada para la energa cintica:

1
Ec mv 2
2

en la que podemos determinar el valor de la masa si tenemos en cuenta que en 4 g de part-


culas habr el nmero de Avogadro de stas, por lo que la masa en kg de una sola de ellas se
obtendr como:

m = (4/NA)10-3 en donde, como sabemos, NA = 6021023 partculas/mol

Sustituyendo ahora en la expresin de la energa cintica:

1 4
Ec 10-3 (107 ) 2 y operando, obtenemos: Ec = 3310-13 J = 2 MeV.
2 6' 02 1023

b) Para una partcula , como la rapidez con que se desplaza es del orden de c, tendremos
que determinar la energa cintica utilizando su expresin relativista. En esta situacin po-
demos determinar la energa cintica como la diferencia entre la energa total de la partcula
y su energa propia (correspondiente a cuando se encuentra en reposo), de modo que:
Fsica nuclear 390

1
Ec = E - E0 = mc2 - mc2 = mc2 (-1) y teniendo en cuenta que 2
v
1
c


obtenemos: Ec = m c 1
2 1
y sustituyendo valores, obtenemos:
1 v/c 2

1
Ec = 9'1 10 31 9 1016 1 = 3310-13 J = 206 MeV
1 0'982

c) En el caso del fotn, sabemos que su energa es cintica y que sta viene dada siempre
por la expresin E = h, siendo h la constante de Planck y la frecuencia de la radiacin,
de modo que basta sustituir los valores correspondientes para obtener:

Ec = E = h = 66310-34 1020 = 66310-14 J = 041 MeV

2. Hallad el nmero atmico y la masa atmica del elemento producido a partir del
84 Po despus de emitir 4 partculas y 2 partculas .
ncleo 218

Dado que una partcula es el ncleo del tomo de 24 He , cuando un ncleo de un elemento
radiactivo emita una de estas partculas, el nmero de sus nucleones (A) deber disminuir en
4 unidades y el nmero de sus protones (Z) en 2, por lo que se transformar en otro ncleo
cuyo nmero msico ser cuatro veces inferior y que estar situado dos lugares antes en el
sistema peridico. El proceso se puede esquematizar como:

A
Z XAZ42Y 24 He

Anlogamente, cuando lo que se emite por un ncleo es una partcula (un electrn), se
comprueba experimentalmente que el nmero de protones Z aumenta en una unidad mien-
tras que el nmero msico A no vara. (El proceso corresponde a la transformacin de un
neutrn en un protn y en un electrn, emitindose este ltimo):

A
Z XZ1Y 10e

De las ecuaciones anteriores se deduce que en una transformacin radiactiva el nmero total
de nucleones y la carga total han de conservarse. Ambas se conocen con el nombre de leyes
de Soddy, ya que fueron establecidas de forma experimental por este cientfico y sus colabo-
radores en la segunda dcada del siglo XX.

Las ecuaciones anteriores aplicadas al caso descrito en el enunciado conducen a:


Fsica nuclear 391

218
84 Po 424 He 201e AZY

De donde resulta sencillo concluir que el nmero msico y el nmero atmico del elemento
Y vendrn dados por:

218 = 16 + A A = 218-16 = 202 nucleones


84 = 8 + 2(-1) + Z Z = 84-8+2 = 78 protones

202
Por tanto, el elemento Y que se forma es el platino 78 Pt

3. Determinad los nmeros msicos y atmicos del nclido que resultar a partir del
92 U despus de que ste emita 3 partculas y 2 partculas .
238

sol: 226 u.m.a y 88 respectivamente.

4. En la serie radiactiva del 232


90Th se emiten las siguientes radiaciones hasta llegar al
Pb: , , , ,,,. Cul es la masa y el nmero atmico del elemento que precede
al Pb en la serie?

sol: 216 u.m.a y 84 respectivamente

5. Dos elementos radiactivos tienen periodos de semidesintegracin de 60 y de 180 aos


respectivamente. Si partimos de 2 moles de tomos de cada uno de ellos, se pide:

a) Trazad las curvas de desintegracin correspondientes.


b) Cul ser el valor de su constante de desintegracin?
c) Cuntos moles de tomos de cada elemento quedarn al cabo de 500 aos?

Experimentalmente se comprueba que dada una muestra radiactiva con un nmero inicial de
tomos N0, dicho nmero decrece de forma exponencial con el tiempo (al ir desintegrndo-
se). El nmero de tomos del elemento radiactivo que quedarn al cabo de un tiempo t viene
dado por la funcin:
N = N0e-t

Si analizamos dicha expresin, apreciaremos que el decrecimiento en el nmero de tomos


del nclido considerado presentes en la muestra inicial, depende de una constante (llama-
da constante de desintegracin) caracterstica de cada elemento radiactivo, de modo que
cuanto mayor es su valor, tanto ms rpidamente decrece la funcin (disminuye el nmero
de tomos del elemento en cuestin).

Tambin podemos ver que el tiempo necesario para que una muestra radiactiva de un ele-
mento cualquiera se desintegre totalmente (esto es, todos los tomos de dicho elemento se
hayan transformado en otros distintos con lo que N = 0), es infinito. Por tanto, dicho tiempo
no sirve para caracterizar una sustancia radiactiva. Sin embargo, el tiempo T necesario para
que el nmero de tomos existentes en un instante inicial (N0 a los 0 s) se reduzca a la mitad
Fsica nuclear 392

(N0/2 a los T s), s que es caracterstico de cada sustancia, tal y como se aprecia en la figura
adjunta (para dos sustancias A y B). A dicho intervalo de tiempo se le denomina Periodo de
Semidesintegracin.

Qu relacin existir entre el periodo de semidesintegracin T y la constante ?

De acuerdo con el significado de ambas magnitudes parece claro que, cuanto mayor sea una,
menor tendr que ser la otra (cuanto ms rpidamente decrezca N, en menor tiempo se redu-
cir a la mitad el nmero de tomos del nclido). Para encontrar la relacin matemtica
existente entre ellas bastar considerar en la expresin N = N 0e-t un tiempo de un periodo
(t = T) y sustituir N por N0/2 de modo que:

N0/2 = N0 e-T Simplificando y tomando logaritmos neperianos: ln 1/2 = ln e -T

de donde: T = ln 2/ = 0693/

Como vemos se trata, de dos magnitudes inversamente proporcionales y si conocemos una


de ellas podemos determinar fcilmente la otra. Aplicando la expresin anterior a nuestro
caso nos queda:

TA = 60 aos; A = 0693/60 = 00115 aos-1


TB = 180 aos; B = 0693/180 = 00038 aos-1

Si ahora queremos conocer el nmero de moles de tomos del elemento radiactivo conside-
rado, que habr en cada una de las muestras cuando hayan transcurrido 500 aos, hemos de
tener en cuenta que en la expresin N = N0 e-t lo que se maneja es el nmero de tomos.

Cmo podemos introducir en la expresin anterior el nmero de moles de tomos?

Basta con tener en cuenta que en cada mol de tomos hay un total de N A tomos (NA es el
nmero de Avogadro 6021023 part/mol). Por tanto, si dividimos por NA en ambos miem-
bros de la igualdad, nos queda:

N N
0 e t n n0 e t
NA NA
Fsica nuclear 393

siendo n el nmero de moles de tomos sin desintegrar existentes en el instante t. Como en


el enunciado se nos indica que partimos de 2 moles de tomos en cada caso:

nA 2 e0' 0115 500 = 66310-3 moles, de tomos del nclido A que habr a los 500 aos.
n B 2 e 0'0038 500 = 0299 moles, de tomos del nclido B que habr a los 500 aos.

Otra cuestin que nos podemos plantear es cmo transformar la funcin que venimos mane-
jando para que en ella aparezca la masa de la muestra en lugar del nmero de tomos o el
nmero de moles de tomos. Como la masa de una cierta muestra de un nclido se puede
expresar en funcin del nmero de moles de tomos (n) y de la masa molar de dicho ncli-
do (M) mediante: m = n M, la expresin anterior (moles de tomos) se puede escribir
tambin en funcin de la masa:

n M n 0 M e t m = m0 e-t

Tenemos as tres ecuaciones de desintegracin:

N = N0 e-t ; n = n0e-t ; m = m0 e-t

En este tipo de funciones, dado que la exponencial no puede tener unidades, ser muy
cmodo trabajar. Bastar con que la unidad en que se mide sea precisamente la inversa de
la unidad en que se mide T y con que en los dos miembros de la igualdad se trabaje con
partculas, moles, gramos, etc.

6. Disponemos de una muestra de 1 g de Ra Cuntos gramos de este elemento tendre-


mos dentro de 100 aos? (El periodo de semidesintegracin del Ra es de 1600 aos).

sol: 09576 g

t
7. Sabiendo que N(t) = N0 e - 0.02 (donde t viene en segundos), es la ley de desintegra-
cin de una determinada sustancia radiactiva Qu tiempo deber transcurrir para
que N0 se reduzca a la mitad? y para que se reduzca a la quinta parte?

Nos interesa conocer el tiempo que debe transcurrir para que una muestra que contiene N 0
nclidos radiactivos pase a tener N0/2. Para ello bastar con sustituir en la ecuacin de des-
integracin correspondiente, que, en este caso, nos dicen que es N = N 0e-002t, el nmero de
nclidos N por N0/2 y determinar el tiempo que debe transcurrir para que esto suceda.

N0/2 = N0 e-002t simplificando, tomando logaritmos neperianos y despejando:

1 ln2
e0' 02t ln1 ln2 0' 02t ln2 0' 02t t 34' 65s
2 0' 02

En este caso, el tiempo obtenido ser el periodo de semidesintegracin del nclido con el
que estemos trabajando.
Fsica nuclear 394

Sugerid un procedimiento para calcular el tiempo necesario para que el nmero inicial de
ncleos N0 se reduzca a la quinta parte y llevadlo a cabo.

El procedimiento ser, como es lgico, similar al anterior, de forma que bastar con utilizar
la misma expresin pero esta vez sustituyendo N por N0/5, de modo que:

N0/5 = N0 e-002t de donde obtenemos: t = 8047 s

Un aspecto importante a considerar es el valor del numero de ncleos sin desintegrar N 0


presentes en la muestra inicialmente. Si este hubiera sido muy pequeo, por ejemplo de solo
10, seguiran siendo vlidos los resultados obtenidos?

En principio podramos responder afirmativamente y decir que para que la muestra se redu-
jera a 5 ncleos o a 2, deberan de transcurrir 3465 s y 8047 s respectivamente. Pero si se
procediese a comprobarlo experimentalmente, nos sorprenderamos de los resultados ya que
podra darse una multitud de posibilidades distintas, desde, por ejemplo, que todos se hayan
desintegrado al cabo de un segundo hasta que ninguno de los 10 lo haya hecho en todo un
ao. A qu puede deberse esto?

La ley de la desintegracin radiactiva N = N0 e-t se obtiene a partir de muestras que con-


tienen un nmero mnimo de ncleos (de cierto nclido) que sea lo suficientemente grande
como para resultar estadsticamente significativo, de modo que las predicciones que se
hacen respecto a la desintegracin son de tipo estadstico, es decir, no sabemos si un nclido
determinado va a tardar un segundo o mil aos en desintegrarse sino que, cuando hay un
gran nmero de ellos (muestra estadstica), al cabo de un tiempo t, de N 0 que haba al prin-
cipio quedarn N sin desintegrar.

210
8. La desintegracin del istopo 84 Po viene dada por la reaccin:

210
84 Po 24He 206
Pb
82

Suponiendo que en el instante inicial hay N0 ncleos de Po presentes en la muestra y


que el nmero de ncleos en el instante t es N y = 0005 das -1 se pide:

a) Periodo de semidesintegracin.
b) La fraccin de ncleos sin desintegrar cuando t = 2T y t = 3T.

sol: a) 13863 das; b) 1/4 y 1/8 respectivamente.

9. El periodo de semidesintegracin de un elemento radiactivo es de 28 aos. Cunto


tiempo tiene que transcurrir para que su cantidad de tomos se reduzca al 75% de los
que hay en una muestra inicial?

sol: 1162 aos


Fsica nuclear 395

10. Una muestra contiene, en un instante dado, 16 g de un elemento radiactivo cuyo


periodo de semidesintegracin es de 23 das. Cuntos gramos de ese elemento que-
darn cuando hayan transcurrido tres periodos?

sol: 02 g

11. El Radn (222) se desintegra con un periodo de semidesintegracin de 39 das. Si


inicialmente se dispone de 20 mg Cuanto quedar al cabo de 76 das?

sol: 52 mg

12. Qu proporcin de los ncleos de 53Co de una muestra se desintegrarn en un


mes, sabiendo que su periodo es de 708 das?

Si consideramos que la muestra inicial contiene N0 ncleos (de cierto nclido) al cabo de 30
das nos quedarn N, que podremos evaluar mediante la expresin:

N = N0 e 30 .
-

Cmo podramos determinar qu proporcin de los ncleos iniciales se habrn desinte-


grado?

El nmero total de nclidos desintegrados vendr dado por: N = N0 - N

La proporcin respecto al nmero inicial ser:

N/N0 = (N0-N)/N0 .

Si queremos expresar la proporcin anterior en % haremos:

N0 N N 0 N 0 e 30
100 100 (1 e 30 ) 100
N0 N0

y como = 0693/T = 0693/708 = 00098 das-1, sustituyendo obtenemos que:

N0 N
100 (1 e 0'009830 ) 100 25'5
N0

As pues, al cabo de un mes se habr desintegrado el 255 % de los ncleos de Cobalto 53


que existan en la muestra inicial.

13. Una muestra radiactiva contiene 108 tomos de perodo de semidesintegracin 5


das Cul ser su actividad inicial? Y al cabo de 2 das?
Fsica nuclear 396

Cada vez que un ncleo se desintegra para convertirse en otro distinto, lo hace emitiendo
una partcula o , dependiendo de su naturaleza. (La emisin no corresponde a una des-
integracin sino que es posterior a ella y se debe a un ajuste energtico en el ncleo resul-
tante). Parece lgico identificar la actividad de una muestra radiactiva con el nmero de
partculas que sta emite por unidad de tiempo (A). De acuerdo con el razonamiento anterior
el valor numrico de esta magnitud coincidir con el nmero de desintegraciones producidas
por unidad de tiempo, es decir, con la velocidad de desintegracin (v) ya que cada desinte-
gracin supone la emisin de una partcula. Como el nmero de ncleos que se desintegran
en un tiempo dado es N = N0 - N, su variacin temporal ser:

v = dN/dt = d (N0 -N )/dt = -dN/dt

Vemos que el valor de v o nmero de desintegraciones por unidad de tiempo coincidir con
la variacin de N (nmero de ncleos que quedan) con el tiempo, que siempre es negativa
(ya que N decrece), cambiada de signo.

En adelante la ecuacin anterior tambin la expresaremos en la forma: A = -dN/dt

Qu relacin existir entre la actividad A de una muestra radiactiva y el nmero de to-


mos sin desintegrar presentes en un instante dado?

En principio, cabe esperar que para un nclido dado, cuanto mayor sea el nmero de tomos
de la muestra (sin desintegrar) en un cierto instante, mayor resulte la actividad A de la
muestra. Al transcurrir el tiempo, como el nmero de tomos decrece, tambin deber
hacerlo la actividad de la muestra. Para obtener la relacin operativa que liga a ambas mag-
nitudes haremos lo siguiente:

De la ecuacin N = N0 e t podemos obtener la derivada cambiada de signo:


-

v = -dN/dt = -N0 e t = -N y como A y v coinciden numricamente, concluimos:


-

A = N

En nuestro caso la actividad inicial de la muestra la determinaremos como:

A0 = N0 = (0693/T)N0 = (0693/5)108 =139 107 d/s (desintegraciones/segundo)

Cmo podremos calcular la actividad de la muestra al cabo de cierto tiempo?

Si en la ecuacin de desintegracin N = N0 e t multiplicamos en ambos miembros por :


-

N = N0 e t de donde: A = A0 e t
- -

-
y sustituyendo: A = 139107 e 01392 = 105107 d/s
Fsica nuclear 397

14. Cul es el periodo de semidesintegracin de una muestra radiactiva que en un


momento determinado da 3850 seales/s en un contador y que al cabo de 5 minutos da
solamente 2600 seales/s?

sol: T = 883 min.

15. Calculad la actividad de una muestra de 1 mg de radio sabiendo que el periodo de


semidesintegracin es de 1590 aos y su masa atmica 226 u.m.a. (El nmero de Avo-
gadro NA = 6022 10 23 tomos/mol).

La actividad inicial de una muestra de un nclido radiactivo se evala mediante la expresin


A0 = N0. Sin embargo, en este caso no nos indican el nmero de ncleos que existen en la
muestra sino su masa.

Cmo podemos obtener N0 ?

Podemos obtener en primer lugar el nmero de moles de ncleos mediante la expresin n =


m/M (m masa de la muestra y M masa molar) y a continuacin multiplicar por el nmero de
Avogadro NA para obtener el nmero total de ncleos presentes.

N0 = n0NA = (m0/M)NA

A0 = N0 = (m0/M)NA . Teniendo en cuenta que = 0693/T y sustituyendo:

0'693 0'001
A0 = 6'02 1023 = 1161015 d/ao
1590 226

16. Si el periodo de semidesintegracin del 146 C es de 5570 aos Cul es la actividad de


una muestra de 5 g de 146 C ? (NA = 6022 1023 tomos mol -1 )

sol: A0 = 271019 d/ao

83 Bi tiene un periodo de 605 minutos. Cuntos tomos se desintegran por se-


17. El 212
23
gundo en 50 gramos de 212 83 Bi ? (NA = 6022 10 tomos/mol)

sol: 27 1019 tomos.

18. Completad las siguientes reacciones nucleares:

13 Al 0 n 2 He
a) 27 11 Na 12 Mg 92 U 0 n 56 Ba 36 Kr
1 4
c) 24 24
e) 235 1 141 92

b) 126 C 42 He 48 Be 12 Mg 1 H 12 Mg
d) 26 1 26

Fsica nuclear 398

En este ejercicio se pide que completemos ciertas reacciones nucleares. Para ello bastar
con que tengamos en cuenta la conservacin del nmero de nucleones y de la carga (convie-
ne puntualizar que cuando aqu se habla de la carga nos estamos refiriendo siempre a la car-
ga nuclear). As pues:

13 Al 0 n 2 He Z X
a) 27 1 4 A

Conservacin del nmero de nucleones: 27 + 1 = 4 + A A = 24


Conservacin de la carga: 13 = 2 + Z Z = 11

24
Al tratarse de un ncleo con 11 protones, X corresponder a un istopo del sodio: 11 Na

b)
12
6
C 24He 48Be
En este caso, como se puede apreciar, se conserva ya el nmero de nucleones y la carga. Por
tanto el proyectil utilizado ha de ser un fotn ya que necesariamente deber de ser muy
energtico para provocar una reaccin nuclear.

c) 24 Na 24 Mg A X
11 12 Z

Conservacin del nmero de nucleones: 24 = 24 + A A = 0


Conservacin de la carga: 11 = 12 + Z Z = -1

Se trata de un proceso radiactivo en el que se emite un electrn1, es decir ZA X = 0


1 e.

d)
26
12
Mg 11H 1226Mg ZAX

En este caso es evidente que ZA X 11H y que, por tanto, solo habr modificacin en la canti-
dad de movimiento y energa de las partculas.

e)
235
92
U 01n 141
56
Ba 3692Kr ZAX

Conservacin del nmero de nucleones: 235 + 1 = 141 + 92 + A A = 3


Conservacin de la carga: 92 = 56 + 36 + Z Z = 0

Como no conocemos ninguna partcula que teniendo nmero atmico 0 tenga de nmero
msico 3, hemos de pensar que en este caso se trata de 3 neutrones (3 01 n ).

1
En realidad este proceso se explica admitiendo que un neutrn se transforma en un protn, un
electrn y una tercera partcula denominada antineutrino de acuerdo con la siguiente ecuacin:

1
0 n11 H 01 e (antineutrino).
Fsica nuclear 399

19. Completad las siguientes reacciones nucleares indicando el nombre, smbolo qu-
mico, nmero de nucleones y carga de cada partcula o nclido que ajuste cada reac-
cin:
a) 27
13 Al +
12
25
Mg + 42 He 31
c) 15 P 21 H 11 H
13 Al +
b) 27 30
15 P +
15 P 14 Si +
d) 30 30

20. Completad la siguiente reaccin nuclear: 27


13 Al 30
15 P

sol: un neutrn ( 01 n ).

21. Cmo podemos obtener energa en un proceso nuclear?

Sabemos que todo aumento o disminucin de energa interna de un sistema (la energa de
todo tipo medida por un observador que est en reposo respecto al centro de masas del sis-
tema, tambin llamada energa propia) va acompaada, respectivamente, de una ganancia o
prdida de masa, cuyo valor vendr dado por la ecuacin de Einstein:

Eo = mc2

Tanto en la formacin de un ncleo a partir de los nucleones que lo forman como en una
reaccin nuclear, las variaciones de masa y de la correspondiente energa interna que se
producen pueden ser muy grandes, lo que plantea la siguiente cuestin:

Cunta energa aprovechable se puede obtener en una reaccin nuclear?

En las reacciones nucleares se produce un cambio de identidad de las partculas, de manera


que, en algunas ocasiones, las masas de los productos de la reaccin son tan distintas de las
masas de los reaccionantes que la variacin de la energa propia del sistema es enorme.
Puesto que la energa total se conserva en un sistema aislado, la energa total antes de la
reaccin (de los reaccionantes) ha de ser igual a la energa total despus de la reaccin (de
todos los productos, incluyendo la posible radiacin). Es decir:

Eor + Ecr = Eop + Ecp

En la igualdad anterior Eo representa la energa propia, dada por Eo = mc2.

De modo que podemos expresarla como: mr c2 + Ecr = mp c2 + Ecp y de aqu obtener que:

Ec = Ecp - Ecr = (mr - mp)c2

As pues: siempre que en una reaccin nuclear la masa de los productos sea inferior a la
masa de los reaccionantes, la disminucin de energa debida a esta disminucin de masa se
transformar en un aumento de la energa cintica que, no olvidemos, es energa aprovecha-
ble para producir cambios. Esa energa cintica es la energa cintica de todos los productos
de la reaccin (ncleos y, en su caso, otras especies posibles como, por ejemplo, fotones).
Fsica nuclear 400

A la cantidad m = (mr - mp) se la denomina defecto de masa del proceso. (No confundir
con un incremento matemtico que siempre es final inicial).

As pues, siempre que el defecto de masa producido en una reaccin nuclear sea una canti-
dad positiva (es decir, que mr > mp), podremos obtener energa (reaccin exotrmica).

22. Hallad la energa en MeV correspondiente a un defecto de masa de 1 u.m.a.

Cuando se mide individualmente la masa de los nucleones (protones y neutrones) que for-
man un ncleo, la suma de la masas individuales de todos ellos es mayor que la masa del
ncleo constituido por ese mismo nmero de nucleones. Eso significa que en el proceso de
unin de los nucleones para formar el ncleo se ha debido desprender (mediante radiacin,
por ejemplo) una cantidad de energa igual a mc2, siendo m, la diferencia entre la masa
de todos los nucleones libres y la del ncleo que forman.

Qu habra que hacer para producir el proceso contrario (descomponer un ncleo en los
protones y neutrones que lo forman)?

Lgicamente, para descomponer el ncleo separando todos los nucleones que lo forman ser
necesario suministrar la misma energa que se ha desprendido en el proceso de su forma-
cin. Es por ello que a dicha energa se le denomina energa de enlace del ncleo en cues-
tin (cada uno se caracterizar por un valor de la energa de enlace, E b). As pues:

Energa de enlace = Eb = (m nucleones m ncleo)c2

Naturalmente, estas variaciones de masa no deben entenderse como que cambia el nmero
de nucleones, que siguen siendo los mismos aislados o integrando el ncleo.

Cmo se podra evaluar la variacin de energa cintica que acompaa a un defecto de


masa de una unidad de masa atmica?

Bastar manejar la expresin Ec = mc2 pero si queremos trabajar en el sistema interna-


cional de unidades deberemos expresar la unidad de masa atmica en kg. Para ello hemos de
recordar que 1 mol de tomos de 12 C posee una masa de 12 g y contiene el nmero de Avo-
gadro de tomos (6021023), de modo que:

1 u.m.a = 1210-3/126021023 = 166010-27 kg y sustituyendo

E= m0c2 = 166010-27 (2998108)2 = 149210-10 J y expresndolo en MeV:

E = 149210-10/160210-19 = 9'313108 eV = 9313 MeV

23. El 168 O tiene una masa atmica de 1599491 u.m.a Determinad su energa de enlace
y su energa de enlace por nuclen en MeV. Datos: masa del protn 100783 u.m.a;
masa del neutrn 100867 u.m.a; masa del electrn 000055 u.m.a.
Fsica nuclear 401

La energa de enlace de un ncleo, viene dada por:

Eb = (m nucleones m ncleo)c2

En ocasiones, por comodidad, se da como dato la masa total del tomo y no la del ncleo

Cmo podremos calcular en ese caso la energa de enlace?

Naturalmente, deberemos considerar la masa de los electrones y la masa total del tomo de
modo que:
Eb mp mn m e m at c 2

Para calcular el valor de Eb, hemos de hallar en primer lugar el defecto de masa producido
(en kg) y luego aplicar la expresin anterior.

m m m m
p n e at
= (8100783+8100867+8000055) -1599491= 014149 u.m.a

Dado que 1u.m.a = 166010-27 kg (ved ejercicio 22), el defecto de masa producido en este
caso ser: m = 0'14149166010-27 = 0'2348710-27 kg.

Y sustituyendo en la expresin de la energa de enlace:

Eb mp mn m e m at c 2 = 0'2348710-27 (2'998108)2 = 2'11110-11 J =

2'111 10 11 6
10 = 131'77 MeV
1'602 10 19

Tambin podramos haber obtenido la energa de enlace de una forma ms rpida, aplicando
directamente la relacin entre un defecto de masa de 1 u.m.a y la variacin de energa aso-
ciada al mismo en MeV (931'3 MeV), con lo que basta con realizar una multiplicacin:

Eb = 14149931'3 = 131'77 MeV

Para obtener la energa de enlace por nuclen hemos de dividir dicha energa entre el nme-
ro de nucleones que conforman el ncleo o nmero msico (A), de modo que:

Eb/A = 13177/16 = 824 MeV/nuclen.

98
24. La masa del ncleo de 42 Mo es 9788310 y las masas del protn y neutrn son respec-
tivamente 100727 y 100867 (todas ellas en uma). Hallad la energa (en MeV) despren-
dida en la formacin de un ncleo (Eb) a partir de sus nucleones aislados y en reposo.

Datos: una variacin de masa de 1 uma equivale a una variacin de energa de 931'5 MeV

sol: Eb = 845'58 MeV


Fsica nuclear 402

25. Al bombardear ncleos de Li con protones, se forman dos partculas . Calclese


la energa cintica en MeV de esas partculas. Datos (en u.m.a): masa del protn
100783, masa del Li 701601, masa del He 400260.

La ecuacin correspondiente al proceso nuclear del que nos hablan ser: 7


3 Li 11 H 2 42 He

En este proceso se produce una variacin de la masa que nos permitir conocer la variacin
de la energa cintica mediante la expresin ya manejada anteriormente:

Ec = (m r - m p )c2

En este caso:

m r - m p = (m Li + m H )- 2 m He = (701601 + 100783) - 2 400260 = 001864 u.m.a

La variacin de energa cintica producida se podr calcular sustituyendo en la expresin


anterior correspondiente a la variacin de energa cintica utilizando unidades internaciona-
les (teniendo en cuenta que 1 uma = 1660510-27 kg y c = 3108 m/s) .

Otra posibilidad es que, como ya hemos calculado la variacin de energa correspondiente a


un cambio de masa igual a 1 uma (9315 MeV), podemos hacer directamente:

Ec = 001864 931'5 = 17363 MeV

Si suponemos que la energa cintica de los reactivos es despreciable y que los productos de
la reaccin solo son las dos partculas que se forman, toda la energa anterior estar aso-
ciada al movimiento de esas dos partculas.

La energa anterior en julios es de solo 28 10-12 J, un valor muy pequeo, lo que plantea
una duda importante: A qu se debe entonces que se atribuya a las reacciones nucleares
una gran capacidad de obtencin de energa til?

El inters de las reacciones nucleares como proceso para obtener energa til radica en que
si bien la energa anterior es muy pequea, hay que tener en cuenta que corresponde a la
desintegracin de un solo tomo de Litio. Podemos plantearnos:

Qu energa se obtendra si se desintegrasen 10 moles (o 70 g) de 73 Li ?

En solo 70 g del istopo 73 Li hay ms de 61024 ncleos. Cuando se desintegre este nmero
de ncleos por bombardeo con protones se liberar una energa de:

Ec = 28 10-12 6'021024 = 1681013 J

en forma de energa cintica de las partculas . Podemos darnos cuenta de la enorme can-
tidad de energa que ello significa si pensamos que para obtener la misma energa quemando
carbn se necesitaran ms de 500000 kg.
Fsica nuclear 403

Aunque este tipo de procesos eran conocidos desde 1930 lo cierto es que no se han utilizado
para obtener energa ya que era mayor la energa que se precisaba para conseguir que se
produjera el proceso que la que se obtena a partir del mismo. As en el ejemplo analizado,
para romper un solo ncleo de Li haba que bombardear la muestra con ms de 1 billn de
protones (que previamente es necesario acelerar empleando una energa), por lo que en con-
junto se tena que gastar ms energa que la que se produca.

26. Cmo se explica que en un ncleo puedan permanecer a tan pequesima distan-
cia muchos protones, siendo que por tener todos ellos carga positiva se repelen?

En la naturaleza existen varios tipos de interacciones. Habitualmente se habla de interaccin


gravitatoria, electromagntica, nuclear fuerte y nuclear dbil. (En la actualidad la interac-
cin electromagntica y la nuclear dbil se han unificado en una sola llamada electrodbil).
Las interacciones gravitatoria y electromagnticas se conocen desde hace mucho tiempo y
han podido ser formuladas matemticamente. Sin embargo, la existencia de la interaccin
nuclear no se plante hasta ya iniciado el siglo XX cuando al investigar la estructura de los
tomos se comprob que en su ncleo coexistan protones a pequesimas distancias. Dado
que los protones tienen todos la misma carga elctrica (positiva) no se entenda cmo era
posible que a pesar de la intensa fuerza de repulsin electrosttica entre ellos (muchsimo
mayor que su atraccin gravitatoria) pudiesen permanece juntos en un espacio tan inmen-
samente pequeo. Esto sugera la existencia de otro tipo de interaccin que proporcionase
estabilidad a los ncleos atmicos.

En la actualidad sabemos que esa interaccin existe (nuclear fuerte), aunque todava no se
ha podido describir matemticamente, de modo que no es posible explicar todas las propie-
dades del ncleo mediante la interaccin nuclear. Entre los conocimientos que se tienen
sobre la interaccin nuclear fuerte conviene destacar los siguientes:

-Son fuerzas atractivas mucho ms intensas que las electromagnticas pero de un alcance
muchsimo menor de tal modo que para distancias superiores a 1 Fermi (10 -15 m) son prcti-
camente despreciables.

-Son independientes de la carga del nuclen, por lo que se manifiestan igualmente entre
protones que entre neutrones o que entre protones y neutrones.

-Solo se dan entre los miembros de una familia de partculas denominadas hadrones (proto-
nes, neutrones, mesones, etc.).

Teniendo en cuenta la existencia y caractersticas de la interaccin nuclear fuerte, resulta


inmediato comprender la estabilidad de los ncleos. No obstante, se presentan algunas ano-
malas, como el hecho de que cuando un ncleo tiene un nmero de neutrones demasiado
grande o demasiado pequeo respecto al nmero de protones, la fuerza nuclear se debilita y
esos ncleos resultan ms inestables (se produce su desintegracin dndose transformacio-
nes radiactivas).
Fsica nuclear 404

27. Una bomba atmica que contiene 20


kg de plutonio explota. La masa de los
productos de la reaccin es inferior en
una diezmilsima al valor de la masa
inicial. Qu cantidad de energa se libe-
ra en la explosin?

sol: 181014 julios.

28. Suponed que la fisin del 235U ocurre segn: 235U + n 137Te + 97Zr + 2 n y que pa-
samos de energas de enlace por nuclen en el 235U de 76 MeV a energas del orden de
86 MeV para los ncleos del 137Te y del 97Zr resultantes. Calculad la energa total libe-
rada en la fisin del ncleo del 235U.

Si representamos grficamente la energa de enlace por nuclen de los distintos nclidos


conocidos, en funcin del nmero de nucleones, se obtiene aproximadamente la funcin de
la figura adjunta. En ella se aprecia que hasta 20 nucleones la energa de enlace por nuclen
va aumentando muy rpidamente con el nmero de nucleones y que entre 40 y 100 nucleo-
nes presenta su mximo (unos 86 MeV) para, a continuacin decrecer lentamente.

Eb/A
(MeV)

0
0 100 150 200 A
El cambio de energa que se produce en una reaccin nuclear, viene dado por:

Ec = Ecp - Ecr = (mr - mp)c2 (1)

De acuerdo con dicha ecuacin, siempre que m rmp podremos obtener energa.
Fsica nuclear 405

No obstante, el cambio energtico en una reaccin nuclear tambin puede ser expresado en
funcin de las energas de enlace de reaccionantes y productos.

Sabemos que la energa de enlace viene dada por E b = (m nucleones m ncleo)c2.

Podemos hacer que en la ecuacin Ec = (mr - mp)c2 aparezca la energa de enlace si la


escribimos como:

Ec = (mr - mp + mnucleones - mnucleones)c2 Ec = (mnucleones - mp)c2 - (mnucleones - mr)c2

As pues:
Ec = Ebp - Ebr (2)

El resultado anterior permite comprender que en una reaccin nuclear cuando la energa de
enlace por nuclen de los productos sea mayor que la de los reactivos, se trate de una reac-
cin exotrmica en la que se producir un desprendimiento de energa. Siguiendo con este
razonamiento, la grfica anterior nos sugiere dos procedimientos posibles para obtener
energa utilizable Cules podran ser?

Puede pensarse en fraccionar ncleos muy pesados (bombardendolos con partculas lige-
ras) en otros ms ligeros cuya energa de enlace por nuclen sea mayor, o por el contrario,
en unir ncleos ligeros para formar otros medios de mayor energa de enlace por nuclen.
En la prctica se han conseguido llevar a cabo ambos procesos y se les ha denominado, res-
pectivamente, fisin y fusin nuclear.

La fisin se ha conseguido bombardeando ncleos muy pesados con neutrones trmicos


(poco energticos). Este proceso presenta el inconveniente de que los productos de la reac-
cin (residuos) son tambin radiactivos y pueden estar emitiendo radiaciones nocivas para
los seres vivos durante mucho tiempo, sin que se conozca la forma de evitarlo, por lo que se
recurre a encerrarlos en envolturas de hormign y depositarlos en minas o en fosas oceni-
cas, con el grave peligro que ello implica. Se trata, por tanto de una forma muy discutible de
obtener energa.

La fusin requiere elevadsimas temperaturas para producirse (hay que acercar ncleos lige-
ros a una distancia cortsima, venciendo la repulsin elctrica, para que acten las fuerzas
nucleares atractivas y se pueda producir la fusin), pero presenta la ventaja de que sus pro-
ductos no son radiactivos y que los reactivos (al contrario que en la fisin) son abundantes y
baratos. Este tipo de reaccin nuclear es el que se produce en las estrellas. La energa que se
obtiene es superior a la que se produce mediante la fisin y, lo que es ms importante, ms
limpia, aunque todava no se disponen de los medios tcnicos necesarios para llevar a cabo
el proceso de forma industrial.
235
En el ejercicio se pide la energa utilizable que se obtendr en la fisin de un ncleo de U
mediante la reaccin:

235
U 01 n 137 Te 97 Zr 2 01 n
Fsica nuclear 406

Para evaluar dicha energa, segn hemos explicado bastar con restar a la energa de enlace
de los productos la energa de enlace de los reactivos, de modo que:

Ec = Ebp - Ebr = (137+97)86 - 23576 = 20124 -1786 = 2264 MeV

29. Teniendo en cuenta los resultados de la actividad anterior, calculad la energa libe-
rada por 1 kg de 235U al fisionarse y las toneladas de carbn que se necesitan para libe-
rar la misma energa (1 kg de carbn de hulla proporciona al quemarse 8000 kcal).

sol: 96 1013 J y 2875 t respectivamente.

30. Determinad la masa en kg de 235U que se ha de fisionar para obtener una bomba de
1 Mt (106 toneladas de trilita) sabiendo que 1 Mt (megatn) equivale a 510 18 J.

Si sobre una muestra de 235 U se lanzan neutrones trmicos, se iniciar el proceso de fisin
segn el cual el ncleo se dividir en dos y los neutrones que se producen en estas primeras
fisiones (tres por trmino medio en cada una) provocarn a su vez nuevas fisiones dando
lugar a lo que se denomina una fisin en cadena en la que se desprende una gran cantidad
de energa.

No todos los neutrones que se liberan en la


fisin de cada ncleo, inciden en otros
ncleos (se trata de partculas sumamente
pequeas y muchas pueden pasar entre los
ncleos sin producirse ningn impacto), por
lo que siempre se producir una prdida de
neutrones que ser tanto menor cuanto mayor
sea el tamao de la muestra (ms ncleos
haya en el camino que recorran los neutro-
nes). De hecho, existe un tamao para el cual
el nmero de neutrones que se pierden por la
superficie es igual al de los neutrones que se
producen, lo que permitir que el proceso se
automantenga. Se le llama tamao o masa
crtica. Si la masa es superior se produce el
proceso en cadena, si es inferior se detiene.

En las centrales nucleares, se produce una reaccin nuclear de fisin en el reactor. Dicha
reaccin se controla mediante unas barras de un material que absorbe neutrones, haciendo
que la energa que se desprende no sea excesiva y se pueda extraer para su utilizacin. Por
el contrario, en una bomba atmica, se busca un desprendimiento de energa lo ms brusco
posible. Una forma de conseguirlo es utilizar dos masas de material fisionable inferiores el
valor crtico, pero que juntas lo superen, de forma que cuando se juntan (en el momento del
impacto) se desarrolle una reaccin en cadena totalmente descontrolada, producindose un
brusco desprendimiento de una cantidad de energa enorme que provocar una onda trmica
y de presin. Conviene tener en cuenta tambin que tanto en las centrales nucleares como en
Fsica nuclear 407

las bombas atmicas se producen grandes cantidades de residuos radiactivos que estarn
activos durante mucho tiempo y que se dispersan en grandes extensiones (con efectos muy
nocivos sobre los seres vivos por las radiaciones que emiten) siendo la eliminacin de estos
uno de los problemas ms graves que tiene planteados la humanidad.

En el ejercicio se pide qu masa de 235 U hay que fisionar para obtener una bomba cuyos
efectos destructores inmediatos (despus son todava peores: alteraciones genticas, mal-
formaciones, cnceres y contaminacin radiactiva en grandes reas, durante muchos aos),
sean equivalentes a los de la explosin de 1000 millones de kg de trilita (510 18 J).

Como en el ejercicio 28 hemos obtenido que con un ncleo de 235 U se obtienen 2264 MeV,
ser sencillo determinar el nmero de tomos necesarios para obtener 51018 J.

EcT N Ecatom 51018 = N2264106 1610-19 N = 138031029 tomos y para


conocer su masa, bastar multiplicar los moles de tomos por la masa molar M:

N 1' 3803 1029


m= M 235 103 = 53882 kg
NA 6' 02 1023

31. Calculad la energa liberada en la formacin de un ncleo de 4He a partir de dos


ncleos de deuterio. Qu energa se liberar al formarse por fusin 1 g de 4He a partir
de deuterio? (Eb/A = 1 MeV para el 2H y Eb/A = 7 MeV para el 4He). La masa de un
ncleo de deuterio es 2'01355 uma y la de un ncleo de helio es de 4'00150 uma. (Una
variacin de masa de 1 uma supone una variacin de energa de 931'5 MeV).

El proceso que tendr lugar es: 21 H 21 H 42 He

y la variacin de energa que se producir ser:

Ec m c 2 = (2m H - m He )c2 = 00256 c2

y si consideramos que una variacin de masa de 1 u. supone un cambio de energa de 9315


MeV, podemos escribir:

Ec = m 9315 = 00256 9315 = 238 MeV

Otra forma de resolver el ejercicio hubiera sido a partir de los datos de energa de enlace por
nuclen en el deuterio (1 MeV) y en el Helio (7 MeV). Cmo se procedera en ese caso?

Bastara con restar la energa de enlace de los reactivos de la reaccin, de la energa de enla-
ce de los productos, es decir:

Ec = Ebp - Ebr = 47- 41 = 24 MeV (el resultado no coincide totalmente con el anterior
porque los valores de las energas de enlace por nuclen utilizados son aproximados).

Cul sera la variacin de energa producida al obtener, mediante la reaccin anterior,


una masa de 1 g de Helio?
Fsica nuclear 408

Para realizar el clculo que se pide ser necesario conocer el nmero N de ncleos de 42 He
que hay en un gramo de dicha sustancia y luego multiplicar N por el valor de la variacin de
energa correspondiente a la formacin de uno de dichos nclidos, es decir:

Ec = N Ec y como N = n NA = (m/M) NA nos queda que:

Ec = (m/M) NA Ec = (1/4) 6021023 238 = 361024 MeV = 571011 J

Este incremento de energa cintica (til) proviene de la transformacin de parte de la


energa asociada a la masa que poseen los reaccionantes en energa cintica de los trillones
de ncleos de helio y fotones generados por la fusin de los mismos. Conviene darse cuen-
ta de la enorme cantidad de energa directamente aprovechable que se produce al formarse
slo 1 g de helio (ms de medio billn de julios) equivalente a la que se obtendra al quemar
unas 17 toneladas de carbn de hulla.

32. Comparad la energa obtenida en la fisin de 1 g de 235U con la obtenida en la fusin


de 2H y 3H para formar 1 g de 4He segn el proceso: 2H + 3H 4He + n + 176 MeV

sol: 96 1010 J y 42 1011 J (Se obtiene una energa 4 veces superior en la fusin).

33. En un hueso procedente de un animal que acaba de morir, la


velocidad de desintegracin del 146 C es de 15d/min por cada gramo
de este nclido. Por otra parte, sabemos que el periodo de semides-
integracin del 146 C es de 5730 aos. Teniendo en cuenta estos da-
tos, determinad la antigedad de un hueso que contiene 250 g de
14
6 C si su velocidad de desintegracin es de 400 de/min.

Como sabemos, las plantas verdes absorben el CO2 del aire mediante el proceso de la fo-
tosntesis. Los tomos de C de este compuesto son fundamentalmente de 126 C pero una pe-
14
quesima parte (aproximadamente uno por cada billn) son de 6 C que es radiactivo y se
desintegra.

La cantidad de 146 C en la atmsfera se mantiene estable, pese a que ste se va desintegrando,


porque los neutrones de la radiacin csmica al colisionar con los ncleos de los tomos de
nitrgeno presentes en la atmsfera hacen que se genere 146 C segn el proceso:

14
7 N 01n 146 C 11H

compensando as la desintegracin del carbono (14). Esto hace que la proporcin de este
istopo en los seres vivos (plantas y animales que las ingieren) se mantenga constante. Sin
embargo cuando un ser vivo muere deja de reponer 146 C , por lo que la proporcin de este
Fsica nuclear 409

nclido en sus restos va disminuyendo con el tiempo (al ir desintegrndose con un periodo
de 5370 aos) y, en consecuencia, tambin lo har su actividad.

Cmo es posible utilizar estos datos para conocer la antigedad de un fsil?

Sabemos que la actividad de una muestra radiactiva viene dada por la expresin:

A = A0 e-t

Si tomamos como instante inicial 0 s el correspondiente a cuando el organismo muri y


conocemos su actividad A0 en ese instante, podemos utilizar la ecuacin anterior para, co-
nocida su actividad presente poder calcular el tiempo transcurrido desde entonces.

En efecto, tomando logaritmos neperianos y despejando t obtenemos:

A0
ln
ln A/A0 = -t t A

En este ejercicio el hueso presenta una actividad A = 400 d/min. En el momento de morir el
ser al cual perteneca dicho hueso, su actividad deba ser:

A0 = 20015 = 3000 d/min.

Este primer resultado nos muestra que el hueso debe tener ms de 5730 aos (tiempo que ha
de transcurrir para que el nmero de nclidos de 14 C (y por tanto su actividad) se reduzca a
la mitad.

Por otra parte = 0693/T = 0693/5730 = 1209410-4 aos-1

Sustituyendo en la expresin de t anterior obtenemos as la antigedad del hueso:

A0 3000
ln ln
A = 400 ln 7'5
t = 16660 aos
12905
' 10 4
12905
' 104

Conviene tener en cuenta que la determinacin de la antigedad con ste mtodo slo es
factible para muestras orgnicas (huesos, tejidos, restos de plantas, etc.) de menos de unos
50000 aos. Ello se debe a que la cantidad de 14C presente en muestras anteriores a ese
tiempo es demasiado pequea como para poder hacer medidas aceptablemente precisas.

Para objetos ms antiguos, en ocasiones se recurre a otros radioistopos que tengan un pe-
riodo de semidesintegracin mucho mayor. ste es el caso, por ejemplo, del 238U ( cuyo pe-
riodo es T = 4510 millones de aos). Podemos pensar en la formacin de una roca hace mi-
llones de aos a base de la solidificacin de un material fundido en el que haba presente
una cierta cantidad del radioistopo anterior. ste es muy pesado y queda fijado a la roca
para lentamente ir produciendo una serie de desintegraciones hasta llegar al istopo estable
Fsica nuclear 410

del plomo Pb-206. Hallando la razn existente entre el Pb-206 y el U-238 los cientficos
pueden determinar aproximadamente los aos que hace desde que la solidificacin tuvo
lugar (instante t = 0). El U-238 antes de la solidificacin tambin produca Pb-206 pero ste
(ms ligero que el U-238) se escap del magma hirviente de modo que slo comienza a fi-
jarse en la roca a partir de que sta solidifica.

De esta forma se ha podido determinar que la edad de las rocas ms antiguas de la Tierra es
del orden de 4000 millones de aos. Tambin se ha podido comprobar que la edad de las
rocas ms antiguas que contienen fsiles procedentes de mamferos es de unos 200 millones
de aos. Todo ello suministra informacin muy valiosa respecto a cundo se form la Tierra
y cuando aparecieron determinadas especies de animales.

34. El 40
19 K radiactivo se desintegra trans-
40
formndose en 18 Ar estable. El contenido en
40
19K de ciertas rocas puede ser utilizado para
averiguar la fecha de su formacin (mientras
el material estaba fundido todo el Ar que se
40
produca por desintegracin del 19 K se esca-
paba, pero en cuanto se solidific dando lu-
40
gar a la roca, el 18 Ar fue quedando retenido
en ella).

Sabiendo que en una roca la proporcin de Ar/K actualmente es de 510 -3, determinad
si es posible que un dinosaurio tropezase alguna vez con esa roca. (Considerad que los
dinosaurios se extinguieron hace unos 60 millones de aos).

Para resolver este ejercicio supondremos que en el momento de formarse la roca haba N 0
tomos de 40 K y ninguno de Ar. Debido a la desintegracin que sufre el potasio radiactivo,
el nmero de nclidos de 40 K ir disminuyendo mientras que el de Ar ir aumentando ya
que por cada tomo de 40 K que desaparezca se formar un tomo de Ar. La ley de desinte-
gracin que gobierna este proceso es:

N = N0 e t
-

siendo N el nmero de tomos de potasio al cabo de un tiempo t.

Cmo podemos hallar el tiempo transcurrido desde que se form la roca?


Si consideramos que en el momento de formarse la roca haba N 0 tomos de 40 K y ninguno
de Ar, al cabo de un cierto tiempo t, nos quedarn N tomos de 40 K y se habrn producido
N0 - N tomos de Ar, de modo que la proporcin ser:

N0 N
que en la actualidad vale, segn se dice en el enunciado: 510 -3.
N
Fsica nuclear 411

Para conocer el tiempo t necesario para que se de esta proporcin podemos partir de las
ecuaciones:

N0 N
= 510-3
N

N = N0 e t
-

N0
De la segunda ecuacin, tomando logaritmos neperianos obtenemos: ln t
N
1 N0
De donde t ln .
N

N0
Despejando ahora en la primera ecuacin: 1 5 103 1'005
N

1 N0 1
Sustituyendo en la expresin anterior: t ln = ln1' 005
N

El clculo de la constante de desintegracin es sencillo a partir del periodo de semidesin-


tegracin ya que, como sabemos, = 0693/T = 0693/13109 = 5310-10 aos-1 con lo que:

1 N0 1
t ln = t ln1'005= 9'356 10 6 aos
N 5'3 10 10

Por tanto, ningn dinosaurio pudo tropezar nunca con la roca ya que esta se form mucho
despus de que se extinguieran.

35. Una muestra de madera contiene 10 g de C y posee una velocidad de desintegracin


del 14 C de 100 cuentas/min. Cul es la edad de la muestra?

sol: 13307 aos.

60
36. Qu utilidad tiene el 27 Co en medicina nuclear?

El dao que las radiaciones producen en los seres vivos se debe a la ionizacin que provo-
can al atravesar las clulas en las sustancias que las constituyen, que hace que el funciona-
miento normal de esas clulas se vea alterado. En general se pueden diferenciar dos casos:
a) La irradiacin no es demasiado intensa: Las clulas afectadas seguirn viviendo aunque
sern defectuosas y al reproducirse de forma anmala pueden provocar procesos cancerosos.
Fsica nuclear 412

b) La irradiacin es muy intensa: Las clulas irradiadas mueren. Esto, en s mismo, puede
no ser importante ya que el organismo tiene capacidad para reemplazarlas, aunque si la do-
sis afecta a un nmero demasiado elevado de clulas puede suceder que el organismo no
pueda recuperarse y muera.

Los seres vivos estamos sometidos a radiaciones de diverso origen: csmico, terrestre, algu-
nas pruebas mdicas, contaminacin radiactiva procedente de reacciones nucleares artificia-
les (centrales, bombas, etc.) y si el nivel de radiacin aumenta demasiado podemos padecer
graves enfermedades (por ejemplo, cncer de piel debido a las radiaciones solares). No obs-
tante, algunas radiaciones se utilizan, precisamente, para combatir el cncer. Para ello se
recurre a irradiar las clulas cancerosas con radiacin muy energtica con el fin de destruir-
las (el problema est, actualmente, en que tambin se destruyen otras sanas).

La radiacin ms utilizada con fines mdicos por su poder de penetracin es la radiacin


60
(muy energtica). Para su obtencin suele utilizarse como fuente el 27 Co (que se produce
fcilmente en los reactores nucleares segn el proceso: 59
27 Co 01n 27
60
Co ). Este nclido es
radiactivo y se descompone segn:

60
27 Co 28
60
Ni 01 e

emitiendo cada ncleo un fotn , de gran poder de penetracin, y un electrn.

-2
88 Ra produce al cabo de 1 da 10810
37. Una muestra de 100 g de radio 276 cm3 de
helio (medido en condiciones normales de presin y temperatura). Determinad el valor
del nmero de Avogadro NA, sabiendo que 1 g de 276 88 Ra emite 3410
10
partculas
cada segundo.

El proceso vendr descrito por la ecuacin:

226
88 Ra 222
86 Rn 42 He .

Como conocemos el volumen de helio obtenido en condiciones normales, podemos saber


fcilmente el nmero de moles de ncleos de He producidos en 1 da.

Sabemos que el nmero de Avogadro NA ser el nmero de ncleos de He presentes en una


mol de dicha sustancia. Por tanto N = n NA siendo n el nmero de moles de ncleos y N el
total de ncleos.

Cmo podemos obtener NA?

De la relacin anterior est claro que NA = N/n de modo que lo que hay que hacer es deter-
minar N y n.

Como 1 mol de cualquier gas, en condiciones normales, ocupa un volumen de 224 l, el


nmero de moles n de ncleos de He que se producirn al da ser:
Fsica nuclear 413

V(l)
n 1
= 10810-5/224 = 48210-7 moles de 42 He
22' 4(l mol )

Por otra parte, como en el enunciado nos dicen que 1 g de Ra emite 3410 10 partculas
cada segundo, en un da (86400 s) 100 g emitirn:

N = 100341010 86400 = 29381017 partculas (ncleos de He).

Sustituyendo en la expresin de NA obtenemos:

NA = N/n = 29381017/48210-7 = 6091023 partculas/mol

38. El Sol radia energa con una potencia aproximada de


41026 W. Admitiendo que el proceso se debe a la conversin
de 4 protones en Helio segn el siguiente proceso:

411 H24 He 2 01e 26' 7MeV

Cunto tiempo tardara el Sol en agotarse si continuara radiando a ese ritmo? (Admi-
tid que los protones constituyen la mitad de la masa del Sol y que sta vale aproxima-
damente 21030 kg).

La potencia nos proporciona el valor de la energa emitida por el Sol cada unidad de tiempo.
Si suponemos que la potencia se mantiene siempre constante, podemos calcular su valor
como P = E/t de modo para saber en cuanto tiempo se emitir una cierta cantidad de
energa, bastar despejar el tiempo de la ecuacin anterior: t = E/P.

Cmo podemos saber el tiempo en que tardara el Sol en dejar de irradiar de acuerdo con
las condiciones expuestas en el enunciado?

Si conocemos la masa de hidrgeno existente en el Sol (1030 kg) podemos calcular el nme-
ro de ncleos de hidrgeno existentes y a partir de este obtener la energa total que podr
emitir, correspondiente a las reacciones de fusin de todos esos ncleos de hidrgeno (espe-
cificada en el enunciado). Una vez que tengamos este dato, bastar dividir por la potencia,
para tener el tiempo que se pide.

En efecto: El nmero de nclidos 11 H ser:

m
N = nNA = NA
M

en donde, como es habitual, n sern los moles de H, M su masa molar y NA el nmero de


Avogadro.
Fsica nuclear 414

N
La energa total que se podra liberar sera por tanto: E T = E
4

En la ecuacin anterior E es la energa de una de las reacciones de fusin (267 MeV) y N se


divide por 4 porque en cada reaccin participan 4 ncleos de H.

Sustituyendo en la expresin t = ET/P obtenemos que:

m 1030 103
NA E ' 1019
6'02 1023 26'7 106 16
t M = 1 =161017 s = 51109 aos
4P 4 4 1026

As pues, de acuerdo con los clculos anteriores al Sol le quedaran todava unos 5000 mi-
llones de aos de vida.
ANEXO: VECTORES

1. Determinad grficamente los vectores compo-



nentes de s , segn las direcciones de la figura:
s

Se entiende por vectores componentes de un vector dado, el conjunto de vectores cuya suma
da como resultado dicho vector. De acuerdo con esto, un vector cualquiera admitir ser ex-
presado como suma de otros vectores, de infinitas formas distintas. As, por ejemplo, el vec-

tor h de la figura adjunta, se puede expresar como:

hab h
c
b
hcde e
a
d

y de tantas otras formas como queramos imaginar, con la nica condicin de que la suma de

los vectores seleccionados de h .

Quiere esto decir que, en el caso que se plantea en este ejercicio, el vector s puede ser
descompuesto en infinitas formas distintas?

Digamos, en principio, que si, pero dmonos cuenta que en el enunciado se aaden condi-
ciones que, sin duda, restringen estas posibilidades. En efecto, como nos sealan dos direc-
ciones, ello nos limita a que sean dos vectores cuyas direcciones estn predeterminadas, de
modo que, como podr comprenderse, slo habr una forma posible.

Cmo determinar los dos vectores?

Podemos aplicar la regla del paralelogramo que consiste en colocar los dos vectores (sin
cambiarlos) con origen comn y trazar la diagonal del paralelogramo que forman:
416 Vectores

Como en nuestro caso conocemos ya el vector



suma s y las direcciones de los vectores busca-

dos, bastar trazar desde el extremo de s las para-
lelas a las direcciones que nos dan y buscar los
puntos de corte con ellas, de esta forma tendremos
definido el paralelogramo que determina los dos

vectores s1 y s2 cuya suma es el vector s dado.

2. Determinad las compo-


nentes cartesianas de los vec-
tores representados en la fi-
gura adjunta:

sol: a (3,4,-5); b (2, 6, 0);


c (0, -2, -3); d (0,-2,0).

3. En el paralelogramo de la figura adjunta se conocen las coordenadas de los vrtices:


A (-1,3,2) m; B (2,-1,5) m y C (0,-3,-4) m. Determinad las coordenadas del vrtice D.

ste es un ejemplo de ejercicio de aplicacin del lgebra vectorial a la resolucin de un pro-


blema que, en principio, nada tiene que ver con vectores.
Vectores 417

Para obtener las coordenadas del vrtice D, podemos considerar la pareja de vectores AB y
CD que se caracterizan por tener igual mdulo, direccin y sentido, es decir: AB = CD. Esto
hace que ambos vectores tengan las mismas componentes escalares. Por otra parte, las com-
ponentes escalares de cualquier vector pueden obtenerse restando a las coordenadas de su
extremo las de su origen, por lo que realizando esta operacin con los vectores AB y CD e
igualando, podremos determinar las coordenadas del vrtice D.

CD = (Dx-Cx) i + (Dy-Cy) j + (Dz-Cz) k ; AB = (Bx-Ax) i + (By-Ay) j + (Bz-Az) k ,
como CD = AB, las componentes escalares han de ser las mismas, por lo que podemos igua-
larlas y despejar:

Dx - Cx = Bx - Ax Dx = Bx - Ax + Cx = 2 + 1 = 3 m
Dy - Cy = By - Ay Dy = By - Ay + Cy = -1 - 3 - 3 = -7 m
Dz - Cz = Bz - Az Dz = Bz - Az + Cz = 5 -2 - 4 = -1m
Por tanto, las coordenadas del punto D son (3, -7, -1) m.

4. Cmo podemos hallar los ngulos directores (ngulos que un vector forma con ca-
da uno de los ejes) cuando expresamos un vector en sus componentes cartesianas?

Pensemos, por ejemplo, en la posicin de un objeto que se desplaza por el espacio. En ese

caso podemos localizarlo mediante un vector de posicin r cuyas componentes escalares
cartesianas van cambiando con el tiempo. La representacin grfica es la siguiente:

Z(m
) z


r x2 y 2 z 2
r
k

j
i y Y(m
)
90
x

X(m) x2 y 2

En la figura anterior se ha representado el vector de posicin de un mvil en un instante


dado, en los ejes de coordenadas cartesianas X, Y, Z. Tambin se han representado los vec-

tores unitarios i , j , k . Dichos vectores tienen de mdulo 1, su origen coincide con el origen
418 Vectores

de coordenadas y siempre tienen la direccin de los ejes X. Y, Z respectivamente (y sentido


positivo).

El anlisis de la figura anterior permite comprender fcilmente (basta aplicar el teorema de



Pitgoras) que r x 2 y 2 z 2 .

Por otra parte, el vector r se puede expresar en funcin de sus componentes escalares carte-

sianas (que pueden ser positivas o negativas), simplemente como: r = (x , y , z).

Tambin se puede expresar en funcin de sus vectores componentes como: r = x y z
o bien (lo que es equivalente) en funcin de los vectores unitarios como:

r = x i y j z k (siempre acompaado de la unidad utilizada).

En cuanto a la direccin del vector, sta (al igual que ocurra cuando nos limitbamos a re-
presentar los vectores en el plano XY), se expresa mediante los ngulos directores (, , ).
En la figura siguiente hemos representado tales ngulos:

Z(m)
z

90

r

k
90

y Y(m)
i j

X(m)

x y z
De la figura anterior queda claro que: cos ; cos ; cos
r r r


O lo que es equivalente: x r cos ; y r cos ; z r cos

Naturalmente, todo lo que hemos dicho aqu respecto al vector de posicin r , es tambin
vlido para cualquier otro vector que se represente en los ejes cartesianos.
Vectores 419

F3 = 15 N
5. Calculad la fuerza resultante del conjunto de
fuerzas coplanarias representadas en la figura
adjunta.
110
F1 = 10 N
45

F2 = 7 N

En este problema, nos dan una serie de vectores


expresados
en mdulo-direccin
y nos pi-
den que calculemos el vector suma Fres F1 F2 F3 . Una forma de hallar Fres sera
sumar
grficamente los tres vectores, con lo que podramos ver la direccin y sentido de Fres . Des-
pus, para conocer el mdulo de Fres , bastara medir la longitud del vector y compararla con
la unidad utilizada al representar los vectores dados. Sin embargo, este procedimiento, no es
muy exacto, ya que estara afectado por la imprecisin con que hubisemos dibujado los
vectores y medido la longitud de los mismos.

Para superar el inconveniente anterior, deberemos sumar los vectores analticamente. Para
ello, es necesario expresar, previamente, los vectores en componentes. Si no nos dan, como
ser habitual, un sistema de coordenadas cartesianas al que podamos referirnos, ser conve-
niente escoger uno tal que el clculo de las componentes de los vectores sea lo ms sencillo
posible (lo que se puede lograr haciendo que sus ejes coincidan con el mayor nmero posi-
ble de vectores). As en este caso podramos hacer:

Ahora, fijndonos en los ngulos que cada vector forma con los semiejes positivos (ngulos
directores), podemos expresar fcilmente cada uno de los tres vectores de forma analtica,
en componentes cartesianas:

F1 F1x i F1y j F1z k F1 cos 0 i F1 cos 90 j F1 cos 90 k

F2 F2 x i F2 y j F2 k F2 cos 45 i F2 cos 135 j F2 cos 90 k

F3 F3x i F3y j F3 k F3 cos110 i F3 cos 20 j F3 cos 90 k
420 Vectores

Sustituyendo y operando nos queda que:



F1 10 i N ; F2 4'95 i 4'95 j N ; F3 5' 13 i 14' 10 j N

Fres F1 F2 F3 = 9'82 i 9'15 j N

Conviene sealar que para expresar analticamente el vector F1 , en realidad, no nos haca
falta ningn clculo ya que al encontrase sobre un eje, resulta inmediato conocer cules son
sus componentes. Cuando esto no es as (como ocurre, por ejemplo, con los otros dos vecto-
res) hay que determinar las componentes escalares haciendo uso de las funciones trigo-
nomtricas. A este respecto, hemos de resaltar que, aunque por encontrarnos en un plano
sera posible haber trabajado conel seno y el coseno de un nico ngulo, p.e. 45 en el caso
de F2 y escribir: F2 F2 cos 45 i F2 sen 45 j N, ste procedimiento no tendra utilidad en
el espacio, por lo que hemos optado por trabajar nicamente con la funcin coseno de cada
ngulo que el vector forma con los semiejes positivos (cosenos directores). ste procedi-
miento, adems de su mayor generalidad, ofrece la importante ventaja de que al calcular las
componentes escalares del vector, stas se obtienen con el signo correspondiente. Finalmen-
te, sealar que podramos haber prescindido de calcular la tercera componente (Fz), ya que,
al encontrase todos los vectores en el plano XOY dicha componente debe ser nula, y que
aunque en este primer problema no lo hemos hecho para facilitar la comprensin del proce-
dimiento general, s lo haremos de aqu en adelante.

Cmo deberamos proceder para expresar el vector resultante en mdulo y direccin?

Para obtener el mdulo basta aplicar la expresin: Fres Fres


2
x Fres y , es decir:
2

Fres 9'822 9'152 18015


' 13'42 N

En cuanto a la direccin y sentido, utilizaremos los cosenos directores (recordemos que es


el ngulo que forma el vector con el semieje Ox+, el que forma con el semieje Oy+ y fi-
nalmente es el ngulo que forma con el semieje Oz+ (en los tres casos yendo por el camino
ms corto).

Fres x 9 '82
cos = 0'73 ; de donde = arc cos 073 = 43
Fres 13'42

Fresy
cos 0'68; de donde = arc cos 0'68 = 47
Fres

Fresz
cos = 0; de donde = arc cos 0 = 90
Fres

As pues: La resultante
de las tres fuerzas representadas viene expresada por medio del vec-
tor Fres = 9'82 i 9'15 j N , su mdulo vale Fres = 1342 N y se halla situado en el plano
Vectores 421

+ + +
XOY, formando un ngulo = 43 con Ox , de = 47 con Oy y de = 90 con Oz (ngu-
los directores).


6. Dado el vector a = 2 i + 4 j + 3 k , se pide:

a) Dibujad dicho vector en un sistema de coordenadas cartesianas.


b) Calculad su mdulo.
c) ngulo que forma con cada uno de los ejes.
d) Obtened un vector de mdulo 5 que tenga su misma direccin y sentido.

10 20 15
Sol: b) a = 29 ; c) = 682, = 420, = 561; d) , ,
29 29 29

7. Un cuerpo de 2 kg de masa permanece en equili-


brio en la situacin de la figura adjunta. Determinad
el valor de la tensin de la cuerda.


sol: T ( 0, 20) N (segn un sistema de coordenadas en
el que el eje Y coincida con la vertical y sentido positivo
hacia arriba).

8. Si al cuerpo
del ejercicio anterior se le aplica una
fuerza F horizontal, pasa a ocupar la situacin de
equilibrio que se aprecia en la figura de la derecha
Qu valores presentan tanto la tensin de la cuerda
como la fuerza aplicada? 30
F

En la situacin representada en la figura, la esfera est interaccionando con la Tierra, la


cuerda y el agente exterior que ejerce la fuerza horizontal. Como consecuencia de esas in-
teracciones sobre la esfera actan las siguientes fuerzas:

-El peso P de la esfera o fuerza gravitatoria con que la Tierra la atrae.
-La tensin T de la cuerda. Debido al peso de la esfera, a la fuerza aplicada y a que la cuer-
da se halla sujeta por un extremo del techo que permanece fijo, dicha cuerda se pone tensa y
ejercer una fuerza sobre
la esfera, a la que llamaremos tensin.
-La fuerza horizontal F . (Ejercida por un agente externo sobre la esfera).

Que tendr que ocurrir con la fuerza resultante que acta sobre la esfera?

Dado que la esfera permanece en equilibrio, la resultante de todas las fuerzas que se ejercen
sobre la misma debe de ser nula, es decir:
422 Vectores


Fres P T F 0

Cmo podramos calcular, a partir de aqu, los valores F y T?

En principio, podra parecer que la ecuacin anterior es insuficiente para poder resolver el
problema ya que hay dos incgnitas. Sin embargo, conviene tener en cuenta que, como
ecuacin vectorial que es, se puede descomponer en tres ecuaciones escalares, de donde
podramos despejar T y F. Para ello, lo primero que hemos de hacer es elegir un sistema de
coordenadas apropiado y a continuacin expresar los vectores fuerza en funcin de sus
componentes y proceder a sumar analticamente:

30

T
F
X
P

P = (Px, Py) = (P cos 90, P cos 180) = (0, -20) N

T = (Tx, Ty) = (T cos 120, T cos 30) = (-05 T, 087 T) N

F = (Fx, Fy) = (F cos 0, F cos 90) = (F, 0) N

Sumando las tres ecuaciones anteriores obtendremos la expresin de la fuerza resultante que
acta sobre la esfera, que como ya hemos dicho ha de valer 0. Por tanto:

Fres = (-05 T + F, -20 + 087 T) = 0

de donde se obtiene un sistema de dos ecuaciones con dos incgnitas:

0'5 T F 0 20
Resolviendo sale que T = 23 N y que F = 0'5 T = 11'5 N
20 0'87 T 0 0'87


9. Si la fuerza F presente en el ejercicio anterior tuviese un mdulo de 23 N cul sera
el ngulo de equilibrio?

sol: 4880
Vectores 423

10. Si un cuerpo de 2 kg de masa se suspende del techo mediante dos cuerdas de forma
que queda en equilibrio en la situacin de la figura cul es el valor de la tensin de
cada cuerda?

20 30
sol: T1 = 24,5 N ; T2 = 22,7 N

11. Un automvil es arrastrado haciendo uso de dos cuerdas, segn se aprecia en la


figura. Determinad el valor T2 sabiendo que T1 vale 500 N y la fuerza resultante R vale
800 N.

sol: T2 = 3424 N


a
12. Los vectores representados en la figura adjunta tienen
de mdulo: a = 6 y b = 3 (ambos en unidades internaciona-

les). Sabiendo que el tringulo es equiltero, determinad el b

vector suma s a b . Expresad el vector suma en mdulo
y direccin.

En este caso se trata, de nuevo, de sumar varios


vectores que se dan expresados en mdulo y
direccin. Para ello, conviene establecer un
sistema de referencia apropiado que nos permi-
ta expresar los vectores en componentes carte-
sianas para poder sumarlos analticamente.

Observemos que como el tringulo es equilte-


ro, los dos vectores forman un ngulo de 60
con OX+. Conociendo este dato y el mdulo de
dichos vectores, es fcil obtener sus componen-
tes cartesianas.
424 Vectores

Observemos que como el tringulo es equiltero, los dos vectores forman un ngulo de 60
con OX+. Conociendo este dato y el mdulo de dichos vectores, es fcil obtener sus compo-
nentes cartesianas.

Otra cuestin de inters es prever la orientacin del vector suma s a b .

Dada la simetra existente en la figura es fcil darse cuenta que si ambos vectores tuviesen
el mismo mdulo, el vector suma debera encontrarse
sobre el eje X y estar dirigido hacia la

derecha. Como el vector a es mayor que el b , hemos de pensar -antes de hacer ningn

clculo- que el vector suma s a b tendr una orientacin ms prxima al primero que al
segundo.

a = (ax, ay) = (a cos 60, a cos 30) = (605, 6087) = (3, 522) U.I.

b = (bx, by) = (b cos 60, b cos 150) = (305, - 3087) = (15, -261) U.I.

s a b = (3, 522) + (15, -261) = (45, 261) U.I.

Cmo podramos calcular el mdulo y la direccin del vector suma?



Como conocemos sus componentes, para calcular el mdulo de s aplicamos la

expresin s sx2 sy2 con lo que nos queda que s 4'52 2'612 = 52 U.I.

En cuanto a su direccin, podemos determinarla calculando los ngulos directores:

s x 4'5
cos 0'87 de donde = arc cos 0' 87 = 30
s 5' 2

sy 2' 61
cos 0'5 de donde = arc cos 0' 5 = 60
s 5' 2

As pues el vector suma s a b es el vector s 4'5, 2'61 U.I. se encuentra en el plano

XOY, forma un ngulo de 30 con el semieje OX+ y de 60 con OY+ y su mdulo es de 52 U.I.

13. Un cuerpo de masa m se desliza libremente por un plano inclinado que forma un
ngulo con la horizontal. Sabiendo que no existe rozamiento, expresad la fuerza
ejercida por el plano sobre el cuerpo y la fuerza resultante que acta sobre el mismo,
en funcin de su peso P y de .

Para resolver este problema conviene realizar en primer lugar un esquema en donde figuren
todas las fuerzas que actan sobre el cuerpo cuando este se desliza por el plano, junto con
Vectores 425

un sistema de referencia que nos permita expresarlas fcilmente en funcin de sus compo-
nentes cartesianas.

Debido a la interaccin existente entre el cuerpo y la superficie sobre la que este desliza,
dicho cuerpo ejerce una fuerza N sobre el plano y a su vez el plano ejerce otra fuerza de
igual valor y sentido contrario R sobre el cuerpo. Se trata de una pareja accin-reaccin, en
la que cada fuerza se ejerce sobre un cuerpo distinto y ambas se encuentran sobre la misma
lnea de accin (perpendicular al plano), aunque en el dibujo hemos desplazado N para ma-
yor claridad.

A causa de la interaccin entre el cuerpo y la Tierra, sta ejerce una fuerza de atraccin P
sobre aqul y el cuerpo ejerce otra del mismo valor y sentido contrario sobre la Tierra (que
no hemos dibujado).

As
pues,
en la situacin descrita en el enunciado, sobre el cuerpo slo actan dos fuerzas:
R y P . Dado que la trayectoria que sigue el cuerpo es rectilnea, la fuerza resultante que
acta sobre l tiene que ser en la direccin del plano. Analizando la figura, se comprende
que dicha fuerza estar dirigida hacia la base del plano (sin importar que el cuerpo est su-
biendo o bajando) y que es la responsable de la aceleracin que el cuerpo experimenta (as-
cienda cada vez ms despacio o descienda cada vez ms aprisa).

Podemos calcular la fuerza resultante que acta sobre el cuerpo haciendo: Fres P R . En

la ecuacin anterior, conocemos P en mdulo y direccin, aunque de Fres y R slo conoce-
mos su direccin. En consecuencia, las incgnitas sern Fres y R, que es lo que vamos a tra-
tar de poner en funcin de P y , tal y como se nos pide en el problema. Para ello expresa-
remos los vectores fuerza en forma analtica (segn el sistema de referencia escogido) y
procederemos a calcular la expresin de la fuerza resultante:
426 Vectores


P P cos i P cos (90 ) j

R R cos 90 i R cos 0 j R j

Fres Fres i

Teniendo en cuenta que los ngulos y son complementarios (sumados dan 90), pode-
mos escribir que cos = sen . Por otra parte al ser los ngulos (90 + ) y , suplementa-
rios (sumados dan 180), podemos escribir que cos (90 + ) = -cos , con lo que nos queda:

P P sen i P cos j y sumando con R , nos sale:

Fres P R P sen i R P cos j Fres i . Descomponiendo la igualdad vectorial
en dos escalares, tendramos:

Psen = Fres
R- Pcos = 0

De acuerdo con lo anterior: Fres = P sen y R = P cos

Al mismo resultado se puede llegar considerando directamente que al no haber aceleracin


en la perpendicular a la superficie del plano, la fuerza que tira del cuerpo segn la normal y
hacia arriba, ha de equilibrarse con la que tira del cuerpo en la misma direccin y hacia aba-
jo (y por tanto R ha de valer lo mismo que P cos ), de manera que el valor de la fuerza re-
sultante deber, en este caso, coincidir siempre con el de la componente segn el plano de la
fuerza peso (P sen ).

14. Al cuerpo A de la figura se le aplica una fuerza F, vertical y hacia arriba de 10 N.


Dibujad cada una de las fuerzas que actan sobre los cuerpos A, B y M y determinad
sus valores (mA =25 kg, mB =3 kg y mM = 4 kg).

Para dibujar cada una de las fuerzas que actan sobre los cuerpos de la figura, hemos de
analizar cules son las interacciones ms relevantes en las que intervienen. Considerando el
sistema formado por A, B, M y la Tierra, podemos escribir:
Vectores 427

a) Interacciones entre los cuerpos


y la Tierra: El cuerpo A es atrado por la Tierra gravi-
tatoriamente con una fuerza PA (peso de A) y a su vez ste atrae a la Tierra con otra fuerza
de igual valor y sentido contrario cuyo origen podemos situar en el centro del planeta.

Mediante consideraciones
similares, podemos justificar la existencia de las restantes fuerzas
gravitatorias PB y PM (ved figura siguiente).

b) Interacciones
(no gravitatorias) existentes entre los cuerpos: El cuerpo A ejerce una
fuerza N BA sobre el cuerpo B y ste hace otra fuerza RAB sobre el A. Ambas se encuentran
sobre la misma lnea de accin aunque a menudo (como se ha hecho aqu) se separe una de
ellas para poder distinguirlas con claridad, tienen el mismo mdulo y sentidos contrarios.

La fuerza N BA (fuerza que sobre B hace A) es el peso PA ?

Mucha gente pensara que la fuerza que hace el cuerpo A sobre el B es el peso PA . Aunque
en los problemas sobre dinmica volveremos a tratar esta cuestin, podemos adelantar ya
que se trata de dos fuerzas distintas (en ciertos casos, como el que nos ocupa, ni siquiera
tienen el mismo valor) ya que corresponden a dos interacciones de diferente naturaleza:

PA es la fuerza con que la Tierra atrae al cuerpo A, es de naturaleza gravitatoria y acta
sobre el cuerpo A.

N BA es la fuerza que sobre B ejerce el cuerpo A, no es gravitatoria (aunque A y B se atraen
gravitatoriamente, esa fuerza es despreciable frente a las restantes y se ignora) y se ejerce
sobre el cuerpo B.

El cuerpo B, interacciona no solo con el A sino tambin con la mesa M, de manera que la
fuerza que sobre M hace
B hade tener el mismo valor y sentido contrario que la que sobre
B hace M, es decir: N MB RBM . La mesa M interacciona tambin sobre el suelo de modo

que podemos escribir: N SM RMS (fuerza que sobre el suelo hace la mesa igual y de senti-
do contrario que la fuerza que sobre la mesa hace el suelo).

En la figura que viene a continuacin hemos representado (no a escala) las fuerzas que aca-
bamos de enumerar, incluyendo tambin un sistema de coordenadas cartesianas para expre-
sar analticamente los vectores representativos de dichas fuerzas:
428 Vectores

Cmo podemos calcular el valor de las fuerzas que actan sobre cada cuerpo?

Teniendo en cuenta que A, B, y M permanecen en reposo, podemos concluir que la fuerza


resultante que acta sobre cada uno de esos cuerpos ha de ser nula. Expresando los vectores
representativos de las fuerzas en funcin de sus componentes y procediendo a sumar analti-
camente e igualar a cero, podemos calcular los valores que no conozcamos de dichas fuer-
zas:

Sobre A: F RAB PA 0 de donde RAB F PA ( 10, 0) ( 25, 0) (15, 0) N

(Luego N BA RAB (15 , 0) N , que como puede verse es distinta que PA )

Sobre B: RBM PB NBA 0 de donde RBM PB NBA ( 30, 0) ( 15, 0) ( 45, 0) N

Sobre M: RMS PM N MB 0 RMS = - PM - N MB ( 40, 0) ( 45, 0) (85, 0) N

Analizando los resultados anteriores nos damos cuenta que el valor de la fuerza que sobre A
hace B, es de 15 N (inferior a su peso) y que dicha fuerza sumada a la fuerza exterior de 10
N, es la que equilibra al peso del cuerpo A (25 N), de forma que la resultante de todas las
fuerzas que actan sobre A es 0 (lo cual explica que si estaba en reposo, contine estndo-
lo). Anlogas consideraciones podemos hacer respecto a B y respecto a M.

Cul es la fuerza que la mesa ejerce sobre el suelo?


Vectores 429

De acuerdo con el tercer principio de la dinmica (o principio de accin y reaccin), la fuerza


que ejerce M sobre el suelo ha de tener el mismo valor que la que el suelo ejerce sobre M y
como esta ltima la conocemos (85 N), concluimos que la mesa hace sobre el suelo una fuerza
que vale 85 N (que, como podemos ver, es mayor que el peso de la mesa, que vale 45 N).

15. En la figura adjunta los vectores tienen de mdulos a = 9107 y b = 3107 (ambos en
unidades internacionales). Obtened el vector suma, su mdulo y ngulos directores.

6 cm a

8 cm X


sol: s (7'2 10 7 , 2'4 10 7 ) U.I.; s = 76 107 U.I.; = 182 ; =1084 ; = 90


16. Dado el vector a 3i 2 j 4k calculad el vector h 6 a y comprobad que

h = 6a y que la direccin y sentido de h coinciden con las de a .

Sol: h 18i 12 j 24k

17. Dado el vector a representado en la figura adjunta, determinad sus componentes
sabiendo que su mdulo vale 15. (Todos los datos en unidades internacionales).
430 Vectores

En este problema nos piden que calculemos las componentes de un vector, del que conoce-
mos su mdulo, sabemos que se encuentra en el plano XOZ y que tiene la direccin y senti-
do indicados en el dibujo.

Cmo se pueden calcular las componentes del vector a ?
Una posibilidad es utilizar las expresiones a x a cos y a z a cos (ay = 0). Para
poder aplicar este mtodo tendramos que conocer el valor de los ngulos directores a partir
de los datos que se nos dan en el problema y esto, en general, en el espacio presenta cierta
dificultad.

Otra forma, ms cmoda, de enfocar el problema consiste en expresar el vector PQ en fun-



cin de sus componentes cartesianas. Dicho vector no es el a , pero tiene su misma direc-
cin y sentido, de manera que si dividimos PQ por su mdulo, obtendremos un vector uni-

tario u de la misma direccin y sentido que a , despus, bastar multiplicar dicho vector

unitario por el mdulo de a , para tener resuelto el problema. Utilizaremos este segundo
mtodo por su mayor sencillez:

El vector PQ se puede expresar de forma analtica sin ms que restar las coordenadas del
punto P (origen) de las coordenadas del punto Q (extremo):

PQ = (Qx - Px, Qy - Py, Qz - Pz) = (3-0, 0, 0-4) = (3, 0, -4) U.I.

PQ 32 42 25 5 U.I.

PQ 3, 0,4
u 0'6, 0, - 0'8 U.I.
PQ 5

Luego el vector a ser a = a u = 15 (06, -08) = (09, -12) U.I.

18. Dado el vector: a 3i 2 j 3k , determinad el vector de igual direccin, sentido
contrario y mdulo 4.

sol: (-12/ 22 , 8/ 22 , -12/ 22 )


19. Dados los vectores: a i 2 j 3k y b i 2 j 5k , calculad su producto esca-

lar y la proyeccin de a sobre b .

sol: a b 12 ; proyeccin =12/ 30


20. Dados los vectores: a 2i j 3k y b i j , calculad el ngulo que forman
entre s.
sol: 5540

21. Demostrad que a 2i 4 j k y b 3i j 2k son perpendiculares.
Vectores 431

22. Calculad un vector unitario que sea perpendicular al plano definido por los vecto-

res concurrentes: a i j k y b i 2 j 3k .

Hemos de obtener un vector que cumpla dos condiciones: que sea perpendicular al plano
formado por otros dos y que tenga de mdulo 1. Para que cumpla la primera, basta con efec-

tuar el producto vectorial de los dos vectores que figuran en el enunciado, c a xb ya que
siempre que se realiza este tipo de producto entre dos vectores se obtiene un vector perpen-
dicular al plano en el que estn contenidos ambos. En cuanto a la segunda condicin, solo

habr que dividir el vector c por su propio mdulo.


i j k

c a x b 1 1 1 ( 3 2) i (3 1) j (2 1) k 5 i 2 j 3 k
1 2 3

c cx2 cy2 cz2 25 4 9 38


c 5i 2 j 3k 5 2 3
u i j k
c 38 38 38 38

Cmo podramos analizar el resultado para comprobar que el vector obtenido es efecti-
vamente perpendicular a los otros dos?

Podemos utilizar el producto escalar entre el vector u y los vectores a y b :

Como u a = u a cos ( es el ngulo que forman u y a ) si u y a son perpendiculares
su producto escalar tendra que ser 0 (y anlogamente con el vector b).

En efecto:

u a 5 / 38, 2 / 38 , 3 / 38 1, 1, 1 5 / 38 2 / 38 3 / 38 0


Anlogamente ocurrir si efectuamos el producto escalar u b


23. Dados los vectores concurrentes: a i 2 j 3k U.I y b i 5 j 2k U.I., calcu-
lad la superficie del paralelogramo que determinan.

sol: 411 UI.


432 Vectores


24. Hallad el producto escalar a b , el mdulo del vector c a x b , y representad dicho

vector c , en cada uno de los siguientes casos (en todos ellos a = 3 y b = 4 ):

1) 2) 3)
b a
a b 90
a 150 b

4) 5)
a
150
a b
b

Se trata este de un ejercicio de manejo que presenta un doble inters:

En primer lugar contribuye a comprender las diferencias existentes entre el producto escalar
y el producto vectorial.

En segundo lugar, constituye un ejemplo concreto en donde se puede apreciar la importan-


cia que tiene en el resultado del producto (escalar o vectorial), no solo el mdulo de los vec-
tores implicados, sino tambin su direccin y sentido.

Para resolver el problema hay que tener presentes las formas de calcular el producto escalar
y vectorial en funcin de los mdulos y del ngulo que forman los dos vectores que se mul-
tiplican (para lo cual es conveniente situarlos con origen comn):

Producto escalar: a b = a b cos

Producto vectorial: c a x b ; vector de mdulo c = a b sen ; direccin perpendi-

cular al plano formado por a y b y sentido segn el avance de un tornillo que girase en el

mismo sentido que resulta de llevar a sobre b por el camino ms corto.

A continuacin resolveremos y comentaremos brevemente cada uno de los cinco casos pro-
puestos.

1)

a b a b = a b cos = 3 4 cos 0 = 12

c a x b = 0 porque c = a b sen = 3 4 sen 0 = 0
Vectores 433

Vemos que el producto vectorial de dos vectores paralelos ( = 0) es nulo, mientras que el
escalar toma su valor mximo (ya que cos = cos 0 = 1). Puede utilizarse para comprobar
si dos vectores son paralelos.

2) a
30 a b = a b cos = 3 4 cos 30 = 1039
b c = a b sen = 3 4 sen 30 = 6


Aqu conviene darse cuenta de que cuando se determina el sentido del vector c , es necesa-
rio que, si no lo estn ya, se dispongan los dos vectores de forma que tengan un origen
comn trasladando uno de ellos (manteniendo su mdulo, direccin y sentido). Esto es lo

que se ha hecho
aqu con el vector a que se ha desplazado hasta que su origen ha coincidido
con el de b .

3)
a
90 a b = a b cos = 3 4 cos 90 = 0
b c = a b sen = 3 4 sen 90 = 12

En este caso el producto escalar es nulo, mientras que el mdulo del



vector c a x b toma su valor mximo. Puede utilizarse para compro-
bar si dos vectores son perpendiculares.

c
4)
150 a b = a b cos = 3 4 cos 150 = -1039
b a c = a b sen = 3 4 sen 150 = 6


En este caso al llevar a sobre b por el camino ms corto (una vez que hemos comprobado
que ambos vectores tienen el mismo origen), vemos que equivale al movimiento de extraer
un tornillo, de modo que el vector c, se dirige ahora hacia arriba.

Tambin hay que resaltar que cuando el ngulo es mayor de 90, el producto escalar sale
negativo, como ocurre en este caso:

5) 180
a
b a b = a b cos = 3 4 cos 180 = -12
c = a b sen = 3 4 sen 180 = 0
434 Vectores

En este ltimo caso, podemos comprobar que cuando el ngulo es de 180 (vectores opues-
tos), el producto escalar toma el valor mximo negativo, y el vectorial es nulo.

As pues, el mdulo del producto vectorial de dos vectores que mantengan sus mdulos
constantes, vara con el ngulo existente entre ambos vectores (dispuestos con origen
comn) entre un valor mnimo de 0 (para un ngulo de 0 o 180) y un valor mximo igual
al producto de sus mdulos a b (para un ngulo de 90), sin que se den valores negativos
(el mdulo de un vector no puede ser negativo).

En cuanto al producto escalar de dos vectores a y b , ste oscila entre un valor mximo da-
do por a b cuando el ngulo es 0 (vectores paralelos) y un valor mnimo que es el ante-
rior pero negativo (- a b) cuando el ngulo es de 180, pasando por el valor 0 en el caso de
que sean perpendiculares.


25. Suponiendo que los vectores a y b del ejercicio nmero 2, representen fuerzas
(medidas en N), calculad el momento de cada uno de dichos vectores respecto del pun-
to de coordenadas: (1,-1,0) m.

sol: Ma 25i 10 j 7k Nm; Mb 0

26. Demostrad que el momento de un vector respecto de un punto permanece cons-


tante, si dicho vector se desplaza a lo largo de su recta de aplicacin.


Consideremos un vector a aplicado en un punto O y el mismo vector desplazado hasta otro
punto O de su misma recta de aplicacin.

Cul sera la expresin del momento de dicho vector respecto del punto P en cada uno de
esos puntos?

Para el punto O, tendramos que : M O PO x a

Para el punto O tendramos que: M O ' PO' x a

Cmo podramos relacionar los momentos anteriores entre s?


Vectores 435

En la figura, podemos ver que: PO PO ' O' O



sustituyendo esta expresin en M O PO x a nos queda que:

M O PO ' O' O x a PO' x a O' O x a .



Como los vectores O' O y a tienen la misma direccin, su producto vectorial ha de ser nulo
y por tanto nos queda que:



M O PO ' O' O a PO' a M O' tal y como queramos demostrar.


27. Dados los vectores de la figura y sabiendo que el mdulo de a es 77 / 4 N, de-

terminad: a) Cosenos directores de b . b) Vector a en componentes. c) ngulo forma-

do por los vectores a y b . d) Momento de b respecto al punto Q (-1, 5, 2) m.

sol: a) cos = 07; cos = 057; cos = -042.



b) a (2'5, 3, 2) N
c) = 779
d) M (11, 8, 29) N m

28. Determinad el momento de la fuerza F de mdulo 200 N, respecto del punto A
de la figura adjunta (L en m).
436 Vectores

sol: ( 10L, 10L,10L) Nm


29. Sabiendo que el vector r es funcin del escalar t en la forma: r (t2, -t, 2t-1) m (si

t en s); determinad la funcin: d r /dt.

sol: (2t, -1, 2) m/s


30. Dado el vector a 3t 2 i 5 j 2t k , se pide:

da
a) Obtened el vector
dt
b) ngulo que forman entre s ambos vectores para t = 1.

da
sol: = (6t, 0, -2); = 556
dt


31. Demostrad que si un vector v cambia continuamente con el tiempo, pero su mdulo
dv
permanece constante en todo momento, el vector a y el vector v han de ser per-
dt
pendiculares.

Un vector viene definido por un mdulo, una direccin y un sentido, de modo que cuando
cambia alguno de estos tres elementos lo que se tiene es otro vector diferente del primero.

En ocasiones la direccin de un vector puede cambiar continuamente con el tiempo y su



mdulo permanecer constante. Este es el caso, por ejemplo, del vector velocidad v en un
Vectores 437

movimiento circular uniforme, tal y como se representa en la figura siguiente (en donde se
ha dibujado el vector velocidad para cuatro posiciones distintas):

Aunque el cuerpo gire siempre igual de aprisa, la velocidad va cambiando continuamente


con el tiempo (aunque su mdulo permanezca constante) y los vectores velocidad represen-

tados en la figura son diferentes ( v1 v2 v3 v4 ).

La demostracin que se demanda en el enunciado de este problema, es una aplicacin del


producto escalar de dos vectores:

Si hacemos el producto v v v 2 y despus derivamos con respecto al tiempo, nos queda:

dv dv
v v 0
dt dt

ya que al tratarse de un movimiento circular y uniforme, el mdulo de v es constante (v =
constante) por lo que tambin lo ser su cuadrado, y la derivada de una constante es 0.

Operando en la expresin anterior:



dv dv
2 v 0 v 0
dt dt

dv
Al ser el producto escalar nulo, los vectores y v tienen que ser perpendiculares entre s.
dt
Este resultado permite comprender mejor el hecho (ya estudiado en cursos anteriores) de

dv
que en un movimiento circular uniforme, el vector aceleracin ( a ) y el vector veloci-
dt
dad v , sean en todo momento perpendiculares.
438 Vectores


32. Dado el vector r R cos wt i sen wt j


dr
a) Calculad el mdulo de r y tambin el vector
dt

b) ngulo que forman r y dr / dt


Si r R cos wt i sen wt j lo representamos en unos ejes XOY, tendremos:

r
Rsenwt
wt

Rcoswt X

Como se aprecia en la figura, al variar t va modificndose el ngulo y en consecuencia las


componentes del vector. As pues la direccin del vector va cambiando, pero qu sucede
con su mdulo? Si queremos evaluarlo tendremos:

r R 2 sen 2 wt R 2 cos2 wt R 2 (sen 2 wt cos2 wt ) R 2 R

Se trata pues de un mdulo constante y por tanto, el extremo del vector que hemos dibujado
en la figura anterior, describe una trayectoria circular, de modo que aunque su mdulo no
vare, el vector s que cambia con el tiempo. Como r es constante, su derivada temporal dr/dt
ser 0.

dr
Podemos calcular: Rw sen wt i Rw cos wt j .
dt

dr
Como r es constante, los vectores r y tienen que ser perpendiculares (tal y como hemos
dt
visto en el problema anterior).

Cmo podramos comprobar que, efectivamente, son perpendiculares?

Bastara multiplicar escalarmente los dos vectores y ver si el resultado es 0. En efecto:


Vectores 439


R cos wt i sen wt j ( Rw sen wt i Rw cos wt j )
dr
r
dt

dr
r = R2 w cos(wt) sen(wt) - R2 w cos(wt) sen(wt) = 0.
dt

dr dr
Como r =r cos = 0,
dt dt

nos queda que cos = 0 y por tanto los dos vectores son perpendiculares.

33. Cmo se puede expresar un vector en sus componentes intrnsecas? En qu casos


interesa trabajar con los vectores expresados en dichas componentes?

En el caso de objetos que se mueven siguiendo una trayectoria previamente conocida como,
por ejemplo, un coche por una carretera o el movimiento circular de un satlite alrededor de
la Tierra, puede interesarnos expresar los vectores en funcin de dos nuevos vectores bsi-

cos unitarios: uno tangencial, que designaremos por y que siempre es tangente a la tra-
yectoria y en el sentido positivo (valores de e crecientes), y otro perpendicular, que desig-

naremos por n y que siempre es perpendicular a la trayectoria y en el sentido escogido co-
mo positivo. En el caso de que la trayectoria sea curva, se conviene que el sentido positivo

de n sea, precisamente, sealando hacia el centro de la curva (si es una recta es arbitrario).

En los dos ejemplos siguientes se representa el caso de dos cuerpos que estn movindose
segn una trayectoria curva. El de la izquierda va cada vez ms aprisa, mientras que el de la
derecha va cada vez ms despacio. En cada uno de los esquemas se ha escogido (arbitrariamen-
te) como sentido positivo de la trayectoria el del movimiento y se han dibujado los vectores

unitarios y n , junto con el vector v (velocidad) y el vector a (aceleracin) en cada caso.

Si nos damos cuenta, el vector aceleracin puede ser expresado en los dos casos en funcin de

sus componentes intrnsecas (aceleracin tangencial y aceleracin normal) a at an n en
donde las componentes at y an son escalares.

De acuerdo con los esquemas utilizados el mdulo del vector a se puede calcular como:
440 Vectores

a at2 an2

34. Una persona arrastra por el suelo un trineo de 500 kg mediante una cuerda que
forma un ngulo de 30 con la horizontal. Sabiendo que el trineo se desplaza con mo-
vimiento rectilneo y uniforme y que la fuerza de rozamiento vale 897 N, determinad
las fuerzas actuantes sobre el mismo en componentes intrnsecas.


La fuerza peso ejercida por
la Tierra, la fuerza que hace la cuerda T , la fuerza de rozamien-
to Fr y la fuerza normal R que ejerce el plano. Estas son pues las fuerzas que hemos de de-
terminar. Para ello hemos de tener en cuenta que:
El trineo se desplaza con movimiento rectilneo y uniforme y, en consecuencia no habr
aceleracin (vector) por lo que la fuerza resultante sobre el trineo ser nula:

Fres P + T + Fr + R = 0

Para expresar en componentes intrnsecas las fuerzas anteriores haremos lo siguiente:

a) Tomar sobre la trayectoria un origen de espacios y escoger un sentido como positivo.



b) Establecer los vectores y n . Recordemos que es un vector unitario tangente a la trayec-

toria y siempre en el sentido escogido como positivo mientras que n se define como un vector
unitario siempre perpendicular a la trayectoria y en el sentido positivo de esa direccin.
c) Expresar las fuerzas actuantes en componentes intrnsecas, segn los vectores unitarios
correspondientes.

Si representamos las fuerzas y tomamos sentido positivo el del movimiento, tenemos:

R T n
Fr (+)
30

O
P

Como puede verse se han dibujado los vectores unitarios desplazados a la derecha para no
complicar la figura. De acuerdo con el enunciado el mvil se desplazar hacia la derecha
(sentido que hemos escogido como positivo). Si denominamos al ngulo que forma un
Vectores 441


vector con el vector unitario por el camino ms corto y al que forma con el vector uni-

tario n (tambin por el camino ms corto), podemos escribir que:


P= P cos + P cos n = P cos 90 + P cos 180 n = -5000 n

T = T cos + T cos n = T cos 30 + T cos 60 n = 087 T + 05 T n

R= R cos + R cos n = R cos 90 + R cos 0 n = R n

Fr = Fr cos + Fr cos n = Fr cos 180 + Fr cos 90 n = -897

Sumando obtenemos la expresin:



Fres = (087 T - 897 ) + ( R + 05 T - 5000) n = 0

y descomponiendo esta ecuacin:



087 T - 897 = 0

R + 05 T - 5000 = 0


Resolviendo el sistema anterior obtenemos que: T =10308 N y R = 44846 N.

As pues, las fuerzas (en newtons) que actan sobre el trineo en componentes intrnsecas
sern:

P = -5000 n ; T = 897 + 5154 n ; R = 44846 n ; Fr = = -897 . Tambin es posible:

P = (0 , -5000) N; T = (897 , 5154) N; R = (0 , 44846) N; Fr = (-897 , 0) N

Conviene tener en cuenta que las componentes anteriores no son cartesianas sino intrnse-
cas. Una de las ventajas de trabajar con este tipo de componentes es que pueden utilizarse
tambin en trayectorias no rectilneas, como vamos a ver a continuacin.

35. Un bloque de masa mA se halla unido a otro de masa


mB mediante una cuerda que pasa por la garganta de
una polea tal y como se observa en el sistema de la figu-
ra adjunta, de forma que el bloque B se mueve hacia la
derecha y el A desciende. Sabiendo que entre el bloque
B y el plano existe rozamiento y considerando la masa
de la polea despreciable, se pide:

a) Dibujad todas las fuerzas que actan sobre cada uno de los bloques.
b) Expresad cada una de dichas fuerzas en componentes intrnsecas.
442 Vectores

c) Obtened la expresin analtica de la fuerza resultante que acta sobre el sistema


formado por ambos bloques.

Analizando la figura vemos que podemos considerar que existe una sola trayectoria y que
est previamente determinada, siendo independiente de las caractersticas del movimiento
de cada uno de los bloques. Es decir, el bloque A descender y el B se mover sobre la me-
sa, ambos en el mismo sentido y siguiendo la trayectoria que contiene a la cuerda. De
acuerdo pues con lo que sabemos, ser posible abordar el problema escalarmente. Para ello
vamos a escoger como sentido positivo el del movimiento.

En el esquema siguiente hemos dibujado todos los vectores fuerza que intervienen en el

problema. Por comodidad hemos situado los vectores unitarios y n desplazados de los
bloques (aunque ya sabemos que ambos vectores se sitan en cada instante o punto de la
trayectoria en el centro de cada bloque). Para contestar la cuestin que se plantea en el
enunciado, podemos considerar todo el sistema en su conjunto (bloques y cuerda), expresar
cada una de las fuerzas en componentes intrnsecas y sumar.

(-) O RB
Fr TB n
PA = ( PA , 0)

PB T A = (-TA , 0)

T B = ( TB , 0)
TA
Fr = (-Fr , 0)

R B = ( 0 , R)
PA

P B = (0 , -PB)
(+)

En los vectores anteriores se dan las componentes escalares de cada uno ya con el signo
correspondiente (segn el criterio de signos especificado en la figura). Conviene darse cuen-

ta que, por ejemplo, los vectores PA y TB tienen slo componente tangencial y en ambos
casos es positiva. Sumando todos los vectores anteriores obtenemos la expresin de la
fuerza resultante sobre el sistema en componentes intrnsecas:

Fres = (PA TA + TB Fr , R PB)

You might also like